Sunteți pe pagina 1din 250

FISICA 1

Autor: Hugo Medina Guzmn Profesor de la Pontificia Universidad Catlica del Per Agosto 2009

PRESENTACIN
Me agrad saber que Hugo Medina Guzmn estaba por publicar un texto sobre Fsica. Haba dos razones suficientes para este sentimiento. Por un lado, tena curiosidad de saber lo que podra aportar un texto ms de Fsica sobre los otros ya disponibles. Por otro lado, conozco de la larga carrera de Hugo Medina como cultor de la enseanza de [a Fsica, y tena curiosidad de ver cmo este compromiso como docente y experiencia se manifestaran en su texto. Tuve la suerte de conocer al Ing. Jos Castro Mendvil en su taller, donde despleg una destacada labor en el diseo y construccin de equipo de laboratorio para la enseanza de la Fsica. Considero que Hugo es un digno discpulo del Ing. Castro Mendvil e igualmente ha dedicado una fraccin considerable de su tiempo a la docencia, y al diseo y construccin de equipo de laboratorio para resaltar los conceptos bsicos de la Fsica. He revisado el contenido de este texto y veo con gran satisfaccin que su autor utiliza un enfoque muy acertado. Toma como punto de partida una observacin experimental y a partir de all desarrolla los conceptos fsicos que permiten interpretar esta observacin utilizando la formulacin matemtica ms sencilla. Todo esto lo hace con el detalle suficiente de manera que el lector pueda seguir el argumento lgico con facilidad. Considero que ste es un gran aporte de este texto. Este enfoque contrasta con textos que enfatizan la formulacin matemtica y dejan al alumno hurfano de una orientacin para aplicarla a una realidad fsica concreta. El contenido de temas de la Fsica General que son desarrollados en este texto se ajusta al programa de estudios de la PUCP. El desarrollo de cada tema incluye ejemplos bien seleccionados que son desarrollados con un detalle muy esmerado. Al final de cada captulo se incluye un conjunto de preguntas y problemas propuestos; se incluye las respuestas. Algunos problemas plantean configuraciones complejas pero que contienen ciertas propiedades de simetra que permiten su reduccin a configuraciones sencillas. Al final del texto encontramos un listado de referencias bibliogrficas a un buen nmero de textos de Fsica General que han servido de consulta al autor. En general, considero que este texto constituye una representacin grfica de la obra cotidiana que Hugo ha venido desarrollando durante su carrera docente y, por lo tanto, es un aporte muy valioso para la comunidad acadmica y pblico en general. Lima, julio de 2007

PRLOGO
Los estudiantes a menudo se preguntan por qu llevan un curso de Fsica. La mejor razn por la que se estudia Fsica es porque proporciona un mtodo coherente y lgico para comprender el mundo que nos rodea; una persona que comprende lo que sucede a su alrededor, es capaz de convivir en su entorno de manera racional y efectiva. Sin embargo, en ocasiones los estudiantes ignoran el potencial que tiene la Fsica para explicar el entorno en trminos fciles de entender; Este libro tiene por objeto brindar a los estudiantes de la Fsica General una ayuda para dominar los principios fsicos que son la base de la tecnologa moderna. En ste libro se asume que los estudiantes tienen una base de lgebra, geometra, y trigonometra. Es mucho ms compacto que los libros de texto tradicionales, proporciona muchos ejemplos trabajados y pide resolver problemas Este libro ser til tambin como texto para una persona que repasa o que consolida su conocimiento de la Fsica. La discusin y las explicaciones narrativas son suficientemente claras y completas para poder utilizar el libro o como texto, o como suplemento a un texto ms amplio. La forma de aprender la fsica es trabajar realmente con problemas. Al usar este libro, el estudiante debe ser activo. Debe intentar trabajar cada uno de los problemas y los ejemplos. Debe mirar las soluciones solamente si no logra dar con el camino a su solucin. Los ejemplos en este libro estn trabajados exhaustivamente, de modo que puedan servir como modelos para el propio trabajo de los estudiantes. En este sentido se considera que los estudiantes se benefician al observar los clculos realizados en ms de una manera, por lo que se han incluido varios mtodos para efectuar los clculos. Adems, se tuvo especial cuidado en incluir problemas y preguntas que combinan el material del captulo en cuestin, con material de captulos anteriores. Tales problemas y preguntas destacan el hecho importante de que diversas reas de la Fsica se manifiestan de manera simultnea en el mundo real. Adems, este mtodo de temas mltiples proporciona una manera para que los estudiantes repasen lo estudiado y ayuda a mejorar la habilidad para resolver problemas. El diseo grfico es de gran importancia, y para mejorar su funcin se ha intentado enfocar solamente una idea principal en cada figura en lo posible. Por consiguiente, las figuras del libro a menudo se dividen en dos o ms partes, para evitar la confusin de mezclar varias ideas en la misma figura. Los profesores conocen la importancia de los diagramas de cuerpo libre cuando utilizan la segunda ley de movimiento de Newton, y todos los estudiantes aprenden de ellos a medida que estudian Fsica. Tales diagramas se utilizan en todo el libro, no solamente en los primeros captulos en los que se presenta y aplica la segunda ley de Newton. Por ejemplo, cuando se analiza la relacin en las oscilaciones, tambin entre la presin y profundidad en un fluido, el anlisis se simplifica considerablemente por medio de un diagrama de cuerpo libre. De manera semejante, cuando se deduce la expresin para la rapidez de una onda transversal en una cuerda, un diagrama de cuerpo libre es muy til. Cifras significativas. A lo largo de todo el libro se siguen los procedimientos normales para las cifras significativas. Se espera que el esfuerzo en la elaboracin de este libro sea de utilidad tanto para los estudiantes como para los profesores. Toda opinin al respecto ser bienvenida. Hugo Medina Guzmn Lima Per

AGRADECIMIENTOS
El autor agradece primeramente a los estudiantes, quienes han contribuido bastante en la elaboracin de este libro a travs de su influencia en el establecimiento de las tcnicas y principios de enseanza y a los profesores que con sus sugerencias y revisiones a las separatas de los captulos hicieron notar puntos que necesitaban una mayor aclaracin.
Hugo Medina Guzmn

CONTENIDO
CAPTULO 1. Unidades, magnitudes fsicas y vectores Introduccin al curso. Magnitudes fsicas: escalares y vectores. Unidades. Sistema internacional de unidades. Precisin y cifras significativas. CAPTULO 2. Movimiento rectilneo Definicin de partcula. Concepto de movimiento de traslacin y rotacin. Sistemas de referencia. Posicin y desplazamiento. Movimiento en una dimensin. Velocidad. Aceleracin. Movimiento con aceleracin constante. Movimiento vertical con aceleracin de la gravedad. Grficos en cinemtica: obtencin de la velocidad y de la aceleracin por derivacin de la funcin posicin versus tiempo, obtencin de la velocidad y de la posicin por integracin de la funcin aceleracin versus tiempo. CAPTULO 3. Movimiento en un plano y en el espacio Sistemas de referencia y el sistema de coordenadas cartesianas en dos dimensiones. Componentes de los vectores y vectores unitarios en coordenadas cartesianas. Adicin vectorial. Movimiento en un plano. Vector posicin, desplazamiento y trayectoria. Velocidad. Rapidez. Aceleracin. Movimiento parablico. Movimiento circular: descripcin horaria (posicin, velocidad y aceleracin angular) y descripcin vectorial cartesiana. Componentes normal y tangencial de la aceleracin. Velocidad y aceleracin relativas. Generalizacin del movimiento a tres dimensiones en coordenadas cartesianas. CAPTULO 4. Dinmica de una partcula Leyes de Newton del movimiento. Sistemas de referencia inerciales. Masa y fuerza. Masa y peso. Fuerzas de contacto y a distancia (Ley de gravitacin universal). Diagrama de cuerpo libre. Aplicaciones de las leyes de Newton: partculas en equilibrio (Esttica) y en movimiento acelerado (Dinmica), fuerzas de friccin. Dinmica del movimiento circular. Dinmica en sistemas de referencia no inerciales. CAPTULO 5. Trabajo y energa Producto escalar de vectores. Trabajo de una fuerza. Energa cintica. Trabajo y energa cintica. Fuerzas conservativas y no conservativas. Energa potencial gravitacional y elstica. Energa mecnica. Generalizacin de la ley de conservacin de la energa mecnica. Potencia. CAPTULO 6. Sistema de partculas Centro de masa. Posicin, velocidad y aceleracin del centro de masa. Cantidad de movimiento lineal de una partcula y de un sistema de partculas. Impulso de una fuerza. Segunda ley de Newton y la conservacin de la cantidad de movimiento lineal para un sistema de partculas. Energa cintica de un sistema de partculas. Colisin elstica e inelstica. CAPTULO 7. Cuerpo rgido Producto vectorial. Torque. Segunda condicin de equilibrio (Esttica del cuerpo rgido). Cantidad de movimiento angular. Momento de inercia. Rotacin alrededor de un eje fijo. Conservacin de la cantidad de movimiento angular. Energa en el movimiento de rotacin. Energa cintica de rotacin. Rodadura.

CAPITULO 1 INTRODUCCIN AL CURSO


QUE ES LA FISICA? METODOLOGIA DE LA FISICA PARTES DE LA FISICA MAGNITUDES FSICAS: ESCALARES Y VECTORES. UNIDADES. SISTEMA INTERNACIONAL DE UNIDADES. MEDICIN. UNIDADES. Unidades fundamentales Unidades derivadas Prefijos comnmente encontrados. CONVERSION DE UNIDADES Factores de Conversin ANALISIS DIMENSIONAL a) Verificacin de una frmula especfica. b) Desarrollo de ecuaciones. c) Convertir un sistema de unidades a otro. CIFRAS S1GNIFICATIVAS Regla 1: Redondeo de un nmero Regla 2: Suma y Resta Regla 3: Multiplicacin y Divisin ERRORES Error absoluto Error relativo Porcentaje de error Clasificacin de errores. a) Error inherente b) Error de truncado c) Error de redondeo d) Error de interpolacin e) Error de aproximacin PROPAGACION ERRORES a) Suma de dos o ms variables. b) Diferencia de dos variables. c) Producto de dos o ms variables. d) Potencias y races. e) Cocientes. PRECISIN Y EXACTITUD RANGO DE ERROR O INCERTIDUMBRE ESTIMADOS Y CLCULOS DEL ORDEN DE MAGNITUD MODELOS IDEALIZADOS COMO ESTUDIAR FISICA? PREGUNTAS Y PROBLEMAS 1 1 1 1 2 2 2 2 3 3 3 3 4 4 4 4 5 6 6 6 6 7 7 7 7 7 7 7 7 7 8 9 9 9 10 10 11 11 12 13 13 14

CAPITULO 2

Movimiento rectilneo

DEFINICIN DE PARTCULA CONCEPTO DE MOVIMIENTO DE TRASLACIN Y ROTACIN CONCEPTO DE MOVIMIENTO CLASIFICACIN DEL MOVIMIENTO SISTEMAS DE REFERENCIA. POSICIN Y DESPLAZAMIENTO Sistemas de referencia Vector Posicin Desplazamiento Trayectoria y Ecuacin Horaria del Movimiento VELOCIDAD Y RAPIDEZ Rapidez Derivadas de algunas funciones Velocidad Velocidad instantnea ACELERACIN Aceleracin Media Aceleracin Instantnea o simplemente aceleracin MOVIMIENTO RECTILNEO UNIFORME MOVIMIENTO RECTILNEO UNIFORMEMENTE VARIADO La Ecuacin de Torricelli MOVIMIENTO VERTICAL CON ACELERACIN DE LA GRAVEDAD. a) Cada libre b) Lanzamiento hacia arriba c) Lanzamiento hacia abajo PROBLEMA INVERSO - CLCULO INTEGRAL Pequea Tabla de Integrales CINEMTICA DE PARTCULAS LIGADAS. MOVIMIENTOS DEPENDIENTES. PREGUNTAS Y PROBLEMAS

1 1 1 1 1 1 2 2 2 3 3 4 4 5 6 6 7 8 8 9 11 12 12 12 18 19 21 23

CAPITULO 3 Movimiento en un plano y en el espacio

MOVIMIENTO CIRCULAR Posicin angular Velocidad angular Aceleracin angular RELACIN ENTRE LAS MAGNITUDES ANGULARES Y LINEALES Hallar el desplazamiento angular a partir de la velocidad angular. Hallar el cambio de velocidad angular a partir de la aceleracin angular. MOVIMIENTO CIRCULAR UNIFORME MOVIMIENTO CIRCULAR UNIFORMEMENTE ACELERADO COMPONENTES NORMAL Y TANGENCIAL DE LA ACELERACIN Velocidad. Aceleracin. MOVIMIENTO CURVILNEO El radio de curvatura MOVIMIENTO PARABLICO Ecuacin de la trayectoria Tiempo de vuelo El alcance horizontal La altura mxima VELOCIDAD Y ACELERACIN RELATIVAS Movimiento Relativo de Traslacin Uniforme. La Relatividad de Galileo PREGUNTAS Y PROBLEMAS

1 1 1 1 1 2 2 2 2 2 2 2 7 7 10 10 11 11 11 18 18 26

CAPTULO 4

Dinmica de una partcula

INTRODUCC1ON EL ORIGEN DEL MOVIMIENTO PRIMERA LEY DE NEWTON DEL MOVIMIENTO QU ES FUERZA? CAMBIO DE VELOCIDAD SEGUNDA LEY DE NEWON DEL MOVIMIENTO UNIDADES DE FUERZA Y MASA PESO DE UN CUERPO ACCION Y REACCIN TERCERA LEY DE NEWTON DEL MOVIMIENTO APLICACIONES DE LAS LEYES DE NEWTON ESTTICA DE LAS MASAS PUNTUALES. DINMICA CON FRICCIN DESPRECIABLE. FRICCIN Algunos valores tpicos de coeficientes de friccin DINMICA DEL MOVIMIENTO CIRCULAR FUERZA CENTRPETA CURVAS EN LAS PISTAS MOVIMIENTO EN MARCOS DE REFERENCIA NO INERCIALES MARCO CON MOVIMIENTO DE TRASLACION NO UNIFORME MARCO DE ROTACIN FUERZA CENTRFUGA FUERZA DE CORIOLIS PREGUNTAS Y PROBLEMAS

1 1 1 1 2 3 3 4 3 4 4 4 7 11 13 27 27 32 34 34 37 38 39 40

CAPITULO 5

TRABAJO Y ENERGA

INTRODUCCION TRABAJO ENERGIA CINETICA SISTEMAS CONSERVATIVOS Y NO CONSERVATIVOS LA FUNCION ENERGA POTENCIAL CONSERVACION DE LA ENERGA Observadores en movimiento relativo SISTEMAS NO CONSERVATIVOS LA CONSERVACIN DE LA ENERGA Y LA FRICCIN POTENCIA MAQUINAS PREGUNTAS Y PROBLEMAS

1 1 4 6 8 9 13 15 16 16 18 19

CAPTULO 6 SISTEMA DE PARTCULAS


INTRODUCCION SISTEMA DE PARTICULAS SEGUNDA LEY DE NEWTON APLICADA A UN SISTEMA DE PARTICULAS CENTRO DE MASA MOVIMIENTO DEL CENTRO DE MASA. IMPULSO Y CANTIDAD DE MOVIMIENTO CONSERVACIN DE LA CANTIDAD DE MOVIMIENTO SISTEMA DE REFERENCIA CENTRO DE MASA CHOQUES CASOS DE CHOQUE El pndulo balstico MOVIMIENTO CON MASA VARIABLE - PROPULSIN POR REACCIN CANTIDAD DE MOVIMIENTO ANGULAR Y TORQUE MOMENTO DE INERCIA MOMENT0 DE UNA FUERZA o TORQUE CONSERVACION DE LA CANTIDAD DE MOVIMIENTO ANGULAR CONSERVACION DE LA CANTIDAD DE MOVIMIENTO ANGULAR DE UN SISTEMA DE PARTICULAS. PREGUNTAS Y PROBLEMAS 1 1 1 2 2 4 6 9 9 11 18 20 22 23 23 24 26 30

CAPTULO 7 CUERPO RGIDO


INTRODUCCION CUERPO RIGIDO MOVIMIENTO DE UN CUERPO RGIDO TRASLACION ROTACIN CANT1DAD DE MOVIMIENTO ANGULAR DE UN CUERPO RGIDO MOMENTO DE INERCIA DEL CUERPO RGIDO. El teorema de Steiner o de los ejes paralelos. El teorema de la figura plana SEGUNDA LEY DE NEWTON PARA ROTACION Maquina de atwood tomando en cuenta la polea EQUILIBRIO ESTTICO TRABAJO Y ENERGIA EN ROTACIN POTENCIA TRASLACIONES Y ROTACIONES COMBINADAS CONSERVACION DE LA CANTIDAD DE MOVIMIENTO ANGULAR GIROSCOPOS Y TROMPOS - MOVIMIENTO DE PRECESION PREGUNTAS Y PROBLEMAS 1 1 1 1 1 2 2 2 2 5 7 11 15 16 24 35 43 44

BIBLIOGRAFA

THEORETICAL PHYSICS, Mechanics of particles, rigid and elastic bodies, fluids and heat flow. F: Woobridge Constant. Trinity College. Addison Wesley Publishing Company (1959) THEORETICAL PHYSICS,Thermodinamics, electromagnetism,waves, and particles. F: Woobridge Constant. Trinity College. Addison Wesley Publishing Company (1959)
The Feynman LECTURES ON PHYSICS. Volumenes I, II y III. Richard P.Feynman, Robert B. Leighton. California Institute of Technology, Matthew Sands, Stanford University. Addison Wesley Publishing Company (1964) CORRIENTES, CAMPOS Y PARTCULAS. Francis Bitter. Massachussets Institute of Technology. Editorial Revert S. A. (1964).

INTRODUCCIN AL ESTUDIO DE LA MECNICA, MATERIA Y ONDAS. Uno Ingard, William L. Kraushaar. Editorial Revert. (1966). FUNDAMENTOS DE ELECTRICIDAD Y MAGNETISMO. Arthur F. Kip. University of California. Mc Graw Hill Book Company (1967)
CIENCIA FSICA Orgenes y principios Robert T. Langeman, Universidad Vanderbilt. UTEHA, (1968)

PROBLEMS IN ELEMENTARY PHYSICS. B. Bukhotsev, V: Krivchenkov, G. Myakishev, V.Shalnov. Mir Publishers. Moscow (1971)
PROBLEMES DE PHYSIQUE COMMENTES. Tomos I y II Hubert Lumbroso. Mason et Cie, Pars. (1971)

ELECTRICIDAD Y MAGNETISMO PARA ESTUDIANTES DE CIENCIAS E INGENIERA. Luis L. Cant. Instituto Tecnolgico y de Estudios Superiores de Monterrey. Editorial Limusa Mexico (1973)
FSICA PARA LAS CIENCIAS DE LA VIDA Y LA SALUD. Simon G. G. MacDonald / Desmond M. Burns University of Dundee. Fondo educativo interamericano. (1975) MECNICA NEWTONIANA, MIT Physics course. A. P. French. Editorial Revert. (1974).

FSICA I y II. Solomon Gartenhaus. Purdue University. INTERAMERICANA. (1977)


TEACHING TIPS. A guidebook for the beginning College Teacher. Wilbert J. McKeachie (University of Michigan). Seventh edition D. C. Heath and Company (1978) FSICA PARA LAS CIENCIAS DE LA VIDA. Alan H. Cromer. Northeastern University. Editorial Revert. (1978) GENERAL PHYSICS WITH BIOSCIENCE ESSAYS. Jerry B. Marion. University of Maryland. John Wiley & Sons Inc. (1979) Fsica general II: Teora Hugo Medina Guzmn, Miguel Piaggio H. QC 21 M19 (Biblioteca PUCP) (1979) Fsica general II: Problemas resueltos Hugo Medina Guzmn, Miguel Piaggio H. FIS 111 M364 (Biblioteca PUCP) (1979) Fsica general I: problemas resueltos Hugo Medina Guzmn, Miguel Piaggio H. FIS 104 M364 (Biblioteca PUCP) (1981)

FSICA PARA ESTUDIANTES DE CIENCIAS E INGENIERA. 1 y 2. John P. McKelvey, Clemson University Howard Grotch, Pennsilvania State University. HARLA. Mexico. (1981)
Fsica 3: electricidad y magnetismo para estudiantes de ciencias e ingeniera Hugo Medina Guzmn, FIS 141 M36 (Biblioteca PUCP) (1982) EXPLORING PHYSICS Concepts and applications. Roger W. Redding North Texas State University, Stuart Kenter, Wadsworth Publishing Company (1984) PROBLEMAS DE FISICA. J. Aguilar Peris, Universidad Complutense de Madrid - J. Casanova Colas, Facultad de Ciencias de Valladolid. Alambra (1985) PROBLEMAS DE FISICA. dirigido por S. Ksel. Editorial Mir Mosc. (1986) PROBLEMAS DE FISICA Y COMO RESOLVERLOS. Clarence E. Benett Maine University. CECSA (1986) PHYSICS for Engineering and Science. Michael E. Browne, Ph. D. (professor of Physics University of Idaho. Schaums outline series Mcgraw-Hill (1988) FSICA: VOLUMEN 1. Mecnica, ondas y termodinmica. Duane E. Roller, Ronald Blum. Editorial Revert. (1990). FSICA: VOLUMEN 2. Electricidad, magnetismo y ptica. Duane E. Roller, Ronald Blum. Editorial Revert. (1990). PROBLEMAS DE FISICA. dirigido por O. Ya. Svchenko. Editorial Mir Mosc. (1989)

MECNICA. Berkeley physics course volumen 1. Charles Kittel, Walter D. Knight, Malvin A. Ruderman. Editorial Revert SA. (1992).

ELECTRICIDAD Y MAGNETISMO. Berkeley physics course volumen 2. Edward M.Purcell. Editorial Revert SA. (1992).
FSICA. Tomos I y II Tercera edicin revisada (Segunda edicin en espaol), Raymond S: Serway, James Madison University, Mcgraw-Hill, (1993) PROBLEMAS DE FISICA Santiago Burbano de Ercilla, enrique Burbano de Ercilla, Carlos Gracia Muoz, XXVI edicin, Zaragoza, MIRA editores (1994) ONDAS. Berkeley physics course volumen 3. Frank S. Crawford, Jr. Editorial Revert SA. (1994). FSICA Para las ciencias de la vida, David Jou Mirabent Universidad autnoma de Barcelona, Joseph Enric Llebot Rabagliati, Universidad de Girona, Carlos Prez garca, Universidad de Navarra. Mcgraw-Hill, (1994) Fsica uno Hugo Medina Guzmn, FIS 104 M365 (Biblioteca PUCP) (1995) APPLIED PHYSICS. Arthur Beiser, Ph. D. Schaums outline series Mcgraw-Hill (1995) TEACHING INTRODUCTORY PHISICS A Sourcebook. Clifford E: Swartz (State University of New York, Stony Brook) and Thomas Miner (Associate Editor The Physics Teacher 1972 1988). ATP Press Springer. (1996) TEACHING INTRODUCTORY PHYSICS Arnold Arons University of Washington JOHN WILEY & SONS, INC. (1997) FSICA John Cutnell / Kenneth W. Johnson. Southern Illinois University. LIMUSA (1998) FSICA EN LA CIENCIA Y EN LA INDUSTRIA. A . Cromer. Northeastern University. Editorial Revert. (2000) FSICA CONTEMPORANEA Edwin Jones. Richard Childers, University of South Carolina. McgrawHill, (2001) PROBLEMAS Y CUESTIONES DE FISICA. Atanasio Lle, Begoa Betete, Javier Galeano, Lourdes Lle, Ildefonso Ruiz Tapiador. Universidad Politcnica de Madrid. Ediciones Mundi prensa (2002) The PHYSICS of every day phenomena. A conceptual introduction to Physics. W. Thomas Griffith, Pacific University. Mcgraw-Hill, (2004) FSICA UNIVERSITARIA. Francis W.Sears, Mark W. Zemansky, Hugh D. Young (Carnegie Mellon University) y Roger A. Freedman (University of California. Santa Barbara) Volumen 1, Volumen 2. Undecima edicin. Pearson - Addison Wesley (2004) FIVE EASY LESSONS Strategies for successful Physics teaching. Randall D. Knight California Polytechnic State University, San Luis Obispo. Addison Wesley (2004) FUNDAMENTALS OF PHYSICS. David Halliday (Univ. of Pittsburgh), Robert Resnick (Rensselaer Polytechnic Institute), Jearl Walker (Cleveland State Univ.). 7th Edition (2005)

INTRODUCCIN AL CURSO

Hugo Medina Guzmn

Capitulo 1. INTRODUCCIN AL CURSO


QUE ES LA FSICA? La fsica es una ciencia dedicada a la comprensin de los fenmenos naturales que ocurren en el universo. El objetivo principal del estudio cientfico es desarrollar teoras fsicas basadas en leyes fundamentales que permitan predecir los resultados de algunos experimentos. Las leyes de la fsica tratan de describir los resultados de observaciones experimentales y de mediciones cuantitativas de los procesos naturales. La fsica es la ciencia ms simple porque estudia los sistemas ms simples. La fsica es la base de todas las dems ciencias. La relacin entre la fsica y la ingeniera es ms directa que la que existe entre la fsica y cualquier otra ciencia. En la ingeniera se trabaja con sistemas a los que se aplica inmediatamente los principios de la fsica. Cualquiera sea la rama de la ingeniera o de la ciencia a la que uno se dedique, va a encontrar a cada paso la aplicacin de las nociones que aprendi en la fsica. Siempre se encontrarn tiles los conceptos especficos de la fsica, las tcnicas que se emplean para resolver los problemas, la forma de pensar que se adquiere en el estudio de la fsica. METODOLOGIA DE LA FISICA La metodologa que se usa tiene tres formas caractersticas. La primera forma es el anlisis de un sistema fsico que se realiza en base a las propiedades de sistemas ms sencillos, estos sistemas estn relacionados de algn modo importante con el sistema original, pero poseen un nmero menor de factores en su comportamiento. Siendo estos ms sencillos se pueden investigar hasta entender bien sus propiedades, una vez que se obtenga el conocimiento de cada sistema se puede hacer una reconstruccin hasta lograr entender las propiedades del sistema original. La segunda forma parte del principio de que la fsica se fundamenta necesariamente en la experimentacin. A veces la teora sugiere el experimento, pero ms frecuentemente un experimentador realiza el trabajo inicial en un rea particular de la fsica y luego el fsico terico sintetiza los resultados de los experimentos y perfecciona el entendimiento de su significado. La tercera se refiere al uso frecuente de las matemticas. La fsica estudia las interacciones entre objetos. Los objetos interaccionan de acuerdo a ciertas leyes, sean estas conocidas o no. Como las leyes fsicas son casi siempre cuantitativas, es esencial poder establecer relaciones lgicas cuantitativas al estudiar los sistemas fsicos. Las reglas que gobiernan todas estas relaciones son objeto de las matemticas. Por eso se dice que la matemtica es el lenguaje de la fsica. PARTES DE LA FISICA 1 Actualmente la fsica se divide en dos clases: Fsica Clsica y Fsica Moderna. La fsica clsica se ocupa de los fenmenos y las leyes que se conocan hasta la final del siglo XIX. La fsica moderna se ocupa de los descubrimientos hechos desde entonces. La fsica clsica se subdivide en cierto nmero de ramas que originalmente se consideraban autnomas: la mecnica, el electromagnetismo, la ptica, la acstica y la termodinmica. La mecnica se ocupa del estudio del movimiento efectos fsicos que pueden influir sobre este. El electromagnetismo se ocupa del estudio de los fenmenos elctricos y magnticos y las relaciones entre ellos. La ptica se ocupa de los efectos fsicos que se asocian a la luz visible. La acstica al estudio de los efectos fsicos relacionados con los sonidos audibles. La termodinmica se ocupa de la generacin, el transporte y la disipacin del calor. Estas disciplinas que originalmente se desarrollaron independientemente, estn enlazadas por medio de la mecnica y el electromagnetismo. La fsica moderna se inici a fines del siglo XIX, con el descubrimiento de cierto nmero de fenmenos fsicos que entraban en conflicto con algunos conceptos de la fsica clsica. Bsicamente, esas alteraciones conceptuales fueron de dos tipos. Una de ellas estableci el lmite superior para las velocidades de las partculas a las que se aplicaban las leyes de la fsica clsica, esto se asocia a la Teora de la Relatividad de Einstein. El segundo se puede considerar como el establecimiento de un lmite inferior para las dimensiones lineales y de masa de los sistemas fsicos, para los que son vlidas las leyes clsicas, esto se asocia a la Teora de la Mecnica Cuntica. Para poder comprender estas dos teoras modernas y los fenmenos de que se ocupan, es necesario estudiar primeramente las leyes de la fsica clsica. MAGNITUDES FSICAS: ESCALARES Y VECTORES. En la descripcin y estudio de los fenmenos fsicos se han desarrollado (y se desarrollan) conceptos abstractos muy especiales llamados magnitudes fsicas. Estas magnitudes se definen por medio de un conjunto de operaciones experimentales que permiten obtener un nmero como medida de la magnitud en cualquier situacin. Esta definicin comprende dos pasos esenciales: 1) La eleccin de una unidad de medida con mltiplos y submltiplos y 2) un proceso para comparar la magnitud a medir con la unidad de medida y establecer un nmero (entero o fraccionario) como medida de la magnitud. Son ejemplos de magnitudes fsicas: la longitud, el rea, el volumen, el tiempo, la masa, la energa, la

INTRODUCCIN AL CURSO

Hugo Medina Guzmn

temperatura, la fuerza, la potencia, la velocidad, la aceleracin, etc. Llamamos magnitud fsica a aquella propiedad de un cuerpo que puede ser medida. La masa, la longitud, la velocidad o la temperatura son todas magnitudes fsicas. El aroma o la simpata, puesto que no pueden medirse, no son magnitudes fsicas. Las medidas de las magnitudes se realizan mediante las unidades de medida, establecidas por la Unin Internacional de Pesas y Medidas (UIPM), que forman el Sistema Internacional de unidades (S. I.), aunque existen otras unidades que se siguen usando por tradicin (como el kilate, que se emplea para medir la masa de las piedras preciosas). Magnitud escalar. Para muchas magnitudes fsicas basta con indicar su valor para que estn perfectamente definidas. As, por ejemplo, si decimos que Jos Antonio tiene una temperatura de 38 C, sabemos perfectamente que tiene fiebre y si Rosa mide 165 cm de altura y su masa es de 35 kg, est claro que es sumamente delgada. Cuando una magnitud queda definida por su valor recibe el nombre de magnitud escalar. Magnitudes vectoriales. Otras magnitudes, con su valor numrico, no nos suministran toda la informacin. Si nos dicen que Daniel corra a 20 km/h apenas sabemos algo ms que al principio. Deberan informarnos tambin desde dnde corra y hacia qu lugar se diriga. Estas magnitudes que, adems de su valor precisan una direccin se llaman magnitudes vectoriales, ya que se representan mediante vectores. En este tema estudiaremos los vectores y sus propiedades. UNIDADES. SISTEMA INTERNACIONAL DE UNIDADES. MEDICIN. La fsica es una ciencia experimental. Los experimentos requieren mediciones cuyos resultados suelen describirse con nmeros. Cualquier nmero empleado para describir cuantitativamente un fenmeno fsico se denomina cantidad fsica. Dos cantidades fsicas que describen a una persona son su peso y su altura. Algunas cantidades fsicas son tan bsicas que slo podemos definirlas describiendo la forma de medirlas, es decir, con una definicin operativa. Ejemplos de esto son medir una distancia con una regla, o un intervalo de tiempo con un cronmetro. En otros casos definimos una cantidad fsica describiendo la forma de calcularla a partir de otras cantidades medibles. As, podramos definir la velocidad media de un objeto como la distancia recorrida (medida con una regla) dividida por el tiempo de recorrido (medido con un cronmetro). UNIDADES. Al medir una cantidad, siempre la comparamos con un estndar de referencia. Si decimos que un automvil mide 4,29 m, queremos decir que es 4,29 veces ms largo que una regla de 2

medir, que por definicin tiene 1m de largo. Este estndar define una unidad de la cantidad. El metro es una unidad de distancia, y el segundo, de tiempo. Al describir una cantidad fsica con un nmero, siempre debemos especificar la unidad empleada; describir una distancia como "4,29" no significa nada. Las mediciones exactas y fiables exigen unidades inmutables que los observadores puedan duplicar en distintos lugares. El sistema de unidades empleado por los cientficos e ingenieros se denomina comnmente "sistema mtrico", pero desde 1960 su nombre oficial es Sistema Internacional, o SI. Las definiciones de las unidades bsicas del sistema mtrico han evolucionado con los aos. Cuando la Academia Francesa de Ciencias estableci el sistema mtrico en 1791, el metro se defini como una diezmillonsima parte de la distancia entre el Polo Norte y el Ecuador (ver figura). El segundo se defini como el tiempo que tarda un pndulo de 1m de largo en oscilar de un lado a otro. Estas definiciones eran poco prcticas y difciles de duplicar con precisin, por lo que se han sustituido por otras ms refinadas y por acuerdo internacional.

Unidades fundamentales Las fuerzas, velocidades, presiones, energas, en realidad todas las propiedades mecnicas, pueden expresarse en trminos de tres cantidades bsicas: masa, longitud y tiempo. En el sistema SI, las unidades correspondientes son: Masa Kilogramo Longitud Metro Tiempo Segundo Estas unidades se conocen como unidades fundamentales. TIEMPO Desde 1889 a 1967, la unidad de tiempo se defini como una cierta fraccin del da solar medio (el tiempo medio entre llegadas sucesivas del Sol al cenit). El estndar actual, adoptado en 1967, es mucho ms preciso; se basa en un reloj atmico que usa la diferencia de energa entre los dos estados energticos ms bajos del tomo de cesio. Cuando se bombardea con microondas de una determinada frecuencia, los tomos de cesio sufren una transicin entre dichos estados. Se define un segundo como el tiempo requerido por 9 192 631 770 ciclos de esta radiacin. LONGITUD

INTRODUCCIN AL CURSO

Hugo Medina Guzmn

En 1960 se estableci tambin un estndar atmico para el metro, usando la longitud de onda de la luz naranja emitida por tomos de kriptn (86Kr) en un tubo de descarga de luz. En noviembre de 1983 el estndar se modific de nuevo, esta vez de forma ms radical. Se defini que la velocidad de la luz en el vaco es exactamente 299 792 458 m/s. Por definicin, el metro es consecuente con este nmero y con la definicin anterior del segundo. As, la nueva definicin de metro es la distancia que recorre la luz en el vaco en 1/299 792458 s. ste es un estndar de longitud mucho ms preciso que el basado en una longitud de onda de la luz. MASA El estndar de masa, el kilogramo, se define como la masa de un determinado cilindro de aleacin platinoiridio que se guarda en la Oficina Internacional de Pesos y Medidas en Sevres, cerca de Pars. Un estndar atmico de masa, sera ms fundamental, pero an no podemos medir masas a escala atmica con tanta exactitud como a escala macroscpica. Unidades derivadas Las cantidades que interesan a los cientficos no se limitan a masa, longitud y tiempo. A menudo el comportamiento de objetos se describe en trminos de sus velocidades; hay que identificar las fuerzas que actan sobre los cuerpos; se paga por la energa que consumen los aparatos domsticos y nos interesa la potencia que pueda desarrollar un motor; la presin atmosfrica es un indicador til de las condiciones del tiempo. Todas las anteriores propiedades, aparentemente dispares, que se miden en metros por segundo (velocidad), newton (fuerza), joules (energa), watts (potencia) y pascales (presin), finalmente se pueden expresar como productos de potencias de masa, longitud y tiempo. Esas unidades, por tanto, se conocen como unidades derivadas, para distinguirlas de las tres unidades fundamentales. Prefijos comnmente encontrados. Utilizamos con frecuencia prefijos para obtener unidades de un tamao ms conveniente. Ejemplos de prefijos comnmente encontrados: 1 manmetro = 1 nm = 10-9 m (un poco ms grande que el dimetro del tomo) 1 micrmetro = 1 m =10-6 m (una clula de sangre humana es aproximadamente de 7 m) 1 milmetro = 1 mm =10-3 m (el carbn del lpiz es aproximadamente de 0,5 milmetros en dimetro) 1 centmetro = 1 cm =10-2 m (el dimetro de un bolgrafo) 1 kilmetro = 1 km = (1000 m) 1 microgramo = 1 g =10-6 g = 1-9 kg (masa de una partcula pequea de polvo) 1 miligramo = 1 mg = 10-3 g = 10-6 kg (una gota de agua es aproximadamente 2 mg) 1 gramo = l g = 10-3 kg (la masa de un clip para papel es de aproximadamente 1 g) 3

1 nanosegundo = 1 ns =10-9 s (tiempo en el que la luz viaja 30 m) 1 microsegundo = 1 s = 10-6 s (tiempo en el que una bala del rifle viaja 1 m) 1 milisegundo = 1 ms = 10-3 s (cerca de 14 ms entre los latidos del corazn) CONVERSION DE UNIDADES Algunas veces encontramos los datos dados en unidades distintas al sistema SI. En este caso debemos convertir las unidades al sistema SI usando los factores conocidos de conversin. La tabla siguiente muestra tales factores. Factores de Conversin Longitud 1 pulgada (in) = 2,54 centmetros (cm) 1 pie (ft) = 0,3048 metro (m) 1 milla (mi) = 5280 ft = 1,609 kilmetros (km) 1 m = 3,281 ft 1 km= 0,6214mi 1 ngstrom

o A = 10-10 m

1 ao luz = 9,461 x 1015 m 1 unidad astronmica (AU) = 1,496 x 1011m 1 prsec (pc) 3,09 x 1016 m Masa 1 slug = 14,59 kilogramos (kg) 1 kg = 1000 gramos = 6,852 x 10-2 slug 1 unidad de masa atmica (amu) = 1,6605 x 10-27 kg (1 kg tiene un peso de 2,205 lb donde la aceleracin de la gravedad es 32,174 ft/s2) Tiempo 1 dia =24 h= 1,44 x 103 min = 8,64 x 104 s 1 ao = 365,24 das = 3,156 x 107s 1 hora (h) =60min =3600s Velocidad 1 mi/h = 1,609 km/h = 1,467 ft/s 0,4470 m/s 1 km/h = 0,6214 mi/h = 0.2778 m/s 0,9113 ft/s Volumen 1 litro (L) = 10 m3 = 1000 cm3 = 0,353 1 ft3 1 ft3 = 0,02832 m3 = 7,481 U.S. galones (gal) 1 U.S. gal = 3,785 x 10 m3 = 0,1337 ft3 Fuerza 1 pound (lb) = 4,448 Newton (N) 1 N = 10 Dinas = 0,2248 lb Trabajo y Energa 1 joule (J) = 0,7376 ft.lb = 107 ergios 1 kilogramo-calora (kcal) = 4186 J 1 Btu (60F) = 1055 J 1 kilowatt-hora (kWh) = 3,600 x 106 J 1 electron volt (eV) = 1,602 x 10-19 J Angulo 1 radian (rad) = 57,30 1 = 0,0 1745 rad Presin 1 pascal (Pa) 1 N/m2 = 1,450 x 104 lb/in2 1 lb/in2 = 6.895 x 10-5 Pa

INTRODUCCIN AL CURSO

Hugo Medina Guzmn

l atmsfera (atm)= 1,013 x 10 Pa= 1,013 bar = 14,70 lb/in2 = 760 torr Potencia 1 horsepower (hp) = 550 ft.lb/s = 745,7 W 1 watt (W) = 0,7376 ft.lb/s ANALISIS DIMENSIONAL La especificacin numrica de una cantidad fsica depende de las unidades que se empleen. Por ejemplo, aunque una distancia se mida en unidades de metros o pies o millas siempre ser una distancia. Se dice que su dimensin es de longitud, la denominacin no depende del sistema de unidades empleado. Los smbolos usados para especificar la 1ongitud, la masa y el tiempo son L, M y T, respectivamente. Para denotar las dimensiones de una cantidad se usan corchetes, por ejemplo de distancia l = L, de velocidad v = L/T, de rea A = L2. Entre sus aplicaciones tenemos: a) Verificacin de una frmula especfica. El anlisis dimensional utiliza el hecho de que las dimensiones se pueden tratar como cantidades algebraicas (se pueden sumar y restar slo si se tienen las mismas dimensiones). Si una ecuacin se lee A=B+C Los trminos A, B, y C deben tener las mismas dimensiones. Ejemplo 1. Verificar la frmula siguiente

x m a g bt c
Donde a, b y c son exponentes que deben ser determinados y el smbolo indica proporcionalidad. Esta ecuacin es correcta nicamente si las dimensiones de ambos lados son iguales, como la dimensin de x es de longitud, la dimensin, del lado izquierdo tambin debe ser de longitud.

[m

g bt c = L
b

L M 2 Tc = L T a b c - 2b M LT =L
a

[]

[ ]

[]

Igualando exponentes en ambos miembros obtendremos a = 0, b =1, c-2b = 0 De aqu a = 0, b = 1 y c = 2 Por lo tanto la expresin debe tener la forma

x gt 2 o x = kgt 2
El anlisis dimensional puede describir la forma de la ecuacin pero no indica el valor de la constante k. Ejemplo 2. Mediante el anlisis dimensional determinar la expresin para la aceleracin centrpeta de una partcula que describe un movimiento circular uniforme. Solucin. Supongamos que la aceleracin centrpeta depende de la velocidad, del radio de curvatura y el peso

ac = kv a R bW c
aceleracin centrpeta velocidad

x = x 0 + vt +

1 2 at , donde x y x0 representan 2

[ac ] =

L T2

distancias, v es velocidad, a es aceleracin y t es un intervalo de tiempo. Solucin. Como

[v] = L
T

1 2 [x] = [x0 ] + [vt ] + at = L


2
Y las dimensiones de la velocidad son L/T y de la aceleracin L/T2, tenemos:

[] ML peso [W ] = 2
radio v = L

Reemplazando

L [vt ] = (T ) = L

T 1 2 L 2 at = 2 T = L 2 T

L L b ML = (L ) 2 2 T T T LT -2 = L
a +b +c

( )

Podemos ver que esta frmula es correcta porque todos los trminos tienen la dimensin de longitud. b) Desarrollo de ecuaciones. Esto lo podemos ver en el ejemplo de encontrar la distancia recorrida por un cuerpo en cada libre. Pongamos que esta cada puede depender de la masa, la aceleracin de la gravedad y del tiempo.

Igualando exponentes para L: 1 = a + b + c para T: 2 = a 2c para M: 0 = c de donde obtenemos a = 2 , b = 1 y c = 0 por lo tanto

T a 2c M c

ac = kv 2 R 1 = k

v2 R

x = f (m, g , t )

El procedimiento para el anlisis dimensional es poner la expresin en la forma 4

c) Convertir un sistema de unidades a otro. Si tenemos una frmula en un sistema de unidades podemos convertirlo a una frmula en otro sistema de unidades. Sean L1, M1, T1 y L2, M2, T2 sus unidades.

INTRODUCCIN AL CURSO

Hugo Medina Guzmn

Si la cantidad G de una ecuacin tiene dimensiones G = La Mb Tc. Se mide g1 con la unidad G1, y mide g2 con la unidad G2, la relacin es:

g1

2 M1 M2 2 = g 2 2 2 2 L1T1 L 2 T22

g1G1 = g 2 G2 g 2 = g 1

G1 G2

L1 g 2 = g1 L 2

M1 M 2

T1 T 2

g 2 = g1

M1 M 2 L1 L 2
2

Ejemplo 3. Si en el sistema MKS la frmula para el clculo de la variable R de unidades kg/ms aparece como R = 1,782 A + p Donde. p tiene unidades de m/s y A de km/m3. Hallar la frmula en el Sistema Ingls. 1 kg = 2,2 1b l m = 3,28 pie Solucin. Sean en el sistema MKS, L1, M1, T1, y en el sistema Ingls, L2, M2, T2. Las relaciones entre estos sistemas son;

5p

1 2

2 ( 2,2 ) g2 = 5 (3,28)2 (1)2

T1 T 2

= 2,25

Luego en el Sistema Ingls la ecuacin correspondiente es

2,25 p 2 R= 95,75 A + p
Para comprobar esta expresin evaluemos R1 para p1 = 1

M1 L T = 2,2 , 1 = 3,28 , 1 = 1 M2 L2 T2

2 5p En la ecuacin R = 1,782 A + p [R] = M , [ p] = L , [A] = M LT T L3


La cantidad l,782 A tiene las mismas unidades que p

kg m , A1 = 1 3 y R2 para s m pie p 2 = 3,28 , s 2,2 lb lb A2 = = 6,23 10 2 3 pie 3 (3,28 pie ) kg lb y R2 = 0,899 pie.s m.s

Operando en las ecuaciones respectivas obtenemos

R1 = 1,34

Realizando la conversin de unidades R1 encontramos que es equivalente a R2. CIFRAS SIGNIFICATIVAS Cuando e realizan mediciones, los valores medidos se conocen nicamente dentro de los lmites de la incertidumbre experimental, 1o datos medidos inherentemente no son exactos y si se registran en notacin decimal consisten de un conjunto finito de dgitos llamados cifras significativas, la ltima de las cuales es conocida como cifra dudosa. Cuando se mide una longitud mediante una regla se observa la lectura de un instrumento en el cual hay una escala, el punto de observacin para la lectura llega a una posicin como la que se indica en la figura siguiente.

L [1,782 A] = [1,782][A] = [1,782 ] M = 3 L T


Las unidades de 1,782 son

[1,782 ] = [5 ] =

L4 MT

Observando la ecuacin de R, concluimos que las unidades de 5 son las correspondientes a (R)2.

M2 L2 T 2
4

Para obtener el valor correspondiente a 1,7132 en el sistema Ingls

L1 L 4 4 L1 L2 2 g 2 = g1 g1 = g2 M1T1 M 2 T2 M1 T1 M 2 T2

(3,28)4 = 95,75 g 2 = 1,7132 (2,2)(1)

Para obtener el valor correspondiente a 5 en el sistema Ingls

Se puede leer exactamente hasta 11 y apreciar un dgito ms, este ltimo depende de cada persona puede ser 11,6 , 11,5 11,7. Si suponemos que nuestros instrumentos estn adecuadamente construidos, entonces las lecturas que tomemos tendrn significado y sern reproducibles, excepto el ltimo digito, el de los dcimos de la 5

INTRODUCCIN AL CURSO

Hugo Medina Guzmn

divisin ms pequea, ser aunque con significado un poco incierto. Por lo que no hay objeto en aadir una segunda cifra incierta. Una cifra significativa es cualquier dgito que denota la magnitud de la cantidad segn el lugar que ocupa en un nmero. Por ejemplo si escribimos S/. 10,52, todas las cifras son significativas, el uno representa el nmero de decenas en soles, el 0 representa que no hay unidad de sol y es significativo y finalmente sabemos que tenemos 52 cntimos. En la expresin 0,01052 gr. el primer cero de la izquierda sirve para llamar la atencin hacia la coma, el segundo cero muestra que el 1 ocupa el segundo lugar despus de la coma. Estos ceros no son significativos, sin embargo el 0 entre 1 y 5 es significativo. 10,52 tiene cuatro cifras significativas (1, 0, 5 y 2) 0,01052 tiene cuatro cifras significativas (1, 0, 5 y 2) La incertidumbre ms pequea posible con cualquier aparato de medicin es mitad del lmite de la lectura. Sin embargo, la mayora de las investigaciones generan una incertidumbre mayor que esto. La tabla siguiente enumera la incertidumbre de algunos equipos comunes del laboratorio. Regla de metro Calibrador vernier Micrmetro Reloj de segundos Cronmetro Dinammetro

Regla 3: Multiplicacin y Divisin El nmero de cifras significativas del producto cociente ser redondeado a un nmero de Significativas igual a aquel componente de aproximacin como se muestra en los ejemplos: 3,14159 x 21,13 = 66,38179 = 66,38 3,14159 / 21,13 = 0,14868 = 0,1487 Esto es porque 21,13 tiene slo cuatro cifras significativas, el resultado se redondea a cuatro cifras significativas Regla 4. Potencias y races La potencia o raz de un nmero de n cifras significativas se redondea a n cifras significativas. como se muestra en los ejemplos: 2,14 2 = 4,5796 = 4,58 2,14 3 = 9,800344 = 9,80

2,14 = 1,46287 = 1,46

2,14 = 1,288658 = 1,29

0,05 cm 0,005 cm 0,005 mm 0,5 s 0,0005 s 0,1 N

Cuando se anotan y se manipulan nmeros obtenidos por medidas, sern de mucha ayuda las siguientes reglas: Regla 1: Redondeo de un nmero En el proceso de rechazo de uno o varios de los ltimos dgitos. La ltima cifra retenida se incrementar en 1 si la cifra rechazada es 5 o mayor. Ejemplo. Nmero Redondeo a dado Cuatro Tres Dos cifras cifras cifras 62,578 62,58 62,6 63 10 232 10 230 10 200 10 000 329 350 329 400 329 000 330 000 Regla 2: Suma y Resta El nmero de cifras significativas de la suma o diferencia ser redondeado desechando todas las cifras a la derecha del lugar ocupado por la cifra incierta en cualquiera de las cantidades que est ms hacia la izquierda, como se muestra en el ejemplo:

Ejemplo 4. Cules son los resultados en las cifras correctas de las siguientes operaciones indicadas? a) 2,5 x 10-2 x 20 b) 3,32 x 103 + 3,2 x 10 c) 4,52 x 108 + - 4,2 x 103 d) 2,801 x 4 x 10-3 e) 6,2 x 104 / 3,0 x 10 Solucin. Aqu todos los nmeros estn expresados en notacin cientfica. Por ejemplo: 0,025 = 2,5 x10-2 = 2,5(-02), tiene 2 cifras significativas 20 = 2 x 10 = 2(+1), tiene una cifra significativa. a) 2,5 x 10-2 x 20 = 5 x 10-1 b) 3,32 x 103 + 3,2 x 10 = 3,35 x 103 c) 4,52 x 108 - 4,2 x 103 = 4,52 x 108 d) 2,801 x 4 x 10-3 = 11 x 10-3 e) 6,2 x 104 / 3, 0 x 10 = 2,1 x 103 Ejemplo 5. Para determinar la densidad de un lquido se toman 10 cm3 de ste. La masa del lquido medida en una balanza es 15,38g. Cul es la expresin correcta de la densidad? Solucin. La densidad del lquido es

m 15,38 g = = 1,538 3 10 V cm

Siendo 10 el nmero con menos cifras significativas (2), el resultado se redondea a 2 cifras significativas. La expresin correcta de la densidad es

= 1,5

g cm 3

ERRORES Como hemos indicado las mediciones fsicas involucran incertidumbre. El valor exacto de una magnitud medida es algo a lo cual intentamos aproximarnos pero que nunca conocemos. Un nmero de lecturas cuando se promedia se considera como el 6

INTRODUCCIN AL CURSO

Hugo Medina Guzmn

mejor acercamiento al verdadero valor de una lectura, y la diferencia entre una lectura y la verdadera lectura o lectura exacta se llama error. Aqu la palabra error no significa equivocacin sino una incertidumbre. Error absoluto es la diferencia entre el valor aceptado N (asumimos conocido) y el valor aproximado N , obtenido por mediciones o clculos.

Si se usan los dos primeros trminos. 3 2 et = N - N = 1,00000 + = +0,07516 2 3! (+7,5%) Si se usan los tres primeros trminos.

e= N-N
Error relativo es la relacin entre el error absoluto e y el valor aceptado N

2 2 et = N - N = 1,00000 + 2 3! 5!
3

e=

e N = 1 N N

= -0,00453 (-0,5%) Si se usan los cuatro primeros trminos. et = 0,00015 , el error de truncado ya es insignificante. c) Error de redondeo (er ) , es el error introducido por redondeo de un decimal. Por ejemplo. Si = 3,14159 Si redondeamos a = 3,14, entonces:

Porcentaje de error es el nmero de partes por cada 100 en que un nmero est errado

N e% = (100e )% = 1 N %
Cuando calcule el porcentaje de error en fsica elemental no use ms de dos cifras significativas. Por ejemplo si una pista para carreras de 3500 metros tiene 17 metros ms. El error absoluto o simplemente error es

er = 3,14159 - 3,14 = 0,00159 y 0,00159 er = 100 = 0,05% 3,14159 d) Error de interpolacin (e p ) , es el error
introducido por la aproximacin de un valor por su equivalente interpolado. Por ejemplo: Si conocemos la circunferencia de un crculo de l0 metros de dimetro y de otro circulo de 11 metros. C10 = 10 = 31,42 m y

e = 17 m

El error relativo es

e=

17 3500

C11 = 11 = 34,56 m
Por interpolacin lineal la circunferencia de un crculo de 10,6 metros es: Pero el valor exacto es

El porcentaje de error es

17 e% = 100% = 0,49% 3500


Clasificacin de errores. En los clculos numricos pueden ocurrir cinco tipos de errores bsicos. a) Error inherente (ei ) . Es el error en los datos iniciales debido a mediciones, observaciones o registros inexactos. b) Error de truncado et . Es el error creado por representar una funcin con slo unos cuantos trminos de una serie. Por ejemplo: El valor correcto de N = sen

C10,6 = C10 + (C11 C10 ) 0,6 = 33,30 m

C10,6 = 10,6 = 33,31 m e p = 33,31 33,30 = 0,01 m ep % =

De aqu

0,01 100 = 0,03% 33,31 e) Error de aproximacin (ea ) , es el error


o introducido por la aproximacin de una constante o una funcin por un valor elegido. Por ejemplo: La aceleracin debido a la gravedad g = 9,80665 m/s2 puede aproximarse por:

= 1,000

El valor aproximado de N computado por expansin de series es:

g= g=

51 m 10 = 9,80769 2 ea % = 0,01% 52 s

2 2 2 N= ... + 2 3! 5! 7!
3 5 7

mejor por

507 m 10 = 9,80658 2 517 s ea % = 0,00%


(El error aparece en el cuarto decimal)

Si se usa solo el primer trmino.

et = N - N = 1,00000

= -0,57080 (-57%)

2
7

INTRODUCCIN AL CURSO

Hugo Medina Guzmn

Error cuadrtico medio o desviacin normal o estndar En general cuando se realiza una medicin cualquiera siempre se comete error, cuando repetimos las mediciones varias veces, encontramos casi siempre resultados diferentes para cada una, aunque empleemos el mismo mtodo y el mismo aparato. Las mediciones sucesivas de un objeto determinado presentan discrepancias debido a los errores al azar o aleatorios de las medidas. Si la longitud verdadera de una varilla es l 0 la media aritmtica de un gran nmero de medidas sucesivas ser un nmero que representa la longitud media l m . Una medida Individual cualquiera tendr una desviacin de la media e = l l m , cantidad que puede ser positiva o negativa segn l sea mayor o menor que l m , es decir

mm el error absoluto o incertidumbre de la medida es l = 0,05 mm. Ejemplo 6. Un estudiante realiza varias mediciones de la masa de un cuerpo, obteniendo los siguientes resultados: 35,73 g , 35,76 g , 35,80 g, 35,76 g, 35,70 g Cul es el mejor valor estimado de la masa del cuerpo? Solucin. La masa media es: 35,73 + 35,76 + 35,80 + 35,76 + 35,70 mm = 5 = 35,75 g La desviacin de la media de cada medicin es:

l = lm e
Si elevamos al cuadrado cada uno de los valores de e y tomamos la media de todos los

e 2 , obtenemos

2 em que es la varianza de las medidas.

m1 mm = 35,73 35,75 = 0,02 m2 mm = 35,76 35,75 = 0,01 m3 mm = 35,80 35,75 = 0,05 m4 mm = 35,76 35,75 = 0,01 m5 mm = 35,70 35,75 = - 0,05
La varianza de las medidas es:
2 = em

2 em =

e
1=1

2 i

( 0,02 )2 + (0,01)2 + (0,05)2 + (0,01)2 + ( 0,05)2


5

A la raz cuadrada de esta meda se la conoce como el error cuadrtico medio o desviacin normal o estndar .

= 0,0112 La desviacin normal


2 = em = 0,0112

= 0,0334

La incertidumbre o error estndar de la medida es:

= e

2 m

Cuanto mayor sea el nmero n de medidas, menor ser la diferencia entre su media l m y la longitud verdadera l 0 , es decir el error estndar de la media,

m =

0,0334 5

= 0,01496 = 0,02

El mejor valor estimado es:

, ser menor. Por esto el mejor valor estimado

n de l 0 es:
l = lm

m = mm m = 35,75 0,02 m = (35,75 0,02 ) g

= l m l n En donde l es la incertidumbre o error absoluto

Si hubiramos realizado una sola medicin con una balanza cuya menor divisin es de 0,1 g la incertidumbre seria 0,05 y el resultado de la medicin podra expresarse as:

m = (35,75 0,05) g

68 por ciento de las veces dentro de una distancia l del valor verdadero pero desconocido l 0 . De esta forma podemos presentar el resultado final de un experimento en el cual se mide varias veces una magnitud. Sin embargo, muchas veces realizamos slo una medicin de la magnitud. En este caso se considera generalmente que la incertidumbre o error absoluto es igual a la mitad de la divisin menor de la escala del instrumento. Por ejemplo: si para medir longitudes se usa una regla cuya divisin minina es 1

determinado a partir de n mediciones. En el caso de verdaderos errores aleatorios, la media l m cae en un

Observemos que en ambos casos la incertidumbre corresponde al segundo orden decimal (0,02 y 0,05 respectivamente) incidiendo por lo tanto en la cifra 5, que es la cifra dudosa. PROPAGACIN ERRORES La determinacin experimental de algunas cantidades fsicas tales como densidad o volumen se obtienen por medicin directa. Generalmente, la cantidad a determinar se re1aciona de alguna manera conocida a una o ms cantidades medibles. El procedimiento es medir estas cantidades y con estas calcular por medio de relaciones conocidas la cantidad original. Por ejemplo el volumen de un cilindro puede conocerse si tenemos su longitud y Su dimetro. Estas pueden medirse directamente, cada una con su intervalo de 8

INTRODUCCIN AL CURSO

Hugo Medina Guzmn

error asociada, Estos intervalos de error determinan el Intervalo de error de la cantidad calculada. Es importante saber como hacer esta determinacin de la propagacin de errores. A continuacin determinemos los errores para diferentes situaciones. a) Suma de dos o ms variables. Consideremos z = x + y .

magnitudes la incertidumbre en el resultado es la raz cuadrada de la suma en cuadratura de las incertidumbres en las magnitudes. Ejemplo 7. Medimos la masa de un tomillo y obtenemos m1 m1 = (253 5) g , luego medimos tambin la masa de una tuerca, m2 m2 = (48 5) g . Cunto vale la masa M del tornillo y la tuerca juntos? Solucin. Evidentemente, la masa M es

z z = ( x x ) + ( y y )

Puesto que x e y tienen las incertidumbres x y y , cul es la incertidumbre z en z? Los mayores valores posibles para x e y son x + x e y + y , respectivamente, dando un valor superior de z = x + y . Los menores valores posibles para x e y son x x e y y , respectivamente, dando un valor inferior de z = (x + y ) . Es decir, los valores lmites para z son

M = m1 + m2 == 253 + 48 = 301 g
La Incertidumbre en la suma es
2 M 2 = m12 + m2 = 50 = 7 g

y el resultado final es

M = (301 7 ) g

Sin embargo, no utilizamos los (x + y ) como la incertidumbre. La razn es que para que z realmente valga z = ( x + y ) (x + y ) se necesita que la incertidumbre en la medicin, tanto de x como de y, sea tal que los dos resultados experimentales sean subestimaciones. Ms probable es que uno de los resultados sea un poco bajo y el otro un poco alto. Si ste es el caso, la incertidumbre en una de las mediciones puede compensar, en parte, la incertidumbre en la otra. Para tomar en cuenta esta posibilidad, lo que hacemos no es sumar las incertidumbres, sino que calculamos

z = ( x + y ) (x + y )

Ejemplo 8. Cul es la diferencia M entre las masas m1 y m2 del tornillo y la tuerca respectivamente? Solucin. Evidentemente, la masa M es

M ' = m1 m2 == 253 48 = 205 g


La Incertidumbre en la diferencia tambin es
2 M ' 2 = m12 + m2 = 50 = 7 g

y el resultado final es

M ' = (205 7 ) g

z = x 2 + y 2
Esta manera de combinar las incertidumbres, sumndolas elevadas al cuadrado, se llama suma en cuadratura. La incertidumbre z calculada de esta manera es siempre mayor que las a x y y por separado, pero menor que la suma x + y . La diferencia entre simplemente sumar las incertidumbres y sumarlas en cuadratura es que la suma simple da la incertidumbre mxima en el resultado, mientras que la suma en cuadratura da la incertidumbre ms probable. b) Diferencia de dos variables Consideremos z = x y .

z z = ( x x )( y y ) = xy yx xy + xy el error de z es z = yx + xy
considerando el mayor valor posible y no tomando en cuenta xy por se el producto de dos cantidades pequeas. El significado de esto se ms claramente en el error relativo.

c) Producto de dos o ms variables. Supongamos z = xy

z yx + xy x y + = = z xy x y

Ejemplo 9. Cul es el producto de (2,6 0,5) cm

z z = ( x x ) ( y y )

y (2,8 0,5) cm? Solucin. Primero, determinamos el producto de 2,6cm x 2,8cm = 7,28 cm2 Error relativo 1 =

La incertidumbre que queremos es la incertidumbre ms probable, que viene a ser la raz cuadrada de la suma en cuadratura de las incertidumbres

z = x 2 + y 2
Por lo tanto, tenemos una regla para la propagacin de incertidumbres Cuando sumamos o restamos dos 9

0,5 = 0,192 2,6 0,5 Error relativo 2 = =0,179 2,8


Suma de los error relativos = 0,371 o 37,1 %

INTRODUCCIN AL CURSO Error absoluto = 0,37l x 7,28 cm2 o 3,71 % x 7,28 cm2 = 2,70cm2 Los errores son expresados con una cifra significativa = 3 cm2 El producto es igual a 7,3 3 cm2 d) Potencias y races. Sea z = x Donde n es el nmero entero o fraccin positivo o negativo.
n

Hugo Medina Guzmn

Ejemplo 12. Encontrar el error en el clculo de

z=

1 = x 3 3 x x x4

Solucin.

z = 3x 31 x = 3 x 4 x = 3

z z = ( x x )
n

Como los errores son indeterminados debemos elegir el signo de tal manera que ste sea el mximo, por esto:

z = 3
n

Esto se puede escribir

x x4

x z z = x 1 x

y el error relativo es

x Haciendo la expansin binomial de 1 + x


x = 1 + x
n

x 3 4 x z = x =3 1 z x 3 x

e) Cocientes. Supongamos z =
2 3

1+ n

ignorando las potencias mayores que 1 de x

x n(n 1) x n(n 1)(n 2) x + + ... + 2! x 3! x x


n

x y (x x ) z z = ( y y )
1

x x 1 + = 1+ n x x
De aqu

Esto se puede escribir como:

z z = ( x x )( y y )
1

x z z = x n 1 n x n 1 El error de z es z = nx x
Y el error relativo es

x 1 y = x1 1 x y y x x y 1 1 m y x y

z x =n z x

x x y x y 1 + y x y x y

Ejemplo 10. Encontrar el error en el clculo de

z = x2
Solucin.

Ignorando el ltimo trmino por se muy pequeo y tomando el valor mximo para z . El error de z es:

z = 2 x 21 x = 2 xx
E error relativo es

z =

x x y yx + xy = + y y y2 x

z x =2 z x

El error relativo es:

Ejemplo 11. Encontrar el error en el clculo de

z = x = x1 2
Solucin

yx + xy yx + xy x y z y2 = = = + x z xy x y y
Ejemplo 13. Supongamos que queremos calcular la densidad de un cilindro de metal habiendo medido su masa M, su longitud L y su dimetro D. Al mismo tiempo queremos calcular el error relativo resultante de los errores en las cantidades medidas. Sabemos que la densidad est dada por la ecuacin

z =

1 2 1 1 x x x = 2 2 x
1

E error relativo es

z 1 x = z 2 x

10

INTRODUCCIN AL CURSO

Hugo Medina Guzmn

(D 2 ) L
2

4M D 2 L

Solucin.

4M 4 = MD 2 L1 2 D L

Como 4 y

M M 2D El error relativo de D es D L El error relativo de L es L


El error relativo de M es De aqu El error relativo de

son cantidades exactas no tienen error.

Ejemplo 16. La medida de los lados de un rectngulo son (1,53 0,06) cm, y (10,2 0,1) cm, respectivamente. Hallar el rea del rectngulo y el error de la medida indirecta. Solucin. El rea es A = 1,53 10,2 = 15,606 cm2 Como debe de tener solamente 3 cifras significativas

A = 15,6 cm 2
El error relativo del rea

A 0,06 0,1 = + = 0,0404422504 A 1,53 10,2


El error absoluto del rea

es

A = 0,0404422504(1,53 10,2 ) = 0,63083

M 2D L = + + M D L
2

El error absoluto con una sola cifra significativa es 0,6. La medida del rea junto con el error y la unidad se escribir como

Ejemplo 14. El volumen de un cilindro de base circular es V = R L . Cunto vale la incertidumbre o error en el volumen en trminos de las incertidumbres R y L ? Solucin. Como es cantidad exacta no tienen error.

A = (15,6 0,6 ) cm 2

Ejemplo 17. Se mide x con una incertidumbre x y se calcula y = ln x . Cunto vale y ? Solucin.

2R El error relativo de R es R L El error relativo de L es L


De aqu El error relativo de V es

y + y = ln ( x + x )

En este caso podemos usar aproximaciones para cantidades pequeas, cuando x << 1 , tales como:

1 nx , e x 1 + x , ln (1 + x ) x , senx x , cos x 1 , tan x x


En nuestro caso

(1 x )n

V R L =2 + V R L

Y el error absoluto:

R R L V = 2 + V = R 2R + L L L R
Ejemplo 15. Supongamos que queremos medir el periodo T de un oscilador, es decir, el tiempo que tarda en efectuar una oscilacin completa, y disponemos de un cronmetro que aprecia las dcimas de segundo, 0,1 s. Medimos el tiempo que tarda en hacer 10 oscilaciones, por ejemplo 4,6 s, dividiendo este tiempo entre 10 resulta t =0,46 s, cmo se expresa la medida? Solucin.

T=

t t , T = 10 10

x y + y = ln( x + x ) = ln x1 + x x x = ln x + ln1 + ln x + x x x Como << 1 podemos aplicar x x x ln1 + , luego: x x x x y + y = ln x + ln1 + ln x + x x Siendo y = ln x : x y = x
PRECISIN Y EXACTITUD Los trminos "PRECISION " y "ACCURACY" del idioma ingls no son sinnimos, para efectos de lenguaje estadstico traduciremos "Precision" como precisin y "Accuracy" como exactitud, estableciendo diferencias claras entre las dos palabras. 11

0,1 = 0,01 s . Por Obtenemos para el error T = 10


tanto, la medida la podemos expresar como

T = (0,46 0,01) s

INTRODUCCIN AL CURSO

Hugo Medina Guzmn

La precisin es una indicacin de la concordancia entre un nmero de medidas hechas de la manera indicada por el error absoluto. Un experimento de gran precisin tiene un bajo error al azar. La exactitud es una indicacin de cuan cercana est una medida al valor aceptado indicado por el error relativo o del porcentaje de error en la medida. Un experimento de gran exactitud tiene un error sistemtico bajo. As como la obtencin de una serie de medidas con las unidades correctas, se requiere una indicacin del error experimental o el grado de incertidumbre en las medidas y la solucin. Cuanto mayor es la exactitud y la precisin en nuestras investigaciones, ms bajo es el grado de incertidumbre. Las cuatro figuras a continuacin ilustran la diferencia:

En la direccin vertical, dibujamos una lnea arriba y abajo para que cada punto muestre la gama de incertidumbre del valor de la fuerza. Entonces ponemos una pequea lnea marcadora horizontal en el lmite del extremo incierto para el punto. En la direccin horizontal, dibujamos una lnea a la izquierda y a la derecha para que cada punto muestre la gama de incertidumbre del valor de la extensin. Entonces ponemos una pequea lnea marcadora lnea vertical en el lmite del extremo incierto para el punto. Cuando todos los puntos de la tabla se trazan en un grfico, la lnea del mejor ajuste con las barras apropiadas de error se muestra en la figura siguiente y se puede ver que la lnea del mejor ajuste cae dentro del rango de la incertidumbre de la barra del error.

RANGO DE ERROR O INCERTIDUMBRE Cuando una respuesta se expresa como valor con incertidumbre tal como 2,3 0,1 cm, entonces la gama de la incertidumbre es evidente. El valor cae entre 2,4 (2,3 + 0,1) y 2,2 (2,3 - 0,1) cm. En la fsica, determinamos a menudo la relacin que existe entre las variables. Para visin la relacin, podemos realizar una investigacin y trazar un grfico del eje dependiente) contra la variable independiente (eje x). Considere un resorte que tenga varios pesos, unido a l. A mayor peso se une a un resorte, el resorte extiende ms lejos de su posicin del equilibrio. La tabla siguiente muestra algunos valores para esta investigacin de Fuerza/alargamiento. 100 150 200 250 300 Fuerza 5 N 3,0 4,4 6,2 7,5 9,1 Alargamiento 0,2 cm Cuando se traza un grfico de la fuerza contra el alargamiento, la lnea del mejor ajuste no pasa por cada punto. Una barra del error se puede utilizar para dar una indicacin del rango de la incertidumbre para cada punto segn se muestra en la figura a continuacin Fuerza/alargamiento.

ESTIMADOS Y CLCULOS DEL ORDEN DE MAGNITUD Hasta donde hemos visto, es importante cuidar el seguimiento de las incertidumbres en la medicin cuando se calculan las respuestas a los problemas. En algunas ocasiones, tanto en la vida cotidiana como en el quehacer cientfico, es necesario resolver un problema del que no tenemos informacin suficiente para obtener una respuesta precisa. A menudo podemos obtener una respuesta til mediante la estimacin de los valores de las magnitudes apropiadas. Estas estimaciones, realizadas generalmente a la potencia de diez ms cercana, se denominan estimaciones del orden de magnitud. El clculo resultante del orden de magnitud no es exacto, pero generalmente es correcto con un factor de diez. El conocimiento justo del orden de magnitud de las cantidades fsicas con frecuencia nos proporciona informacin suficiente para obtener una comprensin til de la situacin fsica y la capacidad para formarnos un juicio y hacer clculos para la construccin de modelos. Realizar estimaciones de magnitud con frecuencia es sencillo. Por ejemplo, imagine que va a la escuela por 12

INTRODUCCIN AL CURSO

Hugo Medina Guzmn

primera vez y que quiere estimar cunto dinero necesitara para comprar libros. Usted conoce que la carga habitual para la mayor parte de los estudiantes es de cinco materias, y que en cada una se necesita un libro de texto. Con estos datos puede estimar el costo de un solo libro con el razonamiento siguiente. Sabe por experiencia que S/. 1 es demasiado bajo y que S/. 100 es demasiado alto. Incluso S/. 10 es bajo. Una estimacin razonable puede ser S/. 50. As, el costo estimado de los libros para un semestre es de 5 x S/. 50 = S/. 250. Aunque el resultado no es exacto, est dentro del orden de magnitud correcto y proporciona una estimacin razonable a un problema real. El siguiente ejemplo ilustra la aplicacin de las estimaciones del orden de magnitud. Cuando hacemos clculos de este tipo con frecuencia tambin efectuamos otras aproximaciones. Al remplazar por 3 o remplazar 2 por 3/2 hacemos pocas diferencias en el orden de magnitud, pero hacerlo simplifica mucho los clculos. Los ejemplos siguientes ilustran esta tcnica. Ejemplo 18. Una tienda ofrece un premio al cliente que adivine con la mayor aproximacin el nmero de caramelos de goma que llenan un frasco de un litro exhibido en un mostrador de la tienda. (Un litro es igual a 1000 cm3.) Estime cual ser el nmero.

As, el nmero aproximado de caramelos que hay en el frasco es: Nmero de caramelos

800cm3 240 . 27 3 cm 8

Un conteo realizado de los caramelos que llenan un frasco de un cuarto (0,95 litros) dio 255 caramelos. MODELOS IDEALIZADOS Ordinariamente usamos la palabra "modelo" para referimos a una rplica en menor escala (digamos, de un ferrocarril) o a una persona que exhibe ropa (o se exhibe sin ropa). En fsica, un modelo es una versin simplificada de un sistema fsico que sera demasiado complejo si se analizase de forma detallada. Por ejemplo, supongamos que nos interesa analizar el movimiento de una pelota de bisbol lanzada en el aire. Qu tan complicado es el problema? La pelota no es perfectamente esfrica ni perfectamente rgida: tiene costuras, est girando y se mueve en el aire. El viento y la resistencia del aire afectan su movimiento, la Tierra gira, el peso de la pelota vara un poco al cambiar su distancia respecto al centro de la Tierra, etc. Si tratamos de incluir todos estos factores, la complejidad del anlisis nos abrumar. En vez de ello, inventamos una versin simplificada del problema. Omitimos el tamao y la forma de la pelota representndola como objeto puntual, o partcula. Despreciamos la resistencia del aire haciendo que la pelota se mueva en el vaco, nos olvidamos de la rotacin terrestre y suponemos un peso constante. Ahora tenemos un problema sencillo de tratar. Para crear un modelo idealizado del sistema debemos pasar por alto muchos efectos menores y concentramos en las caractersticas ms importantes. Claro que hay que ser cuidadosos para no despreciar demasiadas cosas. Si ignoramos totalmente los efectos de la gravedad, nuestro modelo predecir que si lanzamos la pelota hacia arriba sta se mover en lnea recta y desaparecer en el espacio. Necesitamos algn criterio y creatividad para crear un modelo que simplifique lo suficiente un problema sin omitir sus caractersticas esenciales. Al usar un modelo para predecir el comportamiento de un sistema, la validez de las predicciones est limitada por la validez del modelo. La prediccin de Galileo respecto a la cada de los cuerpos corresponde a un modelo idealizado que no incluye la resistencia del aire. El modelo funciona bien para una bala de can, pero no para una pluma. El concepto de modelos idealizados es muy importante en fsica y en tecnologa. Al aplicar principios fsicos a sistemas complejos siempre usamos modelos idealizados, y debemos tener presentes las suposiciones que hacemos. De hecho, los principios mismos se expresan en trminos de modelos idealizados; hablamos de masas puntuales, cuerpos rgidos, aislantes ideales, etc. Estos modelos desempean un papel crucial en este libro. Trate de

Solucin. Una revisin cuidadosa del frasco (vase la figura) revela varias cosas. Los caramelos de goma pueden aproximarse vagamente a pequeos cilindros de casi 2 cm de largo por aproximadamente 1,5 cm de dimetro. Adems, los caramelos no estn apretados en el frasco; posiblemente tan s1o se ha llenado 80% de ste. Podemos hacer uso de estas observaciones para estimar el nmero de caramelos que hay en el frasco.
Nmero de caramelos = Volumen ocupado del frasco Volumen de un caramelo

EI volumen ocupado del frasco = 0,8 x 1000 = 800 cm3, Volumen de un caramelo =

3 cm 27 3 d cm h 2cm 3 2 2 8 2
2

13

INTRODUCCIN AL CURSO

Hugo Medina Guzmn

distinguirlos al estudiar las teoras fsicas y sus aplicaciones a problemas especficos. COMO ESTUDIAR FISICA? Para estudiar fsica es necesario dar atencin especial a los significados especficos de las palabras para poder entender el material, deben estudiarse detenidamente los grficos, dibujos, tablas y fotografas incluidos para entender claramente los principios fsicos involucrados. Gran parte de lo que se aprender ser en las clases. Debern aprender a tomar apuntes exclusivamente de las partes significativas de cada leccin y concentrarse por completo en lo que el profesor est diciendo, estos apuntes son necesariamente breves y carentes de relacin. Por lo tanto, es recomendable tener un cuaderno ordenado con las notas de clase completando con apuntes tomados del estudio de los libros. Hagan esto tan pronto como sea posible despus de clase, esto permitir tener un conjunto de notas claras e inteligibles para repaso; ayudar a detectar las reas dbiles de conocimiento. La parte ms importante de los apuntes son los problemas resueltos. Resulvanse todos los ejemplos vistos en clase y los dejados como tarea. Richard Feynman premio Nbel en fsica dijo: "usted no sabe nada sobre algo hasta que lo ha practicado". La habilidad para resolver problemas no es slo una prueba del dominio que cada cual posee de la ciencia, sino tambin un ndice del crecimiento de nuestra propia capacidad como herramienta en las futuras tareas del intelecto. Se recomienda desarrollar las habilidades necesarias para resolver un amplio rango de problemas. La

habilidad para resolver problemas puede ser la principal prueba de los conocimientos. Es esencial que se comprendan los principios y conceptos bsicos antes de intentar resolver problemas. En fsica general los exmenes se componen principalmente de problemas a resolver, es muy importante que se entiendan y recuerden las hiptesis que sirven de base a una teora o formalismo en particular. Para la resolucin de problemas se incluyen cinco etapas bsicas: a) Dibuje un diagrama con ejes coordenados si son necesarios y ponga las notaciones identificatorias, con esto podemos eliminar errores de signo. b) Identifique el principio bsico, incgnitas, listando los datos y las incgnitas. c) Seleccione una relacin bsica o encuentre una ecuacin que se pueda utilizar para determinar la incgnita y resulvala simblicamente. En esta forma se evitan errores y ayuda a pensar en trminos fsicos el problema. d) Sustituya los valores dados con las unidades apropiadas dentro de la ecuacin y obtenga el valor numrico de la incgnita. e) Verificacin y revisin del resultado por medio de las siguientes preguntas: Las unidades coinciden? Es razonable el resultado? Es apropiado el signo? Tiene significado? Una vez que el estudiante ha desarrollado un sistema organizado para examinar problemas y extraer la informacin relevante, tendr confianza y seguridad cuando tenga que resolverlos.

PREGUNTAS Y PROBLEMAS 1. Suponga que est planeando un viaje en automvil a otra ciudad y estima el tiempo que se requiere para ir all. Demuestre cmo esta estimacin depende de un modelo. Cmo se ha descrito en el texto y qu tan confiable es? 2. D un ejemplo personal del uso de un modelo para el anlisis de los datos medidos. 3. Explique la idea bsica detrs de la conversin de unidades. 4. Explique la diferencia en significado de las tres cantidades 10 m, 10.0 m y 10.00 m. 5. Cul de los nmeros siguientes se da con tres cifras significativas: 0,003 m, 0,32 cm, 0,320 cm, 3,21 mm o 3,213 mm? 6. Un estudiante mide un rectngulo con una regla cuya medida vara 1 mm. Encuentra que la altura es 37 mm y el acho 46 mm. Por qu debe informar que el rea del rectngulo 1700 mm2 en lugar de 1702 mm2? 14

7 Qu modelo describe en la forma ms sencilla las observaciones siguientes? a) Una pelota colocada en cualquier lugar sobre el piso permanece en reposo. b) Una pelota colocada en cualquier lugar sobre el piso empieza a rodar. c) D otros modelos ms sencillos para estas observaciones. Respuesta. a) Bola esfrica uniforme sobre un piso horizontal. b) Bola esfrica uniforme sobre un piso inclinado. c) Para a) la bola tiene una parte plana o no es uniforme y para b) la bola es asimtrica y empieza a rodar hacia su lado ms pesado. 8. Se lanza un dado muchas veces con los resultados siguientes para el nmero que aparece en su cara superior: 1, 63 veces; 2, 58 veces; 3, 62 veces; 4, 63 veces; 5, 75 veces y 6, 61 veces. Qu modelo puede hacer para el dado? Respuesta. El dado es ms pesado hacia el punto 2.

INTRODUCCIN AL CURSO

Hugo Medina Guzmn

9. Un cubo de metal flota en un lquido. Cul es el modelo ms sencillo del cubo y del lquido? Hay otros modelos? Respuesta. El cubo tal vez sea hueco si flota en el agua. Alternativamente, el cubo es slido pero flota en un lquido que es ms denso que l. 10. Un litro (L) es un volumen de 10 cm3. Cuntos centmetros cbicos hay en 2,5 mililitros? Respuesta. 2,5 cm3 11. Qu tan lejos viaja la luz en un vaco en 1,0 nanosegundos (Velocidad de la luz = 3,0 x l08 m/s.) Respuesta 30cm 12. Los granos negros en algunos tipos de pelculas fotogrfica son de aproximadamente 0,8 m de seccin. Asuma que los granos tienen una seccin transversal cuadrada y que todos quedan en un solo plano de la pelcula. Cuntos granos se requieren para oscurecer completamente 1 cm2 de pelcula? Respuesta. 1,6 x 108 13. Una frmula se lee y = at2, donde y est en metros y t en segundos. Cules son las dimensiones de a? Respuesta. m/s2 14. Cul es la altura en centmetros de una persona cuya estatura es 5l1? Respuesta. 180cm 15. Cmo es 40,2 mi expresado en kilmetros? Respuesta 64,7 km 16. Exprese 130 km/h en trminos de millas por hora. Respuesta. 80,8 mi/h 17 Una tienda anuncia un tapete que cuesta US $18,95 por yarda cuadrada. Cunto cuesta el tapete por metro cuadrado? Respuesta. 22,66 dlares/m2 18. Cuando la gasolina se vende a US $1,609 por galn, cul es el precio en dlares por litro? (1 gal = 3,l7853 L) Respuesta. 0,282 dlares/L

19. Cul es el rea en centmetros cuadrados de un pedazo de papel de 8 pulg x 14 pulg? Respuesta. 1.25 768 cm2 20. Los listones de madera en una cerca estn espaciados 6,0 pulgadas, de centro a centro. Cuntos listones estn contenidos en un metro de valla? Respuesta. 6,6 21. La Luna gira sobre su eje cada 271/3 das de modo que la misma cara est siempre hacia la Tierra. A cuntos grados rotar la Luna respecto a su propio eje en una hora? Respuesta. 0,549 22. Cuntas revoluciones hace el segundero de un reloj en tres aos? Suponga que no hay ao bisiesto en el intervalo. Respuesta. 1,58 x 106 revoluciones 23. La Tierra tiene una masa de 5.98 x 1024 kg y un radio de 6,38 x 106 m. a) Cul es la masa por unidad de volumen de la Tierra en kg/m3? b) Cul es la masa por unidad de volumen de un ncleo de oro que tiene una masa de 3,27 x 1025 kg y un radio de 6,98 x 10-15 m? c) Cul sera el radio de la Tierra si su masa no cambiara, pero tuviera la misma masa, por unidad de volumen, que el ncleo de oro? Respuesta. a) 5,50 x 103 kg/m3, b) 2,30 x 1017 kg/m3, c) 184 m 24. Calcule el volumen de la tabla rectangular con altura de 17,5 mm, ancho de 29,4cm y longitud 115,4 cm. Recuerde la regla que se refiere a las cifras significativas.

Respuesta. 5,94 x 103 cm3

25. Si usted mide los lados de un cuadrado y son de diez centmetros con una exactitud de 1 %, cul es el rea del cuadrado y cul es la incertidumbre? Respuesta. (100 2) cm2 26. Sume los nmeros siguientes: 3,57 x 102, 2,43 x 103 y 4,865 x 102. Respuesta. 3,27 x 103

15

INTRODUCCIN AL CURSO

Hugo Medina Guzmn

27. Un legajo de papel copia tiene 5,08 cm de espesor. Cul es el espesor de una sola hoja del papel? Exprese su respuesta en m y mm. Respuesta. 1,02 x 10-4 m o 0,102 mm 28. El piso rectangular de un gimnasio tiene lados de longitud de x x por y y donde x y y son las incertidumbres estimadas en las mediciones y son pequeas comparadas con x e y. Demuestre por clculo directo que el rea del piso y la incertidumbre en esa rea estn dadas por x y cuando se ignoran trminos A = xy xy x + y muy pequeos, del orden de ( x)2. (En la mayor parte de los casos, este resultado sobrestima la incertidumbre en el rea, porque no toma en consideracin que las incertidumbres en las longitudes, x y y, provienen de una serie de medidas, que tienen una dispersin natural en sus valores.)

32. En algunos pases el consumo de gasolina de un automvil se expresa en litros consumidos por 100 km de viaje. Si un automvil logra 27 millas/galn, cul es el consumo de combustible en litros por 100 km? (1 gal = 3,7853 L) Respuesta. 8,7 L/100 km 33. La velocidad del sonido a la temperatura ambiente es 340 m/s. Exprese la velocidad del sonido en unidades de millas por hora. Respuesta. 761 mi/h 34. a) Cuntos milisegundos hay en un minuto? Cuntos gigasegundos hay en un siglo? Respuesta. a) 1 min = 60000 ms, b) 1 siglo = 3,16 Gs 35. a) Calcule la altura de un cilindro de radio R que tiene el mismo volumen de una esfera de radio R. b) Demuestre que el cilindro tiene un rea superficial mayor que la esfera. Respuesta.

29. Estime el espesor de las pginas de un libro. D su resultado en milmetros. Respuesta. Aproximadamente 0,06 mm 30. Alrededor de cuntos ladrillos se requieren para construir una pared de altura hasta el hombro de 100 pies de largo? Los ladrillos estndar tienen 8 pulg de largo por 2 1/4 pulg de alto y estn separados por 3/8 de pulgada de mortero. Respuesta. 3,3 x 103 ladrillos 31. Cul es el volumen en milmetros cbicos de un cubo de 1,00 pulg por lado? Respuesta. 1,64 x 104 mm3

h=

4 R 3

36. Considere una esfera que se ajusta exactamente dentro de un cubo. Cul es la relacin del volumen de la esfera al volumen del cubo? Respuesta.

/6

37. Un vaso cilndrico para malteada tiene un radio interior medido de r r y una altura de h h. Demuestre que el volumen del vaso es

V = r 2 h 2hr r 2 h si se ignoran los 2 trminos muy pequeos del orden (r )

16

Movimiento rectilneo

Hugo Medina Guzmn

CAPITULO 2. Movimiento rectilneo


DEFINICIN DE PARTCULA. El Punto Material Es una idealizacin de los cuerpos que existen en la naturaleza y que llamamos punto material. Es un cuerpo cuyas dimensiones son despreciables al compararlas con las otras dimensiones que intervienen en el movimiento. La Mecnica comienza con el estudio de los puntos materiales y despus extiende estos estudios a los sistemas de puntos materiales, incluyendo cuerpos rgidos y deformables. El punto material, a diferencia de un punto geomtrico, est asociado a una masa inercial; esta propiedad est ntimamente ligada al movimiento de los cuerpos, como podemos ver cuando tratamos de entender cmo se mueven los cuerpos. CONCEPTO DE MOVIMIENTO El movimiento es un fenmeno fsico que se define como todo cambio de posicin que experimentan los cuerpos en el espacio, con respecto al tiempo y a un punto de referencia, variando la distancia de dicho cuerpo con respecto a ese punto o sistema de referencia, describiendo una trayectoria. Para producir movimiento es necesaria una intensidad de interaccin o intercambio de energa que sobrepase un determinado umbral. La parte de la fsica que se encarga del estudio del movimiento es la cinemtica. CLASIFICACIN DEL MOVIMIENTO Segn se mueva un punto o un slido pueden distinguirse distintos tipos de movimiento: Segn la trayectoria del punto: Rectilneo y curvilneo Movimiento rectilneo: La trayectoria que describe el punto es una lnea recta. Movimiento curvilneo: El punto describe una curva cambiando su direccin a medida que se desplaza. Casos particulares del movimiento curvilneo son la rotacin describiendo un crculo en torno a un punto fijo, y las trayectorias elpticas y parablicas. Segn la trayectoria del slido: Traslacin y rotacin. Traslacin: Todos los puntos del slido describen trayectorias iguales, no necesariamente rectas. Rotacin: Todos los puntos del slido describen trayectorias circulares concntricas. Segn la direccin del movimiento: Alternativo y pendular. Alternativo: Si la direccin del movimiento cambia, el movimiento descrito se denomina alternativo si es sobre una trayectoria rectilnea o pendular. Pendular: Si lo es sobre una trayectoria circular (un arco de circunferencia). Segn la velocidad: Uniforme y uniformemente variado. Movimiento uniforme: La velocidad de movimiento es constante Movimiento uniformemente variado: La aceleracin es constante, como es el caso de los cuerpos en cada libre sometidos a la aceleracin de de la gravedad. SISTEMAS DE REFERENCIA. POSICIN Y DESPLAZAMIENTO. El movimiento es una nocin esencialmente relativa. As resulta que el movimiento como el reposo son hechos relativos, no se puede decir que algo se mueve o que est en reposo sin aadir respecto a qu. En consecuencia necesitamos un sistema de referencia para descubrir el movimiento. Sistemas de referencia. Desde el punto de vista estrictamente matemtico, un sistema de referencia en un espacio vectorial de dimensin n est formado por n vectores linealmente independientes, formando una base del espacio, y por un punto, definido por n coordenadas, que suele llamarse origen del sistema de referencia. En el dominio de la fsica, el espacio suele ser la

, base ms habitual la llamada ortonormal ( i j, ), y el origen se sita a conveniencia del k


observador. Los vectores de la base son

= (0,0,1). = (1,0,0), i j = (0,1,0) y k

Atendiendo a su posible estado de reposo o movimiento, los sistemas de referencia pueden ser clasificados siempre y cuando hablemos de su relacin respecto a otro sistema de referencia que arbitrariamente supongamos inmvil. En efecto, debe tenerse en cuenta que cualquier sistema de referencia est movindose respecto a otro (este papel gira y se traslada con la Tierra alrededor del Sol, el cual a su vez se desplaza en la galaxia, que a su vez se expande en el Universo...), por lo que no cabe hablar de un sistema de referencia absoluto. De acuerdo con lo anterior, un sistema de referencia puede estar: a) en reposo respecto a otro

b) movindose con velocidad constante al supuestamente fijo c) con una aceleracin respecto al fijo.

v respecto

Un buen ejemplo del primer caso podemos encontrarlo en un sistema de referencia como la pizarra, que se encuentra en reposo relativo respecto a las paredes del aula (en condiciones normales). Un ejemplo de sistema de referencia inercial podemos encontrarlo en un tren que se mueve en un tramo de va rectilneo con una velocidad sensiblemente constante. 1

Movimiento rectilneo Y por ltimo, la propia Tierra constituye un sistema de referencia no inercial, ya que gira con una aceleracin normal, que si bien es pequea, en ciertos fenmenos se observa con claridad. Vector Posicin.- Para fijar la posicin de un punto en el espacio respecto a un origen de coordenadas bastan tres nmeros que pueden ser las proyecciones sobre los ejes de un sistema cartesiano ortogonal.

Hugo Medina Guzmn una recta, circunferencia, espiral, parbola o curvas tan complicadas como se nos ocurra. La trayectoria no define el movimiento, pues no sabemos en que instante de tiempo ocup cada punto. Sabemos dnde estuvo, pero no cuando y si estuvo varias veces en cada punto o no. Hace falta la ecuacin horaria. Para encontrar la ecuacin horaria debemos medir las distancias en funcin del tiempo.

El vector posicin del punto P es:

OP = r

En la figura P0 es un origen fijo sobre la curva (C) que porta la trayectoria. Sea P la posicin de la partcula en el instante t sobre la trayectoria definida por el arco

El movimiento quedar especificado si conocemos el vector posicin para cada instante, es decir:

P0 P = S
La ecuacin horaria del movimiento de la partcula P es

r = r (t )

Esto se conoce como ley de movimiento. El vector posicin puede ser expresado a travs de las ecuaciones paramtricas de sus componentes en funcin del tiempo: x = x(t ) , y = y (t ) , z = z (t )

S = S (t )

+ y (t ) r = x(t )i j + z (t )k

Desplazamiento. La figura muestra una partcula que se est moviendo a lo largo de la trayectoria curvilnea C.

Ejemplo experimental. Estudio del movimiento de la cada libre de un cuerpo. Solucin. Si dejamos caer un objeto, obtenemos que la trayectoria sea una recta vertical. Para encontrar la ley del movimiento podemos intentar medir a partir de dnde la dejamos caer, distancias sucesivas para diferentes tiempos. Una forma experimental es usando una pelcula fotogrfica y una flash electrnico que se encienda por ejemplo cada 1/30 de segundo. En una habitacin oscura dispondremos el cuerpo, la pelcula y un disparador que deje caer el cuerpo y simultneamente accione el flash. Paralelamente a la trayectoria a seguir por el objeto se fija una regla.

Sean P1 y P2 las posiciones de la partcula en los instantes t1 y t 2 = t1 + t . Los vectores posicin correspondientes son

OP 1 y OP 2 = r 2 = r 1 + r .
Siendo r el vector desplazamiento y describe el desplazamiento de la partcula de la posicin P1 a la posicin P2. Trayectoria y Ecuacin Horaria del Movimiento.Se llama trayectoria de una partcula en movimiento al lugar geomtrico de las posiciones efectivamente ocupadas por la partcula en el transcurso del tiempo. De acuerdo al tipo de movimiento podr ser

Movimiento rectilneo

Hugo Medina Guzmn

r = kt 2 k

Las ecuaciones paramtricas son

x = 0 , y = 0 y z = kt 2
En esencia para cualquier movimiento debemos ingeniarnos para obtener la ecuacin horaria y conocida su trayectoria, queda determinado el movimiento. VELOCIDAD Y RAPIDEZ La fotografa mostrada permite conocer las cotas de la foto en los diferentes instantes bien determinados. La tabla muestra los resultados de la fotografa: Tiempo t0 t1 t2 t3 t4 t5 t6 t7 t8 t1 t 10 Cota(m) 0,2480 0,3250 0,4130 0,5130 0,6235 0,7450 0,8875 1,0215 1,1760 1,3405 1,5155 Rapidez. La rapidez (que en el lenguaje comn se denomina simplemente velocidad) se define como el cociente entre la distancia recorrida y el tiempo transcurrido. La distancia s recorrida a lo largo de una trayectoria es una magnitud escalar, independiente de la direccin. Como el tiempo tambin es un escalar, la rapidez es tambin un escalar. La rapidez se designa mediante el smbolo v y sus dimensiones son:

[v] = LT -1

La unidad en el sistema SI es el metro por segundo (m/s). La figura muestra una partcula que se est moviendo a lo largo de la trayectoria curva C. En el instante t1 esta en P1, a una distancia S1 de un punto P0 de referencia. En el instante t 2 est en P2 a una distancia S 2 del punto de referencia.

Tracemos la curva representativa del la funcin

z = f (t )

En el tiempo que transcurre entre t1 y t 2 ,

t = t 2 t1 , la partcula ha recorrido una distancia S es la diferencia entre S 2 y S1 , esto es S = S 2 S1 .


Se define como rapidez media dentro de este intervalo Esta curva corresponde a una parbola y su expresin matemtica es

z = kt 2 z est en segundos m Donde k = 4,9 2 s t est en segundos


Luego la ecuacin horaria es

s = kt 2
Si fijamos el origen del movimiento en z = 0, la ley del movimiento es

magnitud fsica. Si la rapidez de la partcula vara a lo largo de la trayectoria, para conocer con mejor precisin el movimiento debemos hacer los intervalos S ms pequeos y tomar la rapidez media de cada uno de ellos. La figura a continuacin nos muestra el grfico distancia recorrida versus tiempo, observen que cuando t 2 tiende a t1 , t tiende a cero. Mediante este proceso llamamos a la rapidez instantnea v en el instante t. Este proceso se expresa matemticamente como 3

S 2 S 1 S = t 2 t1 t El smbolo (delta) significa un incremento de una vm =

Movimiento rectilneo

Hugo Medina Guzmn

v = lim

S dS = t 0 t dt

S (t ) = Asen (t )
Solucin. En el intervalo de tiempo de t hasta t + t la partcula que se mueve:

S = S (t + t ) S (t ) = Asen (t + t ) Asent = Asent cos(t ) + Acostsen(t ) - Asent

dS La cantidad se llama derivada de S con dt respecto a t y el proceso de encontrarla se llama derivacin o diferenciacin. La notacin dS , dt ,
expresa incrementos infinitesimalmente pequeos que se conocen como diferenciales. Ejemplo 1. a) Hallar una expresin para la rapidez de una partcula que se mueve de acuerdo a la ley horaria

La rapidez en un instante t cualquiera es

v = lim
= lim
t 0

S t 0 t

v = A cos t

Asen t cos (t ) + Acostsen (t ) Asent t

S = At 2
b) Si A = 1,4 m/s , hallar la distancia a la que se encuentra la partcula y su rapidez para 10 segundos despus de iniciado su movimiento. Solucin. a) Si en el tiempo t est en S(t ) :
2

El proceso desarrollado en los dos ejemplos anteriores se hace simple con la prctica. Hay muchas reglas o frmulas para derivar diferentes tipos de funciones. Estas pueden memorizarse o encontrarse en tablas. La tabla siguiente es una pequea muestra de estas. Derivadas de algunas funciones Funcin Derivada

S(t ) = At 2
Transcurrido un tiempo t , la partcula estar en

S =t

S =c
S = cu

S(t + t ) S(t + t ) = A(t + t )


= Como
2

At 2 + 2 At t + A(t ) ,
2

S =u+v

S = S(t + t ) St
=

= 2 At t + A(t ) La rapidez en el instante t es:


2

At 2 + 2 At t + A(t ) At 2

S = uv
S = Asent S = Acost

S t 0 t 2 2 Att + A(t ) = lim = 2 At t 0 t b) Para t = 10 es m 2 S(10 ) = 1,4 2 (10s ) = 140 m s v(t ) = lim
m m 2 v(10 ) = 21,4 2 (10s ) = 28 2 s s

dS dt dS dt dS dt dS dt dS dt dS dt dS dt

= nt n 1 =0 du dt du dv = + dt dt du dv =v +u dt dt =c = Acost = Asent

Ejemplo 3. Hallar una expresin para la rapidez de una partcula que se mueve de acuerdo a la ley horaria S = At , usando frmulas de la tabla anterior. Solucin. Tenemos que:
2

y su rapidez es

v=

Ejemplo 2. Hallar una expresin para la rapidez de una partcula que se mueve segn la ecuacin horaria 4

dS d At 2 dt 2 = =A = 2 At dt dt dt

( )

Ejemplo 4. Hallar una expresin para la rapidez de una partcula que se mueve de acuerdo a la ley

Movimiento rectilneo horaria S (t ) = Asen (t ) , usando frmulas de la tabla anterior. Solucin. Tenemos que

Hugo Medina Guzmn

La magnitud del vector velocidad instantnea, v , es decir v o simplemente v es igual a la rapidez instantnea en ese punto. La velocidad es la pendiente del grfico de x versus t, como se muestra en la figura.

dsent dS d ( Asent ) = =A = A cos t v= dt dt dt

Velocidad. La velocidad (que ms apropiadamente sera vector velocidad), a diferencia de la rapidez debemos incluir el concepto de direccin en nuestro estudio; para esto debemos emplear vectores. La figura muestra una partcula que se est moviendo a lo largo de la trayectoria curvilnea C. Cuando la pendiente es positiva, el objeto se est moviendo a la derecha. Cuando la pendiente es negativa, el objeto se est moviendo a la izquierda. Cuando la pendiente es cero, el objeto se detiene. Sean P1 y P2 las posiciones de la partcula en los instantes t1 y t 2 = t1 + t . Los vectores posicin correspondientes son OP1 = r 1 y

OP 2 = r 2 = r 1 + r . Siendo r el vector
desplazamiento y describe el desplazamiento de la partcula de la posicin P 1 a la posicin P 2 . Velocidad media. El cociente entre el vector desplazamiento r y el intervalo de tiempo t es el vector velocidad media.

Ejemplo 5. Entre dos observadores hay una distancia de 1050 m, uno de ellos dispara un arma de fuego y el otro cuenta el tiempo que transcurre desde que ve el fogonazo hasta que oye el sonido, obteniendo un valor de 3 s. Despreciando el tiempo empleado por la luz en hacer tal recorrido, calcular la velocidad de propagacin del sonido.

vm =

r t

Solucin. La velocidad es: c = s/t = 1050/3 = 350 m/s Ejemplo 6. Nos encontramos en una batalla naval, en un buque situado entre el enemigo y los acantilados de la costa. A los 3 s de ver un fogonazo omos el disparo del can, y a los 11 s del fogonazo percibimos el eco. Calcular la distancia a que estn de nosotros el enemigo y la costa. Velocidad del sonido, 340 m/s.

Como el desplazamiento es un vector y el tiempo es un escalar positivo, la velocidad es una magnitud vectorial que tiene la misma direccin y sentido que el desplazamiento. Esto significa que si una partcula sufre un desplazamiento negativo, su velocidad ser tambin negativa. Velocidad instantnea. Como en el caso de la

rapidez obtendremos la velocidad instantnea v tomando la velocidad media en un intervalo de tiempo cada vez menor t medido desde un cierto tiempo t1 . En el lmite, cuando t tiende a cero:

r 2 r1 r d r v (t1 ) = lim = lim = t 2 t1 t 0 t dt t

La direccin de este vector es la direccin lmite del vector cuando t 0 de la figura anterior. Es evidente que en este lmite la direccin de r es la de la tangente la trayectoria en P1. 5

Solucin. Despreciando el tiempo empleado por la luz en su recorrido, la distancia a que se encuentra el enemigo es: S = 340 x 3 = 1020 m El sonido emplea para ir y volver a la costa, desde nuestra posicin, un tiempo que es: t = 11 - 3 = 8 s 2S= 340 x 8 S = 1360 m La costa est a 1020 + 1360 = 2380m.

Movimiento rectilneo

Hugo Medina Guzmn

Ejemplo 7. La posicin de una partcula en coordenadas cartesianas est dada por la ecuacin

b) de la ecuacin v x = lim

x dx , la velocidad = t 0 t dt

+ y (t ) r (t ) = x(t )i j + z (t )k

instantnea en funcin del tiempo es

donde

v x = 2bt 3ct 2 = (4,80)t (0,360)t 2


tal que i) v x (0) = 0, ii) v x (5) = (4,80 )(5) (0,360)(5) = 15,0 y
2

x(t ) = 5 + 6t 2 , y (t ) = 3t , z (t ) = 6

t en segundos, x, y, z en metros. a) Determinar el desplazamiento entre t = 0 y t = 1 s. b) Determinar la velocidad media c) Determinar la velocidad y la rapidez para t = 1 s. Solucin. a) para t = 0 s, x = 5m, y = 0m, z = 6m

iii) v x (10) = ( 4,80)(10) (0,360)(10) = 12,0


2

c) el auto est en reposo cuando v x = 0 . Por consiguiente ( 4,80)t (0,360)t = 0 .


2

+ 6k r 0 = 5i

El nico tiempo despus de t = 0 en que el auto se encuentra en reposo es t =

Para t = 1s, x = 11m, y =3m, z = 6m

+ 3 r 1 = 11i j + 6k

4,8 = 13,3 s 0,360

El desplazamiento es

+ (3 0) r = r 1 r 0 = (11 5)i j + (6 6 )k +3 = 6i j
b) la velocidad media es

+3 r 6i j + 3 = = 6i vm = j t 1 0
c) la velocidad instantnea es

+ 3t d r d 5 + 6t 2 i j + 6k v= = dt dt = 12ti + 3 j La magnitud de v es

[(

Ejemplo 9. Un ciclista marcha por una regin donde hay muchas subidas y bajadas En las cuestas arriba lleva una rapidez constante de 5 km/h y en las cuestas abajo 20 km/h. Calcular: a) Cul es su rapidez media si las subidas y bajadas tienen la misma longitud? b) Cul es su rapidez media si emplea el mismo tiempo en las subidas que en las bajadas? c) Cul es su rapidez media si emplea doble tiempo en las subidas que en las bajadas? Solucin. a) vm =

v = 122 + 32 = 153
= 12,4 m/s Valor que corresponde a la rapidez instantnea para t = 1s. Ejemplo 8. Un auto est parado ante un semforo. Despus viaja en lnea recta y su distancia respecto al semforo est dada por x(t) = bt2 - ct3 , donde b = 2,40 m/s2 y c = 0,120 m/s3. a) Calcule la velocidad media del auto entre t = 0 y t = 10,0 s. b) Calcule la velocidad instantnea en i) t = 0; ii) t = 5,0 s; iii) t = 10,0 s. c) Cunto tiempo despus de arrancar vuelve a estar parado el auto? Solucin. a) En t1 = 0, x1 = 0 , tal que la ecuacin

stotal ssubida + sbajada = ttotal ttotal 2v1v 2 2s = = 8 km / h = s s v1 + v 2 + v1 v 2 v t + v2t v1 + v2 b) vm = 1 = 2t 2


= 12,5 km/h

c) vm =

2 5 + 20 v1 2t + v2t 2v1 + v2 = = 3t 3 3

= 10 km/h (Obsrvese que la rapidez media es la media aritmtica de las rapideces uniformes nicamente en el caso de que el tiempo que duran los distintos recorridos sea el mismo). ACELERACIN En el lenguaje ordinario el trmino aceleracin se refiere slo a incrementos del mdulo de la velocidad (rapidez), pero en Fsica se utiliza con un sentido ms amplio para designar un cambio del vector velocidad. En Fsica se dice que un cuerpo est siendo acelerado no slo cuando aumenta su velocidad sino tambin cuando disminuye o cambia de direccin. Se llama aceleracin al cambio de la velocidad (vector velocidad) en el tiempo. 6

vm =

vm =

x2 (2,4)(10) (0,120 )(10) = t2 (10)


2

x 2 x1 x = t 2 t1 t

= 12,0 m/s

Movimiento rectilneo

Hugo Medina Guzmn Observe que la aceleracin negativa no significa necesariamente bajar la velocidad. Cuando la velocidad y la aceleracin ambas tienen el mismo signo, el objeto aumenta su velocidad. Cuando la velocidad y la aceleracin tienen signos opuestos, el objeto disminuye su velocidad. Los grficos de la figura siguiente ilustran el desplazamiento, la velocidad, y la aceleracin para un objeto en movimiento.

Aceleracin Media.

La razn en la cual la velocidad cambia se mide por la aceleracin. As si un objeto tiene la velocidad

v 1 en el t1 del tiempo y velocidad v 2 en el t 2 , su


aceleracin media es

v 2 v 1 v v = = am = i t t t 2 t1

Supongamos que una partcula que se mueve en la trayectoria C de la figura anterior en el instante t1 est en P1 con una velocidad v1 y en el instante

t 2 = t1 + t est en P2 con una velocidad v 2 . Por definicin el vector aceleracin media de la partcula entre los instantes es t1 y t 2 es

Ejemplo 10. Una partcula se mueve a lo largo de una lnea curva

v 2 v1 v am = = t2 t1 t

+ (2t 1) r (t ) = t 2 + t i j + t 3 2t 2 k

Las dimensiones de la aceleracin son a = LT La unidad de la aceleracin en el sistema SI est en metros / segundo por segundo:

[]

ms m = 2 s s
Aceleracin Instantnea o simplemente aceleracin. Cuando t 2 t1 o t 0 llegaremos al valor de la aceleracin en el instante t1 . Este proceso para el lmite se expresa matemticamente como

Encontrar: a) La velocidad para t = 1 s y para t = 3 s . b) La aceleracin media entre t = 1 s y para t = 3 s . c) La aceleracin y su magnitud para t = 1 s . Solucin. a) Las ecuaciones paramtricas son:

x(t ) = t 2 + 1 , y (t ) = 2t 1 , z (t ) = t 3 2t 2

Las componentes de la velocidad son:

dx dy = 2t + 1 , v y = = 2. dt dt dz vz = = 3t 2 + 4t dt vx =
La velocidad es:

a (t1 )

v 2 v1 v d v = lim = lim = t 2 t1 t t t 0 t dt 2 1

+2 v (t ) = (2t + 1)i j + 3t 2 4t k

+2 Para t = 1 s : v (1) = 3i jk +2 Para t = 3 s . v (3 ) = 7i j + 15k b) La aceleracin media entre t = 1 s y t = 3 s .

dr Como v = , tenemos: dt d v d d r d2 r = a= = dt dt dt dt 2
Es mejor evitar el uso de la palabra comn desaceleracin. Describa la aceleracin simplemente como positiva o negativa. 7

v v (3) v (1) = am = = t 3 1 (7 3)i + (2 2) j + (15 + 1)k 2

+ 8k a m = 2i

Movimiento rectilneo c) la aceleracin instantnea es


Hugo Medina Guzmn

a (t ) =

4 )k para t = 1s

+ (6t = 2i

dv d (2t + 1)i + 2 = j + 3t 2 4t k dt dt

)]

= r = x i = x x0 i v = vi t t t t0 x x x0 v= = = tan t t t0

+ 2k a (1) = 2i

Diagrama velocidad-tiempo

la magnitud de la aceleracin es

a(1) = 2 2 + 2 2 = 2 2 m s 2
Ejemplo 11. Una persona que se asoma por la ventana de un edificio alto de oficinas observa lo que sospecha es un ovni. La persona registra la posicin del objeto en funcin del tiempo y determina que est dada por

El grfico velocidad-tiempo del movimiento uniforme es una recta paralela al eje del tiempo. De v

+ 10,0t r (t ) = 5,0t i j + 7,0t 3,0t 2 k

a) Obtenga los vectores de: desplazamiento, velocidad y aceleracin del objeto en t = 5,0 s. b) Hay algn tiempo en que la velocidad del objeto sea cero? c) La aceleracin del objeto es constante o cambia con el tiempo? Solucin. a) El vector desplazamiento es:

x x0 x = x 0 + v(t t 0 ) t t0 Si el instante inicial t 0 = 0 , tenemos x = x0 + vt

Diagrama espacio-tiempo

+ 10,0t r (t ) = 5,0t i j + 7,0t 3,0t 2 k

El vector velocidad es la derivada del vector desplazamiento:

d r (t ) + 10,0 = 5,0i j + [7,0 2(3,0)t ]k dt


y el vector aceleracin es la derivada del vector velocidad:

El grfico indica las posiciones instantneas del mvil en cada instante

d 2 r (t ) = 6,0 k dt 2 en t = 5,0 s:

2 + 10,0(5) r (5 ) = 5,0(5)i j + 7,0(5) 3,0(5) k = 25,0i + 50,0 j 40,0k

d 2 r (5) = 6,0 k dt 2
b) la velocidad en ambas direcciones x e y es constante y diferente de cero, luego la velocidad nunca puede ser cero c) La aceleracin del objeto es constante, ya que t no aparece en el vector aceleracin. MOVIMIENTO RECTILNEO UNIFORME. Para que un movimiento sea rectilneo uniforme su velocidad debe ser constante, es decir, que la aceleracin sea siempre igual a cero. Estudio del Movimiento Como el movimiento es uniforme v m = v , y considerando que su trayectoria est en el eje x 8

MOVIMIENTO RECTILNEO UNIFORMEMENTE VARIADO. Para que un movimiento sea rectilneo uniformemente variado su aceleracin debe ser constante y diferente de cero. Estudio del Movimiento

Como la aceleracin es constante, a m

a=

dv v = = constante dt t

=a

= v = v i a = ai t t v v v0 a= = t t t 0 v v 0 = a(t t 0 )
Si el tiempo inicial t 0

=0

v = v0 + at

Movimiento rectilneo Diagrama velocidad-tiempo

Hugo Medina Guzmn

a=

v v v0 = = tan t t t 0

La velocidad media:

y la posicin en t Si la posicin en t 0 es r 0 = x 0 i
es r

Ejemplo 12. Demostrar que el rea encerrada bajo la curva de la velocidad del diagrama velocidad-tiempo es igual al mdulo del desplazamiento x = x x 0 .

, la velocidad media en este intervalo es = xi x x x0 vm = = t t t0

x x0 = vm (t t0 ) y x = x0 + vm (t t0 )
Por otra parte como la velocidad es una funcin lineal, la velocidad media v m es

La posicin. De lo anterior:

Solucin. El rea encerrada es igual al rea de un trapecio cuyas bases son b1 = v y b2 = v 0 con altura

v + v0 2 y como v = v 0 + a(t t 0 ) vm =
resulta

h = (t t 0 ) .

vm =

v 0 + [v 0 + a (t t 0 )] a(t t 0 ) = v0 + 2 2

finalmente

a(t t 0 ) (t t 0 ) x = x 0 + v 0 + 2 1 2 x = x 0 + v0 (t t 0 ) + a(t t 0 ) 2

h 2 (v + v0 ) (t t 0 ) = 2 1 = v 0 (t t 0 ) + (v v 0 )(t t 0 ) 2 (v v0 ) Pero como a = tan = (t t 0 ) (v v0 ) = a(t t 0 )


Luego

Area del trapecio =

(b1 + b2 )

Si el tiempo inicial t 0

=0

Area del trapecio = v0 (t t 0 ) +

1 2 a(t t 0 ) 2

x = x0 + v0 t +

1 2 at 2

Valor que precisamente corresponde al desplazamiento x = x x 0 . LA ECUACIN DE TORRICELLI. Podemos obtener una relacin muy til eliminando el tiempo como variable en la ecuacin

Diagrama espacio -tiempo

x = x 0 + v0 (t t 0 ) +
Como a =

(v v0 ) (v v0 ) (t t 0 ) = (t t 0 ) a

1 2 a(t t 0 ) 2

Movimiento rectilneo Sustituyendo

Hugo Medina Guzmn

x = x0 + v0
2 2 0

(v v0 )
a

1 (v v0 ) + a 2 a2

De donde se puede despejar:

v = v + 2a( x x 0 )

Los diagramas aceleracin-tiempo, velocidadtiempo y espacio-tiempo correspondientes son los siguientes:

Conocida como la ecuacin de Torricheli. Descripcin del movimiento de una partcula con aceleracin constante. Consideramos una aceleracin constante a > 0 en el sentido positivo de la trayectoria. 1er Caso: La partcula tiene una velocidad inicial v0

a 0 = constante

0.

La partcula se desplaza de P 0 al infinito con un sentido constante y aumentando su velocidad.

v = v 0 + at
Los diagramas aceleracin-tiempo, velocidadtiempo y espacio-tiempo correspondientes son los siguientes:

a 0 = constante

x = x0 + v0 t +

1 a0 t 2 2

v = v 0 + at

x = x0 + v0 t +

1 a0 t 2 2

2do. Caso: La partcula tiene una velocidad inicial v 0

< 0.

Ejemplo 13. Una tortuga camina en lnea recta sobre lo que llamaremos eje x con la direccin positiva hacia la derecha. La ecuacin de la posicin de la tortuga en funcin del tiempo es x(t) = 50,0 cm + (2,00 cm/s)t - (0,0625 cm/s2)t2 . a) Determine la velocidad inicial, posicin inicial y aceleracin inicial de la tortuga. b) En qu instante t la tortuga tiene velocidad cero? c) Cunto tiempo despus de ponerse en marcha regresa la tortuga al punto de partida? d) En qu instantes t la tortuga est a una distancia de 10,0 m de su punto de partida? Que velocidad (magnitud y direccin) tiene la tortuga en cada uno de esos instantes? e) Dibuje las grficas: x-t, vx-t y ax-t para el intervalo de t = 0 a t = 40,0 s. Solucin.

La partcula se desplaza de P 0 en sentido negativo con movimiento retardado (desacelerado) hasta detenerse en P 1 y cambia de sentido. A partir de ese instante la velocidad aumenta constantemente (acelerado) y se desplaza al infinito con un sentido constante.

dx = 2,00 cm s (0,125 cm s 2 )t dt dv ax = x = 0,125 cm s 2 dt a) En t = 0, x = 50,0 cm, v x = 2,00 cm s , vx = a x = 0,125 cm s 2 .


b) Hagamos vx = 0 y resolvamos para t: t = 16,0 s 10

Movimiento rectilneo c) Hagamos x = 50,0 cm y resolvamos para t. Esto da: t = 0 y t = 32,0 s . La tortuga regresa al punto de partida despus de 32,0 s. d) La tortuga est a 10,0 cm del punto de partida cuando x = 60,0 cm o x = 40,0 cm. Hagamos x = 60,0 cm y resolvamos para t: t = 6,20 s y t = 25,8 s En t = 6,20 s, vx = + 1,23 cm/s. En t = 25,8 s, vx = - 1,23 cm/s. Hagamos x = 40,0 cm y resolvamos para t :

Hugo Medina Guzmn

t2 +10t -11 = 0 t = 1 s v D = vC + at = 50 + 10 = 60 cm/s


b) v B

= 2aAB
2 vB 400 = = 20cm 2a 20

AB =
c)

v B = at 20 =2 s t= 20 = 10t 10
d) Ser la suma de los tiempos parciales: t = 2 + 3 +1 = 6 s MOVIMIENTO VERTICAL CON ACELERACIN DE LA GRAVEDAD. La variacin de la magnitud de la aceleracin g debido a la gravedad en la superficie de la tierra con la latitud est dada por la frmula internacional de la gravedad adoptada en 1930 por el Congreso Geofsico Internacional:

t = 36,4 s
(la otra raz de la ecuacin cuadrtica es negativa y por lo tanto sin significado fsico). En t = 36,4 s, vx = - 2,55 cm/s. e)

g = 978,049000 (1 + 0,0052884 sen 2


- 0,0000059 sen 2 )
2

g en cm/s ,

en grados

Ejemplo 14. Un mvil parte del reposo y de un punto A, con movimiento rectilneo y uniformemente acelerado (a =10 cm/s2); tarda en recorrer una distancia BC = 105 cm un tiempo de 3 s, y, finalmente, llega al punto D (CD = 55 cm). Calcular: a) La velocidad del mvil en los puntos B, C y D. b) La distancia AB. c) El tiempo invertido en el recorrido AB y en el CD. d) El tiempo total en el recorrido AD.

Donde es la latitud de la tierra medida en el ecuador Para = 0 (ecuador), g 0 = 978,0490 Para

= 90 (polos), g 90 = 983,2213

La variacin de la aceleracin gravitacional con la altura sobre el nivel del mar es aproximadamente

g = g 0,000002860h
h en metros y g en m/s Donde h
2

40 000 m

Cerca de la superficie de la tierra la magnitud de la aceleracin debido a la gravedad vara muy poco con la altura y en los clculos tcnicos ordinarios se toma g = 9,81 m/s2 (dirigido verticalmente hacia abajo). Un cuerpo que se deja caer est sometido a la aceleracin de la gravedad y su movimiento corresponde a un movimiento rectilneo uniformemente variado en el eje vertical perpendicular a la tierra,

Solucin. a)

1 2 at 2 vB =20 cm/s 1 2 105 = v B 3 + 10 3 2 vC = v B + at = 20 + 30 = 50 cm/s BC = v B t + 1 CD = vC t + at 2 2 1 2 55 = 50t + 10t 2


11

y = y 0 + v0 t + v = v 0 + at a = g
a) Cada libre

1 2 at 2

Movimiento rectilneo

Hugo Medina Guzmn

2v 1 h = h + v0 t + gt 2 t P = 0 y por 2 g supuesto t P = 0 , que corresponde al tiempo inicial. Observamos que t P = 2t m


La velocidad es Si se deja caer un cuerpo desde una altura h sobre el nivel del piso y consideramos despreciable la resistencia del aire. En este caso y 0 = h , v 0 = 0 , luego:

y =h
v = gt a = g

1 2 gt 2

2v0 v P = v0 g g = v0 2v0 = v0 Finalmente toca piso cuando y = 0 2v 1 2h h + v 0 t gt 2 = 0 t 2 0 t =0 g 2 g


cuya solucin es
2 + 2h v0 v0 t= g g

El cuerpo toca tierra cuando y = 0

toca el piso al tiempo

1 2 Luego h gt = 0 t = 2
y la velocidad es v =

2h g

2 gh

2 + 2h v0 v0 + t= g g

con una velocidad


2 v = v0 + 2 gh

b) Lanzamiento hacia arriba Si el mismo cuerpo desde la misma altura h se lanza hacia arriba con velocidad v 0 , se mueve con un movimiento rectilneo uniformemente retardado (desacelerado).

c) Lanzamiento hacia abajo Si el mismo cuerpo desde la misma altura h se lanza hacia abajo con una velocidad v 0 , el cuerpo se mueve en un movimiento rectilneo uniformemente acelerado.

1 y = h + v 0 t + gt 2 2 v = v0 gt a = g
El cuerpo sube hasta que alcanza la altura mxima y m . Esta corresponde a cuando la velocidad disminuye a cero.

y = h v0 t v = v0 gt a = g

1 2 gt 2

El cuerpo alcanza el piso cuando

v0 gt = 0 t m =
De aqu

v0 g
1 v0 + + 2 g g
2

y = 0. 2v 2h 1 h v0 t gt 2 = 0 t 2 + 0 t =0 g 2 g

v0 y m = h + v0 g
=

cuya solucin es
2 + 2h v0 v0 t= g g

h+

2 v0 2

toca el piso al tiempo


2 + 2h v0 v0 t= + g g

Cuando el cuerpo pasa por el punto de lanzamiento

y=h

con una velocidad 12

Movimiento rectilneo

Hugo Medina Guzmn y2 = h - 30(t - 2) - 5(t - 2)2 = 0 De aqu t = 4 s; h = 80m Ejemplo 19. Desde el piso, se lanza hacia arriba una pelota con una rapidez de 40 m/s. Calcule: a) El tiempo transcurrido entre los dos instantes en que su velocidad tiene una magnitud de 2,5 m/s. b) La distancia respecto al piso que se encuentra la pelota en ese instante. Solucin.

2 v = v0 + 2 gh

Ejemplo 15. Desde lo alto de un edificio, se lanza verticalmente hacia arriba una pelota con una rapidez de 12,5 m/s. La pelota llega a tierra 4,25 s, despus. Determine: a) La altura que alcanz la pelota respecto del edificio. b) La rapidez de la pelota al llegar al suelo. Solucin. La altura en funcin del tiempo ser

Con g = 10m/s2, v0 = 12,5 m/s y = h + 12,5t - 5t2 a) Al tiempo t = 4,25 s, y = 0, luego: h + 12,5(4,25) - 5(4,25)2 = 0, h = 37,19 m b) vy = 12,5 - 10t = 12,5 - 10(4,25) = -30,0 m/s Ejemplo 16. Se deja caer un cuerpo desde una altura de y0 = 33 m, y simultneamente se lanza hacia abajo otro cuerpo con una rapidez inicial de 1 m/s. Encontrar el instante en que la distancia entre ellos es de 18 m. Solucin. y1 = 33 - 5t2 y2 = 33 - t - 5t2 y1 - y2 = t Entonces la distancia entre ellos es 18m a los 18 s Ejemplo 17. Un cuerpo que cae, recorre en el ltimo segundo 68,3 m. Encontrar La altura desde donde cae. Solucin. Suponiendo que se solt del reposo y = h - 5t2 El tiempo en que llega al suelo es

1 y = h + v0 t gt 2 2

1 2 gt (1) 2 v y = v0 gt ( 2)

y = v0t

a) De la ecuacin (2):

v y = v0 gt1 = 2,5 v y = v0 gt 2 = 2,5

Restando obtenemos:

t = t2 t1 =

5 = 0,5s g

b) De la ecuacin (2):

v y = v0 gt1 = 2,5

40 gt1 = 2,5 37,5 t1 = = 3,83 s. 9,8


Con t1 en (1):

h = 40(3,83)

1 2 g (3,83) = 81,41 m. 2

Con t2 se obtiene la misma altura, porque es cuando la pelota est de bajada. Ejemplo 20. Una roca cae libremente recorriendo la segunda mitad de la distancia de cada en 3(s). Encuentre a) la altura desde la cual se solt. b) El tiempo total de cada. Solucin.

t=

h 5

La distancia recorrida en el ltimo segundo ser

h h y y 1 5 = 5 h h 5 = 68,2 5 1 5 5 h = 268,6 m
Ejemplo 18. Desde lo alto de un acantilado, se deja caer una piedra, desde la misma altura se lanza una segunda piedra 2 s ms tarde con una rapidez de 30 m/s. Si ambas golpean el piso simultneamente. Encuentre: La altura del acantilado. Solucin. y1 = h - 5t2 y2 = h - 30(t - 2) - 5(t - 2)2 Siendo al mismo tiempo y1 = h - 5t2 = 0 13
2 2

y =h

1 2 gt 2 t1 = h y el g

El tiempo en que alcanza h/2 es tiempo en que h = 0 es

t2 =

2h g

a) por lo tanto el tiempo empleado en la segunda parte de recorrido es

2h g
b)

h = 3 h = 524,6 m g 2h 524,6 = = 10,2 s g 5

t=

Movimiento rectilneo

Hugo Medina Guzmn

Ejemplo 21. Se deja caer una piedra desde un globo que asciende con una velocidad de 3 m/s; si llega al suelo a los 3 s, calcular: a) Altura a que se encontraba el globo cuando se solt la piedra. b) Distancia globo-piedra a los 2 s del lanzamiento.

Solucin. Primer mtodo: En el instante en que empieza a caer el cuerpo el ascensor lleva una velocidad vertical hacia arriba v. Solucin. Tomaremos el origen de coordenadas en el punto en que se suelta la piedra. Magnitudes positivas son las que tienen direcci6n hacia arriba. a) El espacio vertical y hacia abajo que debe recorrer la lmpara es:

v0 = 3m/s 1 g 10m/s 2 y = h + 3t 10t 2 2 t = 3s


Cuando la piedra toca suelo, y = 0 Luego

(h = altura del ascensor) y (vt + at2/2) ascenso del suelo de ste. La lmpara al desprenderse lleva una velocidad inicial hacia arriba v. Aplicando la ecuacin:

1 h vt + at 2 2

s = vt +

1 2 h = 3(3) 10(3) 2
= 36 m b) t = 2 s. h1: distancia al origen del globo en t'. h2: distancia al origen de la piedra en t'.
h1 = v 0 t ' = 3 2 = 6m 1 1 h2 = v 0 t '+ gt ' 2 = 3 2 10 40 14m 2 2

1 2 at 2

Siendo positivas las magnitudes hacia arriba y negativas las descendentes, tendremos:

h + vt + t=

1 2 1 at = vt gt 2 2 2 2h 23 = 0,74 s = g+a 9,8 + 1

d = 6 + 14
= 20 m

Segundo mtodo: La aceleracin de la lmpara respecto al ascensor, considerando magnitudes positivas hacia abajo, es: aBA = aB - aA = 9,8 (-1) = 10, 8 m/s2

Ejemplo 22. La cabina de un ascensor de altura 3 m asciende con una aceleracin de 1 m/s2. Cuando el ascensor se encuentra a una cierta altura del suelo, se desprende la lmpara del techo. Calcular el tiempo que tarda la lmpara en chocar con el suelo del ascensor.

1 a BA t 2 2 2h 23 = 0,74 s t= = a BA 10,8 h=
Ejemplo 23. Una bola es lanzada verticalmente hacia arriba con una velocidad de 20 m/s de la parte alta de una torre que tiene una altura de 50 m. En su vuelta pasa rozando la torre y finalmente toca la tierra. a) Qu tiempo t1 transcurre a partir del instante en que la bola fue lanzada hasta que pasa por el borde de la torre? Qu velocidad v1 tiene en este tiempo? b) Qu tiempo total t 2 se requiere para que la bola llegue al piso? Cul es la velocidad v 2 , con la que toca el piso? 14

Movimiento rectilneo c) Cul es la mxima altura sobre el suelo alcanzada por la bola? d) Los puntos P1 y P2 estn a 15 y 30 m, respectivamente, por debajo del techo de la torre. Qu tiempo se requiere para que la bola viaje de P1 a P2? e) Se desea que despus de pasar el borde, la bola alcance la tierra en 3s, con qu velocidad se debe lanzar hacia arriba de la azotea?

Hugo Medina Guzmn Ejemplo 24. Una maceta con flores cae del borde de una ventana y pasa frente a la ventana de abajo. Se puede despreciar la resistencia del aire. La maceta tarda 0,420 s en pasar por esta ventana, cuya altura es de 1,90 m. A qu distancia debajo del punto desde el cual cay la maceta est el borde superior de la ventana de abajo? Solucin. Si la velocidad de la maceta en la parte superior de la ventana es v 0 , podemos encontrarla en funcin de la altura h de la ventana y el tiempo que tarda en pasarla::

1 2 2h gt 2 gt v 0 = 2 2t 2 m 2(1,90 ) (9,8)(0,42) = 2,47 Luego: v 0 = 2(0,42) s La distancia y desde la azotea al borde superior de h = v0 t +
la ventana es: Solucin. a) Para el sistema de coordenadas mostrado en la figura, y = v 0 t +

y=

1 2 at . 2

2 v0 2,47 2 = = 0,311 m 2 g 2(9,8)

Pero en el borde del techo y = 0, luego

0 = v0 t1 +

1 2 at1 , 2

Otra forma de encontrar la distancia es: como t = 0,420 s es la diferencia entre los tiempos tomados en caer la las alturas ( y + h ) e y , tenemos

De la cual t1 = 0, indica el instante en el cual la bola es lanzada, y tambin t1 = 4,08 s, la cual es el tiempo en que la bola retorna al borde. Luego, de v = v 0 + at

t=

2( y + h) g

2y g

v1 = 20 + ( 9,8)(4,08) = 20m / s , que es el

gt 2 + y= 2

y+h

negativo de la velocidad inicial. b) 50 = 20t2 +

Elevando al cuadrado:

1 ( 9,8)t22 t2 = 5,8 s 2 v2 = 20 + ( 9,8)(5,8) = 37 m / s c) Mxima altura sobre tierra: h = y max + 50 .


De v 0 + 2ay max = 0 ,
2

gt 2 + 2 gyt 2 + y = y + h 2 gt 2 + 2 gyt 2 = h 2 Resolviendo para y :


1 2h gt 2 y= 2g 2t
Con los datos
2 1 2(1,9) (9,8)(0,42) y= = 0,311 m 2(9,8) 2(0,42) 2

ymax

(20) = = 20,4 m 2(9,8)


2

Luego, h = 70,4 m. d) Si t1 y t2 son los tiempos para alcanzar P1 y P2, respectivamente,


2 15 = 20t1 4,9t12 y 30 = 20t 2 4,9t 2

Resolviendo, t1 = 4,723 s, t2 = 5,248 s, y el tiempo de P1 a P2 es (t2 - tl) = 0,525 s. e) Si v0 es la velocidad inicial deseada, entonces v0 es la velocidad cuando pasa el borde. Luego aplicando y = v 0 t +

1 2 at al viaje hacia abajo de 2

la torre, encontramos: -50 = (- v0)(3) 4,9(3)2, v0 = 1,96 m/s.

Ejemplo 25. Malabarismo. Un malabarista acta en un recinto cuyo cielorraso est 3,0 m arriba del nivel de las manos. Lanza una pelota hacia arriba de modo que apenas llega al techo. a) Qu velocidad inicial tiene la pelota? b) Cunto tiempo tarda la pelota en llegar al techo? En el instante en que la primera pelota est en el cielorraso, el malabarista lanza una segunda pelota hacia arriba con dos terceras parte de la velocidad inicial de la primera. 15

Movimiento rectilneo c) Cunto tiempo despus de lanzada la segunda pelota se cruzan las dos pelotas en el aire? d) A qu altura sobre la mano del malabarista se cruzan las dos pelotas Solucin. a) Tomemos el sentido positivo hacia arriba. Tenemos que v y = v 0 y 2 g ( y y 0 )
2 2 2

Hugo Medina Guzmn

Luego: 0 = v 0 y 2(9,8)(3) v 0 y = 7,7 m s . b) Tambin tenemos: v y = v0 y gt = 0 = 7,7 9,8t

En el cielorraso, v y = 0 , y y 0 = 3,0 m .

t = 0,78 s . c) Tomemos el sentido positivo hacia abajo. La primera bola viaja hacia abajo una distancia d en el tiempo t . Como comienza desde su mxima altura, v0 y = 0.
2 2 2 d = v0 y t + 1 2 gt d = ( 4,9 m s )t

La segunda bola tiene . v' 0 y = 1 3 (7,7 m s) = 5,1 m s En el tiempo t habr viajado hacia arriba (3,0 m d ) y estar en el mismo lugar que la 2 primera bola. (3 d ) = v' 0 y t 1 2 gt

Ejemplo 27. En el salto vertical, un atleta se agazapa y salta hacia arriba tratando de alcanzar la mayor altura posible. Ni los campeones pasan mucho ms de 1,00 s en el aire (tiempo de suspensin). Trate al atleta como partcula y sea y mx su altura mxima sobre el suelo. Para explicar por qu parece estar suspendido en el aire, calcule la razn del tiempo que est sobre y mx / 2 al tiempo que tarda en llegar del suelo a esa altura. Desprecie la resistencia del aire. Solucin. El tiempo al caer para alcanzar y mx es:

(3 d ) = 5,1t 4,9t 2

Tenemos dos ecuaciones con dos incgnitas. Resolvindolas obtenemos: t = 0,59 s y d = 1,7 m. d) 3,0 m d = 1,3 m Ejemplo 26. Una manzana cae libremente de un rbol, estando originalmente en reposo a una altura H sobre un csped crecido cuyas hojas miden h. Cuando la manzana llega al csped, se frena con razn constante de modo que su rapidez es 0 al llegar al suelo, a) Obtenga la rapidez de la manzana justo antes de tocar el csped. b) Obtenga la aceleracin de la manzana ya dentro del csped. c) Dibuje las grficas: v-t y a-t para el movimiento de la manzana. Solucin. a) La rapidez de un objeto que cae una distancia H en cada libre una distancia H h es:

t1 =

2 y mx =1 s . g 2 y mx / 2 = g

El tiempo al caer para alcanzar y mx / 2 es:

y mx t 1 = 1 = s. g 2 2 1 El tiempo debajo de y mx / 2 es 1 , de tal 2 t2 =


manera que la razn entre el tiempo que est sobre la mitad de la altura mxima y el tiempo que est por debajo de la altura mxima es.

1/ 2 1 1/ 2

1 2 1

= 2,4.

Esto explica porque el atleta parece estar suspendido en el aire. Ejemplo 28. Un excursionista despierto ve caer un peasco desde un risco lejano y observa que tarda 1,30 s en caer el ltimo tercio de la distancia. Puede despreciarse la resistencia del aire. a) Qu altura (en m) tiene el risco? b) Si en (a) obtiene dos soluciones de una ecuacin cuadrtica y usa una para su respuesta, qu representa la otra? Solucin. a) Sea h la altura y toma un tiempo t en caer:
2 h= 1 2 gt

v = 2 g ( H h).
b) La aceleracin para llevar a un objeto desde la rapidez v al reposo sobre una distancia h es:

a=
c)

v2 2 g ( H h) H = = g 1. 2h 2h h

16

Movimiento rectilneo

Hugo Medina Guzmn

Si tarda 1,30 s en caer el ltimo tercio h :


2 3 2 h=1 2 g (t 1,3)

Eliminando h de estas dos ecuaciones obtenemos:

t 7,8t + 5,07 = 0
Resolviendo

1 3 2

2 gt 2 = 1 2 g (t 1,3)

t = 7,08s t = 3,9 3,18 1 t 2 = 0,73s

Ejemplo 30. Una piedra que cae libremente pasa a las 10 horas frente a un observador situado a 300 m sobre el suelo, y a las 10 horas 2 segundos frente a un observador situado a 200 m sobre el suelo. Se pide calcular: a) La altura desde la que cae. b) En qu momento llegar al suelo. c) La velocidad con que llegar al suelo.

La primera es la solucin correcta porque es mayor que 1,30 s,

h=

1 2

(9,8)(7,08)2 = 245,6 m

b) Con la segunda solucin para t encontramos h = 2,6 m. Esto correspondera a un objeto que estaba inicialmente cerca del fondo de este "acantilado" que era lanzado hacia arriba y tomando 1,30 s la subida a la cima y la cada al fondo. Aunque fsicamente es posible, las condiciones del problema imposibilitan esta respuesta. Ejemplo 29. Desde la cornisa de un edificio de 60 m de alto se lanza verticalmente hacia abajo un proyectil con una velocidad de 10 m/s. Calcular: a) Velocidad con que llega al suelo. b) Tiempo que tarda en llegar al suelo. c) Velocidad cuando se encuentra en la mitad de su recorrido. d) Tiempo que tarda en alcanzar la velocidad del apartado c).

Solucin.

h1 = 300m h2 = 200m h3 = 100m

t1 = 2 s g 10m/s 2

a)

v2 = v1 + gt1 v2 = v1 + 10 2 1 2 1 gt1 100 = 2v1 + 10 4 2 2 2 2 v v h4 = 2 h4 = 2 2g 2 10 H = h2 + h4 h3 = v1t1 +

Solucin. Tomamos corno origen de coordenadas el punto de lanzamiento y como sentido positivo el del eje vertical descendente. Las ecuaciones de este movimiento sern:

v1 = 40m/s De aqu se obtiene v2 = 60m/s , h = 180m 4 Finalmente H = 200 + 180 = 380 m


b) Llamando t2 al tiempo que tarda en recorrer hl:

v = v 0 + gt s = v0 t +

1 2 gt g 10m/s 2 2

v0 = 10 m/s

h1 = v1t 2 +

1 2 gt 2 2

a) y b) h = 60 m

v = 10 + 10t t = 2,6 s 1 2 60 = 10t + 10t v = 36m/s 2

1 2 300 = 40t 2 + 10t 2 2 t 2 = 5s


Luego llega al suelo a las 10 horas 5 segundos c)

c) y d) h = 30 m

v' = 10 + 10t ' t ' = 1,65 s 1 2 v' = 26,5m/s 30 = 10t '+ 10t ' 2
17

v = 2 gH =
= 87 m/s

2 10 380

Movimiento rectilneo

Hugo Medina Guzmn El desplazamiento total para el intervalo (t t 0 ) es la suma de todas las reas de todos los rectngulos de tal modo que:

PROBLEMA INVERSO - CLCULO INTEGRAL Conociendo la ley del movimiento x = x (t ) es

posible sin mayores dificultades calcular v(t ) y

a (t ) tal como fue mostrado

x = v m (t i )t
i

x(t ) v(t ) =

dx(t ) dv(t ) d 2 x(t ) a(t ) = = dt dt dt 2

La regla para los tiempos es que t i +1 = t i + t . La distancia que obtenemos con este mtodo no ser la correcta porque la velocidad cambia durante el tiempo del intervalo t . Si tomamos los intervalos muy pequeos la suma tiene mayor precisin. As es que los hacemos tan pequeos a fin de tener una buena aproximacin. Obtendremos la distancia real en el lmite:

Como hemos visto, el clculo diferencial proporciona la herramienta para determinar la velocidad y aceleracin en cualquier instante del tiempo. En esta seccin veremos cmo el clculo integral, que es el inverso del clculo diferencial, puede utilizarse para deducir las frmulas que ya hemos visto. Por ejemplo, hallar la posicin de una partcula en un instante cualquiera, dado su velocidad inicial y su aceleracin conocida. Ya hemos demostramos que el rea encerrada bajo la curva de la velocidad del diagrama velocidadtiempo es igual al desplazamiento.

x = lim v(t i )t
t 0

Obsrvese que hemos reemplazado la velocidad promedio v m por la velocidad instantnea v , porque en el lmite esta aproximacin es vlida. Los matemticos han inventado un smbolo para este lmite, anlogo al smbolo para la diferencial. El smbolo se convierte en d , v(t i ) se llama v(t ) y el smbolo sumatoria se escribe como una "s grande la cual se conoce el signo integral Luego escribimos

Area del trapecio = v0 (t t 0 ) + x x 0 = v0 (t t 0 ) + 1 2 a(t t 0 ) 2

1 2 a(t t 0 ) 2

En el caso de un movimiento con velocidad constante el desplazamiento entre los tiempos t y

x = v(t )dt
t t0

t 0 es x x 0 = v0 (t t 0 ) o x = v 0 (t t 0 )
Para un movimiento cualquiera con aceleracin variable el diagrama velocidad-tiempo ser el mostrado en la figura siguiente

El proceso de integracin es el inverso del proceso de derivacin. Con un diferencial obtenemos una frmula integral si la invertimos. Ejemplo 31. Encontrar la velocidad de un mvil a partir de la aceleracin. Solucin.

a=

dv dv = adt dt
v v0

dv = adt = a dt
t0 t0

Integrando obtenemos

v v0 = a(t t0 ) v = v0 + a(t t0 ) v= dx dx = vdt dt

Para encontrar la posicin

Si descomponemos el tiempo total desde t 0 hasta t en segmentos pequeos t , entonces cada tramo vertical que baja desde la curva de velocidades hasta el eje de absisas tiene un rea

x0

dx = vdt
t0

x0

dx = [v0 + a(t t0 )]dt


t t0

Integrando obtenemos

A = v m t Donde v m es la velocidad media del intervalo. Esta


rea corresponde al desplazamiento en ese intervalo que como se puede observar el rea faltante se complementa con el excedente del otro lado.

1 2 x x0 = v0 (t t0 ) + a(t t0 ) 2 1 2 x = x0 + v0 (t t0 ) + a(t t0 ) 2

18

Movimiento rectilneo Tambin se puede encontrar la ecuacin del movimiento expresando la integral de la siguiente manera:

Hugo Medina Guzmn

Integrando:

x = [v0 + a(t t 0 )]dt + C 2

v = adt + C1 , x = vdt + C2
Los valores de C1 y C2 dependen de las condiciones iniciales del movimiento. Pequea Tabla de Integrales

dx = x
n x dx =

x n +1 (n 1) n +1

dx = lnx x e ax ax e dx = a cos(ax ) sen(ax ) = a (u + v )dx = udx + vdx

1 2 at at 0 t + C 2 2 Como para t = t 0 se tiene x = x 0 , tenemos 1 2 2 x0 = v0 t 0 + at 0 at 0 + C2 2 1 2 C 2 = x0 v0 t 0 + at 0 2 Reemplazando el valor de C 2 obtenemos 1 1 2 x = v0 t + at 2 at 0 t + x0 v0 t 0 + at 0 2 2 1 2 x = x0 + v0 (t t0 ) + a(t t0 ) 2

x = v0 t +

Ejemplo 33. La aceleracin de una motocicleta est dada por a (t ) = 1,5t 0,12t , con t en s m/s3. La moto est en reposo en el origen en t = 0. a) Obtenga su posicin y velocidad en funcin de t. b) Calcule la velocidad mxima que alcanza. Solucin. a) Para encontrar v(t ) .
2

Ejemplo 32. Encontrar las ecuaciones del movimiento para una partcula que se mueve con

aceleracin constante inicial

y que para el tiempo a = ai

dv dv = adt = (1,5t 0,12t 2 )dt dt Integrando con v0 = 0 y t 0 = 0 : a=


Para encontrar x(t ) .

y tena una t 0 se encontraba en r 0 = x 0 i

v = 1,5t 0,12t 2 dt = 0,75t 2 0,40t 3


0

. velocidad inicial v 0 = v 0 i
Solucin. El movimiento es en el eje x . La aceleracin es

a=

dv dt

La velocidad se puede encontrar en trminos de una integral como

dx = 0,75t 2 0,40t 3 dt dx = (0,75t 2 0,40t 3 )dt Integrando con x0 = 0 y t 0 = 0 : v=


x = 0,75t 2 0,40t 3 dt = 0,25t 3 0,10t 4
0
t

t = t 0 se tiene v = v0 , tenemos v0 = at 0 + C1 C1 = v0 at 0 Reemplazando el valor de C1 obtendremos la


Como para ecuacin de la velocidad:

v = adt + C1 v = at + C1

b) Para que la velocidad sea mxima la aceleracin debe ser cero,

v = v0 + a(t t 0 ) dx dt

t = 0 a(t ) = 1,5t 0,12t = 0 1,5 t = 0,12 = 12,5s


2

Para t = 0 la velocidad es mnima Para t = 12,5 la velocidad


2 3

Ahora consideremos la definicin de la velocidad

v = 0,75(12,5) 0,40(12,5) = 39,1 m/s

v=

Tambin se puede escribir en forma integral

x = vdt + C2
Reemplazando el valor de v : 19

Ejemplo 34. Salto volador de la pulga. Una pelcula tomada a alta velocidad por M. Rothschild, Y. Schlein. K. Parker, C. Neville y S. Sternberg (3500 cuadros por segundo, The Flying Leap of the Flea, en el ScientificAmerican de noviembre de 1973) de una pulga saltarina de 210 g produjo los

Movimiento rectilneo datos que se usaron para dibujar la grfica de la figura. La pulga tena una longitud aproximada de 2 mm y salt con un ngulo de despegue casi vertical. Use la grfica para contestar estas preguntas. a) La aceleracin de la pulga es cero en algn momento? Si lo es, cundo? Justifique su respuesta. b) Calcule la altura mxima que la pulga alcanz en los primeros 2,5 ms. c) Determine la aceleracin de la pulga a los: 0,5 ms, 1,0 ms y 1,5 ms. d) Calcule la altura de la pulga a los: 0,5 ms, 1,0 ms y 1,5 ms.

Hugo Medina Guzmn

= 0,11 cm

1 (1,3 10-3 )(133) + (0,2 10-3 )(133) 2

Ejemplo 35. La grfica de la figura describe, en funcin del tiempo, la aceleracin de una piedra que baja rodando por una ladera, habiendo partido del reposo. a) Determine el cambio de velocidad de la piedra entre t = 2,5 s y t = 7,5 s. b) Dibuje una grfica de la velocidad de la piedra en funcin del tiempo.

Solucin. a) Pendiente de a = 0 para t 1,3 ms b) La altura mxima corresponde al recorrido hasta cuando la aceleracin se hace cero y llega al tiempo t = 2,5 ms, y es el rea bajo la curva v versus t. (Dibujado aproximndolo a Un tringulo y un rectngulo).

Solucin.

dv dv = adt dt Como a (t ) es la ecuacin de la recta: 84 a2 = = 0,8 a = 0,8t + 2 t 0 7,5 2,5 dv = (0,8t + 2 )dt
a)

a=

Integrando:

hmax = rea bajo (v t ) ATringulo + ARectngulo


1 [(1,3)(133) + (2,5 1,3)(133)]10 3 2 0,25 cm

2 ) + 2(t t 0 ) v v0 = 0,4(t t 0 Con t 0 = 2,5s , t = 7,5s , y v = v v0 : 2

v0

dv = (0,8t + 2)dt
t t0

v = 0,4(7,52 2,52 ) + 2(7,5 2,5) cm = 30 s

c) a = pendiente del grfico v t.

a (0,5 ms) a (1,0 ms) 133 2 = 1,0 105 cm s -3 1,3 10 a (1,5 ms) = 0 porque la pendiente es cero.

Otra manera de encontrar el cambio de velocidad es encontrando el rea bajo la curva a versus t, entre las lneas en t = 2,5 s y t = 7,5 s. El rea es:
1 2

(4 + 8)(7,5 2,5) = 30

cm s

d) h = rea bajo el grfico v t.

Como la aceleracin es positiva, el cambio de velocidad es positivo. b)

h (0,5) ATringulo =

1 (0,5 10 -3 )(33) 2 = 8,3 10 3 cm 1 h (1,0) ATringulo = (1,0 10 -3 )(100) 2 = 5,0 10 2 cm h (1,5) ATringulo + ARectngulo

Ejemplo 36. La velocidad de un punto que se mueve en trayectoria recta queda expresada, en el SI por la ecuacin: v = 40 - 8t. Para t = 2 s, el punto dista del origen 80 m. Determinar: 20

Movimiento rectilneo a) La expresin general de la distancia al origen. b) El espacio inicial. c) La aceleracin. d) En qu instante tiene el mvil velocidad nula? e) Cunto dista del origen en tal instante? f) Distancia al origen y espacio recorrido sobre la trayectoria a partir de t = 0, cuando t = 7 s, t = 10 s y t = 15 s. Solucin. a)

Hugo Medina Guzmn

s = vdt = (40 8t )dt = 40t 4t 2 + C

e) s5 =16 + 40x5 - 4x52 = 116 m f) s7 =16 + 40x7 - 4x72 = 100 m sl0 =16 + 40x10 - 4x102 = 16 m s15 = 16 + 40x15 - 4x152 = -284 m Clculo de caminos sobre la trayectoria a partir de t = 0: El mvil cambia el sentido de su velocidad para t = 5s El recorrido en los 5 primeros segundos es: C5 = s s0 = 116 - 16 = 100 m A ellos hay que sumar el recorrido en los segundos restantes que se obtienen de la integral de la ecuacin general de la velocidad, en valor absoluto, entre los limites t = 5 s y t = instante final.

s = s 0 + 40t 4t 2 b) 80 = s0 + 80 - 16 s0 = 16 dv m c) a = = 8 2 dt s d) 0 = 40 - 8t t =5 s

Representacin grfica de la velocidad origen en funcin del tiempo

En la grfica de la velocidad frente al tiempo, el rea limitada por el eje de abscisas y la grfica entre dos instantes coincide numricamente con el camino recorrido por el mvil entre esos dos instantes. Ejemplo 37. El vector velocidad del movimiento de una partcula viene dado por

C7 = 100 +

(40 8t )dt = 116m


7 5

+ (6t 2 - 5) v = (3t - 2)i j m/s. Si la posicin del

mvil en el instante t =1 s es Calcular

2 r = 3i j m.

a) El vector posicin del mvil en cualquier instante.

C10 = 100 +

(40 8t )dt = 200m


10 5

b) El vector aceleracin. c) Las componentes tangencial y normal de la aceleracin en el instante t = 2 s. Dibujar el vector velocidad, el vector aceleracin y las componentes tangencial y normal en dicho instante. Solucin.

C15 = 100 +

(40 8t )dt = 500m


15 5

a) Para el movimiento horizontal

vx = 3t - 2
Representacin grfica de la distancia al origen en funcin del tiempo

ax =

dvx m =3 2 dt s

Como

vx =

dx dx = v x dt , integrando dt

21

Movimiento rectilneo

Hugo Medina Guzmn

dx = (3t 2 )dt
t

an = asen ( ) = 2m/s 2
CINEMTICA DE PARTCULAS LIGADAS. MOVIMIENTOS DEPENDIENTES. Observemos los sistemas fsicos de la figura. Podramos decir que estos sistemas se componen de varias partculas ligadas (conectadas).

7 3 x = t 2 2t + m 2 2
Para el movimiento vertical

v y = 6t 2 - 5

ay =

dv y dt

= 12t

m s2

Como

vy =
t

dy dy = v y dt , integrando dt

y = (2t 3 5t + 1) m

dy = 6t 2 5 dt
1

7 3 r = t 2 2t + i - 2t 3 5t + 1 j 2 2

Las partculas podran ser las poleas y los cuerpos a desplazar (bloques, baldes). La ligadura la tienen a travs de las cuerdas. Es decir, cuando el hombre desplaza el extremo de la cuerda con una aceleracin a, la aceleracin de las poleas y los cuerpos a desplazar (bloques, baldes) tendrn una dependencia de a. Lo mismo se cumplir para las otras variables cinemticas (desplazamiento y velocidad). Ejemplo 38. Anlisis del montaje de la figura siguiente.

b)

+ 12t a = 3i j

c) Para t = 2 s vx = 4 m/s, vy = 19 m/s ax = 3 m/s2, ay = 24 m/s2


2 2 a = ax + ay = 24,2m / s 2

tan =

vy vx ay ax

19 = 4,75 = 78 o 4 24 = 3 = 83o 3

tan =

Para analizar las relaciones que hay entre las variables cinemticas del bloque m1 , del balde m 2 y de la polea mvil, debemos primero saber cules son sus posiciones. Para ello elegimos un sistema de coordenadas. En nuestro caso elegimos el eje y apuntando hacia abajo y con el origen en el techo. Para el sistema de coordenadas escogido las posiciones del bloque, del balde y de la polea son respectivamente: y1 , y 2 , y p . Estas se representan en la figura siguiente.

at = a cos( ) = 24,1m/s 2
22

Movimiento rectilneo La longitud de la cuerda debe permanecer constante en todo instante. Por tanto debe ser siempre vlida la siguiente relacin: Longitud de la cuerda = constante AB + arco BC + CD +arco DE +EF = constante De la figura podemos concluir que las siguientes relaciones son vlidas:

Hugo Medina Guzmn Anlogamente podramos hacer un anlisis para las aceleraciones, y concluiramos que:

1 a p = a1 2
Es decir, si el bloque por ejemplo, baja con una aceleracin igual a 2,0 m/s2 , la polea subir con una aceleracin igual a 1,0 m/s2 . De esta figura tambin se deduce la siguiente relacin entre la posicin del balde y la posicin de la polea mvil: y 2 = y p + c1 (3) Si el balde se desplaza una cantidad y 2 , y la polea se desplaza una cantidad

AB = y p CD = y p c2
EF = y1 c2
Por tanto,

y p + arcoBC + ( y p c2 ) + arcoDE + y1 = constante

Como los arcos BC y DE permanecen constantes podremos escribir la relacin anterior as: 2 y p + y1 = k (1) Siendo k una constante. Esta ecuacin relaciona las variables cinemticas de la polea mvil y del bloque. Si el bloque se desplaza una cantidad y1 y la polea en una cantidad

y p . y p + y p .

El balde pasa a ocupar la posicin: y 2 + y 2 , La polea pasa a ocupar la posicin Sin embargo, la relacin anterior se debe seguir cumplindose. ( y 2 + y 2 ) = y p + y p + c1 (4)

y p . y p + y p ,

La nueva posicin de la polea: La nueva posicin del bloque: y1 + y1 . Sin embargo, la relacin anterior debe seguir cumplindose: 2 y p + y p + ( y1 + y1 ) = k (2)

Restando (3) y (4) obtenemos,

y 2 = y p
Los desplazamientos de la polea y el balde son iguales. Si dividimos la ecuacin anterior por el intervalo de tiempo t obtenemos como se relacionan las velocidades: v2 = v p . Las velocidades de la polea y del balde son iguales. Lo mismo podremos concluir para las aceleraciones:

Restando (1) de (2), obtenemos:

2y p + y1 = 0
y y p = 1 2
Por ejemplo, si el bloque baja 1,0 m, la polea solo sube 0,50 m. La polea solo se desplaza la mitad de lo que se desplaza el bloque.

a2 = a p
En definitiva si el bloque baja con una aceleracin igual a 4 m/s2, el balde y la polea mvil subirn con una aceleracin igual a 2 m/s2.

PREGUNTAS Y PROBLEMAS 1. Un acelerador atmico emite partculas que se desplazan con una rapidez de 2,8x108 m/s. cunto demoran estas partculas en recorrer una distancia de 5,6mm? Respuesta 2x10-11 s. 2. Se desea calcular cul es la profundidad de un lago, para tal efecto se usa un instrumento conocido como sonar que mide el tiempo que tarda un pulso sonoro en ir y volver desde la superficie del agua. Si se sabe que la rapidez del sonido en el agua es de 1450m/s y el instrumento marc 0,042s cuando se hizo la medicin, calcule la profundidad del lago. Respuesta. 30,45m 3. Una cucaracha se desplaza en lnea recta y su posicin con respecto al tiempo se expresa de acuerdo al siguiente grfico. De acuerdo a la informacin dada se pide calcular. a) distancia recorrida entre 4s y 9 s b) distancia recorrida entre 9 s y 14s c) distancia recorrida entre 0 y 16s. d) velocidad media entre 0s y 16s. e) velocidad media entre 9s y 16s.

23

Movimiento rectilneo

Hugo Medina Guzmn

a)

v = 2t 2 t 4 ; b) x = 2 + 2t 3 / 3 t 5 / 5
3 2

9. El movimiento de una partcula se define mediante la relacin x = t / 3 3t + 8t + 2 , donde x se expresa en metros y t en segundos. Determinar a) el momento en que la velocidad es nula; b) la posicin y la distancia total recorrida cuando la aceleracin es nula. Respuesta a) 2s, 4s; b) 8m, 7,33m 10. El movimiento de una partcula est dado por la ecuacin horaria x = t + 4t + 5 x sobre el eje x, x en metros t en segundos. a) Calcular la velocidad y la aceleracin de la partcula en el instante t. b) Encontrar la posicin, la velocidad y la aceleracin de la partcula para t 0 = 2s y t 1 = 3s.
3 2

Respuesta a) 4m b) 8m c) 22m d) 5/8 m/s e) 0

4. Un hombre camina con una velocidad v constante pasa bajo un farol que cuelga a una altura H sobre el suelo. Encontrar la velocidad con la que el borde de la sombra de la cabeza del hombre se mueve sobre la tierra. El alto del hombre es h. Respuesta

Hv H h
5. Un tren arranca en una estacin y acelera uniformemente a razn de 0,6 m/s hasta alcanzar una velocidad de 24 m/s. Determinar el tiempo empleado y la distancia recorrida en ese perodo si la velocidad media fue: a) 16 m/s, b) 22m/s. Respuesta a) 60s, 960m, b) 240s, 5280m 6. Un ciclista recorre 100 km en 2 horas. El viaje de vuelta dos das ms tarde lo realiza en el tiempo usual de 6 horas. a) Cul es su rapidez media a la ida? b) Cul es su rapidez media al regreso? c) Su rapidez media en e viaje completo? d) Su velocidad media en e} viaje entero? Respuesta. a) 50 km/h , b) 16,7 km/h c) 25 km/h d) 0 7. Un automvil que viaja con una velocidad de 50 km/h hacia el oeste repentinamente empieza a perder velocidad a un ritmo constante y 3 segundos ms tarde su velocidad es de 25 km/h hacia el oeste. a) Cunto tiempo tardar en detenerse el auto, contando a partir del momento en que empez a desacelerar? b) Cul es la distancia total que recorrer antes de detenerse? c) Cul sera el tiempo necesario para detenerse y la distancia recorrida el) la frenada con la misma aceleracin, pero con una velocidad inicial de 100 km/h? Respuesta. a) t = 6s ; b) 41,7m ; c) 125; 125m 8. La aceleracin de una partcula est dada por:
2

c) Cules son la velocidad media y la aceleracin media de la partcula entre t 0 y t1 ? Respuesta. a) v = (3t + 8t)m/s , a = ( 6t + 8 ) m/s b)
2 2

x0 = 29m, v0 = 27 m/s, a 0 = 20 m/s 2

x1 = 68 m v1 = 51 m/s, a1 = 26 m/s 2 2 c) v m = 39 m/s , a m = 23 m/s


11. La posicin de una partcula que se mueve en el eje x est dada por 8 t + 5, x es la distancia a origen en metros y t es el tiempo en segundos. a) Para t = 2, encontrar la posicin, velocidad y aceleracin b) Grafique x versus t c) Encuentre la ley horaria, la ley del movimiento y la trayectoria. d) Analizar el movimiento. Respuesta. a) x = -3, v = 0 , a = 4 b) s = 2t 8t + 5 , r = 2t Trayectoria rectilnea en el eje x.
2

8t + 5)i

a = 4t 4t 3 , t 0 .
a) Hallar la velocidad de la partcula en funcin del tiempo. b) Hallar su posicin en funcin del tiempo. Respuesta 24

12. Un automvil se encuentra detenido frente a un semforo, le dan luz verde y arranca de modo que a los 4s su rapidez es de 72 km/hora. Si se movi en trayectoria rectilnea, con aceleracin constante, I.- Determine: a) La rapidez inicial en metros por segundo. b) El mdulo de la aceleracin en ese tramo. c) La rapidez que lleva a los 3s. d) La distancia que recorre en los tres primeros segundos e) La distancia que recorre entre t = 2s y t = 4s. II.- Haga un grfico representativo de posicin versus tiempo y de la rapidez versus tiempo. Respuesta. d) 22.45m a) 20m/s b) 5 m/s2 c) 15m/s e) 30m

Movimiento rectilneo

Hugo Medina Guzmn movimiento para retornar hacia A con una aceleracin constante de mdulo 6(m/s2). Calcular: a) La distancia total cubierta hasta que la partcula retorne al punto A. b) El tiempo total para el recorrido completo hasta volver a dicho punto A. c) El intervalo de tiempo que transcurre entre los pasos de la partcula por el punto situado a 1/3 de AB, medido desde A. 17. Desde una altura de 45m se deja caer un objeto A. simultneamente se lanza un objeto B verticalmente desde una altura de 5m. Calcular: a) la velocidad inicial de B para que los objetos se crucen a una altura de 20m. b) la distancia que separa a los objetos cuando B alcanza su altura mxima. 18. Sobre un mismo eje x se mueven dos partculas A y B. En t = 0 la partcula A parte desde P con (m/s2). Un segundo aceleracin constate de 15i despus, B pasa por Q con una velocidad de 20i (m/s). Encuentre las retardaciones constantes que deben aplicar A y B a partir de este ltimo instante para que ambas partculas se detengan simultneamente antes de chocar.

13. Una partcula A, se mueve en el eje X, de acuerdo a la siguiente grfica. Determinar a partir del grfico de la partcula: a) b) c) d) e) f) g) Velocidad media entre t = 0 y t = 4 s Velocidad instantnea en t = 2 s Aceleracin media entre t = 0 y t = 4 s Intervalos de tiempo en que se acerca al origen Intervalos de tiempo en que se aleja del origen Ecuacin Itinerario de la partcula A Qu tipo de movimiento tiene esta partcula?

Respuesta. a) ( -8;0)m/s b) (-8;0)m/s d) (0-3)s e)(3-....) f) x(t ) = 24 8t g) Movimiento rectilneo uniforme.

c) 0

14. Un vehculo se mueve en el eje x de acuerdo con la siguiente ecuacin de itinerario:

x(t ) = 20 36t + 6t 2 . Con x medido en metros y

t en segundos. a) Identifique a posicin inicial, la velocidad inicial y la aceleracin. b) Determine la ecuacin que entregue la velocidad para cualquier instante. c) Determine el instante en que cambia de sentido d) La velocidad de la partcula en t = 2 s y en t = 4 s e) Posicin de la partcula en t = 6 segundos f) Grfico x versus t. Describa la curva g) Grfico v x versus t. Describa la curva h) Grfico a versus t. Describa la curva Respuesta. a) (20,0)m (-36,0)m/s (12,0)m/s2 b) v(t ) = 36 + 12t c)3s d) (-12,0)m/s (12,0)m/s e) (20,0)m 15. Se lanza un cuerpo hacia arriba con una rapidez de 16m/s, a) Qu altura alcanza a subir? b) Qu tiempo demora en volver al punto de partida? Respuesta. a) 3,2m b) 6,4s 16. Una partcula se mueve sobre una recta horizontal; parte hacia la derecha desde un punto A con una rapidez de 28 (m/s) y una retardacin constante de mdulo 12(m/s2). En el punto B, es donde se anula su rapidez, invierte el sentido de 25

19. Una partcula se mueve a lo largo del eje x con aceleracin constante. En t = 0 pasa por la

posicin

m con una velocidad x 0 = 10i

m/s y en t =3s su posicin es v 0 = 20i

m. Calcule: x = 52i a) La ecuacin itineraria de la partcula b) La distancia recorrida en el intervalo (3-6) s. c) La velocidad media en el intervalo (4-7) s. d) Intervalos de tiempo en que la partcula se aleja del origen del sistema.

20. Sobre el eje x de un sistema de coordenadas se mueven dos partculas A y B. El grfico (a) es una parbola cuadrtica que muestra la variacin de la componente x de la posicin en funcin del tiempo de la partcula A. El grfico (b) muestra la variacin de la componente v x de la velocidad en funcin del tiempo de la partcula B. Si en t = 0, ambas partculas tienen la misma posicin, determinar: a) Ecuacin horaria de las partculas A y B.

Movimiento rectilneo b) Posicin de B cuando A cambia de sentido de movimiento. c) Instante en que se encuentran. d) Distancia recorrida por A y B entre 3 y 9 s.

Hugo Medina Guzmn

23. Un cuerpo que se ha dejado caer desde cierta altura, recorre 72 m en el ltimo segundo de su movimiento. Calcule la altura desde la cual cay el cuerpo y el tiempo que emple en llegar al suelo. 24. Un hombre parado en el techo de un edificio tira un cuerpo verticalmente hacia arriba con una rapidez de 14m/s. El cuerpo llega al suelo 4,7s ms tarde. a) Cul es la mxima altura alcanzada por el cuerpo? b) Qu altura tiene el edificio? c) Con qu rapidez llegar el cuerpo al suelo?

21. En el grfico de la figura estn representadas la componente v x del vector velocidad de dos partculas, A y B, que se mueven a lo largo del eje x Calcular: a) La aceleracin de B. b) Camino recorrido por A y B cuando B alcanza la

m/s. velocidad v B = 30i c) Desplazamiento de B en el intervalo (0-10)s. d) Ecuacin horaria de A si en t0 = 0 su posicin es

m. x 0 = 8i

25. Un malabarista mantiene cinco bolas continuamente en el aire, lanzando cada una de ellas hasta una altura de 3m. a) Cul es el tiempo que debe transcurrir entre lanzamientos sucesivos? b) Cules son las alturas de las otras pelotas en el momento en que una de ellas vuelve a su mano? Respuesta. a) 0,31s ; b) 1,91; 2,87; 2,87 y 1,91 m. 26. Dos cuerpos son lanzados uno despus de otro con las mismas velocidades

v0

desde una torre alta. El primer

cuerpo es lanzado verticalmente hacia arriba, y el segundo verticalmente hacia abajo despus del tiempo . Determinar las velocidades de los cuerpos una con respecto al otro y las distancias entre ellos en el instante t > . Respuesta. La velocidad del primer cuerpo relativa al

22. Dos partculas A y B se mueven sobre el mismo eje x. En t = 0, B pasa por Q con m/s v B (0 ) = ( 5,0 ) m/s y 2s despus A pasa

segundo es:

v1 v 2 = 2v0 g

La distancia es

S = 2v0 t v0 gt +

1 2 g 2

m/s. Encuentre las retardaciones por P a 6i constantes que deben aplicar A y B a partir de este ltimo instante para que ambas partculas se detengan simultneamente justo antes de chocar. Determine la ecuacin itinerario de A y B (diga cul es su origen).

26

Movimiento en un plano y en el espacio

Hugo Medina Guzmn

CAPITULO 3. Movimiento en un plano y en el espacio


MOVIMIENTO CIRCULAR Se define movimiento circular como aqul cuya trayectoria es una circunferencia. Una vez situado el origen O de ngulos describimos el movimiento circular mediante las siguientes magnitudes. Posicin angular, En el instante t el mvil se encuentra en el punto P. Su posicin angular viene dada por el ngulo , que hace el punto P, el centro de la circunferencia C y el origen de ngulos O. El ngulo , es el cociente entre la longitud del arco S y el radio de la circunferencia r, = S / r . La posicin angular es el cociente entre dos longitudes y por tanto, no tiene dimensiones. es

1 . La velocidad angular del mvil ha cambiado = 1 0 en el intervalo de tiempo


t = t1 t 0 comprendido entre t 0 y t1 .

Se denomina aceleracin angular media al cociente entre el cambio de velocidad angular y el intervalo de tiempo que tarda en efectuar dicho cambio.

m =

La aceleracin angular en un instante, se obtiene calculando la aceleracin angular media en un intervalo de tiempo que tiende a cero.

= lim

d = t 0 t dt

Velocidad angular,

1 . El mvil se habr

En el instante t1 el mvil se encontrar en la posicin P1 dada por el ngulo

RELACIN ENTRE LAS MAGNITUDES ANGULARES Y LINEALES De la definicin de radin (unidad natural de medida de ngulos) obtenemos la relacin entre el arco y el radio. Como vemos en la figura, el ngulo se obtiene dividiendo la longitud del arco entre su radio

desplazado = 1 0 en el intervalo de tiempo

t = t1 t 0 comprendido entre t 0 y t1 .

s s' = r r'

Se denomina velocidad angular media al cociente entre le desplazamiento y el tiempo.

m =

, con las unidades en el SI de rad/s. t

Derivando s = r respecto del tiempo obtenemos la relacin entre la velocidad lineal y la velocidad angular

ds d =r v = r dt dt

Como ya se explic en el movimiento rectilneo, la velocidad angular en un instante se obtiene calculando la velocidad angular media en un intervalo de tiempo que tiende a cero.

La direccin de la velocidad es tangente a la trayectoria circular, es decir, perpendicular a la direccin radial


0H

= lim

d = t 0 t dt

Aceleracin angular, Si en el instante t la velocidad angular del mvil es y en el instante t1 la velocidad angular del mvil 1

Aceleracin tangencial Derivando esta ltima relacin con respecto del tiempo obtenemos la relacin entre la aceleracin tangencial a t y la aceleracin angular.

dv d =r at = r dt dt

Movimiento en un plano y en el espacio Existe aceleracin tangencial, siempre que el mdulo de la velocidad cambie con el tiempo, es decir, en un movimiento circular no uniforme Hallar el desplazamiento angular a partir de la velocidad angular. Si conocemos un registro de la velocidad angular del mvil podemos calcular su desplazamiento 0 entre los instantes definida.

Hugo Medina Guzmn

d d = dt , integrando dt obtenemos el desplazamiento 0 del mvil entre los instantes t 0 y t :


Siendo

d = [
t
0

t0

+ (t t0 )] dt
1 2

t 0 y t , mediante la integral

= 0 + 0 (t t0 ) + (t t0 )2
Habitualmente, el instante inicial

0 = dt
t0

t 0 se toma como

Hallar el cambio de velocidad angular a partir de la aceleracin angular. Del mismo modo que hemos calculado el desplazamiento angular del mvil entre los instantes t 0 y t , a partir de un registro de la velocidad
2H

cero. Las frmulas del movimiento circular uniformemente acelerado son anlogas a las del movimiento rectilneo uniformemente acelerado. = constante , = 0 + t ,

= 0 + 0 t + t 2
Despejando el tiempo t en la segunda ecuacin y sustituyndola en la tercera, relacionamos la velocidad angular con el desplazamiento 0 .

1 2

angular

d
dt

en funcin del tiempo t .

=
t t0

d dt

0 = dt 0 = dt
t0

2 = 02 + 2 ( 0 )
COMPONENTES NORMAL Y TANGENCIAL DE LA ACELERACIN. Cuando el sistema de referencia se sita sobre la partcula tal como se indica en la figura, pero no de cualquier modo. Uno de los ejes siempre est perpendicular a su trayectoria, y el otro siempre es tangente a la misma. As pues,

MOVIMIENTO CIRCULAR UNIFORME Un movimiento circular uniforme es aqul cuya velocidad angular es constante, por tanto, la aceleracin angular es cero.

La posicin angular del mvil en el instante t podemos calcularla integrando

d d = dt dt
t

O grficamente, en la representacin de en funcin de t. Habitualmente, el instante inicial t 0 se toma como cero. Las ecuaciones del movimiento circular uniforme son anlogas a las del movimiento rectilneo uniforme
1H

0 = (t t 0 )

d = dt
t0

= 0 = constante = 0 + t

El primero siempre pasar por el centro de la circunferencia. Al primer eje se le denomina eje normal, con vector unitario (r =n ) y al segundo eje

MOVIMIENTO CIRCULAR UNIFORMEMENTE ACELERADO Un movimiento circular uniformemente acelerado es aqul cuya aceleracin es constante. Dada la aceleracin angular podemos obtener el cambio de velocidad angular 0 entre los instantes t 0 y t , mediante integracin de la velocidad angular

. Debemos tangencial, con vector unitario t estudiar ahora que componentes tienen la velocidad y la aceleracin en este sistema de referencia.
Velocidad. Con anterioridad se ha deducido que el vector velocidad siempre es tangente a la trayectoria descrita. Por tanto es fcil afirmar que en este

()

en funcin del tiempo = 0 + (t t0 ) .

movimiento la velocidad ser de la forma Aceleracin. 2

v = vt

Movimiento en un plano y en el espacio No es tan obvio que la aceleracin tenga una sola componente, de manera que adoptar la expresin

Hugo Medina Guzmn

, por lo que parntesis es efectivamente n

+ an n general a = at t
Sabemos por la definicin de aceleracin que

dt dt

dv , luego. a= dt

dt v = n = n . dt R dv v2 t n Finalmente: a = dt R
quedar como As, en esta expresin, se denomina aceleracin tangencial (at ) al trmino at

d v dvt dv dt a= = = t +v dt dt dt dt

Estudiemos el ltimo trmino de esta expresin

Si se define el ngulo , como el ngulo formado por el eje normal con el eje de abscisas (eje x), tal como se muestra en la figura.

dt dt

normal (a n )

dv y aceleracin dt v2 a la ecuacin an = R

desde el No es difcil darse cuenta que el vector t sistema de referencia situado en el centro de la circunferencia tendr la forma + cos al ser t = sen i j , mientras que n perpendicular a este adoptar la expresin
+ sen n j = cos i Derivando t dt d d sen i = cos j dt dt dt dt d ( cos i sen = j) dt dt

De esta expresin para la aceleracin pueden concluirse cosas sustancialmente importantes: Existen dos componentes: Una tangente a la trayectoria y una perpendicular y orientada hacia el centro de la circunferencia. La aceleracin tangencial slo se dar en aquellos movimientos en los que el mdulo de la velocidad vare con el tiempo. Por tanto, en el caso particular del MCU, su aceleracin tangencial ser nula. La aceleracin normal siempre existir, salvo que el radio de curvatura fuera muy grande, con lo cual tendera a cero, que es el caso extremo de los movimientos rectilneos. Concluyendo pues, en un MCU, la aceleracin

v2 n tendr la expresin a = es decir slo R

presentar aceleracin normal. Un objeto puede experimentar la aceleracin normal o centrpeta y la aceleracin tangencial. En las figuras siguientes se muestran algunas combinaciones posibles para v y a para un auto en movimiento. Para entender la aceleracin, descompngala en las componentes paralela y perpendicular a v . Para decir si el auto est dando vuelta a la derecha o a la izquierda, imagnese que usted es el conductor que se sienta con el vector de la velocidad dirigido hacia adelante de usted. Un componente de la aceleracin hacia adelante significa que la velocidad est aumentando.

Ahora bien, si tomamos un desplazamiento diminuto sobre la circunferencia, al que denominamos ds , teniendo en cuenta que arco = ngulo x radio, del esquema adjunto se deduce que ds = Rd , y adems el mdulo de la velocidad instantnea lo

ds , utilizando estos dt d v dos ltimos llegamos a = = , dt R dt reemplazando en : dt dt v + sen = (cos i j ) , si observamos dt R


podemos expresar como v

detenidamente esta ecuacin, comprobaremos que el 3

Movimiento en un plano y en el espacio

Hugo Medina Guzmn rapidez lineal de un pasajero en el borde es constante e igual a 7,00 m/s. Qu magnitud y direccin tiene la aceleracin del pasajero al pasar a) por el punto ms bajo de su movimiento circular? b) por el punto ms alto? c) Cunto tarda una revolucin de la rueda?

Ejemplo 1. Un avin a chorro militar de combate volando a 180 m/s sale de una picada vertical dando la vuelta hacia arriba a lo largo de una trayectoria circular de 860 m de radio cul es la aceleracin del avin? Exprese la aceleracin como mltiplo de g. Solucin.

a=

v 2 180 2 m = = 37,7 2 r 860 s g a = 37,7 = 3,8 g 9,8

Solucin. a) a =

v 2 7,00 2 m = = 3,50 2 . La aceleracin el R 14,0 s


2

Ejemplo 2. Una rueda de 75 cm de dimetro gira alrededor de un eje fijo con una velocidad angular de 1 rev/s. La aceleracin es de 1,5 rev/s2. a) Calclese la velocidad angular al cabo de 6 segundos. b) Cunto habr girado la rueda en ese tiempo? c) Cul es la velocidad tangencial en un punto de la periferia de la rueda en t = 6 s? d) Cul es la aceleracin resultante de un punto de la de la periferia para t = 6 s? Solucin.

punto ms bajo del crculo es hacia el centro, hacia arriba. b) a = 3,50 m/s , dirigida hacia abajo., hacia el centro. c) Como v =

2R T 2R 2 (14,0) T= = = 12,6 s 7,00 v

rad rad , = 3 2 R = 37,5 cm , 0 = 2 s s a) (t ) = 0 + t

(6 ) = 2 + 3 (6) = 20
b)

rad s

1 2 1 (6 ) = 2 (6) + (3 )(6 2 ) = 66 rad 2 66 Habr girado = 33 vueltas. 2 c) v(t ) = R(t )

(t ) = 0 t + t 2

Ejemplo 4. La rueda de la figura del problema anterior, que gira en sentido antihorario, se acaba de poner en movimiento. En un instante dado, un pasajero en el borde de la rueda que est pasando por el punto ms bajo de su movimiento circular tiene una rapidez de 3,00 m/s, la cual est aumentando a razn de 0,500 m/s2. Calcule la magnitud y la direccin de la aceleracin del pasajero en este instante. Solucin.

m v 2 3,00 2 m ac = = = 0,643 2 , y at = 0,5 2 R 14,0 s s


Luego:

= 0,643 + at t j + 0,5i a = ac n 0,643 2 + 0,5 2 = 0,814 m s2

v(6 ) = 37,5(20 ) = 750


d) an =
2 (6 )

cm s

a = a c2 + at2 =

R
2

= tan 1

0,5 = 37,9 0,643

at = R an = (3 )(37,5) = 353,25 cm/s2.


2 a = an + at2 = 147894,42 cm/s2.

an = (20 ) (37,5) = 147894 cm/s2.

Ejemplo 3. Una rueda de la fortuna de 14,0 m de radio gira sobre un eje horizontal en el centro. La 4

Movimiento en un plano y en el espacio

Hugo Medina Guzmn

Como

H =

2 2 , M = donde TH = 12 h y TH TM H =

TM = 1 h y bajo la condicin que estos formen un


ngulo de 90, es decir, M De (2) - (1), con 0 H Ejemplo 5. Una partcula se mueve sobre una circunferencia de radio R con aceleracin angular constante partiendo del reposo. Si la partcula realiza n vueltas completas a la circunferencia en el primer segundo, determine la aceleracin angular de la partcula. Determine adems el nmero de vueltas que realiza la partcula durante el siguiente segundo del movimiento. Solucin. Aqu Se encuentra para t:

= 0M = 0 , M H = ( M H )t
t= 2(M H )

3 h, 11

Es decir, en t = 16,36 min. Por lo tanto forman 90 a las 12:16:22 h. Ejemplo 7. Dos partculas describen movimientos circulares de radio R = 1m, como lo muestra la figura. El primero (1) parte de O con rapidez angular = 10 rad/s constante en sentido antihorario y el segundo (2) parte del reposo del mismo punto en sentido horario con aceleracin tangencial constante de 2 m/s . Determine cuando y donde se cruzan ambas partculas.
2

= t 2
1 = 4n 2

1 2

Entonces 2n =

1 2 2 Como = (4n )t = 2nt , 2 Nmero de vueltas para t = 1


n(1) =
2

Nmero de vueltas para t = 2

(1) 2

n(2) =
2

(2 ) 2

Durante el siguiente segundo (dos) realiza

(2 ) (1) = n(2 2 12 ) = 3n 2

vueltas.

Ejemplo 6. En un reloj anlogo el horario y el minutero coinciden a las 12:00:00 horas. A qu hora minutero y horario formarn un ngulo de 90? Solucin.

Solucin. Como el cuerpo (1) se mueve con M.C.U., la posicin angular de este ser: 1 = 0 + 1 t = 10 t . (1) El cuerpo (2) posee una aceleracin tangencial constante y por lo tanto, se trata de un M.C.U.A. Debido que at = R = 2 m/s , = 2rad/s . Por
2 2

otro lado, como parte del reposo,

0 = 0.

2 = 2 t 2 = t 2
El recorrido se muestra en la figura siguiente:

1 2

Como los movimientos del horario y minutero son circulares uniformes, encontramos para la posicin angular del horario: H = 0 H + H t . (1) Anlogamente para el minutero se tiene: M = 0 M + M t . (2)

El encuentro se produce cuando:

1 + 2 = 2
10t + t 2 = 2
5

Movimiento en un plano y en el espacio

Hugo Medina Guzmn

t = 0,59 s t 2 + 10t 2 = 0 1 t 2 = 10,59 s La solucin significativa es: t = 0,59 s


Reemplazando este valor de t en ecuacin (1), se obtiene para el ngulo de encuentro: encuentro = 5,9 rad = 338,04 . Ejemplo 8. Dos vehculos describen la misma trayectoria circular de radio 0,75 m. El primero est animado de un movimiento uniforme cuya velocidad angular es de 60 rpm. y sale de la posicin A cuando se empieza a contar el tiempo. El segundo mvil est animado de un movimiento uniformemente acelerado cuya aceleracin angular vale - /6 rad/s2, pasa por B dos segundos ms tarde llevando una velocidad angular de 120 rpm. a) Escribir las ecuaciones del movimiento de cada uno de los mviles. Hallar el instante y la posicin de encuentro por primera vez de ambos mviles. b) La velocidad lineal, la velocidad angular, las componentes tangencial y normal de la aceleracin de cada uno de los mviles en el instante de encuentro. c) Realcese un esquema en el que se especifique los vectores velocidad, aceleracin, en dicho instante de encuentro.

2 2 = rad / s 6 2 = 4 t rad / s 6 2 2 = + 4 t t rad 12 2


Los mviles se encontrarn cuando

1 = 2

2 t =

+ 4 t

12

t2

12

t 2 2 t

=0

t 2 24t 6 = 0
Resolviendo

t = 0,25 s t = 24,25 s

La solucin es 24,25 s. El punto de encuentro es

1 = 2 (24,25) = 48,5 rad

2 = 0,5 + 4 (24,25)

12

(24,25)2 = 48,5 rad

Los valores son iguales, tal como esperbamos. Solucin. a) Para t = 2 s el mvil 1 como su velocidad angular es 2 rad/s estar en el punto A, y podemos considerar ese instante como tiempo inicial, con lo que: Mvil 1: Como 1 = 2 = 48,5 rad , equivalente a 24 vueltas mas 1/4 de vuelta, el encuentro es en punto B. b) La velocidad lineal, la velocidad angular, las componentes tangencial y normal de la aceleracin de cada uno de los mviles en el instante de encuentro. Mvil 1

1 = 0 1 = 2 rad / s = 2 t rad 1
Mvil 2:

1 = 2 rad / s v = r = 1,5 m/s 1 1 1 = 0 at1 = 1 r = 0 a = 2 r = 3 2 m / s 1 n1


Mvil 2

Movimiento en un plano y en el espacio

Hugo Medina Guzmn

24,25 2 = 4 6 = 0,04 rad / s v = r = 0,03 m/s 2 2 at 2 = 2 r = 0,125 m/s 2 2 2 2 a n 2 = 2 r = 0,0012 m/s


El mvil 2 tiene velocidad negativa, porque a l tiempo t = 24 s su velocidad se hizo cero e inicia el retorno, al tiempo t = 24,25 s se produce el encuentro. c) Esquema especificando los vectores velocidad, aceleracin, en el instante de encuentro. En el instante del encuentro el esquema sera el siguiente:

Como at = a cos m/s y a n = asen m/s ,


2 2

La aceleracin tangencial en cualquier instante, se obtiene a partir del producto escalar del vector aceleracin a y el vector velocidad v .

v a = va cos = vat

at =

v a vx ax + v y a y = 2 2 v vx + vy

MOVIMIENTO CURVILNEO El movimiento curvilneo es aquel en el que pueden combinarse tramos rectos y/o curvos. La extensin de las ecuaciones en el sistema intrnseco es inmediata sufriendo slo una ligera modificacin respecto a la aceleracin. Esta adopta la expresin

La aceleracin normal, se obtiene a partir del mdulo de la aceleracin a y de la aceleracin tangencial a t .


2 2 2 a 2 = ax + ay = an + a t2 2 2 2 an = ax + ay at2

dv v 2 donde es el denominado radio a= t+ n dt

de curvatura y corresponde al radio de una hipottica circunferencia en cada uno de los puntos de la trayectoria. Es evidente que en el caso del movimiento circular ste no vara ya que coincide con el radio de la circunferencia en cada uno de esos

v a +v a x x y y 2 2 2 an = ax + ay 2 2 vx + v y
Finalmente a n =

v2 dv puntos. at = y an = dt

v y ax vx a y
2 2 vx + vy

El radio de curvatura

an =

v2

v2 an

Ejemplo 9. El vector velocidad del movimiento de una partcula viene dado por La figura siguiente muestra la velocidad y la aceleracin con las coordenadas x e y para un determinado instante. 7

+ 6t 2 5 v = (3t 2)i j m/s. Calcular las

Movimiento en un plano y en el espacio componentes tangencial y normal de la aceleracin y el radio de curvatura en el instante t =2 s. Solucin.

Hugo Medina Guzmn b) las componentes normal y tangencial de la velocidad y aceleracin. c) la ecuacin de la trayectoria en coordenadas cartesianas. Solucin.

dv v x = (3t 2) m/s a x = x = 3 m/s 2 dt v y = (6t 2 5) m/s a y =


En el instante t = 2 s
2 v x = 4 m/s a x = 3 m/s 2 v y = 19 m/s a y = 24 m/s

x = 3t , y = 2t 5t 2
a) vx = 3; vy = 2 - 10t; ax = 0; ay = -10;

dv y dt

= 12t m/s 2

= b) t

+ (2 10t ) v 3i j = , 2 v 9 + (2 10t ) = 3 j + (2 10t )i entonces k =t n 2 9 + (2 10t )

2 vt = v t = v = 9 + (2 10t )

v = 4 2 + 19 2 = 19,49 m/s a = 3 2 + 24 2 = 24,19 m/s 2


La aceleracin tangencial es:

vn = 0

= aT = a .t
= aT = a .t
= an = a n
c) y =

10t (2 10t ) 9 + (2 10t ) 10t (2 10t )


9 + (2 10t ) 30
2

at =

v a v x a x + v y a y 4(3) + 19(24 ) = = = 24 m/s 2 2 2 v 19 , 49 vx + v y

La aceleracin normal es: v y a x v x a y 19(3) 4(24) an = = = 2 m/s 2 2 2 19,49 vx + v y El radio de curvatura

9 + (2 10t )

2 5 x x2 3 9

an =

v2

v2 an

Ejemplo 11. Una partcula se mueve en el plano xy de acuerdo con la ley ax = 0, ay = 4cos(2t) m/s2. En el instante t = 0, el mvil se encontraba en x = 0, y = -1 m, y tena la velocidad vx = 2, vy = 0 m/s. a) Hallar las expresiones de r(t) y v(t). b) Dibujar y calcular las componentes tangencial y normal de la aceleracin en el instante t = /6 s. Solucin.

a = 3 2 + 24 2 = 24,19 m/s 2 v = 4 2 + 19 2 = 19,49 m/s

v 2 = 377 , an = 2 m/s 2

a) En t = 0

v 2 377 = = 188,5 m an 2

ax = 0 , vx = 2 a y = 4 cos(2t )

m , x=0 s

Ejemplo 10. Una partcula se mueve de modo que sus coordenadas cartesianas estn dadas como funciones del tiempo

m , v y = 0 , y = 1 m s2 m , s

x = 3t , y = 2t 5t 2
Determine a) las componentes cartesianas de la velocidad y de la aceleracin. 8

En el eje x el movimiento es uniforme v x = 2

x = 2t m

Movimiento en un plano y en el espacio Para encontrar el movimiento en y hay que integrar

Hugo Medina Guzmn

vy

v y = 4 cos(2t )dt v y = 2sen (2t )


t 0
t

m s

Para t = 2 s

v y = 0 v x = 4

dy = 2sen (2t )dt y ( 1) = 1 cos(2t )


0

y = cos(2t ) m
b) Las componentes tangencial y normal de la aceleracin en el instante t = /6 s.

vx = 2 , ax = 0 vy = 3 , ay = 2 at = ax = 2
v2

m s2

an = a y = 10

m s2

an =

v 2 42 = = 1,6m an 10

Ejemplo 13. El vector velocidad del movimiento de una partcula viene dado por

+ 6t 2 5 v = (3t 2)i j m/s.

Si la posicin del mvil en el instante t = 1 s es

m at = 2 cos = 1,31 2 , s v m 2 a n = 2sen = 1,51 2 , tan = x = vy s 3 = 49,1


Ejemplo 12. Un mvil se mueve en el plano xy con las siguientes aceleraciones: ax=2 m/s2, ay =10 m/s2. Si en el instante inicial parte del origen con velocidad inicial vx = 0 y vy =20 m/s. Calcular las componentes tangencial y normal de la aceleracin, y el radio de curvatura en el instante t = 2 s. Solucin.

2 r = 3i j m. Calcular

a) El vector posicin del mvil en cualquier instante. b) El vector aceleracin. c) Las componentes tangencial y normal de la aceleracin en el instante t = 2 s. Dibujar el vector velocidad, el vector aceleracin y las componentes tangencial y normal en dicho instante. Solucin. a) Para el movimiento horizontal

m a y = 10 2 s ax = 2 m s2

v y = 20 + ( 10 )t v x = 2t

vx = 3t - 2
Como v x =

ax =

dv x m =3 2 dt s

dx dx = v x dt , integrando dt

Movimiento en un plano y en el espacio

Hugo Medina Guzmn Considere un objeto que se desplaza en el aire sin ninguna fuerza con excepcin de la gravedad y de la resistencia del aire. La fuerza de la gravedad produce una aceleracin constante hacia abajo de magnitud 9,80 m/s2. Como primera aproximacin, no tomemos los efectos del aire y de variaciones en g . Asumiremos que la tierra es plana para el rango horizontal de los proyectiles. A pesar de estas simplificaciones, podemos an obtener una descripcin bastante buena del movimiento del proyectil. El recorrido de un proyectil se llama su trayectoria. Si se desprecia la resistencia del aire, no hay entonces aceleracin en la direccin horizontal, y a x = 0 . La aceleracin en la direccin de y es

dx = (3t 2 )dt
t 1

7 3 x = t 2 2t + m 2 2
Para el movimiento vertical

v y = 6t 2 - 5
Como v y =

ay =

dv y dt

= 12t

m s2

dy dy = v y dt , integrando dt
2

dy = 6t 5 dt y = 2t 5t + 1 m
3 1

debido a la gravedad. Es constante y dirigida hacia abajo, as que a y = g . Es conveniente elegir

x0 = 0 y y 0 = 0 (es decir, poner el origen en el


punto donde el proyectil comienza su movimiento). Adems, nos referimos tpicamente a v 0 como la rapidez inicial del proyectil. Si el proyectil es lanzado con un ngulo sobre la horizontal, la velocidad inicial en la direccin x y la velocidad inicial en la direccin y se pueden expresar en trminos de g y de y usando la trigonometra.

7 3 r = t 2 2t + i - 2t 3 5t + 1 j 2 2

+ 12t b) a = 3i j
c) Para t = 2 s vx = 4 m/s, vy = 19 m/s ax = 3 m/s2, ay = 24 m/s2
2 2 a = ax + ay = 24,2m / s 2

tan =

vy vx ay ax

19 = 4,75 = 78 o 4 24 = 3 = 83o 3

v0 x = v 0 cos , v0 y = v0 sen ax = 0 , a y = g
Con esto:

tan =

v x = v0 cos = constante , v y = v0 sen gt x = (v0 cos )t , y = (v0 sen )t


1 2 gt 2

Ecuacin de la trayectoria. De la ecuacin para x obtenemos t = Sustituyendo en la ecuacin para y

x . v0 cos

at = a cos( ) = 24,1m / s 2 an = a(sen ) = 2m


MOVIMIENTO PARABLICO. 10

g y = (tan )x 2v 2 cos 2 0

2 x

Corresponde a la ecuacin de una parbola que pasa por el origen. Una caracterstica dominante del movimiento del proyectil es que el movimiento horizontal es independiente del movimiento vertical. As un proyectil se mueve a una velocidad constante

Movimiento en un plano y en el espacio en la direccin horizontal, independiente de su movimiento vertical. Esto se ilustra en la figura.
2 v0 sen (2 ) g

Hugo Medina Guzmn

La altura mxima que alcanza un proyectil se obtiene con v y = 0 .

v y = v0 sen gt = 0 , despejando t.
Podemos entender mejor el significado de la

1 2 ecuacin y = (v 0 sen )t gt viendo el 2


movimiento del proyectil de esta manera: Primero, si no hubiera fuerza de la gravedad y aceleracin hacia abajo, en el tiempo t el proyectil movera una distancia v 0 t en una lnea inclinada recta. Si ahora imaginamos con la gravedad el efecto sera hacer que el proyectil se aleje de la trayectoria recta por una distancia gt2. De la superposicin de estos dos efectos resulta la trayectoria parablica como se muestra en la figura.

t=

v0sen , como vemos es igual a la mitad del g


1 2 gt 2
1 v0sen 2 g g
2

tiempo de vuelo.

y mx = (v0 sen )t

v0sen = (v0sen ) g
Finalmente:

y mx =

2 v0 sen 2 2g

Tiempo de vuelo. Poniendo y = 0

Su valor mximo se obtiene para el ngulo de disparo = 90. Ejemplo 14. UN BLANCO EN CADA LIBRE (Tiro al mono) Se deja caer una botella desde el reposo en el instante en que una piedra es lanzada desde el origen. Determinar los valores del ngulo y de la velocidad de disparo para que la piedra rompa la botella. (Tmese g = 9,8 m/s2)

1 y = (v 0 sen )t gt 2 = 0 , despejando t, 2

t2

2v0sen t=0 g

Resolviendo obtenemos dos soluciones t = 0, que corresponde al disparo del proyectil y

t=

2v0sen g

El valor mximo de t se obtiene para = 90. Cuando el proyectil se lanza verticalmente hacia arriba, describiendo una trayectoria rectilnea a lo largo del eje y. El alcance horizontal de cada uno de los proyectiles se obtiene para y = 0.

2v0sen xmx = (v0 cos )t = (v0 cos ) g


11

Solucin. Movimiento de la piedra: El movimiento curvilneo de la piedra se realiza bajo la aceleracin constante de la gravedad, es decir, es la composicin de dos movimientos - Uniforme a lo largo del eje horizontal
3H

Movimiento en un plano y en el espacio

Hugo Medina Guzmn De estas ecuaciones, obtenemos:

a px = 0 Horizontal v px = v0 cos x p = v 0 cos t


- Uniformemente acelerado a lo largo del eje vertical.

g 2x 2y y = x Ecuacin de una = ax ax g
lnea recta. b) En tierra, y = 124 , tal que

a px = g Vertical v px = v 0 sen gt 2 y p = v 0 sen t gt / 2


Movimiento de la botella: La botella se mueve verticalmente bajo la aceleracin constante de la gravedad.
4H

2( 124) t = 5,03 s 9,8 c) v y = v0 y gt = 0 (9,8)(5,03) t2 = m s vx = v0 x + axt = 0 + (1,10)(5,03) m = 5,53 s


= 49,3
2 2 v = vx + vy =

abx = g

vbx = gt y b = H gt 2 / 2
Choque de la piedra con la botella: Cuando se produce el choque, la posicin de la piedra y de la botella coincide.

(5,53)2 + ( 49,3)2

= 49,6

A = v0 cos t

m s

H gt 2 / 2 = v0 sen t gt 2 / 2 H = v0 sen t
Dividimos la segunda ecuacin entre la primera.

tan =

H A

Para romper la botella debemos de apuntarla directamente y en el instante en el que se deja caer, se debe lanzar la piedra. La velocidad debe tener un valor mnimo para hacer el recorrido A, mientras la botella est en el aire. Esto sucede para el tiempo t =

Ejemplo 16. Disparamos un proyectil desde el origen y ste describe una trayectoria parablica como la de la figura. Despreciamos la resistencia del aire. Dibuja en las posiciones A, B, C, D y E el vector velocidad, el vector aceleracin y las componentes normal y tangencial de la aceleracin. (No se trata de dar el valor numrico de ninguna de las variables, slo la direccin y el sentido de las mismas) Qu efecto producen an y at sobre la velocidad?

2H , y el g
g 2H

recorrido horizontal de la piedra debe cumplir:

2H v0 cos g

A v0 A cos

Solucin.

Ejemplo 15. Una bolsa de arena cae del reposo de un globo de aire caliente desde una altura de 124 m est soplando un viento horizontal, y el viento da a bolsa de arena una aceleracin horizontal constante de 1,10 m/s2. a) Demuestre que la trayectoria de la bolsa de arena es una lnea recta. b) Cuanto tiempo toma para llegar la tierra? c) Con qu velocidad llega a la tierra? Solucin. a) x =

v es tangente a la trayectoria

1 2x axt 2 t 2 = 2 ax 1 2y y = gt 2 t 2 = 2 g
12

Cuando sube

Movimiento en un plano y en el espacio

Hugo Medina Guzmn y = 150 + 180(sen /6)t - 5t2 a) Punto de cada 150 + 180(sen /6)t - 5t2 = 0, t = 19,5 s x = 180(cos /6)(19,5) = 3039,8m b) Tiempo para la altura mxima 180(sen /6) - 10t = 0, t = 9,0 s entonces ymax = 150 + 180(sen /6)(9) - 5(9)2 = 555,0m El vector unitario tangente es

at y v tienen sentidos opuestos.


Cuando baja

at y v tienen el mismo sentido at modifica el mdulo de la velocidad con el


tiempo.

a n modifica la direccin de v

v cos + t = = i jsen 6 6 v

Ejemplo 17. Una bala del rifle se dispara con una velocidad de 280 m/s hacia arriba de una superficie plana inclinada 30 sobre la horizontal. La bala se dispara con un ngulo de elevacin inicial de 45 sobre la horizontal (es decir, 15 sobre la superficie plana). Cul es el alcance de la bala sobre el plano? Solucin. La ecuacin del plano inclinado es

a = 10 j

Entonces

at = a t = 10sen = 5 m/s 2 6
2 an = a 2 an = 100 25 = 8,66 m/s 2

y x = tan 30 y = x 3
La ecuacin de la trayectoria parablica.

g y = (tan )x 2 2v cos 2 0

2 x

La interseccin de la parbola y la lnea recta ocurre cuando

Ejemplo 19. Un can de artillera lanza proyectiles con una rapidez de 300 (m/s). El artillero debe darle a un blanco que se encuentra a 8640 (m) detrs de un cerro, cuya altura es de 1000 (m) ubicado a 1200 (m) del can. Demuestre que es posible darle al blanco y determine el ngulo de elevacin para cumplir el objetivo. Solucin. Supondremos que damos en el blanco entonces

x g = (tan )x 2 x2 2 2v0 cos 3 v2 1 Para = 45 : x = 0 1 g 3


Para un tringulo 30, 60, 90 vemos que

5(8649) 8649 tan =0 (300)2 cos 2


2

gx 2 =0 y = x tan 2 2v0 cos 2

x = S cos 30 =
De aqu S =

2 3

3 S. 2 v2 v2 3 1 0 = 0,49 0 , arriba del g g

plano. Con y 0 = 280 m/s, S = 3,90 km. Ejemplo 18. Se dispara un proyectil desde la cima de una colina de 150 (m) de altura con una rapidez de 180 (m/s) y formando un ngulo de 30 con la horizontal. Calcule: (a) La distancia horizontal entre el punto de lanzamiento y el punto de cada del proyectil. (b) La altura mxima del proyectil con respecto al suelo. (c) Las componentes normal y tangencial de la aceleracin al salir en el punto de disparo. Solucin: x = 180(cos /6)t 13

Tiene dos races reales 1 = 53,03 2 = 36,97 Debemos verificar que el disparo pasa sobre el cerro, para ello evaluamos en ambos ngulos y(1200) y1 (1200) = 1373,0 m y2 (1200) = 777,95 m La altura del cerro es excedida en el primer caso. Ejemplo 20. Se dispara un proyectil de modo que su alcance horizontal es igual al triple de la altura mxima. Encuentre el ngulo de lanzamiento. Solucin. Sabemos que

xmax ymax

2 v0 sen 2 = g 2 v0 sen 2 = 2g

Entonces

Movimiento en un plano y en el espacio


2 v0 v 2sen 2 sen 2 =3 0 g 2g 2 cos = 3sen 2 tan = =33,69 3

Hugo Medina Guzmn

Ejemplo 21. Un lanza granadas tiene un alcance mximo de 300 m. Para dar en un blanco que se encuentra a una distancia de 400 m del lanza granadas. Determine: a) La altura mnima que debe subirse el lanza granadas. b) La rapidez de lanzamiento. c) El ngulo de lanzamiento, Solucin. La ecuacin de la parbola de seguridad es

Solucin. a) y b)

y = h+

2 v0 gx 2 2 2 g 2v 0

Sabemos tambin que para h = 0 la distancia mxima alcanzable es

x (0 ) =

2 v0 = 300 g

ax = 0

v x = 45 cos

y para una altura h la distancia horizontal mxima ser

x(h ) =

(v

2 0

+ 2hg

) vg = 400m
0

de la primera b)

v0 = 3000 = 54,77
y de

m s 10
= 400

(54,77 )2 + 2h(10) 54,77

a) h = 116,701m c) El ngulo de lanzamiento cuando el blanco est sobre el lmite de la parbola de seguridad es

a y = 10 v y = 45sen 10t x = 45 cos .t 1 y = 45sen .t 10t 2 2 Punto de impacto x = 45 , y = 45 3 45 = 45 cos .t 2 45 3 = 45sen .t 5t 1 1 45 3 = 45sen . 5 cos cos 2 tan 2 9 tan + 1 9 3 = 0 1 = 84,5 o t1 = 10,45s

tan =

2 v0 entonces = 36,87o gx

2 = 54,5 o t 2 = 1,72 s
c) Para t =

Ejemplo 22. Un patinador desciende por una pista helada, alcanzando al finalizar la pista una velocidad de 45 m/s. En una competicin de salto, debera alcanzar 90 m a lo largo de una pista inclinada 60 respecto de la horizontal. a) Cul ser el ngulo (o los ngulos) que debe formar su vector velocidad inicial con la horizontal?. b) Cunto tiempo tarda en aterrizar? c) Calcular y dibujar las componentes tangencial y normal de la aceleracin en el instante t/2. Siendo t el tiempo de vuelo. Tomar g =10 m/s2

ax = 0 t1 v x = 4,31 2 v y = 7,46 a y = 10 = 30 o

tan =

vx vy

at = g cos 30 o = 5 3 m s 2 a n = gsen30 o = 5 m s 2

14

Movimiento en un plano y en el espacio Punto de encuentro

Hugo Medina Guzmn

20015 cos .t 1 1 30 9,8t 2 = 15sen t 9,8t 2 2 2 o 30 = 56,3 tan = 20 y = 1,69m

b) Ejemplo 23. Se deja caer una botella desde el reposo en la posicin x =20 m e y =30 m. Al mismo tiempo se lanza desde el origen una piedra con una velocidad de 15 m/s. a) Determinar el ngulo con el que tenemos que lanzar la piedra para que rompa la botella, calcular la altura a la que ha ocurrido el choque. b) Dibujar en la misma grfica la trayectoria de la piedra y de la botella. (Tomar g = 9,8 m/s2). Solucin: a) Ejemplo 24. Desde un can que est sobre un plano inclinado un ngulo con la horizontal se formando un ngulo con el plano horizontal. Encontrar. a) El punto ms alto al que llega el proyectil. b) El alcance del proyectil. Solucin. dispara un proyectil. Este sale con una velocidad v 0

Movimiento de la botella

a) v 0 x = v 0 cos v 0 y = v 0 sen

ax = 0

1 a y = 9,8 v y = 9,8t y = 30 9,8t 2 2


Movimiento de la piedra

vx = 0

x = 20

v y = v0 sen gt
La altura mxima se produce cuando v y = 0
2 v0 sen 2 = 2g

y mx

Con ese valor,


2 v 0 sen v 0 x = v0 cos = g 2 g sen 2 2 v y = x tan = 0 sen 2 tan 2g v2 h = y mx y = 0 (sen 2 sen 2 tan ) 2g b) El alcance mximo S . 2 x = v0 cos t y = v 0 sen t 1 2 gt Ecuacin del plano en funcin de t y = x tan Dividiendo y x :

ax = 0

vx = 15 cos

a y = 9,8 v y = 15sen 9,8t x = 15 cos t 1 y = 15 sen t 9,8t 2 2

15

Movimiento en un plano y en el espacio


2 y v0 sen t 1 2 gt = = tan x v 0 cos t 2 v0 sen t 1 2 gt = tan v 0 cos t

Hugo Medina Guzmn c) El punto de cada ocurre para z = 0 y la distancia vale x( ) =

Resolviendo encontramos el tiempo para el que el proyectil toca tierra:

1 + cos 2 tan + sen 2 dx( ) La distancia mxima ocurre para = 0. d


2 v0 gsen

t=

2v0 (sen cos tan ) g


Ejemplo 26. Se lanza una pelota verticalmente hacia arriba con una velocidad de 20 m/s desde la azotea de un edificio de 50 m de altura. La pelota adems es empujada por el viento, produciendo un movimiento horizontal con aceleracin de 2 m/s2. Calcular: a) La distancia horizontal entre el punto de lanzamiento y de impacto. b) La altura mxima c) El valor de las componentes tangencial y normal de la aceleracin cuando la pelota se encuentra a 60 m de altura sobre el suelo. Tmese g =10 m/s2.

El valor de x cuando el proyectil toca tierra es:


2 2v0 x = v0 cos t = cos (sen cos tan ) g

Y el alcance S es:

S=

2 2v 0 x cos (sen cos tan ) = cos g cos

forma un ngulo con el eje z (perpendicular al plano). El eje x se toma tangente al plano apuntando hacia abajo. a) Tome el sistema de referencia indicado en la figura y halle las componentes de los vectores aceleracin, velocidad y posicin del proyectil en funcin del tiempo. b) Halle la mxima separacin entre el proyectil y la colina. c) Halle la distancia entre el origen y el punto de cada del proyectil sobre la colina. Demuestre que esa distancia es mxima si = / 2 .

Ejemplo 25. La figura muestra una colina inclinada un ngulo respecto a la vertical y la trayectoria de un proyectil. El proyectil se lanza desde el origen O con una velocidad inicial de mdulo v 0 y que

Solucin.

a x = 2 , v x = 2t , x = a y = 10 ,
Solucin. a) a x = g cos , v x = g cos t + v 0 sen ,

1 2 2t 2

v y = 20 + ( 10 )t ,

y = 20t +

1 g cos t 2 + v0 sen t 2 a z = gsen , v z = gsen t + v0 cos , 1 z = gsen t 2 + v 0 cos t 2 b) La mxima separacin ocurre para v z = 0 y vale x=
z=
2 v0 cos 2 2 g 2sen

1 ( 10)t 2 2

a) Punto de impacto y = -50 t = 5,74 s x = 32,97 m b) altura mxima

v y = 0 t = 2 s y = 20 m
hmxima = 70 m sobre el suelo. 16

Movimiento en un plano y en el espacio c) h = 60 y = 10 m t1 = 0,59 s t2 = 3,41 s

Hugo Medina Guzmn

= 2 1 = 14,5o 2 a n = a. cos = 9,87 m, /s


2 at = a.sen = 2,55 m/s

v x = 1,17 a x = 2 t1 = 0,59 s v y = 14,14 a y = 10 a = 2 2 + 10 2

Ejemplo 27. Nos encontramos en la antigua Suiza, donde Guillermo Tell va a intentar ensartar con una flecha una manzana dispuesta en la cabeza de su hijo a cierta distancia d del punto de disparo (la manzana est 5 m por debajo del punto de lanzamiento de la flecha). La flecha sale con una velocidad inicial de 50 m/s haciendo una inclinacin de 30 con la horizontal y el viento produce una aceleracin horizontal opuesta a su velocidad de 2 m/s2. a) Calcular la distancia horizontal d a la que deber estar el hijo para que pueda ensartar la manzana. b) Hllese la altura mxima que alcanza la flecha medida desde el punto de lanzamiento. (g = 9,8 m/s2)

tan 1 = tan 2 =

vx 1,17 = = 0,08 1 = 4,7 o v y 14,14 ay ax = 10 = 5 2 = 78,7 o 2

= 2 1 = 73o 2 a n = a. cos = 2,81 m/s


2 at = a.sen = 9,80 m/s
Solucin.

v x = 6,83 m/s a x = 2 t 2 = 3,41s v y = 14,14 m/s a y = 10 a = 2 2 + 10 2

tan 1 = tan 2 =

vy vx ay ax

ax = 2 , v x = 50 cos 30 o 2t , 1 x = 50 cos 30 o 2t 2 2 a y = 9,8 , v y = 50sen30 o 9,8t , 1 y = 50sen30o 9,8t 2 2


Punto de impacto x = d, y = -5 -5 =25 t -4,9 t2 t = 5,29 s x = 201,23 m 17

= =

14,14 = 2,07 1 = 64,2 o 6,83 10 = 5 2 = 78,7 o 2

Movimiento en un plano y en el espacio Mxima altura vy = 0 50sen30 - 9,8t = 0 t = 2,55 s y = 31,89 m Ejemplo 28. Un paraguas abierto mojado se sostiene hacia arriba como se muestra en la figura y se gira sobre la manija a razn uniforme de 21 revoluciones en 44 s. Si el borde del paraguas es un crculo 1 m de dimetro, y la altura del borde sobre el piso es 1,5 m, hallar donde las gotas del agua al hacer girar del borde tocan el piso.

Hugo Medina Guzmn

Sea un objeto P determinado por un observador en el

y por un + y j + zk sistema S por r = xi observador en el sistema S' por como se muestra en la figura. + y' r ' = x' i j + z' k

Solucin. La velocidad angular del paraguas es

21 2 rad = 3rad / s 44s

La velocidad tangencial de las gotas de agua que salen del borde del paraguas es

v0 = r = (0,5)(3) = 1,5m / s

Las ecuaciones de transformacin de Galileo que relacionan las observaciones desde los sistemas S y S' son x = x'+ Vt , y = y ' , z = z ' t = t ' Aqu se supone que puede establecerse una escala de tiempo absoluta aplicable a ambos marcos de referencia de manera que t = t ' . Esto sucedera si la velocidad de la luz fuera infinita (Debemos reconocer que las escalas de tiempo asociadas a dos marcos de referencia no son los mismos si existe movimiento relativo entre ellos es uno de los principios fundamentales de la teora especial de la relatividad propuesta por Einstein en 1905). Vectorialmente podemos representar la transformacin de Galileo como

Para calcular el tiempo en que la gota llega al piso usamos h =

t=

1 2 gt 2 2h 2(1,5) = = 0,553m 9,8 g

r = r '+ V t . dx dx' = + V v x = v' x ' + V dt dt dy dy ' v y = v' y ' = dt dt dz dz ' = v z = v' z ' dt dt

Derivando las relaciones anteriores podemos obtener la relacin de la velocidad.

El alcance horizontal de la gota es x = v0t = (1,5)(0,55) = 0,83 m; y el locus de las gotas es un crculo de radio

R=

(0,5)2 + (0,83)2

= 0,97 m.

VELOCIDAD Y ACELERACIN RELATIVAS. Movimiento Relativo de Traslacin Uniforme. La Relatividad de Galileo Consideramos dos sistemas de referencia S y S', S' tiene un movimiento de traslacin rectilneo uniforme con respecto a S; S' se aleja de S con una

Vectorialmente v = v '+ V Derivando nuevamente obtenemos la relacin de la aceleracin

velocidad V = vi
18

dv x dv' x ' dV dV = + a x = a' x ' + dt dt dt dt dv y dv' y ' = a y = a' y ' dt dt

Movimiento en un plano y en el espacio

Hugo Medina Guzmn

dv z dv' z ' a z = a' z ' = dt dt


Si la velocidad V del sistema S' es constante,
dV = 0 y a = a' dt

Estas relaciones encontradas son de aplicacin general si S y S' estn animadas por un movimiento relativo cualquiera, como se muestra en la figura siguiente

b) Si se observa el mismo proyectil desde un sistema de referencia situado en el suelo S con un origen en el lugar de lanzamiento (para t 0 = 0 , O = O'), entonces las posiciones, las velocidades y las aceleraciones respecto de O estarn dadas por la transformacin de Galileo. En este caso la velocidad inicial v 0 vista desde el suelo ser

r 2 + v y0 v 0 = Vi j v0 = V 2 + v y 0
Las ecuaciones son:

r = r '+ V t , r ' = r V t

0 = tan 1

v y0 V

v = v'+ V

La trayectoria ser una parbola tal como se ve en la figura siguiente

a = a'
r

Ejemplo 29. Desde la plataforma de un camin en movimiento horizontal V constante se lanza un proyectil directamente hacia arriba con una r velocidad v 0 . Cmo ser visto el movimiento del proyectil por: a) un observador situado en el camin (sistema S')? b) un observador situado en el suelo (sistema S)? Solucin. a) El tiempo se mide desde el momento del lanzamiento t 0 = 0 , cuando el proyectil se eleva con velocidad v 0 . La componente horizontal de la velocidad coincide con la velocidad V del camin. El observador O' en el camin ver nicamente la componente vertical v ' y '0 , la componente horizontal ser v ' x '0 = 0 . Para un instante t cualquiera La componente horizontal del movimiento del proyectil es igual al movimiento del can, de modo que cuando cae el proyectil coincidir con el can. Ejemplo 30. El observador O suelta una piedra del trigsimo piso de un rascacielos. El observador O, descendiendo en un ascensor a velocidad constante de V = 5,0 m/s, pasa el trigsimo piso justo cuando se suelta la piedra. Al tiempo t = 3,0 s despus de que se suelta la piedra, hallar: a) La posicin, la velocidad, y la aceleracin de la piedra relativa a O. b) La posicin, la velocidad, y la aceleracin de la piedra relativa a O. Solucin. a) Para O, la posicin de la piedra est dada por:

x' = 0

y ' = v' y '0 t gt 2

v' x ' = 0 v' y ' = v' y '0 gt a' x ' = 0 a' y = g

x = x0 + v0 t +

1 2 at 2

Donde x = 0 en el trigsimo piso con la direccin hacia abajo como la direccin positiva de x. As, en t = 3,0 s,

x = 0+0+

1 (9,8)(3,0)2 = + 44 m/s 2

Tambin, v = v0 + at da v = 0 + 9,8 m/s2 x 3,0 s = +29 m/s.

19

Movimiento en un plano y en el espacio La aceleracin de un cuerpo en cada libre, segn el observador O que est inmvil con respecto a la tierra, se sabe que la aceleracin gravitacional es constante. (De hecho, esto es la base de la validez de los dos clculos anteriores.) As tenemos: a = +g = +9,8 m/s2. b) O mide la posicin x', relativa a x por medio de la ecuacin x' = x - Vt. Luego, despus de 3,0 s, x' = 44 m 5,0 m/s x 3,0 s = +29 m. Es decir, la piedra se localiza 29 m debajo del observador O despus de 3,0 s. La velocidad de la piedra relativa a O' es v' = v -V; de aqu, en t =3,0s, v' = 29 m/s 5,0 m/s = +24 m/s Puesto que V es constante, a' = a, y a'= +g = +9,8 m/s2. El observador O ve la piedra con la misma aceleracin vista por O. (en general, las aceleraciones son iguales en todos los sistemas inerciales.) Ejemplo 31. Un automovilista viaja hacia el oeste sobre la Ruta Interestatal 80 a 80 km/h y es seguido por un auto patrulla que viaja a 95 km/h. a) Cul es la velocidad del automovilista respecto al auto patrulla? b) Cul es la velocidad del auto patrulla respecto al automovilista? Solucin. Si el Oeste indica el sentido positivo entonces a) 80 - 95 = -15 km/h b) 95 - 80 = 15 km/h Ejemplo 32. Un ro tiene una rapidez uniforme de 0,5 m/s. Un estudiante nada corriente arriba una distancia de 1 km y regresa al punto de partida. Si el estudiante puede nadar con una rapidez de 1,2 m/s en agua tranquila, cunto dura el recorrido? Compare este resultado con el tiempo que durara el recorrido si el agua estuviera tranquila. Solucin. La rapidez absoluta (respecto a la ribera) cuando nada corriente arriba es 1,2 0,5 = 0,7 y cuando nada corriente abajo es 1,2 + 0,5 = 1,7 entonces el tiempo de ida y vuelta ser

Hugo Medina Guzmn Sea W la rapidez del ro y u la rapidez de los botes respecto al agua, (igual en ambos), entonces V1 = u -W V2 = u + W de modo que

W =

V2 V1 2

Ejemplo 34. Un bote cruza un ro que mide de ancho a en el cual la corriente fluye con una rapidez uniforme de u. El botero mantiene una orientacin (es decir, la direccin en la cual apunta el bote) perpendicular al ro y al motor fijo para dar una rapidez constante de v m/s con respecto al agua. De acuerdo a los datos (a) Cul es la velocidad del bote respecto a un observador detenido en la orilla? (b) Hasta dnde estar el bote, medido corriente abajo paralelamente al ro, desde la posicin inicial hasta cuando alcance la orilla opuesta? Solucin. a)

+ v V = ui j
b) La componente de la velocidad absoluta perpendicular al ro determine el tiempo de cruce de acuerdo a t =

a v

Por lo tanto el bote avanza paralelamente al ro una distancia

d = ut =

u a v

Ejemplo 35. Un comprador que est en una tienda puede caminar sobre una escalera mecnica en 30 s cuando est detenida. Cuando la escalera mecnica, funciona normalmente, puede llevar al comprador sin caminar al siguiente piso en 20 s. Cunto tiempo le tomara al comprador al subir caminando con la escalera mecnica en movimiento? Suponga que el comprador hace el mismo esfuerzo al caminar sobre la escalera mecnica en movimiento o cuando est parada. Solucin. Sea L el largo de la escalera. Entonces la velocidad de la persona respecto a la escalera es

1000 1000 t= + = 2016,81 s = 0,56 h 0,7 1,7


Ejemplo 33. Dos remeros en idnticas canoas ejercen el mismo esfuerzo remando en un ro, uno corriente arriba (y se mueve corriente arriba), mientras que el otro rema directamente corriente abajo. Un observador en reposo sobre la orilla del ro determina sus rapideces que resultan ser de V1 y V2 respectivamente. Determine en trminos de los datos la rapidez de las aguas del ro. Solucin. 20

v' =

L . 30 L 20 L L L + = 20 30 t

Sea ve la velocidad de la escalera. Ella corresponde a la de la persona cuando no camina, es decir

ve =

Si la escalera funciona y la persona camina, entonces

v = ve + v' =

de donde el tiempo ser t = 12 s

Movimiento en un plano y en el espacio

Hugo Medina Guzmn

Ejemplo 36. El piloto de un avin observa que la brjula indica que va dirigindose hacia el oeste. La rapidez del avin respecto al aire es de 150 km/h. Si existiera un viento de 30 km/h hacia el norte, calcule la velocidad del avin respecto a la Tierra.

a) La direccin en la que debe dirigirse el avin est dada por el ngulo . Solucin. La velocidad del viento es vv = 30 km/h y la rapidez del avin respecto al aire es v = 150 km/h. Pero

cos =

50 vv = = 0,25 = 75,5 v 200

+ v' v = v j = 30i

Debe dirigirse 75,5 direccin N-O. b) Su velocidad respecto a la Tierra es:

v = v' 50i

De donde v' = v j 30i y si tomamos magnitudes 150 = v 2 + 302


v = 146,969 km/h Ejemplo 37. El piloto de un avin desea volar hacia el oeste en presencia de un viento que sopla hacia el sur a 50 km/h. Si la rapidez del avin cuando no sopla el viento es de 200 km/h, a) en qu direccin debe dirigirse el avin? b) cul debe ser su rapidez respecto a la Tierra?

Y su rapidez respecto a tierra es:

v = v'2 502 = 200 2 502


= 193,6 km/h Ejemplo 38. Un nio en peligro de ahogarse en un ro est siendo llevado corriente abajo por una corriente que fluye uniformemente con una rapidez de 2,5 km/h. El nio est a 0,6 km de la orilla y a 0,8 km corriente arriba de un embarcadero cuando un bote de rescate se pone en camino. a) si el bote procede a su rapidez mxima de 20 km/h con respecto al agua, cul es la direccin, relativa a la orilla, que deber tomar el conductor del bote? b) Cul es el ngulo que hace la velocidad, v, del bote con respecto a la orilla? c) Cunto tiempo le tomar al bote para alcanzar al nio? Solucin. a) Considerando al bote y al nio dentro del ro se encuentran en un sistema inercial S. En este sistema el nio esta en reposo y el bote se mueve con su velocidad, para poder alcanzar en el menor tiempo el bote de enfilar con un ngulo relativo a la orilla dado por:

Solucin. La velocidad del viento es vv = 50 km/h hacia el sur y la rapidez del avin respecto al aire es v = 200 km/h. Para poder volar directamente hacia el oeste con respecto a tierra debe compensar el arrastre producido por el viento, tal como se muestra en la figura siguiente.

tan =

0,6 = 1,5 = 37 0,8

b) La velocidad del bote v, con respecto a la orilla vx = 20 cos 37 +2,5 = 13,5 (1)

21

Movimiento en un plano y en el espacio

Hugo Medina Guzmn

v y = 20sen37 = 12
Dividiendo (2) : (1)

(2)

h=

vx 12 = tan = = 0,89 vy 13,5 = 41

V 2 cot 2 2g

Ejemplo 41. La brjula de un avin indica que se est dirigiendo hacia el este con una velocidad de 400 km/h. La informacin de tierra indica que el viento sopla hacia el norte con una velocidad de 300 km/h. cul es la velocidad del avin con respecto a tierra? Solucin. En este caso tenemos dos sistemas, el sistema tierra (S) y el sistema aire (S') que se mueve con una velocidad de 300 km/h respecto a tierra.

c) El tiempo que le tomar al bote para alcanzar al nio:

d = vt t =

d v

Siendo v = 20 km/h y

d = 0,82 + 0,62 = 1,0 km 1 t= = 0,05 h = 3 min 20


Ejemplo 39. Desde el techo del carro de un tren que est acelerando hacia el norte a una razn de 2,5 m/s2 se suelta y cae un perno. Cul es la aceleracin del perno con respecto a: a) el carro del tren? b) la estacin? Solucin: Si y es la vertical hacia arriba y x es la direccin de la aceleracin del tren, entonces a)

V = 300 j
v' = 400i

R =V t r ' = v' t
La posicin del avin visto desde O es

r = R+ r ' = V t + r '

9,8 a ' = 2,5i j.


b)

La velocidad es

a = 9,8 j

dr = V + v' v= dt

Ejemplo 40. Un estudiante de la Facultad de Ingeniera pasea sobre el vagn de un tren que viaja a lo largo de una va horizontal recta a una rapidez constante de V m/s. El estudiante lanza una pelota al aire a lo largo de una trayectoria que inicialmente forma un ngulo de con la horizontal y est en lnea con la va. El profesor del estudiante, que est parado cerca sobre la tierra, observa que la pelota sale verticalmente. Qu altura subir la pelota? Solucin. Si V es la rapidez inicial de lanzamiento relativa al tren, entonces en la direccin x tenemos: Vx = V cos V = 0 Porque el profesor observa que sale verticalmente.

j + 400i Luego v = 300 Su magnitud

v = 300 2 + 400 2 = 500


= tan -1 300 = 37 o 400

km h

El avin se dirige hacia el NE formando un ngulo de 37 con la direccin este, el mdulo de la velocidad es 500 km/h. Ejemplo 42. Un nadador recorre una piscina de 100 m en 2 min. Va a nadar en un ro observando antes de lanzarse e al agua, que un trozo de madera que flota en ella recorre 20 m en 1 minuto. Calcular el tiempo que tardar el nadador en recorrer 100 m en el ro, segn vaya a favor o en contra de la corriente. Solucin.

V '=

V cos

Luego Vy = Vy = Vsin= V cot Subir una altura h dada por 22

Movimiento en un plano y en el espacio La velocidad del nadador es:

Hugo Medina Guzmn

vn =

m s 100 = = 50 2 min t m min


Si en la figura y es el ancho del ro y x el avance producido por la corriente, el camino recorrido por la lancha es s.

La velocidad del agua del ro es: vr = 20

La velocidad nadando a favor de la corriente es: v1 = v n + v r = 50 + 20 = 70 m/min Y el que tarda en recorrer 100 m es:

t1 =

s 100 = = 1 min 26 s 70 v1

s = x 2 + y 2 = 60 2 + 50 2
= 78,1 m Ejemplo 45. La velocidad que provocan unos remeros a una barca es de 8 km/h, La velocidad del agua de un ro es 6 km/h, y el ancho de tal ro 100 m. a) Suponiendo la posicin de la proa perpendicular a las orillas, calcular el tiempo que tarda la barca en cruzar el ro y la distancia a que es arrastrada, aguas abajo, por la corriente.

La velocidad nadando en contra de la corriente es: v 2 = v n v r = 50 - 20 = 30 m/min Y el que tarda en recorrer 100 m es:

t2 =

s 100 = = 3 min 20 s 30 v2

Ejemplo 43. Un acorazado navega con rumbo NE a una velocidad de 50,56 km/h. Suena zafarrancho de combate y uno de los tripulantes marcha corriendo de babor a estribor para ocupar su puesto, a una velocidad de 10 km /h. Calcular el valor de la velocidad resultante y su direccin. Solucin.

b) En qu direccin debe colocarse la proa de la barca para alcanzar el punto de la orilla opuesta situado enfrente del de partida? (punto de partida y llegada en la perpendicular comn a las orillas),

km km , vT = 10 h h 2 2 V = 55,56 + 10 = 56,45 km/h 55,56 tan = = 79,8 10 = 79,8 45 = 34,8 = 34 47 49

v A = 55,56

La direccin ser 90 -

= 55 12 11

c) Qu velocidad, respecto a la tierra, lleva la barca en los dos casos estudiados? d) Cunto tarda en atravesar el ro?. Solucin. a) vx = vr = 6 km/h, vy = vb = 8 km/h

Ejemplo 44. Una pequea lancha atraviesa un ro de 50 m de. Anchura, al mismo tiempo la corriente lo arrastra 60 m aguas abajo. Qu camino ha recorrido? Solucin.

y = vyt t = x = vxt = 6

y 0,1 = h = 45 s vy 8

La distancia a que es arrastrada por la corriente:

0,1 Km = 75 m 8 6 8

b) Para que la barca vaya en la direccin de v2 la componente horizontal de vb ha de ser igual a 6 km/h.

vb sen = vr sen = = 48o 35


c) En el primer caso 23

Movimiento en un plano y en el espacio

Hugo Medina Guzmn b y c) La velocidad del bote con respecto a la orilla es v neta = v B + v R . Como v B y v R son perpendiculares, tenemos
2 2 v neta = v B + vR

2 2 v1 = v x + vy = 62 + 82

= 10 km/h En el segundo caso: v2 = vbcos = 8cos 48 35 = 5,3 km/h d) En el primer caso son 45 s ya calculados. En el segundo caso:

t=

y 0,1 = h = 68 s v2 5,3

= 3 + 4 = 5m / s . El ngulo mostrado en la figura se determina por


2 2

tan =

Ejemplo 46. Una canoa de 2,5 m de larga est junto a la orilla de un ro y perpendicularmente a ella. Se pone en marcha con una velocidad de 5 m/s y al llegar a la orilla opuesta ha avanzado en el sentido de la corriente 23,4 m. a) Calcular la velocidad del agua sabiendo que el ro tiene una anchura de 100 m. b) Si la canoa marcha a lo largo del ro, determinar el camino recorrido en 1 minuto segn vaya en el sentido de la corriente o en sentido contrario. Solucin . a) La proa de la canoa debe recorrer un espacio en direccin perpendicular al ro: y = 100 2,5 = 97,5 m siendo y = vc t = 97,5 m el ro arrastra a la canoa x = 23,4 m = vr t dividiendo las dos anteriores

vR . vB
o

Para las velocidades dadas encontramos = 53,1 . El bote se mueve a lo largo de una lnea dirigida 53,1 ro abajo.

d) Haciendo D = distancia ro abajo, tenemos

v D 4 = R = , tal que D = 133 m. 100 v B 3


Ejemplo 48. Un submarino de propulsin convencional (Diesel) sufri un incendio en el Atlntico norte despus de salir de Inglaterra. Debido a un huracn no era posible enviar barcos ni aviones para ayudar al submarino diesel. La marina decidi enviar un submarino de propulsin nuclear para ayudar al de propulsin Diesel. El submarino diesel se encuentra al Sur a 500 km de distancia del submarino nuclear (ver figura). La rapidez del submarino nuclear respecto al agua es de 54 km/h. Adems, hay una corriente marina de 36 km/h que se mueve al NE formando un ngulo de 30 respecto al norte. (Asuma que el eje x es el eje DE, y el eje y es el NS). a) Si V es el mdulo de la velocidad del submarino nuclear visto desde tierra, escriba en forma vectorial, usando el sistema de coordenadas x -y, la velocidad del submarino nuclear respecto a tierra para que llegue al submarino diesel y la velocidad de la corriente marina con respecto a tierra. b) Halle la velocidad del submarino con respecto a la corriente de agua. c) Calcule el mdulo de la velocidad V. d) Halle el tiempo en el cual los marineros son rescatados.

97,5 5 = vr = 1,2m / s 23,4 v r b) v1 = v c + v r = 5 + 1,2 = 6,2 m/s x1 = 6,2 x 60 =372 m v 2 = v c v r = 5 - 1,2 = 3,8 m/s x2 = 3,8 x 60 =228 m
Ejemplo 47. Un bote de remos se dirige perpendicular a la orilla de un ro. Los remos pueden propulsar el bote con una velocidad de 3,0 m/s con respecto al agua. El ro tiene una corriente de 4,0 m/s. (a) Construya un diagrama en el cual las dos velocidades se representen como vectores. (b) Encuentre el vector que representa la velocidad del bote con respecto a la orilla. (c) Qu ngulo forma este vector con la direccin en la cual el bote est sealando? (d) Si el ro tiene 100 m de ancho, determnese cuan lejos ro abajo del punto del lanzamiento el bote llega al orilla opuesta. Solucin. Solucin: a) Diagrama.

24

Movimiento en un plano y en el espacio

Hugo Medina Guzmn a) A qu distancia del poste contada a lo largo de la va, y a qu distancia de esta chocar el cuerpo con el suelo? b) Realcese un esquema de la trayectoria seguida por el cuerpo Dato: la altura inicial del objeto sobre el suelo es de 2,45 m

Solucin. a) Si V es el mdulo de la velocidad del submarino nuclear visto desde tierra, escriba en forma vectorial, usando el sistema de coordenadas x-y, la velocidad del submarino nuclear respecto a tierra para que llegue al submarino diesel y la velocidad de la corriente marina con respecto a tierra.

Solucin. Velocidad del tren v y = 108

km m = 30 , h s km m = 10 h s

Velocidad de la piedra vx = 36

g 10
b) Halle la velocidad del submarino con respecto a la corriente de agua.

m s2

a) El movimiento de la piedra lanzada est dada por las ecuaciones:

sR

= 54 sen i 54 cos j ,

+ 36 cos 30 + 31,18 v c = 36sen30 i j = 18i j 18 1 54sen + 18 = 0 sen = = 54 3 cos = 0,94

1 x = 10t , y = 30t , z = 2,45 10t 2 2


Cuando la piedra llega al suelo z = 0

V sT = ( 54 cos + 31,18) j = ( 50,76 + 31,18) j = 19,18 j


c) Calcule el mdulo de la velocidad V. 19,18 km/hora d) Halle el tiempo en el cual los marineros son rescatados.

1 z = 0 = 2,45 10t 2 t = 0,7 s 2


Distancia del poste medida desde la va:

y = 30t = 30(0,7) = 21m

Distancia de la va al punto de cada:

x = 10t = 10(0,7) = 7m

t=

500 d = V 19,18

b)

= 26 horas Ejemplo 49. Desde el interior de un tren que viaja a 108 km/h, un nio lanza un objeto por una ventana con una velocidad de 36 km/h, horizontalmente y perpendicularmente a la marcha del tren, justo en el momento en que pasa en frente de un poste indicador. 25

Movimiento en un plano y en el espacio

Hugo Medina Guzmn

PREGUNTAS Y PROBLEMAS 1. La velocidad de la corriente de un ro aumenta en proporcin a la distancia de la orilla y alcanza su valor mximo v 0 en el medio. Cerca de la orilla la velocidad es cero. Un bote que navega en el ro tiene una velocidad u relativa al agua, constante y perpendicular a la corriente. a) Encontrar la distancia que fue arrastrando el bote al cruzar el ro de ancho C. b) Determinar la trayectoria del bote Respuesta. a) d = C

+ 36 j , t = 12 b) r 1 = r 2 = 840i
6. Las posiciones de dos partculas P1 y P2 estn

, dadas por r 1 = 5 + 3t + 2t i
2

. r 2 = (t + 5t 2 )i

v0 2u

a) En qu instante chocarn las dos partculas? b) Cul es la diferencia de velocidades en ese instante? Respuesta: a) t = 2 b) 8 7. El movimiento de una partcula est definido por el vector posicin

2. Un automovilista entra en una curva de 150 m de radio, una velocidad de 72 km/h. Accionando los frenos hace disminuir su velocidad de modo uniforme a razn de 1,5 m/s . Determinar el mdulo de la aceleracin del automvil cuando su velocidad es de 63 km/h. Respuesta: 2,53 m/s
2 2

. Determinar. + Ct r = Rsenb t i j + Rcosb t k

a) La velocidad y aceleracin de la partcula. b) La trayectoria de la partcula. c) El radio de curvatura.

3. Las ecuaciones paramtricas del movimiento de una partcula son x = R cos t , y = Rsen t , z = vt . R, , v son constantes. Probar que se trata de un movimiento uniforme, dibujar la trayectoria. Respuesta: Movimiento helicoidal con velocidad angular y subiendo con velocidad v. 4. Dadas las ecuaciones paramtricas de un movimiento x = Asen t , y = A cos t , a) Escribir la ecuacin del movimiento. b) La ley horaria c) La trayectoria

C 2 + R 2 b 2 , a = Rb 2 , C2 b) Helicoide, c) = R + Rb 2
Respuesta. a) v = 8. El movimiento de una partcula est definido por el vector posicin

+ 0,25cos2 t r = 0,1sen t i j , r en metros y t

en segundos: a) Determinar la velocidad y aceleracin para t = l s. b) Demostrar que la trayectoria de la partcula es una parbola. Respuesta. a)

+ Acos t Respuesta. a) r = Asen t i j , b)


s = At , c) x 2 + y 2 = A 2
5. Dos objetos se mueven en el plano xy de acuerdo

m/s , a = 0 v = 0,1 i 2 b) y = 0,025 5 x

9. La aceleracin de un cuerpo es:

cm/s 2 +2 a = 3i j+k

+ (2t + 12) a r 1 = 4t + 3t + 228 i jy


2

+ (5t 24) r 2 = (8t 2 + 11t 444 )i j

a) Si el cuerpo parte del reposo Cul es su velocidad despus de 3 segundos? b) Cul es su posicin despus de 10 segundos? c) Cul es su rapidez media durante los primeros 10 segundos?

respectivamente. a) Cuales son la velocidad y aceleracin de cada objeto? b) Dnde y cuando chocan? Respuesta.

cm/s +6 Respuesta. a) 9i j + 3k
cm + 100 b) 150i j + 50k c) 18,71 cm/s

+2 j , a 1 = 8i a) v 1 = (8t + 3)i

+5 v 2 = (16t + 11)i j , a 1 = 16i


26

10. Si una partcula que se mueve sobre una trayectoria curva tiene una aceleracin total en un

+ 2n cm/s . Hallar: momento dado a = 3t


2

Movimiento en un plano y en el espacio a) La aceleracin tangencial. b) La aceleracin centrpeta. c) El mdulo de la aceleracin total. d) El ngulo que la aceleracin total forma con la tangente a la curva. Respuesta: a) at = 3 cm/s b) a c = 4 cm/s c) a = 5cm/s d)
2 2

Hugo Medina Guzmn

= 53,1o

11. Dos cuerpos se lanzan simultneamente desde un mismo punto con la misma rapidez inicial pero en distintas direcciones, uno verticalmente hacia arriba y el otro formando un ngulo = 60 con la horizontal. Conociendo que la rapidez inicial de ambos cuerpos es v 0 = 25 m/s, a qu distancia se encontrarn cuando hayan pasado 1,7 s? 12. Una partcula se mueve en un plano de tal suerte que su radio vector con respecto a un punto fijo barre ngulos iguales en tiempos iguales mientras que la distancia al punto fijo es variable con el tiempo. Escriba las componentes radial y tangencial de la velocidad y la aceleracin de la partcula mostrando explcitamente cualquier cantidad que se mantenga constante durante el movimiento. 13. Un tren pasa por una estacin con una velocidad de 30 km/h. En el instante en que la locomotora pasa junto al guardagujas este lanza una bolsa a uno de los ingenieros de maquinas. Sabiendo que la rapidez inicial con que el guardagujas lanz la bolsa fue de 45 km/h a) Cul tendr que ser el ngulo de lanzamiento para lograr el objetivo?. b) Describa la trayectoria de la bolsa en el sistema de referencia del maquinista. 14. Un arquero est en una colina cuya pendiente forma un ngulo con la horizontal. Si el arquero dispara la flecha segn una direccin respecto a la colina y con velocidad v 0 , encontrar la distancia, medida a lo largo de la colina, a la cual caer la flecha.

partcula 1 con aceleracin constante a = a j , y la partcula 2 con aceleracin angular constante , en sentido contrario al movimiento de las agujas de un reloj, describiendo una circunferencia de radio R, como se muestra en la figura. Determine en funcin de a y R: a) El tiempo que tardan en encontrarse, suponiendo que lo hacen sobre el eje de las ordenadas, antes que la partcula 2 complete una vuelta completa. Encuentre el valor de que hace esto posible. b) Halle los vectores velocidad y aceleracin de las dos partculas para el instante del encuentro.

16. Un nio hace girar uniformemente una piedra en un crculo horizontal por medio de una cuerda de 1 m de longitud. El nio se encuentra sobre un montculo de tal forma que el plano del movimiento se encuentra a 5 m de altura sobre el suelo. La cuerda se rompe y la piedra sale disparada horizontalmente, golpeando el suelo a 3 m de distancia. Cul fue la aceleracin centrpeta de la piedra mientras estaba en movimiento circular?

17. Desde un sistema de referencia situado en el suelo, con eje horizontal x y vertical y, se observa el movimiento de un objeto sometido a una aceleracin

6 a = 2i j (m/s). Si en el instante inicial el


objeto se encontraba en el punto P = (-3, 2) (m),

j (m/s): movindose con una velocidad v (t = 0 ) = 3 a) Obtenga la ecuacin explcita de la trayectoria del objeto. b) Determine el instante en el que la velocidad y la aceleracin son perpendiculares. c) Calcule las coordenadas del punto ms alto de la trayectoria. d) Calcule el tiempo que tard el mvil desde que sali del punto P hasta que lleg al suelo.
18. La figura muestra una cuenta p que desliza por un alambre plano en forma de parbola. La ecuacin de la parbola es y = x2/b, donde b es una constante positiva con dimensiones de longitud. Llamaremos a 27

15. Dos partculas se encuentran inicialmente en reposo en las posiciones que muestra la figura. Ambas comienzan a moverse al mismo tiempo, la

Movimiento en un plano y en el espacio al ngulo entre la tangente a la curva y el eje x. en el punto donde se encuentra la cuenta. a) Halle tan en funcin de la coordenada x de P. b) Suponga que la cuenta tiene rapidez v y se mueve hacia la derecha. Halle las componentes x e y de la velocidad de la cuenta en funcin de y y de la coordenada x de P. Ayuda: recuerde que el vector velocidad es tangente a la trayectoria.

Hugo Medina Guzmn

Respuesta.

y N a los vectores unitarios en Llamaremos E direccin Este y Norte respectivamente.

+ 60 N ) km/h, v = 60 26 km/h. v = (300 E

21. Un hombre gua su automvil bajo lluvia a una velocidad constante respecto a Tierra de mdulo y direccin. Mientras conduce el hombre observa que la trayectoria de cada gota es una lnea recta que se aparta un ngulo de la vertical y al detenerse observa que la lluvia cae verticalmente y prcticamente con velocidad constante. Halle el vector velocidad de las gotas de lluvia respecto al auto en movimiento y respecto a Tierra (tome vertical hacia arriba). Respuesta.

Respuesta.

2x a) tan = b 2 xv bv b) v x = , vy = 2 2 2 b + 4x b + 4x 2
19. Un ascensor parte del reposo y desciende con aceleracin constante de 1 m/s2 respecto a Tierra. Dos segundos despus de iniciarse el descenso se cae la lmpara del techo del ascensor. La distancia del techo al piso del ascensor es de 2 m. Definimos el referencial del ascensor como aqul con origen en su techo y direccin y positiva apuntando hacia abajo. a) Halle los vectores aceleracin, velocidad y posicin de la lmpara respecto al ascensor. b) Determine el tiempo que tarda la lmpara en caer. c) Encuentre la distancia recorrida por el ascensor mientras cae la lmpara. Respuesta. Todas las unidades estn expresadas en el sistema MKS. L indica lmpara, A ascensor y T Tierra. a) Tomaremos como t = 0 el instante para el cual se desprende la lmpara.

v gota ,Tierra =

v gota , Auto

v j, tan v = j vi tan

22. Un vagn de ferrocarril motorizado va cuesta abajo sobre un plano inclinado un ngulo . La distancia entre el techo y el piso del vagn es H y su aceleracin respecto a Tierra es constante y

a LA = a LT a AT = 9 j , v LA = 9t j, 9 r LA = t 2 j 2 9 2 2 b) y LA = t = 2 t = 2 3 14 c) D = 9
20. Los instrumentos de un aeroplano en vuelo horizontal indican que se dirige hacia el Este con una rapidez de 300 km/h respecto al aire. En Tierra se observa que el aeroplano se encuentra en medio de una corriente de aire que sopla hacia el Norte con rapidez de 60 km/h. Halle la velocidad y rapidez del avin respecto a Tierra. 28

, ver figura. Un pasajero del vagn vale a = ai observa que una lmpara, situada en el centro del techo del vagn, se desprende y choca con el piso en el punto O (en el extremo inferior del vagn). a) Halle la aceleracin de la lmpara respecto a Tierra y respecto al pasajero del vagn. Exprese sus y resultados en trminos de los vectores unitarios i j. b) Escriba las componentes cartesianas de la velocidad y posicin de la lmpara segn el pasajero. Torne el origen en el punto o solidario al vagn y llame L a la longitud del vagn. c) Halle el tiempo que tarda la lmpara en caer y la longitud L del vagn. d) Determine la ecuacin de la trayectoria de la lmpara, y = y ( x ) , segn el pasajero. Qu clase de curva es la trayectoria de la lmpara vista por el pasajero y vista desde Tierra?

Respuesta. Los subndices L, P y T hacen referencia a la lmpara, al pasajero y al referencial inercial de Tierra respectivamente.

Movimiento en un plano y en el espacio

Hugo Medina Guzmn 24. El aro de la figura tiene radio R y rueda sobre una superficie horizontal fija a Tierra. El aro gira en sentido horario mientras su centro e se mueve hacia la derecha con rapidez V respecto a la superficie. Considere un observador con origen en C (se traslada con el aro) y que no rota respecto a Tierra. Suponga que todos los plintos del aro tienen rapidez V respecto al observador (se dice entonces que el aro rueda sin deslizar). En la figura se han marcado cuatro puntos para un cierto instante. El punto A es el punto ms alto del aro, el B el ms bajo, el D el punto del extremo izquierdo y el E con un radio vector que forma un ngulo con la vertical. a) Halle la velocidad angular w del aro. b) Halle los vectores velocidad de los puntos A, B y D respecto a la superficie. c) Halle el vector velocidad del punto E respecto a la superficie y diga para qu ngulo su mdulo es igual a V.

cos j , a) a LT = g sen i g cos a LP = (gsen a )i j b) v x = (gsen a )t , v y = g cos t 1 (gsen a ) t 2 L , 2 2 1 y = g cos t 2 + H 2 2H 2 H (gsen a ) c) t = , L= g cos g cos

x=

d) Vista por el pasajero la trayectoria es una lnea recta de ecuacin

y=

g cos x gsen a

Vista desde Tierra la trayectoria es una parbola. 23. La corriente de un ro fluye de Este a Oeste con rapidez constante v = 2 m/s respecto a Tierra. Un bote atraviesa el ro y de acuerdo a sus instrumentos de a bordo se mueve respecto al ro dirigindose al Norte con rapidez constante = 10 m/s. Respecto al bote un pasajero se desplaza sobre la cubierta en lnea recta desde el punto A hasta el punto G con una rapidez constante v1 = 10 m/s. Suponga que BA = 4 m y apunta hacia el Norte y BC = 3 m y apunta hacia el Este. a) Halle el vector unitario u que apunta de A a C y las velocidades del bote y del pasajero respecto a Tierra. b) Halle el tiempo que tarda el pasajero en ir de A hasta C. Qu distancia recorre el bote en ese tiempo segn un observador en Tierra?

Respuesta. a) La rapidez de cualquier punto del aro respecto a C es V = R , luego = V / R . r r r , VB = 0 , V D = Vi + V j b) V A = 2Vi

+ Vsen c) V E = V (1 cos ) i j , VE = V

= 60
25. Para conocer la rapidez de un avin es necesario determinar cuanto tiempo toma volar en un rizo cerrado de longitud conocida. Cunto tiempo tomar al avin volar alrededor de un cuadrado de lado a, con el viento soplando con una velocidad u?, en dos casos: a) la direccin del viento coincide con uno de los lados del cuadrado; b) la direccin del viento coincide con la diagonal del cuadrado? Sin viento la rapidez del avin es v, mayor que u. Respuesta, a) t1 =

Respuesta. Las letras b, p y T designarn respectivamente el bote, pasajero y Tierra. a) u =

+4 3i j

10 , v b ,T = 2i j m/s,

2 v p ,T = (4i j ) m/s. 1 b) t = s , d = 26m 2

2a v + v 2 u 2 , b) v2 u 2

t2 =

4a v 2 u 2 / 2 v2 u 2

26. Un hombre que viaja en un camin intenta golpear un poste con una piedra, y cuando pasa 29

Movimiento en un plano y en el espacio frente a l arroja la piedra con una velocidad horizontal de 20 m/s respecto al camin. Sabiendo que la velocidad del camin es de 40 km/h, Calcular: a) la direccin en que debe lanzar la piedra. b) la velocidad horizontal de la piedra respecto al suelo. Respuesta. a) 56,3 con relacin a la direccin trasera del camin b) 16,63 m/s 27. El piloto A est volando con un avin con una velocidad de 150 km/h, sobrevolando al piloto B, cuyo avin vuela a 135 km/h, 300 m por debajo Con el mismo rumbo. El piloto A para mandar un mensaje a B lo sujeta a una piedra y la arroja a la cabina de B. Sin tomar en cuenta la resistencia del aire. a) Con qu velocidad deber lanzarla respecto a su avin cuando B est directamente debajo de l? b) Cundo B est todava a 300 metros delante de l? Respuesta, a) v = 15 km/h hacia atrs; b) v = 128 km/h haca adelante. 28. Una partcula describe una circunferencia de radio R = 0,5 m con una frecuencia de 10 r pm. Si en t0= 0 la partcula est en la posicin A movindose en el sentido horario, calcular: a) El perodo T y la rapidez del movimiento b) La velocidad media y aceleracin media en el intervalo (0; 0,75T). c) La aceleracin en t =T / 2

Hugo Medina Guzmn

30. Un automvil viaja hacia el Este con una rapidez de 50 km/h. Est lloviendo verticalmente con respecto a la Tierra. Las marcas de la lluvia sobre las ventanas laterales del automvil forman un ngulo de 60 con la vertical, calcule la velocidad de la lluvia con respecto a: a) el automvil y b) la Tierra. 31. La distancia de A a B es l . Un aeroplano vuela desde A hasta B y vuelve otra vez con una velocidad constante V relativa al aire. Calcular el tiempo, total que emplear en realizar el recorrido si el viento sopla con una velocidad v en las siguientes direcciones: a) Sobre la lnea que une A y B. b) Perpendicular a esta lnea. c) Formando un ngulo con esta lnea. Demostrar que la duracin del trayecto siempre aumenta con la existencia del viento. Respuesta. Poniendo T0 = a)

2l , los resultados son: V

T0 v2 1 V2 T0
v2 1 V 2



12

b) 29. Una partcula P se mueve con aceleracin angular constante sobre una circunferencia de radio R =3m. Parte desde el reposo del punto A y completa la primera vuelta en un tiempo t = 2s. Calcular: a) El mdulo de la aceleracin angular b) La ecuacin r = r(t ) . c) El tiempo que emplea para llegar a la posicin definida por = 3/2 . d) La velocidad lineal en =

vsen 1 V c) T0 2 v 1 V 2

1 2

32. El bloque deslizante A se mueve hacia la m/s, izquierda a una velocidad constante de 0,3i Determinar: a) La velocidad del bloque B; b) las velocidades de los tramos de cable C y D; e) la velocidad relativa de A respecto a D; d) La velocidad relativa del tramo de cable C respecto al tramo D. 30

Movimiento en un plano y en el espacio

Hugo Medina Guzmn

Respuesta. m/s, b) 0,2i m/s, 0,4i m/s, a) 0,2i

m/s, d) 2i m/s, c) 0,1i

31

Dinmica de una partcula

Hugo Medina Guzmn

CAPTULO 4. Dinmica de una partcula


INTRODUCCIN En el captulo anterior estudiamos el movimiento de una partcula con respecto a un sistema de referencia sin preguntarnos sobre la causa del movimiento. Lo describimos simplemente en trminos de los vectores

r , v y a.
En ausencia de una fuerza resultante, el objeto se mantiene en movimiento con velocidad uniforme o permanece en reposo. Esta es la PRIMERA LEY DE NEWTON DEL MOVIMIENTO Ahora podemos pensar acerca de la situacin cuando un objeto era empujado sobre un plano. Cuando la fuerza era pequea no haba movimiento, pero una fuerza debera causar movimiento; la conclusin es que debe haber otra fuerza actuando sobre el cuerpo la cual anula justamente el efecto de la fuerza que aplicamos. Al incrementar nuestra fuerza, la fuerza opuesta tambin se incrementa, hasta que en algn valor particular la fuerza opuesta termina de incrementarse y comienza el movimiento porque hay una fuerza resultante actuando sobre el objeto. La fuerza opuesta es la fuerza de Friccin QU ES FUERZA? En la vida cotidiana se considera fuerza a una sensacin comn asociada con la dificultad para mover o levantar un cuerpo. En Fsica se identifica una fuerza por el efecto que produce. Uno de los efectos de una fuerza es cambiar el estado de reposo o de movimiento del cuerpo, ms concretamente, una fuerza cambia la velocidad de un objeto, es decir produce una aceleracin. Cuando se aplica una fuerza sobre un cuerpo y no se produce movimiento, entonces puede cambiar su forma, an si el cuerpo es muy rgido. La deformacin puede o no ser permanente. Entonces los efectos de la fuerza neta son dos: cambiar el estado de movimiento de un cuerpo o producir una deformacin, o ambas cosas. Normalmente sobre un cuerpo pueden actuar varias fuerzas, entonces el cuerpo acelerar cuando el efecto de la fuerza neta que acta sobre l no es cero. Se llama fuerza neta o fuerza resultante a la suma de todas las fuerzas que actan sobre un cuerpo. Si la fuerza neta es cero, la aceleracin es cero, el movimiento es con velocidad igual a cero (cuerpo detenido) o con velocidad constante. Cuando un cuerpo est en reposo o se mueve con velocidad constante, se dice que est en equilibrio. Se pueden distinguir dos grandes clases de fuerzas: fuerzas de contacto, representan el resultado del contacto fsico entre el cuerpo y sus alrededores, por ejemplo mover un carro o estirar un resorte; y fuerzas de accin a distancia que actan a travs del espacio sin que haya contacto fsico entre el cuerpo y sus alrededores, por ejemplo la fuerza con que la Tierra atrae a los cuerpos que caen en cada libre. Todas las diferentes formas de fuerzas se encuentran dentro de esas dos grandes clasificaciones. 1

Nuestra discusin fue geomtrica, en este captulo discutiremos la causa del movimiento. Seguiremos tratando a los cuerpos como partculas simples. Posteriormente trataremos sobre sistemas de partculas y cuerpos rgidos. EL ORIGEN DEL MOVIMIENTO Qu origina el movimiento? Qu detiene el movimiento? Se necesita causa para mover las cosas? Por qu un objeto al que se le da un empujn pronto se detiene? Por qu los planetas mantienen su movimiento alrededor del sol? Aristteles joven filsofo griego (siglo IV a.c.) deca que un cuerpo permaneciera en movimiento era necesario ejercer alguna accin sobre l ya que el estado natural es el reposo. Esto parece ser razonable, cuando dejamos de empujar un cuerpo, este pronto alcanza el reposo. Parece ser necesaria una accin exterior o fuerza aplicada al cuerpo para mantener el movimiento. Sin embargo, observemos esta situacin con mayor detenimiento. La figura siguiente muestra un bloque de madera sobre un plano.

Aplicamos una fuerza pequea al bloque, no pasa nada. Incrementamos la fuerza y a un valor particular el bloque se mueve. Si seguimos incrementando la fuerza empujando o jalando ms, el objeto se mueve con mayor rapidez, Cuando dejamos de empujar el cuerpo rpidamente vuelve al reposo. Sin embargo si ponemos ruedas al bloque el resultado es diferente, una fuerza muy pequea causa el movimiento. La diferencia son las ruedas debido a la friccin. Para hacer un estudio libre de la friccin busquemos llegar cercanamente a esta condicin, una forma de lograr esto es con una mesa neumtica, se sopla aire sopla hacia arriba a travs de pequeos agujeros manteniendo un disco suspendido sobre un colchn de aire. Qu pasa cuando empujamos un objeto en ausencia de friccin? Este se mantiene en movimiento a velocidad constante.

Dinmica de una partcula Para describir el mundo, la fsica contempornea recurre a cuatro interacciones o fuerzas fundamentales, que actan sobre las partculas de materia (y sobre las antipartculas), son vehiculadas por unas partculas llamadas vectores de interaccin, que son: fotn (interaccin electromagntica), bosn (interaccin dbil), glun (interaccin fuerte) y gravitn (interaccin gravitacional). 1) Fuerzas electromagnticas de atraccin o repulsin entre partculas cargadas en reposo o en movimiento, explica la cohesin de los tomos, es mucho ms intensa que la fuerza gravitacional. 2) Fuerzas nucleares intensas entre partculas subatmicas, responsable de la existencia del ncleo atmico asegura la cohesin interna de los constituyentes del ncleo atmico, protones y neutrones, y es responsable de un gran nmero de reacciones y de desintegraciones; es la de mayor magnitud (102 - 103 veces la fuerza electromagntica). 3) Fuerzas nucleares dbiles de corto alcance, rige algunos procesos radiactivos, establece la estabilidad de algunos ncleos, es varios rdenes de magnitud (1012) menor que la fuerza electromagntica. 4) Fuerza de atraccin gravitacional entre cuerpos debido a sus masas, entre otras cosas hace que caigan las manzanas y que suba la marea, es la fuerza de menor magnitud comparada con las otras. Para que el concepto de fuerza sea exacto se debe establecer un mtodo para medirla. Una fuerza se puede medir por el efecto que produce. Por ejemplo se puede usar la deformacin que una fuerza produce en un resorte, como en la figura. Si se aplica una fuerza verticalmente a un resorte y se estira una unidad, le asignamos a la fuerza una magnitud unitaria F. Se aplica ahora otra fuerza al mismo resorte horizontalmente, producindole un estiramiento de dos unidades, la magnitud de la fuerza ser de 2F. Si se aplican simultneamente las dos fuerzas, el resorte se inclina, y se estira 5 veces. La fuerza equivalente que produce ese estiramiento del resorte es la suma vectorial de F y 2F. Es decir, la fuerza es un vector.

Hugo Medina Guzmn si se estira 2,5 unidades, entonces la fuerza aplicada es 2,5 veces la unidad de fuerza. Este procedimiento es vlido para pequeos alargamientos del resorte, ya que si la fuerza es muy intensa, se puede deformar y no volver a su forma original. CAMBIO DE VELOCIDAD Nuestro siguiente problema es encontrar una relacin entre la fuerza y el cambio en el movimiento producido por sta. Para esto necesitamos lo siguiente: 1. Un carro muy ligero que pueda moverse sin friccin sobre una superficie horizontal. 2. Una fuerza constante. Esta podernos obtenerla mediante un resorte (Si mantenemos un resorte estirado una misma longitud, la fuerza que la estira es constante).

3. Un registrador de tiempo. El movimiento del carro puede estudiarse si una cinta de papel atada a ste pasa a travs del registrador que produce marcas en la cinta a intervalos de tiempo regulares.

La figura siguiente muestra la cinta de papel producida por una fuerza constante.

El instrumento para medir fuerzas se llama dinammetro, es un resorte que se estira sobre una escala. Si se aplica una fuerza de una unidad sobre el dinammetro, el resorte se estira hasta que ejerce una fuerza igual y contraria a la aplicada. En la escala se mide el alargamiento del resorte y se le asigna una unidad de fuerza. De esa manera se calibra el dinammetro y se usa para medir fuerzas, por ejemplo se aplica una fuerza sobre el dinammetro y 2

Con los datos obtenidos en esta experiencia se realiza el grfico distancia - tiempo y se obtiene una curva. Con los datos tambin se puede obtener la velocidad media en cada intervalo de tiempo. El grfico velocidad - tiempo es una lnea recta que indica que

Dinmica de una partcula el movimiento es con aceleracin constante. De aqu podemos concluir que una fuerza constante produce una aceleracin constante.

Hugo Medina Guzmn internacional (S.I.) la unidad de aceleracin es m/s. Cuales son las unidades de fuerza y de masa? Como son dos cantidades que se relacionan slo tenernos que especificar un estndar para una de ellas. El sistema internacional adopta corno unidad una pieza de material llamado KILOGRAMO, cuyo smbolo es kg. El kilogramo es la masa un prototipo de platino iridiado sancionado por la Conferencia General de Pesas y Medidas realizada en Pars en 1889 y depositado en el pabelln de Breleuil en Sevres. La unidad de fuerza es el newton, cuyo smbolo es N y se define as: El newton la fuerza que produce una aceleracin de un metro por segundo al cuadrado a una masa de un kilogramo.

Si duplicamos la fuerza usando dos resortes iguales estirados la misma longitud, como se muestra en la figura.

Duplica la fuerza y produce el doble de aceleracin. Si triplicamos la fuerza se obtiene una aceleracin de valor triple. Concluimos que la aceleracin a del cuerpo es directamente proporcional a la fuerza. Podemos escribir esto como F = ma , donde m es la constante de proporcionalidad. A esta constante la llamaremos MASA. Para una determinada fuerza a mayor constante m la aceleracin es menor. A mayor valor de la constante es ms difcil acelerar el cuerpo. Para conocer qu factores cambian esta constante realicemos el siguiente experimento: en lugar de usar un solo carro jalado por el resorte estirado usemos dos carros uno sobre otro y luego tres carros como se muestra en la figura

N=

kgm s2

aF

Otros sistemas: MKS: igual al S.I. CGS: Masa gramo (g), l g = 10-3 kg Aceleracin cm/s2 Fuerza dina = g.cm/s2 Ingls tcnico: En este sistema la unidad fundamental es la unidad de fuerza. Fuerza libra (lb), 1 lb = 4,45 N Aceleracin pie/s2 Masa slug = lb58 s2/pie PESO DE UN CUERPO. El peso de un cuerpo es la fuerza de atraccin que ejerce la Tierra sobre el cuerpo. Un cuerpo de masa m sometido a cierta fuerza cae con la aceleracin de la gravedad g, el peso P de este cuerpo es

P = mg
La aceleracin que se obtiene con los carros es igual a la mitad y con tres es igual a un tercio. Como el valor de F es igual en todos los casos, quiere decir que la constante con dos carros es igual a 2m y con tres carros es 3m. Como la aceleracin es una cantidad vectorial la fuerza tambin lo es y tiene la misma direccin que la aceleracin, pero un mdulo m veces mayor, de modo que la relacin anterior puede escribirse en la forma

F = ma

Fuerza = masa x aceleracin. Esta expresin constituye la SEGUNDA LEY DE NEWTON DEL MOVIMIENTO. La fuerza que acta sobre un cuerpo es igual al producto de la masa del cuerpo por la aceleracin que le imprime. UNIDADES DE FUERZA Y MASA La relacin F = ma nos da una relacin entre fuerza, masa y aceleracin. En el sistema 3

Su direccin es hacia abajo (hacia el centro de la Tierra). Como el peso es una fuerza debe medirse en Newtons. Debido a que la aceleracin de la gravedad vara de un lugar a otro de la Tierra, el peso de un cuerpo es diferente en lugares distintos, sin embargo la masa de un cuerpo es la cantidad fija que no depende del lugar donde est situado el cuerpo, Aunque el peso de un objeto vara de un sitio a otro, esta variacin es demasiado pequea para ser observada en la mayor parte de las aplicaciones prcticas, por esto, el peso de un cuerpo parece ser una caracterstica constante al igual que su masa. Este

Dinmica de una partcula hecho ha conducido al empleo ordinario de otras dos medidas: KILOGRAMO FUERZA, es el peso de un Kilogramo masa. 1 kgf = 9,8 N LIBRA MASA, es la masa de un cuerpo que pesa una libra. 1 libra masa = 0,454 kg. Estas unidades son prcticas pero incorrectas y no deben ser usadas en Fsica. ACCION Y REACCION. Hagamos una observacin ms detallada cuando jalamos el carro con un resorte estirado una determinada longitud.

Hugo Medina Guzmn aplicacin de las leyes de Newton. APLICACIONES DE LAS LEYES DE NEWTON Cuando estudiamos Cinemtica, encontrarnos las relaciones entre desplazamiento, aceleracin y tiempo. Por ejemplo, conociendo la aceleracin a las condiciones tales como posicin inicial, velocidad inicial, es decir la posicin y la velocidad en el tiempo que llamamos inicial (t = 0), podemos conocer la velocidad y posicin para cualquier tiempo. Las condiciones iniciales las tenemos pero la aceleracin, de dnde? Para esto tenemos F = m a , todo lo que tenemos que hacer es conocer las fuerzas sobre el cuerpo y su masa, y entonces podremos encontrar a . La mejor forma de estar seguros que comprendemos el significado de F = m a , es hacerlo con algunos problemas que involucran las leyes de Newton. Para resolver un problema sugerimos cuatro pasos a seguir: 1. Dibujar un esquema del sistema 2. Identificar el cuerpo a cuyo movimiento se refiere el problema. 3. Dibujar otra figura con solamente el objeto en particular manteniendo el marco de referencia poner todas las fuerzas que actan sobre el objeto mediante flechas. Esto se conoce como DIAGRAMA DEL CUERPO LIBRE (DCL). Si se comete una equivocacin todo lo dems fallar, por eso es conveniente hacerlo bien. Una mejor forma de comenzar es poner la fuerza de gravedad primero y luego preguntarse: Qu toca al cuerpo?, la accin de tos resortes, cuerdas, manos y otros objetos, todos deben ser considerados. As como tambin las fuerzas que actan sin tocar el cuerpo, como la fuerza elctrica, magntica de las cuales no nos preocupamos en este curso. 4. Finalmente, aplicar la segunda ley de Newton a cada componente de fuerza y aceleracin.

Para que el resorte est estirado es necesario jalarlo por los dos lados. Se necesitan fuerzas en sentidos opuestas y en cada extremo del resorte. Cuando jalamos el carro, una fuerza acta sobre el carro y una fuerza en sentido opuesto acta sobre nuestra mano. Cules son las magnitudes de estas fuerzas?

Con el objeto de dar respuesta a esta pregunta pongamos dos resortes iguales al primero y jalemos de tal manera que el carro adquiera la misma aceleracin que antes, esto quiere decir, por la segunda ley de newton que siendo la misma masa m estamos aplicando la misma fuerza (F = ma) que antes y observamos que los resortes estiran la misma longitud, lo que quiere decir que la fuerza sobre la mano es igual a la fuerza sobre el carro.

F =ma Fx = ma x ,
Esto constituye la TERCERA LEY DE NEWTON DEL MOVIMIENTO. Si un cuerpo ejerce una fuerza sobre un segundo, ste ejerce una fuerza igual y opuesta sobre el primero. La fuerza ejercida por el primer cuerpo sobre el segundo es la ACCIN, la fuerza igual y opuesta actuando sobre el primero es la REACCIN, Expresado en smbolos, es:

= ma y , Fz = ma z .

y ahora resolver para la aceleracin. En algunos de los problemas que se presentan ms frecuentemente, las acciones se producen por fuerzas sin contacto; en otros se usan cuerdas y varillas como medios de conexin. Cuando las masas de estos medios de conexin son despreciables su nico efecto es el de transmitir ESTTICA DE LAS MASAS PUNTUALES. Los sistemas en los cuales todas sus partes satisfacen la primera ley son llamados sistemas estticos, es decir si la suma vectorial de todas las fuerzas que actan es nula, el cuerpo esta en equilibrio y permanece en reposo, o si est en movimiento, se mantiene con velocidad constante 4

F sobre 2 debido a 1 = F sobre 1 debido a 2


Fuerza de contacto de un cuerpo a otro con un cambio de direccin o sin l A continuacin presentarnos algunos casos tipo de la

Dinmica de una partcula La condicin de este equilibrio es

Hugo Medina Guzmn

F = 0
y en componentes cartesianas:

= 0,

= 0 , Fz = 0 .

Las fuerzas son ejercidas sobre el objeto o sistemas por. Medios exteriores al sistema. Ejemplo 1. La Fuerza gravitacional Dado que la aceleracin de un cuerpo en cada libre en la tierra es g, cul es la fuerza de la gravedad? Solucin. Como este movimiento es en una sola dimensin, consideramos que este se realiza en el eje z, tal que

a = gk

Segn la Segunda Ley de Newton

F = m a = mgk Fx = 0 , Fy = 0 , Fz = mg .
Siendo esta la respuesta que ya conocamos. Ejemplo 2. El dinammetro. El dinammetro es un instrumento que se utiliza para medir las fuerzas. Consta de un resorte con una escala que indica su estiramiento, la cual est graduada en Newtons. Cuando lo utilizamos para pesar se dispone como lo muestra la figura.

Diagrama del cuerpo libre (DCL) Aplicando la condicin de equilibrio de la masa m

T1 mg = 0 Luego T1 = mg
Si despreciamos la masa del dinammetro, tenemos que: T1 T2 = 0 y T1 = T2 El dinammetro indica en la escala la fuerza

T2 = mg
Ejemplo 3. Se tiene los dispositivos mostrados en la figura. Cunto indica el dinammetro de la figura (a) y cunto el dinammetro de la figura (b)?

Se suspende la masa m, el resorte del dinammetro se estira hasta que alcanza el equilibrio esttico.

Solucin. a) El diagrama de cuerpo libre de la figura (a) es

Dinmica de una partcula En la masa

Hugo Medina Guzmn

T1 mg = 0 T1 = mg
En la polea

T1 = T2
En el dinammetro

T3 = T2 = T1 = mg
Empezando por la derecha El dinammetro es tensionado por la fuerza indicacin ser

T1 y su

T1 mg = 0 T1 = mg
La figura siguiente muestra la polea

T1 = mg
Como se puede ver esta situacin es completamente anloga a la anterior, slo que hemos sustituido una de las poleas por la pared. Ejemplo 4. Un cuerpo de masa m se sostiene por medio de cuerdas como se muestra en la figura. Encontrar las tensiones T1, T2 en las tres cuerdas.

Para que el trozo de cuerda este en equilibrio

F = 0
Descomponiendo las fuerzas sobre el trozo de cuerda en los ejes x e y. Como la cuerda se considera sin masa la suma de fuerzas a lo largo del eje x es

T1 cos T2 cos = 0 T1 = T2

Solucin.

En el dinammetro, considerndolo de masa despreciable.

F = 0
T2 T3 = 0 T2 = T3
En la polea de la izquierda

T4 = T3
En la masa de La izquierda

F = 0
T4 mg = 0 T4 = mg
Como conclusin todas las tensiones son iguales a mg

Tomando un sistema de ejes horizontal y vertical como el mostrado en la figura tenemos:

T1 = mg j
+ T2 sen T2 = T2 cos i j
+ T3sen T3 = T3 cos i j
Con

T4 = T3 = T2 = T1 = mg
El dinammetro es tensionado por la fuerza indicacin ser:

T1 , y su

F = 0

T1 = mg
b) El diagrama de cuerpo libre de la figura siguiente es

T1 + T2 + T3 = 0
Obtenemos:

F F

= T2 cos T3 cos = 0

= T2 sen + T3sen mg = 0

Resolviendo estas dos ecuaciones

T2 =

mg cos mg cos , T3 = sen ( + ) sen ( + )

Ejemplo 5. Un bloque de 50N de peso se ubica sobre un plano inclinado en un ngulo de 30 con la 6

Dinmica de una partcula horizontal. El bloque se sujeta con una cuerda ideal que se encuentra fija en la parte superior del plano inclinado, como en la figura. Estudiar el comportamiento mecnico del bloque.

Hugo Medina Guzmn

Solucin. El D. C. L. del cuerpo:

Del diagrama de cuerpo libre se obtiene:

Solucin. El D. C. L. del cuerpo:

F F

: mgsen = ma x :

N mgcos = ma y = 0

De estas ecuaciones se obtiene: a x = gsen y N = mg cos Se concluye que la aceleracin del bloque en direccin del plano inclinado es la componente de g en esa direccin. Estudiando ahora el movimiento del bloque, considerando que parte del reposo y se desliza una distancia D, se puede calcular la rapidez con que llega a la base del plano. Si se considera que el movimiento del bloque comienza desde el reposo, se puede usar:
2 v 2 = v0 + 2a x x v 2 = 2( gsen )D

Fuerza de atraccin de la Tierra, que es su peso mg. Fuerza de la cuerda que lo sostiene, que es la tensin T Fuerza que el plano ejerce sobre el cuerpo, que es la normal N Como el sistema est en equilibrio, se aplica la primera Ley de Newton: Del diagrama de cuerpo libre se obtiene:

y v=

2 gDsen

Ejemplo 7. Para el siguiente sistema mecnico, calcular la aceleracin de las masas y la tensin de la cuerda.

F F

x y

T + mgsen = 0

: N mgcos = 0 Solucin. Como no se conoce la direccin del movimiento, supongamos que el cuerpo de masa M sube por el plano inclinado, lo que determina el sentido de la aceleracin, entonces aplicando la segunda Ley de Newton se aplica cada masa: El D. C. L. del cuerpo M:

Despejando T y N, y reemplazando los valores numricos, se obtiene:

T = mgsen = 50sen30 = 25 N N = mgcos = 50cos30 = 43,2 N

DINMICA CON FRICCIN DESPRECIABLE.


Los sistemas en los cuales todas sus partes satisfacen la primera ley son llamados sistemas estticos, es decir si la suma vectorial de todas las fuerzas que actan no es nula y la friccin se considera despreciable, Ejemplo 6. Si un bloque de masa m se ubica sobre un plano sin roce, inclinado un ngulo con la horizontal, resbalar una distancia D a lo largo del plano. Describir su movimiento.

Del diagrama de cuerpo libre se obtiene:

T Mgsen = Ma T = Mgsen + Ma Fy : N Mgcos = 0


x

De estas ecuaciones se obtiene: El D. C. L. del cuerpo m: 7

Dinmica de una partcula

Hugo Medina Guzmn

Del diagrama de cuerpo libre se obtiene:

T mg = ma T = mg ma
a) Sumando (1), (2) y (3):

De estas ecuaciones se obtiene

Mgsen + Ma = mg ma (m Msen ) g a= (m + M )

F1 (m1 + m2 + m3 )g = (m1 + m2 + m3 )a

Se observa que el signo de a depende del trmino (m - M sen ). Ahora se calcula el valor de la tensin reemplazando el valor de a en T:

y a=

m Msen T = mg m g m+M mM (1 + sen )g T= (m + M )


Ejemplo 8. Dos bloques de masas m1 = 20 kg y m2 = 8 kg, estn unidos mediante una cuerda homognea inextensible que pesa 2 kg. Se aplica al conjunto una fuerza vertical hacia arriba de 560 N. Calcular: a) La aceleracin del conjunto; b) Las fuerzas que actan en los extremos de la cuerda.

F1 g (m1 + m2 + m3 ) 560 a= 9,8 = 8,87 m/s2 (20 + 8 + 2) b) De (3) FB = m2 ( g + a ) FB = 8(9,8 + 8,87 ) = 149,4 N De (1) FA = F1 m1 ( g + a ) FA = 560 20(9,8 + 8,87 ) = 186,6 N

Ejemplo 9. La mquina de ATWOOD. Es un aparato que se utiliza para determinar con exactitud la gravedad y consiste de dos masas m1 y m 2 , ( m1 > m 2 ), que estn unidas mediante una cuerda que pasa sobre una polea. Considerar la cuerda inextensible y sin masa. Asimismo, no tornar en cuenta la friccin y la masa de la polea. Describir el movimiento y calcular la tensin en la cuerda.

Solucin. En el D. C. L. de m1:

F1 FA m1 g = m1 a (1) En el D. C. L. de la cuerda de masa m3: FA FB m3 g = m3 a (2) FB m2 g = m2 a


(3)

Solucin. Siendo m1 mayor que m 2 , la masa m1 se mover hacia abajo con una aceleracin a y la masa m 2 se mover hacia arriba con la misma aceleracin a . La figura siguiente muestra los diagramas de cuerpo libre de cada una de las partes del sistema.

En el D. C. L. de m2:

Dinmica de una partcula

Hugo Medina Guzmn Ahora la reaccin del piso es R ' . Aplicando la Segunda Ley de Newton al movimiento de la persona

R' mg = ma R ' = m( g + a )

Si el ascensor sube el pasajero se siente ms pesado, como si fuera empujado contra el piso. Si el ascensor desciende con esta aceleracin, R ' mg = ma R ' = m( g a ) , el pasajero se siente ms liviano. La polea cumple la funcin de cambiar la direccin T1 Considerando el sentido de la aceleracin o como positiva. Aplicando la Segunda Ley de Newton a la masa m1 Ejemplo 11. La figura muestra a un hombre elevndose mediante una fuerza vertical que aplica l mismo a la cuerda que tiene en las manos. Si el hombre y la silla juntos tienen una masa de 100 kg. Se pregunta: a) Con qu fuerza debe jalar para, subir con una velocidad constante? b) Con qu fuerza debe jalar para subir con una aceleracin de l m/s2 (considerar g = 10 m/s2?

m1 g T1 = m1 a
Aplicando la Segunda Ley de Newton para la masa m2 :

T1 m2 g = m2 a
De estas dos ecuaciones obtenemos:

a=

(m1 m2 ) g (m1 + m2 )

y T1 =

2m1 m2 g (m1 + m2 )

Si las masas m1 y m 2 fueran casi iguales, el valor de la aceleracin sera pequea y podra determinarse midiendo el tiempo en que una de las masas sube o baja una distancia determinada. La razn (m1 m2 ) se determina pesando los cuerpos. Finalmente, la magnitud de g se obtiene a partir de estas cantidades mediante la ecuacin

(m1 + m2 )

g=

(m1 + m2 ) a (m1 m2 )

Solucin. a) La figura siguiente muestra los diagramas de cuerpo libre de cada una de las partes del sistema.

Ejemplo 10. El peso de un pasajero en ascensor. Consideremos un pasajero de peso mg en un ascensor este peso es equilibrado por la reaccin que el piso ejerce sobre l, si el ascensor estuviera parado R = mg . Si el ascensor sube con aceleracin a. Cul es el peso de la persona? Solucin. La figura muestra el ascensor subiendo con una aceleracin a

Como se considera la cuerda con masa despreciable en el D.C.L. del trozo de cuerda La polea solo cambia la direccin de la tensin T . En el D.C.L .del hombre-silla

T=F

T + F W = 0 2F = W W y F = 2 Como W = 100 10 = 1000 N


9

Dinmica de una partcula

Hugo Medina Guzmn

F=

1000 = 500 N 2

b) Ahora como el hombre debe subir con una aceleracin de l m/s2 tenemos:

W W a 2F = W + a g g W a y F = 1+ 2 g T + F W =
Como W = 1000 N ,

T1 m1 g = m1 a T1 = m1 (a + g ) T1 = 1100(2 + 9,8)
= 12980 N b) Consideremos el D.C.L. de la masa m 2 :

Aplicando la Segunda Ley de Newton

a = 1 m/s 2 y = 1 m/s 2

F=

1000 1 1 + = 550 N 2 10
Aplicando La Segunda Ley de Newton

Ejemplo 12. La figura muestra un ascensor. Este consiste de la caja con masa m1 = 1100 kg , el contrapeso con masa m 2 = 1000 kg . El cable y poleas con masa y friccin despreciables. Cuando el ascensor tiene una aceleracin hacia arriba de 2 m/s2, el contrapeso tiene igual aceleracin pero hacia abajo. a) Cul es el valor de la tensin T1 ? b) Cul es el valor de la tensin T2 ? c) Cul es la fuerza ejercida por el motor sobre el cable?

m1 g T2 = m2 a T2 = m2 ( g a ) T2 = 1000(9,8 2)

= 7800 N c) En el motor Fuerza ejercida por el motor (T1 y T2 pueden considerarse colineales)

FM = T1 T2 = 12980 7800
= 5180 N Ejemplo 13. Demostracin de la tercera ley de Newton mediante el uso de la segunda ley. Se tienen dos cuerpos de masas m1 y m 2 los cuales son empujados sobre un plano sin friccin por una fuerza de magnitud P . Demostrar que aqu se cumple la tercera ley de Newton.

Solucin. a) Consideremos el D.C.L de la masa m1 :

Solucin.

10

Dinmica de una partcula Asumiremos que no hay friccin entre las superficies de contacto de m1 y m 2 . La figura muestra los D.C.L. para los bloques 1, 2 y para el sistema.

Hugo Medina Guzmn a) el valor y sentido de la velocidad del carrito, b) el lugar, donde encontrar c) el desplazamiento del carrito d) el recorrido total del carrito. (Usar g = 9,8 m/s2)

Solucin.

N 1 y N 2 son las fuerzas ejercidas por el plano. F21 es la fuerza que el bloque 2 ejerce sobre el
bloque 1. F12 es la fuerza que el bloque 1 ejerce sobre el bloque 2. La fuerza P solo acta sobre el bloque 1, ya que solo est en contacto con l. Como asumimos que no hay friccin entre los bloques, las fuerzas son normales a la superficie de contacto. Para el bloque 1 tenemos: P F21 = m1 a1x y N 1 m1 g = 0 Similarmente para el bloque 2 F12 = m2 a 2 x y N 2 m2 g = 0 Para el sistema

Para la masa M:

T = Ma

(1) (2)

Para la masa m:

P = (m1 + m2 )a x y N 1 + N 2 (m1 + m2 )g = 0

T mg = ma

Sumando (1) y (2)

En este caso no nos interesan las ecuaciones en y pero si las ecuaciones en x. Como los bloques se mueven juntos:

mg = (M + m )a m 0,2 (9,8) = - 2,8 m/s2 a= g = (M + m ) 0,7

a1x = a 2 x = a x
Sumamos la ecuacin para el bloque 1 con la ecuacin para el bloque 2. P F21 + F12 = m1 a1x + m2 a 2 x = (m1 + m2 )a x Comparando con la ecuacin para el sistema tenemos:

La aceleracin es en sentido contrario al indicado en la figura. a) La velocidad inicial del carrito es v0 = 7 m/s y su aceleracin es a = - 2,8m/s2. De las ecuaciones de cinemtica

x = v0 t +
Hallamos:

P F21 + F12 = P
Esto dice que la magnitud de la fuerza de 1 sobre 2 es igual a la fuerza de2 sobre 1. Como ellas son opuestas resulta ser precisamente la tercera ley de Newton. F21 = F12 , Accin y reaccin. Ejemplo 14.. Un carrito de masa M = 500 gramos est unido a una carga de masa m = 200 gramos mediante una cuerda. En el momento inicial el carrito tenia la velocidad inicial v0 = 7 m/s y se mova a la derecha por un plano horizontal. Determinar para t = 5 s: 11

1 2 at , v = v0 + at , 2

x = 7t 1,4t 2 , v = 7 2,8t
Dentro de 5 s el carrito tendr una velocidad v = - 7 m/s (dirigida a la izquierda).
2

b) x = 7(5) 1,4(5) = 35 35 = 0 El carrito se encontrar en la posicin inicial. c) El desplazamiento es cero. d) El carrito se detiene cuando v = 0 e inicia el camino de vuelta.

Dinmica de una partcula

Hugo Medina Guzmn

0 = 7 2,8t t =

Recorrido total s = 2 7(2,5) 1,4(2,5) = 17,5 m Recorrer un trayecto igual a 17,5 m.

7 = 2,5 s 2,8

a=
2

(F F )
fk

Si incrementamos la fuerza F, punto C, la fuerza neta sobre el bloque F F fk se incrementa y tambin se incrementa la aceleracin. Observacin. Encontramos que con fuerzas menores que 10 N no se produce movimiento. Con 10 N el bloque comienza a moverse. Para fuerzas mayores a 10 N el bloque se acelera. Si medimos la aceleracin podemos conocer la fuerza resultante sobre el bloque aplicando la segunda ley de Newton, F = ma . Cuando el dinammetro indica 12 N la fuerza resultante a partir de la aceleracin medida es 4 N, esto significa que se necesita 12 N 4 N = 8 N, para vencer la fuerza de friccin Si aplicamos 10 N al bloque para que inicie el movimiento, despus de esto es posible reducir la fuerza a 8 N y an mantener el bloque en movimiento. En resumen: Una fuerza de 10 N inicia el movimiento del bloque. Una fuerza de 8 N mantiene el movimiento del bloque. El origen de este fenmeno se debe a la existencia de fuerzas entre las molculas del cuerpo y la superficie; si la superficie de contacto del cuerpo con la superficie fuera perfectamente plana, la fuerza de atraccin podra ser considerable, como es el caso de dos placas de vidrio perfectamente limpias que una vez puestas en contacto, difcilmente pueden ser separadas. Las superficies nunca son perfectamente lisas y las imperfecciones constituyen verdaderos obstculos al desplazamiento como se muestra en la figura. Es preciso vencer estos obstculos para iniciar el movimiento y tambin para mantenerlo.

FRICCIN Cuando un cuerpo sobre una superficie se empuja o se jala ste puede permanecer inmvil, esto sucede porque la fuerza aplicada no ha sido suficiente para vencer la fuerza de friccin. Cuando lograrnos que el cuerpo deslice sobre la superficie es necesario aplicar una fuerza para que ste contine en movimiento. Comportamiento de un cuerpo que descansa sobre un plano horizontal Supongamos que jalamos un bloque con un dinammetro, como se muestra en la figura. Comportamiento de un cuerpo que descansa sobre un plano horizontal

Dibujemos una grfica de la fuerza el bloque versus el tiempo t .

F aplicada sobre

1. Desde el origen hasta el punto A la fuerza F aplicada sobre el bloque no es suficientemente grande como para moverlo. Estamos en una situacin de equilibrio esttico

F = Ffs = s N

En el punto A, la fuerza de rozamiento su mximo valor

F fs alcanza
A esta fuerza se le conoce como FUERZA DE FRICCION O ROZAMIENTO F f .

smx N

F = Ffsmx = smx N
2. Si la fuerza F aplicada se incrementa un poquito ms, el bloque comienza a moverse. La fuerza de rozamiento disminuye rpidamente a un valor menor e igual a la fuerza de rozamiento dinmico,

( )

F = F fk = k N

Con la finalidad de conocer la dependencia de esta fuerza de rozamiento realicemos la siguiente experiencia. Supongamos un plano inclinado con un bloque de masa ni descansando sobre l.

Si la fuerza F no cambia, punto B, y permanece igual a F fsmx , el bloque comienza movindose con una aceleracin

12

Dinmica de una partcula

Hugo Medina Guzmn intermoleculares son tanto mayores, cuanto mayor es la superficie de contacto. En realidad se deba esperar que F f fuera proporcional a la superficie, lo que suceder es que si el cuerpo pesa muy poco, prcticamente no hay puntos de contacto entre las dos superficies (el rea de contacto es despreciable). Cuando N aumenta, la superficie aumenta y F f

Encontramos que el bloque empieza a resbalar para un determinado ngulo . Si colocamos dos bloques juntos, el ngulo con el cual inician el movimiento sigue siendo , lo mismo ocurre con tres bloques. La fuerza que jala al cuerpo es la componente del peso mgsen , paralela al plano. La otra componente es perpendicular al plano mg cos . Esta es la fuerza que sostiene al bloque sobre la superficie (Fuerza Normal). Si duplicarnos el peso mg a 2mg, duplicamos la fuerza que jale al bloque y la fuerza normal tal que:

tambin, por lo tanto

F f = N donde se est

incluyendo ya el aumento de superficie. Es decir, la fuerza de friccin F f es proporcional a la fuerza normal N porque la verdadera superficie de contacto es proporcional a la fuerza normal. Ejemplo 15. Cul es la fuerza mnima F necesaria para mover la masa m , siendo el coeficiente de rozamiento esttico entre el piso y el bloque en cada uno de los casos siguientes?

Fuerza que inicia el movimiento = Constante Fuerza normal


O

mg sen = tan = s = Constante mg cos Ff = s N

Solucin. a) La figura muestra el D.C.L.

A esta constante

se le llama coeficiente de

friccin esttica. Si se toman los datos con el bloque en movimiento, el ngulo para que el movimiento contine es generalmente menor y obtenemos

Fuerza para continuar el movimiento = k Fuerza normal


A esta constante se le llama coeficiente de friccin cintica k . es una constante que depende de la superficie y se puede escribir simplemente. F f = N . Algunos valores tpicos de coeficientes de friccin. Material Acero Cuero Cuero Bronce Aluminio Vidrio Caucho Caucho Caucho Piedra Sobre material Acero Cuero Roble Hierro Aluminio Vidrio Asfalto Concreto Hielo Piedra

F F

: N mg = 0 N = mg : F N = 0 F = N

Luego:

F = mg

b) La figura muestra el D.C.L.

s
0,78 0,64 0,60 0,40 1,05 0,92 0,60 0,80 0,02 0,65

k
0,42 0,56 0,50 0,30 1,40 0,40 0,40 0,70 0,005 0,60

El hecho que la fuerza de friccin es independiente del rea de contacto parece absurdo ya que las fuerzas 13

N = mg Fsen Fx : F cos N = 0 F cos = N


De estas dos ecuaciones obtenemos:

: N + Fsen mg = 0

Dinmica de una partcula

Hugo Medina Guzmn une los bloques m1 y m2. El coeficiente de rozamiento entre los bloques y el plano inclinado es .

F=

mg cos + sen

c) La figura muestra el D.C.L.

N = mg + Fsen Fx : F cos N = 0 F cos = N F=


De estas dos ecuaciones obtenemos:

: N Fsen mg = 0

Solucin.

mg cos sen

Ejemplo 16. Cul es el valor mnimo de F para sostener el bloque de masa m sobre una pared vertical, como se muestra en la figura, es el coeficiente de friccin esttico entre la pared y el bloque?

Para

m0 : {m0 g T1 = m0 a
T1 T2 N 2 = m2 a N 2 m2 g = 0

Para m 2 : Para m1 :

T2 N 1 = m1 a N 1 m1 g = 0

De estas ecuaciones obtenemos: N 2 = m2 g , N 1 = m1 g y Solucin. La figura siguiente muestra el D.C.L. De aqu:

m0 g (m1 + m2 )g = (m0 + m1 + m2 )a

a=

[m0 (m1 + m2 )] g (m0 + m1 + m2 )


m1 m0 (1 + )g (m0 + m1 + m2 )

La tensin del cable que une los bloques m1 y m2:

T2 = m1 (a + g ) =

Ejemplo 18. Se tiene una masa m 2 sobre una masa

Fy : N F = 0 N = F F
F=
x

: N mg = 0 N =

mg

Por consiguiente

mg

m1 sobre un piso horizontal, tal como muestra la figura. Se aplica una fuerza horizontal F sobre la masa m1 . La masa carece de friccin. Cul es el valor mximo de F para que la masa m1 no resbale sobre m 2 . Cul es la aceleracin resultante de los
bloques?

Ejemplo 17. En el esquema de la figura las masas de la polea y del cable son despreciables y no hay rozamiento entre el cable y la polea. Hallar la aceleracin del bloque m0 y la tensin del cable que

14

Dinmica de una partcula

Hugo Medina Guzmn

Solucin. La figura muestra el D.C.L. de las masas m1 y m 2 .

Solucin. La figura muestra el D.C.L. para este caso

Aplicando la Segunda Ley de Newton a la masa m 2 , la que suponemos se mueve con aceleracin a 2 .

F F

: N 2 m2 g = 0 : N 2 = m2 a 2

Las ecuaciones para la masa m 2 son

Aplicando la Segunda Ley de Newton a la masa m1 , la que suponemos se mueve con aceleracin a1 .

F F F F

: N 2 m2 g = 0 : F N 2 = m2 a 2 : N 1 N 2 m1 g = 0 : N 2 = m1 a1

F F

: N 1 N 2 m1 g = 0 : F N 2 = m1 a1

Las ecuaciones para la masa m1 son.


y

Trabajando con estas ecuaciones encontramos que

F = m1 a1 + m2 a 2

Trabajando con estas ecuaciones encontramos que

N 2 m2 g = = g m2 m2 Como el valor de vara desde 0 hasta el valor mximo mx : a 2 = mx g o simplemente a 2 = g .


a2 =
Pero como queremos encontrar el valor mximo posible de F para que las masas vayan juntas, es decir, para que m1 no se quede, se tiene como condicin que;

La aceleracin de la masa m 2 es:

F = m1 a1 + m2 a 2 m N 2 m2 g = = g 2 m1 m1 m1 Como el valor de vara desde 0 hasta el valor mximo mx : a1 =


a1 = mx g m2 m1
La aceleracin de la masa m1 es:

a1 = a 2 = g Luego: Fmx = (m1 + m 2 ) mx g

Como la condicin de que las masas m1 y m 2 vayan juntas es,

a1 = a 2
Luego el valor mximo de F pera que m1 y m 2 vayan juntas es,

Si aplicamos una fuerza mayor el bloque m1 avanzar dejando atrs al bloque m 2 . Ejemplo 19. Usando el dispositivo del ejemplo anterior discuta el caso en ci que la fuerza F se aplica a la masa m 2 .

Fmx =

(m1 + m2 )m2
m1

mx g

Ejemplo 20. En el dispositivo de la figura encontramos el valor mnimo de F para sacar la masa m1 . El coeficiente de friccin entre m1 y la mesa es el coeficiente de friccin entre m1 y m 2 es 15

2 .

Dinmica de una partcula

Hugo Medina Guzmn

Solucin. La figura muestra los D.C.L. de las masas m1 y m 2

Solucin. La figura muestra el D.C.L.de las masas m1 y m 2

Considerando que el equilibrio es la condicin mnima de inicio de movimiento Aplicando la Segunda ley de Newton para la masa m2 .

Considerando que el equilibrio es la condicin mnima de inicio del movimiento. Aplicando la segunda ley de Newton a la masa m 2 :

F F
F F
m1

: N 2 m2 g = 0 : 2 N2 T = 0

F F
F F

: N 2 m2 g = 0 : 2 N2 T = 0

Aplicando la Segunda Ley de Newton para la masa

Aplicando la segunda ley de Newton para la masa m1 :


y

: N 2 N 1 + m1 g = 0 : F 1 N1 2 N 2 T = 0

: N 2 N 1 + m1 g = 0 : F 1 N 1 2 N 2 = 0

Resolviendo estas ecuaciones

Resolviendo estas ecuaciones

N 2 = m2 g T = 2 N 2 = 2 m2 g N 1 = N 2 + m1 g = (m1 + m2 )g F = 1 N 1 + 2 N 2 = 1 (m1 + m 2 )g + 2 m2 g
Siendo este valor de F el mnimo para iniciar el movimiento de la masa m1 . Ejemplo 21. En el dispositivo de la figura, encontrar el valor mnimo de F para sacar la masa m1 . El coeficiente de friccin entre m1 y la mesa es coeficiente de friccin entre m1 y m 2 es

N 2 = m2 g T = 2 N 2 = 2 m2 g N 1 = N 2 + m1 g = (m1 + m2 )g F = 1 N1 + 2 N 2 + T = 1 (m1 + m2 )g + 2 m2 g = [1 m1 + m2 (1 + 2 )]g


Siendo este valor de F el mnimo para iniciar el movimiento. Ejemplo 22. Los bloques m1 y m 2 de 20 y 60 kg, respectivamente, estn unidos por una cuerda de masa despreciable que pasa por una polea sin rozamiento. El coeficiente de rozamiento cintico entre las masas y la superficie es 0,3. Determinar la velocidad del sistema 4 segundos despus de partir del reposo.

2 .

1 ,

el

16

Dinmica de una partcula

Hugo Medina Guzmn Ejemplo 23. En una mesa un plato descansa sobre el mantel, cuyo centro est a 0,25m del borde de la mesa. El mantel se jala sbitamente en forma horizontal con una aceleracin constante de 10 m/s2. El coeficiente de friccin cintico entre el mantel y el plato es k = 0,75 . Asumiendo que el mantel llega justo al borde de la mesa. Cuando el extremo del mantel pasa bajo el centro del plato, encontrar: a) La aceleracin del plato b) La velocidad de! plato c) La distancia del plato al borde de la mesa. Solucin. a) Aplicando la segunda ley de Newton para el plato, la masa del plato es m y su aceleracin a p .

Solucin. La figura muestra el D.C.L. de la masa m1 . Consideremos que el movimiento es de izquierda a derecha con aceleracin a

F F

: N 1 m1 g cos 30 = 0 : T F f 1 m1 gsen30 = m1 a

De estas ecuaciones

3 = 173 N 2 F f 1 = N 1 = 0,3 173 = 51,9 N


N 1 = m1 g cos 30 = 20 10
y

T 51,9 20 10

T = 151,9 + 20a

1 = 20a 2

La figura muestra D.C.L. de la masa m 2 .

F F

V H

= 0 mg N = 0 = ma p F f = ma p

De aqu obtenemos: N = mg y k mg De donde:

= ma p

a p = k g = 0,75 x 9,8 = 7,35 m/s2 a p es menor que 10 m/s2

El plato resbala ya que

F F
Ff 2
y

: N 2 m2 g cos 60 = 0 : m2 gsen 60 F f 2 T = m2 a

b) En el instante en que el extremo del mantel coincide con el centro del plato estn a la misma distancia del borde de la mesa

De estas ecuaciones

N 2 = m2 g cos 60 = 20 10

1 = 150 N 2 = N 2 = 0,3 150 = 45 N


x p = xm 1 1 a p t 2 = 0,25 + 7,35t 2 2 2 1 1 x m = a m t 2 = 10t 2 2 2 x p = 0,25 +
Igualando
2

3 45 T = 30a 2 T = 214,5 30a 30 10 m s2

Igualando los valores de T:

151,9 + 20a = 214,5 30a a = 1,25


Como Siendo

v = v0 + at , v0 = 0 v = 1,25t v = 1,25 4 = 5 m s

Para t = 4 s

1 1 0,25 + 7,35t 2 = 10t 2 2 2


Resolviendo:

t = 0,58 s y

17

Dinmica de una partcula

Hugo Medina Guzmn horizontal con coeficiente de friccin . La polea por donde cuelga otro bloque de masa M no tiene roce y la cuerda se considera inextensible y de masa despreciable. Calcular la aceleracin y la tensin de la cuerda.

v p = v0 + a p t = 0 + 7,35 0,58
= 4,26 m/s. c)

x p = 0,25 +

1 a pt 2 2 1 2 = 0,25 + 7,35 0,58 = 1,49 m 2

Ejemplo 24. El plano inclinado mostrado en la figura tiene una aceleracin a hacia la derecha. Si el coeficiente de friccin esttico entre el plano y el bloque es , encontrar la condicin para que el bloque resbale.

Solucin. Se hacen los DCL y se aplica la segunda ley de Newton, suponiendo que el cuerpo de masa M desciende y tira a m hacia la derecha, lo que define el sentido de la aceleracin. Para m

Solucin. Consideremos que el bloque tiene masa m , la figura a continuacin muestra su DCL.

N = mg F sen y FH = ma
Para que el bloque no resbale debe tener la misma aceleracin a . Aplicando la segunda ley de Newton
V

= 0 N + F sen mg = 0
(1)

T F cos F f = ma (2)

Para M

F = 0 N cos + N sen mg = 0 y F = ma N sen + N cos = ma


H

De estas ecuaciones

N=

mg y cos + sen mg ( sen + cos ) = ma ((cos + sen ))

Finalmente

a=

Este es el va1or crtico de a para que no resbale; el bloque resbalar para valores menores que el indicado. Ejemplo 25. En el siguiente sistema mecnico, se aplica una fuerza F inclinada un ngulo sobre el cuerpo de masa m, ubicado sobre la superficie 18

( cos sen ) g (cos + sen )

T Mg = Ma Adems: F f = N
De la ecuacin (1):

= Ma
(3)

F f = (mg Fsen ) (4)


(5)

De (3) se despeja T:

T = Mg Ma

Ahora 4) y (5) se reemplazan en (2), lo que permite despejar la aceleracin

Dinmica de una partcula

Hugo Medina Guzmn deslizarse sobre la viga? Dentro do cunto tiempo el cuerpo caer de la viga? La longitud do la viga es l .

Mg Ma F cos (mg Fsen ) = ma (M m )g F (cos sen ) a= M +m


y la tensin T

T = Mg M

(M m )g F (cos sen )
M +m
Solucin.

Ejemplo 26. Una viga de masa M est situada en un plano horizontal. Sobre la viga se encuentra un cuerpo do masa m. El coeficiente de rozamiento entre el cuerpo y la viga, as como entre la viga y el plano es k . Analizar el movimiento para diferentes valores do la fuerza F.

Las ecuaciones del movimiento de la viga y del cuerpo tienen la siguiente forma: F fm = ma m , (1)

F k mg = Ma M (2) Donde F fm es la fuerza do rozamiento, am y aM son


Solucin. Si F k (m + M )g , no hay movimiento. Supongamos que las aceleraciones. Supongamos que no hay deslizamiento, entonces am = aM De las ecuaciones del movimiento podemos determinar la aceleracin y la fuerza de rozamiento. La fuerza de rozamiento es

F > k (m + M )g . Analicemos

el caso de ausencia de deslizamiento del cuerpo por la viga. Las ecuaciones del movimiento, en este caso, tendran la siguiente forma: F fm = ma ,

F fm =

mF (m + M )

Ma = F F fm F fM = F F fm k (m + M )g ;

Para que no haya deslizamiento la fuerza de rozamiento debe satisfacer la siguiente desigualdad:

F fm k mg
de donde

F fm k mg , es decir,

F g. (m + M ) k

F a= g, (m + M ) k mF F fm = mg k mg (m + M ) k
que es posible, si k (m + M) g < F < 2k (m + M) g. Si F > 2k(m + M)g, entonces el cuerpo deslizar por la barra. En este caso las ecuaciones del movimiento tendrn la siguiente forma: ma m = k mg ,

Si F > k (M + m) g, entonces surge el deslizamiento. Las ecuaciones (1) y (2) en este caso deben escribirse en la siguiente forma: ma m = k mg , Ma M = F k mg De estas ecuaciones obtenemos am y aM:

am = k g , aM =

(F k mg )
M

Es evidente que aM > am.

Ma M = F k mg k (M + m )g

de donde

(2m + M ) g F k M M Que es fcilmente verificar en el caso de a M > a m


am = k g , aM =

Ejemplo 27. Una viga do masa M est sobre un plano horizontal liso, por el cual puede moverse sin friccin. Sobre la viga hay un cuerpo do masa m. El coeficiente de rozamiento entre el cuerpo y la viga es k . Con qu valor de la fuerza F que acta sobre la viga en direccin horizontal, el cuerpo comienza a 19

1 1 am t 2 , xM = aM t 2 2 2 1 1 xM xm = l = a M t 2 am t 2 2 2 2l 2l t= = (F k mg ) aM am k g M 2lM = F k g (M + m ) xm =

Dinmica de una partcula Ejemplo 28. En la figura, encontrar la aceleracin del carro requerida para evitar que caiga el bloque B. El coeficiente de rozamiento esttico entre el bloque y el carro es k .

Hugo Medina Guzmn

Solucin. Si el bloque no cae, la fuerza de friccin, Ff, debe balancear el peso del bloque: Ff = mg. Pero el movimiento horizontal del bloque est dado por y N = ma. Luego,

F Cuerpo 2: F F Mesa: F
Cuerpo 1:

verticales

= m1 g T = m1 a = T = m2 a

horizontales

verticales

= N 3 N 2 T m3 g = 0 = T Ff 3 = 0

horizontales

Ff N

g g a= Ff a

Donde N3 y Ff3 (friccin) las componentes verticales y horizontales de la fuerza ejercida por el piso sobre la mesa. (Asumimos que las patas de la izquierda y de la derecha comparten la carga igualmente. Esto no afecta nuestro anlisis) De las primeras dos ecuaciones,

N Ff N
es

a=

m1 g (m1 + m2 ) m1 m2 g (m1 + m2 )

Como el valor mximo de tener a

s , debemos

Luego, F f 3 = T = m 2 a = Finalmente,

si el bloque no cae.

N 3 = T + m2 g + m3 g
=

Ejemplo 29. Dos cuerpos, de las masas m1 y m2, se liberan de la posicin mostrada en la figura. Si la masa de la mesa de superficie lisa (sin friccin) es m3, encuentre la reaccin del piso sobre la mesa mientras los dos cuerpos estn en movimiento. Asuma que la mesa permanence inmvil.

m1m2 + m2 + m3 (m1 + m2 )

Ejemplo 30. Se tiene un bloque de 20 kg sobre un plano inclinado que est sujeto a una cuerda (ver figura). Las superficies de contacto entre el bloque y el plano inclinado son rugosas con coeficiente de friccin cintica k = 0,5 y el de friccin esttica s = 0,7. a) Si la tensin de la cuerda es de 150 N, determine la magnitud y sentido de la fuerza de rozamiento. b) Si por un accidente se corta la cuerda, determine la aceleracin del bloque.

Solucin. La figura muestra los diagramas de cuerpo libre de cada uno de los elementos.

Solucin. a)

20

Dinmica de una partcula

Hugo Medina Guzmn

T mgsen30 F f = 0 F f = T mgse30 = 150 100 = 50 N


en el sentido indicado en la figura (hacia abajo). b) La segunda ley de Newton para m1 es T m1 a = 0 , N 1 m1 g = 0 (2) De aqu T = m1 a La segunda ley de Newton para m2 es N 2 m2 a = 0 , T m2 g = 0 De aqu T = m2 g De (2) y (3) se tiene Cuando se rompe la cuerda para iniciar el movimiento debe vencerse a la mxima fuerza de friccin esttica: (3)

a=

m2 g m1

(4)

3 F fs = s mg cos 30 = 0,7 20 g 2 = 173 N


Como 20g sen 30 = 100 N 100 N < 173 N, el movimiento no se inicia , por lo tanto la aceleracin del bloque es cero. Ejemplo 31. Determinar la fuerza F aplicada al bloque de masa M de la figura adjunta, para que los bloques de masas m1 y m2 apoyados en M, no se muevan respecto de M. Todas las superficies son lisas, la polea y el cable tienen masa despreciable.

Sustituyendo (4) en (1) se obtiene la fuerza aplicada a M

F=

m2 (M + m1 + m2 )g m1

Ejemplo 32. Determinar la aceleracin mnima con que debe desplazarse el bloque de masa M en sentido horizontal para que los bloques de masas m1 y m2 no se muevan respecto de M, siendo el coeficiente de rozamiento entre los bloques. La polea y el cable tienen masa despreciable.

Solucin. Consideremos un sistema de referencia fijo en el suelo con el eje x paralelo a la fuerza aplicada

Solucin. Consideremos un sistema de referencia fijo en el suelo con el eje x paralelo a la fuerza aplicada De la segunda ley de Newton aplicada al conjunto se tiene:

F.

F. De la primera ley de Newton aplicada al conjunto se tiene: F = (M + m1 + m2 ) a


F = (M + m1 + m2 ) a

(1)

(1)

Siendo a la aceleracin del conjunto. Las masas m1 y m2 estn en reposo sobre el bloque M, luego en la referencia O su aceleracin es del conjunto. La fuerza que ejerce el cable sobre m1 y la que ejerce sobre m2 tiene el mismo mdulo T.

Siendo a la aceleracin del conjunto. Las masas m1 y m2 estn en reposo sobre el bloque M, luego en la referencia O su aceleracin es del conjunto. La fuerza que ejerce el cable sobre m1 y la que ejerce sobre m2 tiene el mismo mdulo T.

21

Dinmica de una partcula

Hugo Medina Guzmn

Diagrama del cuerpo libre masas separadas

La segunda ley de Newton para m1 es T m1 a F f 1 = 0 , N1 m1 g = 0

F f 1 = N 1 = m1 g
(2) La segunda ley de Newton para m2 es N 2 m2 a = o , T + F f 2 m2 g = 0

T = m1 a + m1 g

a) Consideremos un sistema de referencia fijo en el suelo con el eje x paralelo a la fuerza aplicada F . Sea el instante en que m empieza a deslizar sobre M. Hasta dicho instante t , el conjunto se mueve con una aceleracin comn a . La segunda ley de Newton aplicada al conjunto en el instante t = es k = (M + m )a( ) , N 2 (M + m )g = 0

F f 2 = N 2 = m2 a T = m2 g m2 g De (2) y (3) se tiene m1 a + m1 g = m2 g m2 g (m m1 ) a= 2 g (m1 + m2 )


m2 (M + m1 + m2 )g m1
(3)

(4)

a ( ) =

Sustituyendo (4) en (1) se obtiene la fuerza aplicada a M

k (M + m )

(1)

F=

La segunda ley de Newton aplicada a la masa m en el instante t = es, ( la fuerza de rozamiento sobre m tiene, en ese instante, su valor mximo Ff = m g ) F f = N 1 = ma( ) , N 1 = mg

Ejemplo 33. Un bloque de masa m se encuentra sobre otro bloque de masa M que est apoyado sobre una superficie horizontal lisa. El coeficiente de rozamiento entre los dos bloques es . Al bloque M se le aplica una fuerza horizontal dirigida hacia la derecha que depende del tiempo segn la ley F = k t. Determinar: a) El instante en que m empieza a deslizar sobre M. b) La aceleracin de cada uno de los bloques.

a( ) =

mg
m

= g

(2)

De (1) y (2) queda

(M + m ) g
k

b) De (1) se tiene que la aceleracin del conjunto para t < es

a1(t ) = a(t ) =

k t (M + m )

Para t > . Las fuerzas que actan sobre m son constantes, luego la aceleracin de m es

a1 = a( ) = g

La segunda ley de Newton aplicada a la masa M es kt F f = kt N 1 = Ma 2 (t ) , como N 1 = mg Solucin. Diagrama del cuerpo libre del conjunto

kt mg = Ma 2(t ) y a2(t ) = g m m k + t 2 M M s

Grfica de las aceleraciones en funcin del tiempo

22

Dinmica de una partcula

Hugo Medina Guzmn

El movimiento de B es hacia arriba, luego

mB g < T
El movimiento de A es hacia abajo, luego T + m A cos < m A gsen El movimiento de los bloques es el indicado si m B g < m A gsen m A cos Ejemplo 34. Dos bloques A y B de masas mA y mB estn unidos mediante un cable que pasa a travs de una polea tal como se muestra en la figura adjunta. El coeficiente de rozamiento entre el bloque A y el plano inclinado es . Determinar el sentido del movimiento cuando se dejan en libertad a partir del reposo. El cable es inextensible y las masas del cable y la polea despreciables.

mB < sen cos mA

Los bloques no se mueven si

mB < sen + cos mA Ejemplo 35. Dos bloques A y B de masas m A = 10 kg y m B = 7 kg, estn unidos mediante un cable que

sen cos <

pasa a travs de las poleas tal como se muestra en la figura adjunta. El coeficiente de rozamiento entre el bloque A y el plano inclinado es = 0,10 y = 30. El cable es inextensible y las masas del cable y las poleas son despreciables. Determinar: a) Las aceleraciones de los bloques; b) La tensin del cable. Solucin. Supongamos que el bloque A sube sobre el plano inclinado. Sea T la fuerza que ejercen los extremos del cable sobre los bloques dirigida, en ambos bloques, tal como se indica.

Solucin.

El movimiento de B es hacia abajo, luego mB g > T El movimiento de A es hacia arriba, luego T > m A gsen + m A cos El movimiento de los bloques es el indicado si m B g > m A gsen + m A cos

Supongamos que el movimiento de A es hacia abajo, luego:

m B > sen + cos mA


Supongamos que el bloque A desciende sobre el plano inclinado.

T + m A g cos < m A gsen T < m A gsen m A g cos m B g < 2T

El movimiento de B es hacia arriba, luego: De ambas expresiones queda

1 m B g < m A gsen m A g cos 2 1 (7 ) < (10)sen30 0,10(10) cos 30 2 Con los valores 3,5 < 4,13
Desigualdad que se cumple, luego el movimiento es el previsto. 23

Dinmica de una partcula

Hugo Medina Guzmn

a) Consideremos un sistema de referencia con el eje x horizontal. Las posiciones de los bloques estn relacionadas por la condicin de ligadura s A + 2 y B = constante , Luego sus aceleraciones cumplen

1 a A + 2a B = 0 a B = a A = a 2

(1)

Fuerzas sobre los bloques La segunda ley de Newton aplicada al bloque A es m A a A = m A gsen T N A ,

N A m A g cos = 0

Supongamos que el bloque A asciende por el plano inclinado. Consideremos un sistema de referencia con el eje x horizontal. Las posiciones, por una parte, del bloque A y de la polea mvil, estn relacionadas por las condiciones de ligadura

De estas dos obtenemos:

(2) La segunda ley de Newton aplicada al bloque B es

T = m A g (sen cos ) m A a A
2T m B g = m B a B 1 T = m B (a B + g ) 2

s A + h y p = constante
Las posiciones de la polea y el bloque B, estn relacionadas por las condiciones de ligadura

2 y p y B = constante
De estas dos ecuaciones obtenemos:

(3)

2 s A + 2h y B = constante
Las componentes de las aceleraciones de los bloques satisfacen la condicin 2a A = a B (1)

Igualando las ecuaciones (2) y (3),

m B (a B + g ) = 2m A g (sen cos ) 2m A a A

Teniendo en cuenta la ecuacin (1) y los valores: Resolviendo: a = 0,26 m/s2 Las aceleraciones de los bloques son :

7(a + 9,8) = 2(10 )(9,8)(0,5 0,1 0,87 ) 20(2a )

a A = 0,26 m/s 2 para arriba. a B = 0,52 m/s 2 para abajo.


b) La magnitud de la tensin del cable es el valor de la fuerza que el cable ejerce sobre los bloques. De la ecuacin (3) se tiene Sean T A y TB las fuerzas que los cables ejercen sobre los respectivos bloques. Fuerzas sobre los bloques y sobre la polea mvil. Como la polea superior tiene masa despreciable solo cambia el sentido de la fuerza. La masa de la polea mvil es cero, luego La tensin en ambos lados son iguales (TB ) y

T=

1 (7 )(0,26 + 9,8) = 35,2 N 2

Ejemplo 36. Dos bloques A y B de masas m A y

m B estn unidos mediante un cable que pasa a travs de las poleas tal como se muestra en la figura adjunta. El coeficiente de rozamiento entre el bloque A y el plano inclinado es . El cable es inextensible y las masas del cable y la polea son despreciables. Estudiar el sentido del movimiento de los bloques.

T A = 2TB (2) De la segunda ley de Newton aplicada al bloque A se tiene: TA mA gsen N A = mAa A N A m A g cos = 0

T A = m A g (sen + cos ) + m A a A (3) De la segunda ley de Newton aplicada al bloque B se tiene


(4) De las ecuaciones (1), (2), (3) y (4) obtenemos:

De estas ecuaciones obtenemos:

m B g TB = m B a B TB = m B ( g a B )

Solucin.

m A g (sen + cos ) + m A a A = 2m B ( g 2a A )

aA =

2m B g m A g (sen + cos ) m A + 4m B

24

Dinmica de una partcula El movimiento es el indicado, si se cumple:

Hugo Medina Guzmn

2m B > (sen + cos ) mA


El movimiento es de sentido opuesto, si se cumple:

Las fuerzas exteriores que actan sobre la m1 son la tensin del cable T y el peso m1 g , y sobre m 2 son la tensin del cable T y el peso m2 g . De la ecuacin fundamental de la dinmica en la referencia no inercial se tiene

2 mB < (sen cos ) mA


El signo menos es porque en este caso el peso de la masa A es el que mueve al sistema y la fuerza de rozamiento est en sentido contrario a ste. Ejemplo 37. A los extremos de un hilo que pasa a travs de una polea fija al techo de la cabina de un ascensor se atan los cuerpos de masa m1 y m 2

m1 a '1 = T m1 g m1

g 2
(1)

3 m1 a'1 = T m1 g 2
m1 a ' 2 = T m2 g m2 g 2

(m1 < m2 ) . La cabina comienza a subir con una

aceleracin constante g / 2. Despreciando la masa de la polea y la del hilo, as como el rozamiento, calcular: a) La aceleracin de m1 y m 2 respecto de la cabina y con relacin al foso del ascensor. b) La fuerza con la cual la polea acta sobre el techo de la cabina.

3 m1 a' 2 = T m2 g 2

(2)

De la condicin de ligadura para los bloques se tiene a '1 + a ' 2 = 0 a '1 = a ' 2 = a ' (3) De las ecuaciones (1), (2) y (3) se obtiene

3 3 m1 a' = T m1 g y m1 a' = T + m2 g 2 2
Sumando estas ecuaciones:

(m2 + m1 )a' = 3 (m2 m1 )g


2
Despejando a '

a' =

3 (m2 m1 ) g 2 (m2 + m1 )

Finalmente:

a'1 = a ' j y a' 2 = a' j


En la referencia fija, las aceleraciones de m1 y de Solucin. a) El ascensor constituye una referencia no inercial en traslacin que se mueve con una aceleracin constante en sentido ascendente respecto de una referencia fija. Seleccionemos una referencia con origen O en un punto del ascensor. La aceleracin del origen O respecto de la referencia fija O es la aceleracin

m 2 se obtienen de sumar a las anteriores la


aceleracin del ascensor

a1 =

(2m2 m1 ) g g y + a' = (m2 + m1 ) 2 (2m1 m2 ) g a 2 = a' = g (m2 + m1 ) 2

1 del ascensor g j la aceleracin de m1 j . Sean a '1 2 y a' 2 j la aceleracin de m2 en la referencia O.

b)

La fuerza que la polea ejerce sobre el techo de la cabina es

F 2T = 0 F = 2T

De la ecuacin (1) y (3) se tiene

3 3m1m2 T = m1 a '1 + g = g 2 (m2 + m1 )


25

Dinmica de una partcula Luego

Hugo Medina Guzmn

6m1 m2 F = 2T = (m2 + m1 )
Ejemplo 38. Un nio de masa m = 45 kg se pesa en una bscula de resorte situada sobre una plataforma especial que se desplaza por un plano inclinado de ngulo = 30 como muestra la figura (no hay rozamiento entre la plataforma y el plano inclinado). Cul ser la lectura de la bscula en estas condiciones?

F F

x y

= F f ma cos 30 = 0

(1)

= N mg + masen 30 = 0 (2) g 3 = 191N 2 2

de (1) F f = 45 de (2)

g N = mg masen30 = 45 g 4
= 33,45 Kg. Siendo N la cantidad que marca la bscula. Solucin en una referencia no inercial . Seleccionemos una referencia con origen O (x,y) en un punto de la plataforma. El nio est en reposo sobre la plataforma.

Solucin. Sea M la masa del conjunto nio - cua., y a la aceleracin con la que desliza hacia abajo el conjunto.

Aplicando la segunda ley de Newton al DCL del nio.

F F

= F f = ma cos 30 (1) = N mg = masen 30 (2) g 3 = 191N 2 2

Aplicando la segunda ley de Newton al conjunto nio - cua.

de (1) F f = 45 de (2)

//

= Ma Mgsen30 = Ma

a = gsen 30 =

g 2

g N = mg masen30 = 45 g 4

La aceleracin del conjunto es a =

1 g 2

= 33,45 kg Siendo N la cantidad que marca la bscula. Ejemplo 39. Un ascensor de masa total 3M es levantado bajo la accin de una fuerza F. El piso del ascensor est inclinado un ngulo , con respecto a la horizontal. Adems, un bloque de masa M se apoya sobre el centro del piso rugoso del ascensor (con coeficiente de friccin esttica ). a) Hallar la aceleracin del ascensor. b) Haga el diagrama de cuerpo libre de la masa M. c) Cul es el valor mximo de F para que el bloque dentro del ascensor no resbale respecto del piso del ascensor? d) Si el ascensor pierde contacto con la fuerza F y empieza a caer libremente, calcule el valor de la fuerza normal entre el bloque y el piso del ascensor, y la fuerza de friccin sobre el bloque. Solucin. a) Para hallar la aceleracin del ascensor. 26

Solucin en una referencia inercial. Sobre el nio actan: su peso mg y la reaccin Ff en el apoyo. La indicacin de la bscula el valor de la normal.

Aplicando la segunda ley de Newton al DCL del nio.

Dinmica de una partcula

Hugo Medina Guzmn Por ser proporcional a la aceleracin centrpeta, la fuerza Fc se llama fuerza centrpeta. Su efecto es cambiar la direccin de la velocidad de un cuerpo. Se puede sentir esta fuerza cuando se hace girar a un objeto atado a una cuerda, ya que se nota el tirn del objeto. Las fuerzas centrpetas no son diferentes de otras fuerzas ya conocidas, su nombre se debe a que apunta hacia el centro de una trayectoria circunferencial. Cualquiera de las fuerzas ya conocida pueden actuar como fuerza centrpeta si producen el efecto correspondiente, como ser la tensin de una cuerda, una fuerza de roce, alguna componente de la normal, la fuerza gravitacional en el caso de movimientos de planetas y satlites, etc. Ejemplo 40. Un cuerpo de masa m, sujeto al extremo de una cuerda de longitud L, que describe una trayectoria circular en el plano horizontal, genera una superficie cnica, por lo que se llama pndulo cnico. Determinar la rapidez y el perodo de revolucin de la masa.

F 3Mg Mg = (3M + M )a F 4Mg F = g a= 4M 4M


b) Diagrama de cuerpo libre de la masa M.

c) Para que el bloque dentro del ascensor no resbale respecto del piso del ascensor se debe cumplir

M ( g + a )sen M ( g + a ) cos tan .


Como a depende de F, y a esta en miembros de la igualdad, el que el bloque resbale dentro del ascensor solamente depende del coeficiente de friccin. d) Si el ascensor pierde contacto con la fuerza F y empieza a caer libremente, N = 0, por lo tanto Ff = 0 DINMICA DEL MOVIMIENTO CIRCULAR La primera ley de Newton dice que un objeto permanecer en movimiento uniforme en lnea recta con velocidad constante o en reposo si no acta una tuerza sobre l. Entonces cuando un objeto se mueve en trayectoria circular, debe haber una fuerza sobre l cambindole la trayectoria recta. Esta fuerza puede ser proporcionada por la tensin en una cuerda, para un objeto que se hace girar en una circunferencia horizontal al extremo de una cuerda; por la fuerza de la gravedad para un satlite orbitando la tierra. Los objetos en movimiento circular no estn en equilibrio, debe haber una fuerza resultante, de otro modo slo habra un movimiento en lnea recta. FUERZA CENTRPETA. Una partcula que se mueve sobre una trayectoria circular de radio R con rapidez constante, se encuentra sometida a una aceleracin radial de magnitud v2/R. Por la segunda ley de Newton, sobre la partcula acta una fuerza en la direccin de hacia el centro de la circunferencia, cuya magnitud es:

Solucin. La partcula est sometida a una aceleracin centrpeta, y la fuerza centrpeta correspondiente est dada por la componente de la tensin de la cuerda en direccin radial hacia el centro de la circunferencia. El D. C. L. de la masa m.

Aplicando la segunda ley de Newton:

T cos mg = 0 Tcos = mg y Fx = ma Tsen = ma = m


Dividiendo (2) entre (1):

=0
(1)

v2 r

(2)

v2 Fc = mac = m R
27

tan =

v2 v 2 = rg tan rg

Dinmica de una partcula De la geometra de la figura, r = Lsen , reemplazando se obtiene la rapidez de m:

Hugo Medina Guzmn

v 2 = (Lsen )g tan

v = Lg tan sen
Para calcular el periodo T, esto es el tiempo que demora en dar una vuelta. Se sabe que x = vt , con x = 2r , entonces: Solucin. La figura muestra el D.C.L.

t =

2r = v

2L sen

Lg tan sen

= 2

L cos g

L cos T = 2 g
Ejemplo 41. Una bola de masa m, atada al extremo de una cuerda se hace ir en un plano horizontal formando una circunferencia de radio R. Si tiene una velocidad angular , cul es la tensin en la cuerda?

Aplicando la segunda ley de Newton.

F F

= mac T mgsen = mR 2 = mat mg cos = mR

La tensin en la cuerda es

T = mR 2 mgsen La fuerza tangencial es mg cos y la aceleracin


angular es

g cos R d 2 Como = , obtenemos la ecuacin dt

diferencial: Solucin. La figura muestra el D.C.L.

d 2 g = cos dt R
cuya solucin esta fuera del alcance de este curso. Pero podramos encontrar la tensin y fuerza tangencial para posiciones determinadas, es decir para valores dados de .

Aplicando la segunda ley de Newton a la masa m .

F F

= mac T = mR 2 = mat 0 = mR
2

T = mR 2 , Para = 0 Ft = mg T = mR 2 mg , Para = 90 Ft = 0 T = mR 2 Para = 180 , Ft = mg T = mR 2 + mg , Para = 270 Ft = 0 T = mR 2 Para = 360 Ft = mg

La tensin en la cuerda es T = mR . La fuerza tangencial es cero y la aceleracin tangencial tambin es cero, ya que la velocidad angular es constante. Ejemplo 42. Resolver el problema anterior pero en el caso que el giro sea en el plano vertical.

28

Dinmica de una partcula

Hugo Medina Guzmn b) Cuando el bloque est en A se dirige a B, su velocidad es en el sentido antihorario y su aceleracin en el sentido horario. Luego su rapidez disminuye. c) Si en "B" su velocidad es nula, cul es la trayectoria que seguir la masa m?

Ejemplo 43. Un pequeo bloque de masa m se desliza sobre una superficie lisa circular de radio R como se muestra en la figura. (La pista est sobre un plano vertical y g = aceleracin de la gravedad) a) Trace el diagrama de cuerpo libre del bloque cuando se encuentra en "A" y muestre (dibujando los vectores) la direccin de la fuerza resultante y su aceleracin. b) Cuando est en "A", su rapidez aumenta o disminuye? (Justifique) c) Si en "B" su velocidad es nula, cul es la trayectoria que seguir la masa m? d) Si en "B" su velocidad es seguir la masa m?

N + mg = ma c = m

v2 v2 N = mg + m R R

Si v = 0, el valor de N es negativo, lo que no permite al bloque sostenerse sobre la circunferencia, por consiguiente el bloque caer verticalmente. d) Si en "B" su velocidad es seguir la masa m?

gR , qu trayectoria

gR v2 N = mg + m N = mg + m = 0, R R
el bloque tiene suficiente velocidad para seguir en la trayectoria circular. Ejemplo 44. Un avin describe un rizo (un camino circular en un plano vertical) de 150 m de radio. La cabeza del piloto siempre apunta al centro del rizo. La rapidez del avin no es constante; es mnima en el cenit del rizo y mxima en el nadir. a) En el cenit el piloto experimenta ingravidez. Qu rapidez tiene el avin en ese punto? b) En el nadir, la rapidez del avin es de 280 km/h. Qu peso aparente tiene el piloto aqu? Su peso real es de 700 N. Solucin. a) S el piloto siente ingravidez, est en cada libre, y

gR , qu trayectoria

Solucin. a) Trace el diagrama de cuerpo libre del bloque cuando se encuentra en "A" y muestre (dibujando los vectores) la direccin de la fuerza resultante y su aceleracin.

v2 , luego R v = Rg = (150 )(9,80) = 38,3 m s , o 138 km h . a=g=


b) El peso aparente es la suma de la fuerza neta hacia adentro (arriba) y el peso del piloto, o

P' = P + ma = P + m
Aqu: P = 700 N

v2 R

m=

P 700 = = 71,43 kg g 9,8

v = 280 km/h = 77,6 m/s R = 150 m Luego:

77,6 2 P' = 700 + 71,43 150 = 3579 N


29

Dinmica de una partcula

Hugo Medina Guzmn

Ejemplo 45. Una partcula de masa m que est unida al extremo de un cable de longitud l , cuyo otro extremo est fijo, se mueve en un plano vertical, a partir de un punto A tal que el cable forma con la vertical un ngulo 0 , iniciando el movimiento con velocidad cero. Determinar: a) La velocidad de v de la esfera en funcin de b) La tensin del cable en funcin de . c) La aceleracin a en funcin de .

+ an n a = at t

at = gsen , T mg cos = ma n = 2mg (cos 0 cos ) a n = 2 g (cos 0 cos )


Ejemplo 46. Una partcula de masa m se encuentra en el polo de una semiesfera de radio R, la cual est apoyada sobre una superficie horizontal. Desplazada ligeramente de su posicin de equilibrio, la partcula desliza sobre la superficie, la cual se supone lisa. Determinar: a) La velocidad v de la partcula en funcin del ngulo que forma su radio posicin con el radio inicial. b) El valor de la normal N en funcin de . c) El valor de , en el instante en que la partcula se despega de la superficie. Solucin.

Solucin. En la referencia de origen O, la esfera recorre una circunferencia de radio l con velocidad variable v(t). Las componentes intrnsecas la aceleracin son:

at =

Sobre la masa m actan la tensin del cable peso mg .

dv v2 , an = dt l

T y su

En la referencia de origen O, la partcula m tiene un movimiento circular no uniforme de radio R. Las componentes de la aceleracin son:

y t De la segunda ley de Newton en componentes n se tiene:

at =

F F

t n

= mat mgsen = mat = ma n T mg cos = ma n

Sobre la masa m actan el peso mg y la reaccin en el apoyo N. Aplicando la segunda ley de Newton:

dv v2 , an = dt R

a) Para la componente tangencial se tiene:

Integrando y teniendo en cuenta las condiciones iniciales queda

dv mgsen = m dt ds dv = gsen dt ds vdv = gsen ds = gsen ld v = 2 gl(cos 0 cos )


2

F F

t n

= mat mgsen = mat = ma n N mgcos = ma n

a) De la componente tangencial se tiene:

dv ds dv = gsen dt dt ds vdv = gsen ds = Rgsen d mgsen = m


Integrando y teniendo en cuenta las condiciones iniciales queda

v 2 = 2 Rg (1 cos )

Finalmente:

v = 2 gl(cos 0 cos )
b) Para la componente normal:

v = 2 Rg (1 cos )

T mg cos = m

La tensin del cable es

v = 2mg (cos 0 cos ) l

b) De la componente normal se tiene: N = mgcos ma n =

T = mg (2 cos 0 3 cos )

c) De las ecuaciones anteriores se tiene la aceleracin:

v2 = mg cos 2mg (1 cos ) R La normal es N = mg (3 cos 2 ) mgcos m


c) La masa m deja de estar en contacto con la superficie cuando N = 0

30

Dinmica de una partcula

Hugo Medina Guzmn

N = mg (3 cos 2 ) = 0 2 cos = = 48,19 3


Ejemplo 47. En un parque de diversiones hay un cilindro grande vertical, de radio R que rota alrededor de su eje, con velocidad angular constante . Explicar cmo es posible que las personas que estn dentro, al retirrseles el piso permanezcan pegadas a la pared interior del cilindro.

m 2 . Los bloques estn a la distancia R del eje de


rotacin. El coeficiente de rozamiento esttico entre las masas y entre m 2 y la tornamesa es Considerando el rozamiento y la masa de la polea despreciables, encontrar la velocidad angular de la tornamesa para la cual los bloques justamente comienzan a resbalar.

Solucin. Solucin. En este problema todo depende de tomar correctamente la direccin de la fuerza de friccin entre m1 y m 2 . Consideremos m 2 > m1 , por lo tanto m 2 tender a moverse hacia afuera, jalando a

La figura muestra el D.C.L del hombre.

m1 hacia adentro. La fuerza de friccin actuar en oposicin a su movimiento relativo. La figura muestra los D.C.L. de los componentes del sistema.

Aplicando La segunda ley de Newton: Como el hombre no cae, radialmente est en reposo (R = constante)

F
y

Fr = mac N = m 2 R
z

Aplicando la segunda Ley de Newton

= 0 mg N = 0
2

= ma z ,

= ma r y

= mat

A la masa m1 :

De estas ecuaciones: mg m R

N 1 m1 g = 0 , T + F1 = m1 2 R , Ft = 0
A la masa m 2 :

g R

Esto quiere decir que para que suceda el efecto de suspensin de las personas, la velocidad angular tiene que tener un valor relacionado con el radio R y el coeficiente de friccin . Ejemplo 48. En la tornamesa mostrada en la figura el bloque de masa m1 descansa sobre el bloque de masa

N 2 N1 m2 g = 0 , T F1 F2 = m2 2 R , Ft = 0
De las ecuaciones obtenemos: N 1 = m1 , N 2 = (m1 + m2 )g
2

y 2 F1 + F2 = (m2 m1 ) R

F1 m1 g , F2 (m1 + m2 )g

31

Dinmica de una partcula Corno puede incrementarse hasta que F1 y F2 alcancen sus valores mximos

Hugo Medina Guzmn

2m1 g + (m1 + m2 )g = (m2 m1 ) 2 R

Finalmente

(3m1 + m2 )
R(m2 m1 )

Curvas sin peraltar En estos casos la fuerza de rozamiento es la que nos proporciona toda la componente normal que servir para tomar la curva. Siempre que tengamos que sta es mayor que la aceleracin normal el automvil ser capaz de tomar la curva, es decir, el caso lmite se alcanza cuando

Ejemplo 49. Cmo afectar la rotacin de la tierra al peso aparente de un cuerpo en el ecuador? Solucin. La figura muestra la situacin de un cuerpo situado en la lnea ecuatorial

v2 Fr = ma c = m R
Ejemplo 50. Cul es la velocidad a que puede ir un automvil por una curva sin peralte, de radio R, sin derrapar?, el coeficiente de rozamiento entre las ruedas y el suelo vale . Solucin.

Aplicando la segunda ley de Newton

F F F

= maz Fz = 0 = mar N mg = m R
2

= mac

= 0 ac =

v2 R

= mat Ft = 0

El peso de la masa es representado por la reaccin N

F f = N = mg = m

N = mg m 2 R
Para tener una idea de cunto afecta la rotacin de la tierra es necesario hacer el clculo numrico para esta consideracin: El radio de la tierra en el ecuador: R = 6,378 x l06m La velocidad angular de la tierra

v2 v = gR R

Ejemplo 51. El ciclista tiene que inclinarse al desplazarse por una pista circular (o para pasar por una curva), Encontrar la relacin de la velocidad con el radio de curvatura, el ngulo de inclinacin y coeficiente de friccin.

2 rad 5 rad = 7,27 10 24 3600 s s

La aceleracin de la gravedad en el Ecuador: g = 9,780490 m/s2

Porcentaje =

2R
g

100 = 0,34%

CURVAS EN LAS PISTAS. Para un cuerpo como un vehculo o un vagn de tren que se mueven describiendo una trayectoria curva de radio r, sobre el vehculo debe actuar una fuerza centrpeta para evitar que contine movindose en lnea recta y se salga de la pista; esta es la fuerza para hacer que el vehculo gire por la pista curva. La fuerza centrpeta necesaria la da el roce de las llantas o las pestaas de las ruedas del tren.

Solucin.

La figura muestra el D.C.L. 32

Dinmica de una partcula

Hugo Medina Guzmn Ejemplo 52. Cul es la velocidad a que puede ir un automvil por una curva con peralte, de radio R, sin derrapar, el peralte es de grados? Solucin.

Aplicando la segunda ley de Newton:

= maz N mg = 0 = mar N = m

v R

F = 0 ac =

v2 R v2 R

De las ecuaciones obtenemos

N = mg y mg = m
Finalmente v =

v2 R

F// = mac cos mgsen = m


v = gR tan

gR

Del D.C.L. tambin obtenemos:

tan =

N
N

Curvas peraltadas con rozamiento Este es un caso bastante ms complejo de analizar. Ejemplo 53. Cul es la velocidad a la que puede ir un automvil por una curva con peralte, de radio R, para que no se deslice hacia el exterior, el coeficiente de rozamiento entre las ruedas y el suelo vale ., el peralte es de grados? Solucin.

Esto quiere decir que si el motociclista al realizar una curva no se reclina y el piso no es lo suficientemente spero (friccin), ste caer. Curvas peraltadas sin rozamiento Para no tener que confiar en el roce o reducir el desgaste de los rieles y pestaas, la carretera o la va pueden inclinarse, como en la figura. En este caso la componente de la normal dirigida hacia el centro de curvatura proporciona la fuerza necesaria para mantener al mvil en la pista. A la inclinacin de la pista o va se le llama ngulo de peralte, .

F f = N , a c =

v2 R

//

= ma c cos

v2 mgsen + N = m cos R

= 0 N mg cos = m

v2 sen R

N = mg cos + m

v2 sen R

En estos casos se toma la proyeccin de la normal sobre la horizontal como causante de la fuerza centrpeta. Este caso se tiene, que:

v2 v2 R tan = = mg Rg m
Siendo

v2 v2 mgsen + mg cos + m R sen = m R cos v2 v2 mgsen + mg cos + m sen = m cos R R

mg (sen + cos ) = m

, la inclinacin de la carretera.
33

v = gR

(sen + cos ) (cos sen )

v2 (cos sen ) R

Dinmica de una partcula

Hugo Medina Guzmn conoce como MARCOS DE REFERENCIA INERCIALES. En los problemas trabajados hasta esta parte el primer paso era dibujar un sistema de coordenadas. Elegimos un sistema fijo a tierra, pero no pusimos atencin al hecho que la tierra no es un marco inercial debido a que la tierra al viajar en su orbita casi circular alrededor del sol experimenta una aceleracin centrpeta hacia el centro de la tierra. Sin embargo, estas aceleraciones son pequeas comparadas con la aceleracin de la gravedad y a menudo se pueden despreciar. En la mayora de los casos se supondr que la tierra es un marco inercial. Ahora veremos cmo cambian los resultados cuando se trabaja en un MARCO DE REFERENCIA NO INERCIAL, que es el nombre que se da a un marco de referencia acelerado. MARCO CON MOVIMIENTO DE TRASLACION NO UNIFORME. Consideremos los sistemas S y S tal corno se muestra en la Figura siguiente. El sistema S es inercial y el sistema S se mueve con respecto a S con

v = gR

(tan + ) (1 tan )

Para que no se vaya

Ejemplo 54. Cul es la velocidad a laque puede ir un automvil por una curva con peralte, de radio R, para que no se deslice hacia el interior, el coeficiente de rozamiento entre las ruedas y el suelo vale ., el peralte es de grados? Solucin.

v2 F f = N , a c = R F// = mac cos v2 mgsen N = m cos R v2 F = 0 N mg cos = m R sen v2 N = mg cos + m sen R


v2 v2 cos sen cos mgsen mg + m = m R R v2 v2 mgsen mg cos m sen = m cos R R

, tal que aceleracin constante A = Ai


D=

1 2 At . 2

De la figura obtenemos que la posicin de la partcula P es:

mg (sen cos ) = m v = gR v=

(sen cos ) (cos + sen ) (tan ) gR (1 + tan )

v2 (cos + sen ) R

1 2 At , y = y ' , z = z ' 2 1 r = r '+ At 2 i 2


x = x'+
Derivando con respecto al tiempo encontramos v x = v' x ' + At , v y = v' y , v z = v' z '

Para que no se caiga

v = v'+ Ati

La velocidad debe de estar entre esos valores para permanecer en la carretera.

gR

(tan + ) v (1 tan )

gR

(tan ) (1 + tan )

Derivando nuevamente encontramos a x = a' x + A , a y = a' y ' , a z = a' z '

o a = a'+ A a = a'+ Ai

MOVIMIENTO EN MARCOS DE REFERENCIA NO INERCIALES Hasta este momento nuestro estudio de mecnica clsica lo hemos realizado en sistemas de referencia que estn en reposo o con movimiento con velocidad constante con respecto a un sistema considerado en reposo. A este conjunto de marcos de referencia se le

Si la partcula P tiene una masa m y aplicarnos la segunda ley de Newton del movimiento en el sistema inercial S obtenemos

F = ma

Donde P es la suma de todas las fuerzas de interaccin que actan sobre las partculas. Para relacionar con el sistema no inercial S

34

Dinmica de una partcula

Hugo Medina Guzmn

F = m a '+ Ai o m a ' = F m A

Aqu vemos que para que el observador segn S pueda aplicar la segunda ley de Newton debemos introducir una fuerza extra FA a la llamaremos fuerza de arrastre y debemos incluirla en los diagramas de fuerzas:

El observador ve que el resorte se estira l . La fuerza es Aplicando la segunda ley de Newton:

F = kl
x

FA = m A m a ' = F + FA
De este modo, en el sistema S: Donde F ' es la suma de las fuerzas reales ms la de arrastre

= ma x kl = mA

l =

mA F

F ' = F + FA
Recalquemos el carcter ficticio de FA . Para aplicar una fuerza real sobre un cuerpo debemos ponerlo en interaccin con otro, de manera que, segn la tercera ley de Newton, si A ejerce una fuerza sobre B, FAB , a su vez B ejercer una fuerza sobre A, FBA , tal que

Observador en el vagn: La figura a continuacin muestra el D.C.L. de la masa m que no se mueve para el observador en el vagn. Como es sistema no inercial tenemos que aplicar la fuerza ficticia mA .

Aplicando la segunda ley de Newton

x'

= 0 mA = kl

FAB = FBA . Ahora, es la reaccin de la fuerza de arrastre?, cul es el otro cuerpo que est ejerciendo la fuerza ?. No existe tal cuerpo, la fuerza no tiene reaccin, es una fuerza ficticia que agrega un observador ubicado en un sistema acelerado (respecto a uno inercial) para justificar los fenmenos que observa.
Ejemplo 55. La fuerza para estirar o comprimir un resorte es proporcional a su deformacin lineal, F = kl , donde k es la constante del resorte y el signo menos significa que la fuerza es en oposicin a la deformacin. Si sobre una mesa sin friccin que se encuentra en un vagn se coloca una masa. m sujeta a un resorte de constante k y largo l , como se muestra en la figura. El tren arranca con una aceleracin A que se mantiene constante en la direccin x. Calcular la deformacin del resorte desde el punto de vista del observador en tierra y desde el punto de vista del observador en el vagn.

l =

mA F

Ejemplo 56. Analizar el caso de masa m colgada mediante un hilo del techo de un vagn, que se mueve con una aceleracin A. a) Desde el punto de vista de un observador en tierra (S). b) para un observador dentro del vagn (S).

Solucin. a) Para un observador en S: El D.C.L. de la masa m

Aplicando la segunda ley de Newton: Solucin. Observador en tierra: La figura muestra el D. C. L. de la masa m.

T cos = mg
Dividiendo (1) : (2)

F F

x y

= ma x Tsen = mA (1) = 0 T cos mg = 0


(2)

35

Dinmica de una partcula

Hugo Medina Guzmn

tan =

A g

tan =

b) Para un observador en S El D.C.L..de la masa m

2 g = = 33,7 3 3 g 2

Ejemplo 58. Resolver el caso del peso del hombre en un ascensor cuando asciende con una aceleracin constante A, desde el punto de vista del hombre en el ascensor. Solucin.

Aplicando la segunda ley de Newton

Tsen = mA = 0 (1) Fy ' = 0 T cos mg = 0 T cos = mg (2)


Dividiendo (1) : (2) obtenemos:

x'

= 0 Tsen mA = 0

tan =

A g

Aplicamos la segunda ley de Newton,

Ejemplo 57. Desde el techo de un carrito de juguete cuelga una masa m unida al cielorraso mediante una cuerda ideal. El carrito se encuentra en el piso de un ascensor que sube con aceleracin g/2. A su vez el carrito tiene una aceleracin horizontal de magnitud g respecto al ascensor. Encuentre el ngulo que forma la cuerda con la vertical, resuelva para un observador situado dentro del ascensor.

N = m( g + a )
N = m( g + a )

y'

= ma y ' N mg ma = 0

El peso del hombre ser la reaccin N En caso de subir con aceleracin a: En caso de bajar con aceleracin a:

N = m( g a )

Solucin. Para un observador en el ascensor. El D.C.L..de la masa m

Ejemplo 59. El pasajero de un tren deja caer una piedra en diversos estados de movimiento del tren. Hallar la trayectoria de dicha piedra que ve el pasajero y la trayectoria vista por un observador en tierra. a) El tren acelera con aceleracin A constante. b) El tren frena con aceleracin A constante. Solucin. El tiempo en que la piedra esta en movimiento, es el mismo para todo sistema puesto que el movimiento vertical es independiente del horizontal.

y = y' = h
piso: Aplicando la segunda ley de Newton

1 2 gt , para y = 0 la piedra lega al 2 2h g

Tsen = mg

x'

= ma x '
(1)

1 2 gt = 0 t = 2

g = 0 T cos m g + = 0 2 3 T cos = m g (2) 2

y'

a) Cuando el tren va con aceleracin A, deja caer una piedra. Considerando que en el momento que suelta la piedra el tren tiene una velocidad v 0 .

Dividiendo (1) / (2) 36

Dinmica de una partcula

Hugo Medina Guzmn

xtren = v0 t

1 2 At 2 1 2 At , detrs del punto de 2

La piedra cae a una distancia

x = xtren x piedra =
Observador en tierra Las ecuaciones del movimiento en el sistema S. Movimiento de la piedra

x piedra = v0 t
Movimiento del tren

plomada. Observador en el tren La ecuacin del movimiento en el sistema S Movimiento de la piedra

xtren = v0 t +

1 2 At 2 1 2 At , detrs del punto de 2 x piedra = 1 2 At 2 1 2 At , detrs 2

La piedra cae a una distancia

x = xtren x piedra =

plomada. Observador en el tren La ecuacin del movimiento en el sistema S Movimiento de la piedra

La piedra cae a una distancia x =

del punto de plomada. El grfico siguiente muestra el moviendo visto por un observador en el sistema S y en el sistema S.

x piedra =

1 2 At 2 1 2 At , detrs 2
MARCO DE ROTACIN Veamos el caso de un marco de referencia que est rotando con velocidad angular con respecto a otro marco de referencia. Supongamos que tenemos un objeto movindose alrededor de un punto arbitrario; este es un caso especfico, sin embargo tiene todos los efectos en l. La posicin de la partcula con respecto a un sistema inercial est determinada por un vector r . Consideremos un nuevo sistema de coordenadas tal que siga al objeto, el nuevo origen est determinado

La piedra cae a una distancia x =

del punto de plomada. El grfico siguiente muestra el moviendo visto por un observador en el sistema S y en el sistema S.

b) Cuando el tren desacelera con aceleracin A, deja caer una piedra. Considerando que en el momento que suelta la piedra el tren tiene una velocidad v 0 .

por R contenido en r tal que la posicin de la partcula en este nuevo sistema est ciada por r ' .

Observador en tierra Las ecuaciones del movimiento en el sistema S. Movimiento de la piedra

De la figura tenemos.

x piedra = v0 t
Movimiento del tren 37

r = R + r ' = Rr + r' r = (R + r ')r

Derivando:

Dinmica de una partcula

Hugo Medina Guzmn Si la partcula tiene una masa m y aplicamos la segunda ley de Newton en el sistema inercial

dr d d ( R + r ') dr r = (R + r ')r = + ( R + r ') dt dt dt dt dr Como = t dt d r dR dr ' = r r + + (R + r ')t dt dt dt dr = v es la velocidad de la partcula vista en el dt d r' sistema inercial y = v' es la velocidad de la dt
partcula vista en el sistema no inercial. Tal que
dR r + v'+ (R + r ')t dt

F = ma

donde F es la suma de todas las fuerzas de interaccin que actan sobre la partcula. Para relacionar con el sistema inercia!
F = m a '+ A o m a' = F m A

Para que el observador pueda aplicar la segunda ley de Newton debemos introducir aqu tambin una fuerza extra FA y debemos incluirla en los diagramas de fuerzas

FA = m A
d 2 ( R + r ') d (R + r ') dr d dt d ( R + r ') r + t + (R + r ') t + (R + r ') + dt dt dt dt dt dt 2

v=

FA = FAr r + FAt t

Para encontrar la aceleracin es necesario derivar nuevamente:


d r d d d ( R + r ') = 2 (R + r ')r = r + (R + r ')t dt 2 dt dt dt
2 2

Como

dr dt = t y = r dt dt

d 2 ( R + r ') FAr = m (R + r ') 2 r 2 dt y FAt = 2m d (R + r ') + (R + r ') t dt De este modo, en el sistema no inercial

d ( R + r ') d + t + (R + r ') t (R + r ') 2 r dt dt


= d 2 (R + r ') d ( R + r ') +2 r t 2 dt dt + (R + r ')t (R + r ') 2 r

d 2 r d 2 ( R + r ') d ( R + r ') + r t = 2 2 dt dt dt

F ' = m a ' = F + FA
Recalquemos el carcter ficticio de FA Con el objeto de clarificar esta idea veamos dos casos especiales: a) El origen O rota con velocidad angular constante a una distancia constante b, tal R + r ' = b , R y r son constantes.

d 2 ( R + r ') d ( R + r ') + (R + r ') t + 2 = (R + r ') 2 r 2 dt dt

d2 r donde a = es la aceleracin de la partcula dt 2

d ( R + r ') d 2 ( R + r ') y =0 =0 dt dt 2 d = constante, = =0 dt


2 FAr = m(R + r ') 2 r = mb r

Slo nos queda

vista en el sistema inercial y

a' =

d 2 r' es la aceleracin de la partcula vista en dt 2

e1 sistema no inercial. Llamando a

Que es la fuerza ficticia del centro hacia afuera y se le da el nombre de FUERZA CENTRFUGA, debemos insistir que solo aparece en el marco no inercial. b) El origen O rota con velocidad angular constante y tambin se est alejando del origen fijo en O con una velocidad constante V = Con esto,

d 2 ( R + r ') (R + r ') 2 r Ar = 2 dt

y At = 2

d ( R + r ') + (R + r ') t dt

Tenemos: A = Ar r + At t Tal que: a = a '+ A


d =0 dt

d ( R + r ') . dt

y nos queda

FAr = m(R + r ') 2 r


38

Dinmica de una partcula

Hugo Medina Guzmn

y FAt = 2mVt
Esta ltima fuerza ficticia, cuya direccin es transversal, se conoce como FUERZA DE CORIOLIS. Ejemplo 60. Un cuerpo de masa de masa m unido a un resorte de constante k y longitud l que gira con ve1ocidad angular constante en un plano horizontal sin friccin. Se quiere calcular el estiramiento l del resorte.

F' F'

z' t'

= ma' z ' , = ma' t '

F'

r'

= ma' r ' ,

Como a ' z ' = 0 , a ' r ' = 0 , Tenemos

F'

t'

= ma' t '

N mg = 0 , T + m 2 (l + l ) = 0 , Ft = 0 Como T = kl kl + m 2 (l + l ) = 0
y l =

Visto por un observador no inercial colocado sobre la misma masa Este caso es idntico al caso anterior. Ejemplo 61. Se tiene una plataforma circular de radio R a la cual se le ha pintado un radio y gira con velocidad angular constante . Un hombre camina de afuera hacia adentro de la plataforma siguiendo la lnea con una velocidad de mdulo constante v . Cul es la fuerza que la plataforma ejerce sobre el hombre, en funcin de su posicin?

m 2 l k m 2

Solucin. Visto por el observador inercial. La figura muestra el D.C. L. de la masa

Aplicando la segunda ley de Newton, el resorte estira l , luego su longitud es (l + l )

= ma z ,

= ma r ,
2

Como: a z = 0 , a r = Tenemos

(l + l ) , at

= mat

=0

Solucin. La figura muestra el D.C.L. del hombre

N mg = 0 , T = m 2 (l + l ) , Ft = 0

De aqu obtenemos:

N = mg y T = m 2 (l + l ) Como T = kl kl = m 2 (l + l ) m 2 l k m 2
Aplicando la segunda ley de Newton:

y l =

Visto por un observador no inercial colocado en el centro de rotacin y girando con la misma velocidad angular.

F F

= mat Rt = m( 2v + r )
2

= ma r Rr = ma r mr 2

Como: a r = 0 y

Rr = mr y

= 0: Rt = 2mv

Rt es debido a la aceleracin de coriolis. Rr es el sentido indicado en la figura y Rt en el


sentido contrario.

Aplicando la segunda ley de Newton:

39

Dinmica de una partcula

Hugo Medina Guzmn

PREGUNTAS Y PROBLEMAS 1. Sobre una partcula de masa m que parte del reposo en origen de coordenadas. acta una fuerza Respuesta. 20N opuesta a la velocidad.

+ 3 F = 2i j Despus de l0s la posicin de la


partcula viene dada por las coordenadas (3m; 4,5 m). Cul es su masa? Respuesta. m = 33,3 kg. 2 Hallar las fuerzas que actan sobre cada una de las seis barras rgidas de peso despreciable. Si estn unidas mediante pivotes lisos y cada una de las barras cortas tiene una longitud l .

N y F2 = 5. Qu fuerza en adicin a F1 = 4i N debe aplicarse al cuerpo en la figura, tal que: a) no acelere?


m/s2 b) tenga una aceleracin 4i

2 j

2 Respuesta. a) F = 4i j N, b) 2 F = 16i j N
6. Cul es la mnima aceleracin con la que puede deslizarse hacia abajo un hombre de 75 kg por una cuerda que solo soporta una tensin de 490N, Cul ser la velocidad de la persona despus de deslizarse la distancia de 20m? Respuesta. a = 3,27 m/s2 ; v = 11,4 m/s 7. El libro de Fsica I, est apoyado en el extremo superior de un resorte vertical, que a su vez esta parado sobre una mesa. Para cada componente del sistema libro-resorte-mesa-tierra: a) dibujar el diagrama de cuerpo libre, b) identificar todos los pares de fuerzas de accin y reaccin. 8. De acuerdo con la leyenda, un caballo aprendi las leyes de Newton. Cuando se le dijo que tirara una carreta, se neg argumentando que si l tiraba la carreta hacia delante, de acuerdo con la tercera ley de Newton habra una fuerza igual hacia atrs. De esta manera, las fuerzas estaran balanceadas y de acuerdo con la segunda ley de Newton, la carreta no acelerara. Cmo podra usted razonar con este misterioso caballo? Respuesta. a) La cuerda superior debido a que la tensin es mayor. b) La tuerza de reaccin inercial de la masa superior aumenta la resistencia frente a una aceleracin rpida. 4. Una caja de 40 kg que est resbalando en el piso disminuye su velocidad de 5 m/s a 2 m/s. Asumiendo que la fuerza sobre la caja es constante, encontrar su magnitud y direccin relativa a la velocidad de la caja. 40 9. Dos alumnos de forestal ubicados en los bordes opuestos de un camino recto tiran a un carro por el camino, con fuerzas de 160 N y 200 N, que forman un ngulo de 30 y 60 respectivamente, con la direccin del camino. Calcular la magnitud de la fuerza resultante y la direccin en la que se mover el carro. Respuesta. 256,1N, -21,3

Respuesta. AD = DB = mg ; CB = CA = mg/2, BC = 2mg; CD = 0. CD se puede retirar y no pasa nada. 3. Dos cubos de masa m estn unidos mediante una cuerda y uno de ellos est sujeto al techo mediante otra cuerda igual. a) Si en el cubo inferior se hace presin suavemente hacia abajo. Cul de las cuerdas se romper antes? porqu? b) Si la masa interior se golpea hacia abajo con un martillo, se rompe la cuerda de abajo porqu?

Dinmica de una partcula 10. Una masa de 5kg cuelga de una cuerda de 1m de longitud que se encuentra sujeta a un techo. Calcular la fuerza horizontal que aplicada a la masa la desve 30 cm de la vertical y la mantenga en esa posicin. Respuesta. 15,7 N.

Hugo Medina Guzmn

+2 11. Tres fuerzas F1 = 2i j N, 3 F2 = 5i j N y F3

) ) N que actan = ( 45i

Respuesta: a) a A = a B = 0,738 m/s2, b) 5,68 N 16. Un bloque A de 100 kg est unido a un contrapeo 8 de 25 kg mediante un cable dispuesto como muestra la figura. Si el sistema se abandona en reposo, determinar: a) la tensin en el cable. b) la velocidad de B transcurridos 3 s, c) la velocidad de A cuando ha recorrido 1,2 m.

sobre un objeto le producen una aceleracin de valor 3 m/s2. a) Cul es la direccin de la aceleracin? b) Cul es la masa del objeto? c) Si el objeto esta inicialmente en reposo, calcular su velocidad despus de 10s? 12. Una mano ejerce una fuerza horizontal de 5 N para mover hacia la derecha a dos bloques en contacto entre s uno al lado del otro, sobre una superficie horizontal sin roce. El bloque de la izquierda tiene una masa de 2 kg y el de la derecha de 1 kg. a) Dibujar el diagrama de cuerpo libre para cada bloque. b) Calcular la aceleracin del sistema, c) Calcular la aceleracin y fuerza sobre el bloque de 1 kg, d) Calcular la fuerza neta actuando sobre cada cuerpo. Respuesta. b) 5/3 m/s2, c) 5/3 m/s2, 5N, d) 5 N. 13. Una fuerza F aplicada a un objeto de masa m1 produce una aceleracin de 3 m/s2. La misma fuerza aplicada a una masa m2 produce una aceleracin 1 m/s2. a) Cul es el valor de la proporcin m1/m2? b) Si se combinan m1 y m2, encuentre su aceleracin bajo la accin de F. Respuesta. a) 1/3, b) 0,75 m/s2. 14. Dos bloques de masas M y 3M ubicado a la derecha de M, que estn sobre una mesa horizontal lisa se unen entre s con una varilla de alambre horizontal, de masa despreciable. Una fuerza horizontal de magnitud 2Mg se aplica sobre M hacia la izquierda. a) Hacer los diagrama de cuerpo libre. b) Calcular la aceleracin del sistema. c) Calcular la tensin del alambre. Respuesta. b) 5 m/s2, c) 15Mg N. 15. Dos paquetes se colocan sobre un plano inclinado como muestra la figura. El coeficiente de rozamiento entre el plano y el paquete A es 0,25 y entre el plano y el paquete B es 0,15. Sabiendo que los paquetes estn en contacto cuando se dejan libres, determinar: a) la aceleracin de cada paquete, b) la fuerza ejercida por el paquete A sobre el B. c) Resolver el problema invirtiendo las posiciones de los paquetes.

j m/s, c) -1,346 j m/s Respuesta. a) 302 N, b) 6,79


17. Determinar la aceleracin de cada uno de los bloques de la figura, Que bloque llega primero al suelo? mA=5kg, mB = 15 kg, mC = 10kg

Respuesta. a A ==

4,04 j m/s2,

a B == 0,577 j m/s2, a C == 2,89 j m/s2

C llega primero. 18. En la figura bloque.

= 0,45 , 5 kg . m A = 5 kg, m B =

20 kg mC = 15 Kg. determinar la aceleracin de cada

41

Dinmica de una partcula b) 28 N y 37 N

Hugo Medina Guzmn

Respuesta. a A =

j m/s2, a B = 2,45 j m/s2, 4,91

21. Pepe anda esquiando, cuando en algn momento sube 5 m deslizndose por la pendiente de un cerrito nevado en sus esques, saliendo desde la cima ubicada a 3 m de altura respecto a la horizontal, con una rapidez de 10 m/s. El coeficiente de roce entre la nieve y los esques es 0,1. a) Calcular la rapidez con la cual el esquiador comienza a subir la pendiente. b) Determine la distancia horizontal que vuela Pepe cuando sale de la punta del cerro. Respuesta. a) 13 m/s, b) 16,6 m. 22. El bloque de masa m de la figura parte del reposo, deslizndose desde la parte superior del plano inclinado 30 con la horizontal. El coeficiente de roce cintico es 0,3. a) Calcular la aceleracin del bloque mientras se mueve sobre el plano. b) Calcular la longitud del plano si el bloque sale con una rapidez de 5 m/s. c) Si el bloque cae al suelo a una distancia horizontal de 3 m desde el borde del plano, determine el tiempo total del movimiento.

aC = 0
19. Determinar la aceleracin del cilindro B de la figura, si a) T = 1500 N, b) T = 4000 N. mA=250 kg, mB = 100 kg,

Respuesta. a) 2,4 m/s2, b) 5,2 m, c) 2,8 s.

j N b) -9,81 j N Respuesta. a) -3,11


20. Se tiene un sistema formado por tres bloques y una polea sin friccin. El bloque A tiene una masa de 6,0 kilogramos y est en una superficie spera ( = 0,40). El bloque C tiene una masa de 4,0 kilogramos. Una fuerza externa P = 80 N, se aplica verticalmente al bloque A, la que mantiene el sistema en equilibrio esttico segn como se muestra.

23. En el sistema de la figura, se aplica una fuerza F sobre m. El coeficiente de roce es entre cada cuerpo y los planos. Deducir la expresin de la magnitud de F para que el sistema se mueva: a) con rapidez constante, b) con aceleracin a constante.

Mg ( cos + sen ) + mg + a(m + M ) .


24. En el sistema de la figura, la fuerza F paralela al plano inclinado empuja al bloque de masa m hacindolo subir una distancia D sobre el plano, de coeficiente de roce . Calcular en funcin de m, F, g, D, y , la aceleracin del bloque.

Respuesta. b)

a) Cul es la masa del bloque B? Cul es la fuerza de friccin sobre el bloque A? b) se quita la fuerza externa de 8,0 N. Las masas de los bloques B y C se ajustan, de modo el sistema siga en reposo tal como se muestra, pero estn justo por iniciar el movimiento. La masa del bloque A no se cambia. Las tensiones en las dos cuerdas verticales son: Respuesta. a) 3,1 kg 25.2 N 42

Dinmica de una partcula 25. Una fuerza F se aplica a un pequeo bloque de masa m para hacerlo moverse a lo largo de la parte superior de un bloque de masa M y largo L. El coeficiente de roce es entre los bloques. El bloque M desliza sin roce en la superficie horizontal. Los bloques parten del reposo con el pequeo en un extremo del grande, como se ve en la figura. a) Calcular la aceleracin de cada bloque relativa a la superficie horizontal. b) Calcular el tiempo que el bloque m demora en llegar al otro extremo de M, en funcin de L y las aceleraciones.

Hugo Medina Guzmn

30. Sobre el planeta X un objeto pesa 12 N. En el planeta Y, donde la magnitud de la aceleracin de cada libre es 1,6g, el objeto pesa 27 N. Cul es la masa del objeto y cul es la aceleracin de cada libre en el planeta X? Respuesta. 1,7 kg, 7m/s2. 31. Dos bloques de 1 y 2 kg, ubicados sobre planos lisos inclinados en 30, se conectan por una cuerda ligera que pasa por una polea sin roce, como se muestra en la figura. Calcular: a) la aceleracin de cada bloque, b) la tensin en la cuerda. c) si la aceleracin cuando los planos son rugosos fuera de la calculada en ese problema, calcular: el coeficiente de roce y la tensin en la cuerda.

Respuesta. a) (F- mg)/m, mg/(m+M), b) [2L/(a1-a2)]1/2. 26. Un bloque de masa M se ubica sobre un pequeo plano inclinado un ngulo sin roce, que tiene su extremo inferior fijo a un eje vertical que puede girar. En algn momento el eje gira con el plano con rapidez constante. Demostrar que si la masa asciende desde la base del plano, su rapidez cuando ha subido una distancia L es

v = gLsen .
27. Una fuerza dependiente del tiempo,

4t F = 8i j N (donde t est en segundos), se


aplica a un objeto de 2 kg inicialmente en reposo. a) En qu tiempo el objeto se mover con una velocidad de 15 m/s? b) A qu distancia est de su posicin inicial cuando su velocidad es 15 m/s? c) Cul es la posicin del objeto en este tiempo? Respuesta. a) 3s, b) 20,1m, c)

32. Un trineo de 50 kg de masa se empuja a lo largo de una superficie plana cubierta de nieve. El coeficiente de rozamiento esttico es 0,3, y el coeficiente de rozamiento cintico es 0,1. a) Cul es el peso del trineo? b) Qu fuerza se requiere para que el trineo comience a moverse? c) Qu fuerza se requiere para que el trineo se mueva con velocidad constante? d) Una vez en movimiento, qu fuerza total debe aplicrsele al trineo para acelerarlo a 3 m/s2? 33. La masa m1 sobre una mesa horizontal sin friccin se conecta a la masa m2 por medio de una polea mvil y una polea fija sin masas. Si a1 y a2 son magnitudes de las aceleraciones de m1 y m2, respectivamente. Determinar: a) una relacin entre estas aceleraciones. b) las tensiones en las cuerdas, y c) las aceleraciones a1 y a2 en funcin de m1, m2 y g.

(18i 9 j ) m

28. Una araa de 2 x 10-4 kg est suspendida de una hebra delgada de telaraa. La tensin mxima que soporta la hebra antes de romperse es 2,1 x 10-3 N. Cul es la aceleracin mxima con la cual la araa puede subir por la hebra con toda seguridad? Respuesta. 0,5m/s2. 29. Los instrumentos de un globo meteorolgico tienen una masa de 1 kg. a) El globo se suelta y ejerce una fuerza hacia arriba de 5 N sobre los instrumentos. Cul es la aceleracin del globo y de los instrumentos? b) Despus de que el globo ha acelerado durante 10 segundos, los instrumentos se sueltan. Cul es velocidad de los instrumentos en el momento en que se sueltan? c) cul es la fuerza neta que acta sobre los instrumentos despus de que se sueltan? d) En qu momento la direccin de su velocidad comienza a ser hacia abajo? 43

34. Calcular la fuerza F que debe aplicarse sobre un bloque A de 20 kg para evitar que el bloque B de 2 kg caiga. El coeficiente de friccin esttico entre los bloques A y B es 0,5, y la superficie horizontal no presenta friccin.

Dinmica de una partcula

Hugo Medina Guzmn b) Calcular la tensin de la cuerda. c) Si el sistema da una vuelta en 30 s, determinar El ngulo que forma la cuerda con la vertical.

35. Una bola de masa m se suelta sin velocidad inicial desde un punto A y oscila en un plano vertical al extremo de una cuerda de longitud L. Determinar: a) la componente tangencial de la aceleracin en el punto B. b) la velocidad en el punto B. c) la tensin en la cuerda cuando la bola para por el punto mas bajo. d) el valor de si la tensin en la cuerda es 2 mg cuando la bola pasa por el punto C

Respuesta. a) v = b) mg/cos .

g (l + Lsen ) tan ,

39. Una bola pequea da vueltas con una rapidez y recorriendo una circunferencia horizontal en el interior de un cono recto de base circular. Expresar la rapidez y en funcin de la altura y de la trayectoria sobre el vrtice del cono.

Respuesta. a)

gsen , b)

c) mg (3 2 cos 0 ) , d) 60.

2 gL(cos cos 0 ) ,
Respuesta. v =

36. Tres automviles circulan a la velocidad de 80 km/h por la carretera representada en la figura. Sabiendo que el coeficiente de rozamiento entre las llantas y la carretera es 0,60, determinar la desaceleracin tangencial de cada automvil s sus respectivos frenos sen repentinamente accionados y las ruedas deslizan.

gy

40. Cul es el mnimo radio que un motociclista con velocidad de 21 m/s puede hacer en una pista que tiene un coeficiente de friccin con las llantas igual a 0,3? Cul es el ngulo que har la motocicleta con la horizontal? Respuesta: 147 m; 73 20 41. Un estudiante hace girar un balde que contiene 2 kg de agua en una circunferencia vertical de l,2m de radio, considerar a) Cul es la mxima velocidad para que el agua permanezca en el balde? b) Cul es la fuerza ejercida por el balde sobre el agua en el punto inferior de la circunferencia? c) a la altura de los hombros? d) Si el balde pesa 10k, hallar cada una de las fuerzas que actan sobre el balde en el punto inferior de la circunferencia. Respuesta. a)

Respuesta. a A =3,91 m/s2, a B = 7,86 m/s2, aC = 5.89 m/s2. 37. Con qu ngulo debe peraltarse una carretera en una curva de 50 m de radio, para que un vehculo pueda tomar la curva a 72 km/h, con un coeficiente de rozamiento 0,30? Respuesta: 22,5 55,9 38. En el sistema de la figura, el brazo del pndulo es de longitud l y la cuerda de largo L. a) Calcular la rapidez tangencial para que el sistema gire en torno al eje de rotacin que pasa por la barra vertical, de modo que la cuerda que sostiene a la masa m forme un ngulo de con la vertical. 44

r , b) 2mg , c) g

2mg

d) 10 N debido a la tierra, 40 N debido al agua, 100 N debido al hombre. 42. Una mesa giratoria horizontal tiene una aceleracin angular de = 3 rad/s2. En el instante en que la velocidad angular vale 2,4 rad/s, una partcula

Dinmica de una partcula de masa 1,8 kg descansa sin deslizar sobre la mesa, con tal que est situada a una distancia inferior a 50 cm del eje vertical de rotacin de la mesa, a) Cul es el valor de la tuerza de rozamiento? b) Hallar el coeficiente de rozamiento esttico entre el objeto y la mesa. Respuesta: a) Ff = 7,9 N b) s = 0,45 43. Se tiene una partcula de masa 5g que se mueve sobre una trayectoria curva y su aceleracin en un

Hugo Medina Guzmn

c) a = 5 cm/s2, d)

= 53,1; e) Ft = 15 x 10-5 N,

f) Fr = 20 x l0-5 N, g) F = 25 x 10-5 N. 44. Describir e interpretar las fuerzas que realmente se apreciaran si nos encontrramos con los ojos vendados y: a) de pie sobre una plataforma elevada. b) cayendo libremente en el aire. c) estando sentado en el suelo de una plataforma en rotacin, como la de un carrusel a una cierta distancia de su centro. Respuesta. a) Una fuerza de reaccin de la plataforma hacia arriba. b) Ninguna fuerza. c) Una fuerza de reaccin de la plataforma y una fuerza hacia afuera (radial). 45. Calcular el ngulo de peralte de una carretera en una curva de radio 150 m, para que un camin de 15 toneladas pueda girar con una rapidez de 70 km/hr, sobre un pavimento cubierto de escarcha. Respuesta. 14

+ 4n momento dado vale a = (3t ) cm/s2. Hallar: a) la aceleracin tangencial, b) la aceleracin centrpeta, c) el mdulo de la aceleracin total, d) el ngulo que la aceleracin total forma con la tangente a la curva, e) la componente tangencial de la fuerza aceleradora, f) la componente centrpeta de la fuerza aceleradora, g) la fuerza aceleradora total. Respuesta. a) a t = 3 cm/s2 , b) a t = - 4 cm/s2 ;

45

TRABAJO Y ENERGA

Hugo Medina Guzmn

CAPTULO 5. TRABAJO Y ENERGA


INTRODUCCIN Con lo que hemos visto hasta el momento estamos en condiciones de analizar un movimiento en situaciones en que la fuerza es constante. Una vez aplicada La segunda ley de Newton, determinamos la aceleracin a = F / m . De aqu podemos determinar la velocidad y la posicin. Pero en el caso en que la fuerza no es constante, por ejemplo cuando se jala una masa situada en un extremo de un resorte, el problema se complica. Sin embargo, es importante notar que los conceptos de Trabajo y Energa se fundamentan en las leyes de Newton y por lo tanto no requieren ningn principio nuevo. TRABAJO El trmino trabajo que se usa en la vida cotidiana es para definir una actividad de algn tipo que incluye un esfuerzo fsico o mental y cuya finalidad sea el alcance de algn objetivo definido y bien establecido. En el estudio de la mecnica tiene un significado ms restringido, por ejemplo si subimos cierta altura h con una masa m decimos que hemos realizado un trabajo W, si subimos la misma altura h pero con una masa 2m, se habr realizado un trabajo 2W, igual a que si se hubiese transportado una masa m una altura 2h, o si se hubiese transportado dos veces la masa m, la altura h. Estas observaciones sugieren que el trabajo es una magnitud fsica proporcional a la fuerza y a la distancia, pero que puede sumarse como un escalar. Cuando una fuerza constante Fx mueve un cuerpo realizando un desplazamiento x que tiene la misma direccin que la fuerza, se define la cantidad de trabajo realizado por esta fuerza como:

La figura muestra un cuerpo de masa m sobre una superficie horizontal lisa, conectado a un resorte helicoidal. Si el resorte se estira o se comprime una longitud pequea desde su posicin no deformada o de equilibrio, el resorte ejercer una fuerza sobre el cuerpo F = kx , donde x es el desplazamiento del cuerpo desde la posicin de equilibrio ( x = 0 ) , k es la constante del resorte, el signo negativo (-) significa que la fuerza es en sentido opuesto al sentido del desplazamiento. Esta ley de fuerza se conoce como la ley de Hooke, de la cual nos ocuparemos en el Captulo de Elasticidad Apliquemos la segunda ley de Newton:

W = Fx x

F = ma

Ahora consideremos que sobre la misma masa m acta una fuerza vertical Fy , menor que el peso mg del bloque, como tal no dar origen a ningn movimiento vertical y por lo tanto no estar realizando trabajo.

dv d 2 x Con F = kx y a = = , dt dt 2
Obtenemos:

d 2x kx = m 2 dt 2 d x k + x=0 dt 2 m
A pesar de ser una ecuacin simple esta ltima, todava no tenernos el conocimiento matemtico para resolverla. Es decir, estamos en condiciones de plantear las ecuaciones del movimiento, pero no sabemos resolverlas. Veremos aqu que se puede tomar un atajo y resolver de otra forma el problema. En este capitulo se vern los conceptos de Trabajo y Energa que se pueden aplicar a la dinmica de un sistema mecnico sin recurrir a las leyes de Newton. 1

Si ahora aplicamos al mismo tiempo las dos fuerzas, la fuerza aplicada es:

+ Fy F = Fx i j
Si el desplazamiento del bloque es nicamente en la direccin x,

TRABAJO Y ENERGA

Hugo Medina Guzmn

r = xi

dW = F d r cos
dW = F ds cos dW = Ft ds
Es el trabajo realizado por la componente tangencial de la fuerza Ft . El trabajo de la componente normal

Fn es nulo.

El trabajo realizado es el producto escalar de la tuerza por el desplazamiento es:

+ Fy = Fx x j ) xi W = F r = (Fx i

Para evaluar el trabajo realizado para ir desde el punto P1(x1, y1, z1) a un punto P2(x2, y2, z2) tenemos que integrar el trabajo diferencial.

O W = Fx cos Donde

WP1P2 = dW = F d r
P 1 P 1

P2

P2

F = F +F
2 x

2 y

Para esto tenemos que conocer como vara

es el ngulo formado entre la fuerza aplicada y el desplazamiento.

+ Fy F = Fxi j + Fz k
Siendo

+ dy d r = dxi j + dzk

Consideremos el caso general de una fuerza F cualquiera que mueve a una partcula sobre una trayectoria curva como se muestra en la siguiente figura.

Tenemos: F d r = Fx dx + Fy dy + Fz dz Luego: WP1 P2 =

x2

x1

Fx dx + Fy dy + Fz dz
y1 z1

y2

z2

La unidad de trabajo es una unidad derivada de las unidades de fuerza y de longitud.

[W ] = FL = ML2T 2

En el sistema Internacional la unidad de trabajo es el Joule (J). 1 Joule = (1 Newton)(1 metro) Ejemplo 1. Un hombre levanta una masa m con una fuerza tal que la coloca a una altura h sobre el piso a velocidad constante. a) Cunto trabajo realiza la gravedad? b) Cul es la magnitud de la fuerza que ejerce el hombre? Solucin. a)

Sea P la posicin de la partcula en un instante t , la posicin con respecto al origen de coordenadas O est dada por La partcula en el tiempo t describe la trayectoria

OP = r

PP' , si esta es suficientemente pequea se puede

asimilar como la cuerda

PP' , el desplazamiento de la

partcula en el tiempo t es PP' = Cuando P tiende a P (t 0 ) .

La direccin de la cuerda

PP' es el de la tangente PT

en P, r es d r , la fuerza es constante en direccin y sentido.

Wgravedad =

y =h

y =0

h dyk F d r = mgk 0

= mg

dy = mgh

El trabajo de la fuerza F para el desplazamiento

d r es un trabajo diferencial. dW = F d r

b) Podramos hacerlo directamente por la ley de Newton, pero lo haremos con los conceptos de trabajo. Como la masa se mueve con velocidad constante, el trabajo realizado es cero.

Whombre + Wgravedad = 0
2

TRABAJO Y ENERGA

Hugo Medina Guzmn

Whombre = Wgravedad = mgh


Tambin tenemos:

Calcule el trabajo cuando el cuerpo se mueve desde x = 0 hasta x = 8 m.

Whombre =

y=h

y =0

h dyk Fhombre d r = Fk 0

= F Luego:

Fh = mgh F = mg
Solucin. El trabajo realizado por la fuerza es exactamente igual al rea bajo la curva desde x = 0 hasta x = 8.
W = 1 (5N )(2 0)m + (5N )(5 2)m + 1 (5N )(8 5)m 2 2

dy = Fh

F = mgk

Ejemplo 2. Se arrastra una caja de masa m sobre un piso horizontal, el coeficiente de friccin cintico entre la caja el piso es , mediante una fuerza que forma un ngulo con la horizontal, la caja se desplaza un distancia s haca la derecha, a) Calcule el trabajo realizado por la fuerza b) Calcule el trabajo efectuado por La fuerza de friccin. e) Determine el trabajo neto efectuado sobre la caja por todas las fuerzas que actan sobre ella.

= (5 + 15 + 7,5) Nm = 27,5 J Ejemplo 4. Trabajo realizado por un resorte.

Solucin.

a) El trabajo efectuado por

F es:

WF =
Como

x=s

+ Fsen F = F cos i j y d r = dxi


x=s x =0

x =0

F d r

WF =

(F cos i + Fsen j ) dxi


Fx s

El resorte de la figura, cuando se deforma o estira hasta una cierta posicin x, ejercer una fuerza restauradora F = kx . Solucin. Supongamos que el objeto se empuja hacia la izquierda una distancia x respecto a la posicin de equilibrio y se deja libre. El trabajo realizado desde x1 = x hasta x 2 = 0 por la fuerza del resorte a medida que el objeto se mueve es

= Fs cos =

W =

x2 = 0

x1 = x

Fx dx =

1 ( kx )dx = kx 2 x
0

La componente vertical de

F no realiza trabajo.

b) Como F f = Ni
Y

Y si consideramos el trabajo realizado por el resorte a medida que se estira de x1 = 0 a x2 = x el trabajo es W =

N = mg Fsen

Obtenemos F f = (mg Fsen )i


El trabajo efectuado por F f es = (mg Fsen )s c) El trabajo neto sobre la caja es la suma de los resultados obtenidos en a) y b). Wneto = WF + W f = F cos s (mg Fsen )s = F cos (mg Fsen ) s
dxi W f = F f d r = (mg Fsen )i
s 0 0 s

1 2 kx 2

Este resultado podemos obtenerlo tambin de La grfica F versus x, como se muestra en la figura siguiente.

Ejemplo 5. La posicin de una partcula en el plano

Ejemplo 3. Una fuerza que acta sobre un cuerpo vara con respecto a x como se muestra en la figura. 3

2t 2 est dada por r = 3ti j (t en segundos, r en

TRABAJO Y ENERGA

Hugo Medina Guzmn

metros), la fuerza ejercida sobre la misma es

5 F = 4i j (en Newton).
Qu trabajo se realiza sobre la partcula en el intervalo de t = l s a t = 3 s? Solucin.

P2 v2 1 W12 = dW = d mv 2 P v1 1 2 1 2 1 2 = mv 2 mv1 2 2

2t 2 4tdt r = 3ti j d r = 3dti j


Aqu tenemos una medida para el trabajo realizado sobre la partcula expresada en funcin de la variacin de la magnitud

Luego

5 4tdt dW = F d r = 4i j 3dti j = 12dt + 20tdt


W =
t =3

)(

1 2 mv . 2

Esta magnitud se define como la ENERGIA CINETICA K de la partcula. Entonces: K =

El trabajo W realizado sobre la partcula entre t = 1 y t = 3.


xt =1

dW = (12 + 20t )dt


3 1

1 2 mv 2

= 12t +

1 20t 2 = 126 22 = 104 J 2 1

La energa cintica es una propiedad general del movimiento de la partcula es la ENERGIA DEL MOVIMIENTO. Sus dimensiones son las de trabajo.

[K ] = ML2 T -2

El trabajo realizado sobre la partcula es 104 Joules.

Su unidad es la misma que la del trabajo. Resulta conveniente escribir:

ENERGIA CINETICA Consideremos una partcula de masa m bajo la accin

W12 = K 2 K 1 = K
El trabajo realizado por la fuerza al desplazar una partcula es igual al cambio de energa cintica de la partcula. Ejemplo 6. Encontrar la variacin de la energa cintica de un proyectil en funcin de su altura. Se lanza un proyectil de masa m desde el punto P0 (x0,

de la fuerza F . La segunda ley de Newton afirma que:

dv F = ma = m dt
Tambin sabemos que d r = v dt . Multiplicando escalarmente:

+ v0 y j. y0) con una velocidad inicial v 0 = v 0 x i


Solucin.

F d r = m

dv v dt = m v d v dt

Como F d r es el trabajo diferencial dW y

d 1 2 1 d mv = m v d v dt 2 2 dt
1 dv 1 dv = m v + m v dt 2 dt 2

dv = m v dt
De aqu:
1 d mv 2 = m v d v 2

Para un proyectil la posicin en funcin del tiempo es;

x = x0 + v0 x t , y = y 0 + v0 y t
Y la velocidad v x = v0 x , v y = v0 y gt La energa cintica en P0 es

1 2 gt 2

Reemplazando obtenemos:

1 dW = d mv 2 2
El trabajo para ir de P1 donde la velocidad es v1 al punto P2 donde la velocidad es v 2 ser:

K0 = K=

1 2 1 2 2 mv0 = m v 0 x + v0 y 2 2

La energa cintica en P es

1 2 1 2 2 mv = m v x + vy 2 2 1 2 2 2 2 = m v0 x + v0 y 2v0 y gt + g t 2

TRABAJO Y ENERGA

Hugo Medina Guzmn

La variacin de energa entre P y P0 es:

1 1 2 K = K K 0 = mv 2 mv0 2 2 1 2 2 = m 2v 0 y gt + g t 2 1 2 = mg v 0 y t gt 2 1 2 Como y y 0 = v 0 y t gt 2 1 2 1 2 Resulta K = mv mv0 = mg ( y y 0 ) 2 2

dirigida hacia abajo y la bola se mueve lo ms rpidamente.

fuerza radial = T mg =

mv 2 r

mv 2 1 2 y mhg = mv r 2 h = r r cos T mg = 2mg (1 cos ) 120 80 T mg = 0,75 = 1 cos = 1 2(80 ) 2mg = 41,4

Ejemplo 7. En una demostracin experimental para ilustrar la conservacin de la energa por medio del dispositivo siguiente. Se ata una bola del bowling a un extremo de una cuerda, y se sujeta el otro extremo al techo de la sala de conferencias. Se sostiene la bola parado en una escala tijeras alta, Para la demostracin se suelta del reposo en el extremo de la nariz, la bola volver de la oscilacin ms arriba y golpear violentamente la cara, (intente esto alguna vez si usted desea experimentar un juego para asustar) La demostracin impresiona a la clase, pero no por la razn esperada. Aunque la cuerda es bastante fuerte para sostener la bola cuando est inmvil, cuando la dej ir, la cuerda se rompi en el fondo del arco y la bola fue despedida alrededor del saln "Boing boing, boing" y dispers a los presentes en todas las direcciones. Una bola de bowling realmente rebota en el concreto. Suponga que la bola pesa 80 N y la cuerda tena 4,0 m de largo y tena una resistencia a ruptura de 120 N. Cul es el mximo ngulo con la vertical con el que se habra podido lanzar la bola sin tener la rotura de la cuerda?

Ejemplo 8. Se arrastra una caja de masa m sobre un piso horizontal, el coeficiente de friccin cintico entre la caja el piso es , mediante una fuerza que forma un ngulo con la horizontal. Si se empieza a jalar desde el reposo y considerando que ya se inici el movimiento Cul es la velocidad del bloque despus que recorre una distancia s?

Solucin. En este caso como la fuerza F es constante, por la ley de Newton podramos encontrar la aceleracin, que es constante, pero vamos a hacerlo por conceptos de Energa Cintica y Trabajo. Encontramos que

WNeto = [F cos (mg Fsen )]s

Sabemos que

1 2 1 2 mv2 mv1 2 2 Como: v1 = 0 y v2 = v WNeto = K 2 K1 =


Finalmente:

v=

2 [F cos (mg Fsen )]s m

Solucin.

Ejemplo 9. Para el caso de la masa m atada a un resorte con constante de rigidez k . Cul es la velocidad cuando pasa por la posicin de equilibrio despus de estirarlo una longitud L y soltarlo? Solucin.

La cuerda debe proporcionar suficiente fuerza ascendente para balancear el peso ms la fuerza radial necesaria para que la bola haga la curva hacia arriba. La tensin en la cuerda ser as la mayor en el punto ms bajo del arco, donde la fuerza de la gravedad est 5

El trabajo realizado desde x = L a x = 0 por la fuerza restauradora del resorte F = - kx Es:

TRABAJO Y ENERGA

Hugo Medina Guzmn

WR =

1 2 kL 2 1 2 1 2 mv2 mv1 2 2
Si en un sistema conservativo el trabajo efectuado por la fuerza para desplazar la partcula de A a B es independiente del camino entre A y B, se puede escribir:

Tambin WR = K 2 K1 = Siendo v2 = v0 y v1 = 0 Tenemos

1 2 1 2 kL = mv0 0 2 2 k v0 = L m

WAB = WBA
En un circuito cerrado

Para el caso que mostramos la respuesta correcta es la negativa. Ejemplo 10. Un objeto de masa m se mueve en el eje x sujeto a la fuerza F = m

WAA = WAB + WBA Como WAB = WBA WAA = WAB WAB = 0


El trabajo total efectuado por una fuerza conservativa sobre una partcula es cero cuando la partcula se mueve alrededor de cualquier trayectoria cerrada y regresa a su posicin inicial. Naturalmente la definicin de un sistema no conservativo es aquel que no satisface las condiciones anteriores. Ejemplo 11. Sistema no Conservativo. - La fuerza de friccin. Supongamos que un bloque se mueve del punto P1 (x1, y1) al punto P2 (x2, y1), siguiendo Las trayectorias mostradas en las figuras siguientes, el coeficiente de friccin entre el bloque y la superficie es . Calcular el trabajo realizado por la friccin en ambos casos.

A i donde A es una x2

constante y x es la distancia desde el origen. a) Cunto trabajo realiza esta fuerza si el objeto se mueve de x = a a x = b? b) Si la masa tena una velocidad v en la direccin positiva de x, Cul es su velocidad en b? Solucin. a) El trabajo que realiza la fuerza para mover la masa desde x = a a x = b es:

Wab =
Luego

x =b

x=a

F d r , F = m
b

A i , d r = dxi x2

Wab = m
a
b

b A = mA dx = i dxi 2 a x 2 x

1 1 1 mA = mA a b x a
b) Como

Wab = K b K a =

1 2 1 2 mvb mva 2 2

Solucin. Por la trayectoria (a)

Siendo

va = v0

WP1 P2 = F f d r
x1

x2

1 1 1 2 1 2 Tenemos mA = mvb mv0 2 a b 2


1 1 2 vb = 2 A + v0 a b
SISTEMAS CONSERVATIVOS Y NO CONSERVATIVOS Un sistema conservativo es aquel en el que el trabajo realizado por las fuerzas del sistema es independiente de la trayectoria seguida por el mvil desde una posicin a otra, no existen fuerzas de rozamiento, ni dispositivos que puedan producir prdida de la energa cintica.

, Aqu F f = Ni
Luego

d r = dxi

WP1 P2 =

x2

x1

( N )dx = N (x2 x1 )
x2 y1

Por la trayectoria (b)

WP1P2 = F f 1 d r1 + F f 2 d r2 + F f 3 d r3
y1 x1 y2

y2

Aqu

F f 1 = N j , d r1 = dy j

, d r2 = dxi F f 2 = Ni
F f 3 = N j , d r3 = dy j
Luego 6

TRABAJO Y ENERGA

Hugo Medina Guzmn

WP1 P2 =

y2

= N ( y2 y1 ) N ( x2 x1 ) + N ( y1 y2 )

y1

( N )dy + x ( N )dx + y (N )dy


x2
1

y1
2

WP1P2 = Fg d r + Fg d r
r1 r3

r2

r2

Aqu

= N ( x2 x1 ) 2 N ( y2 y1 ) Obviamente el trabajo realizado por la fuerza de friccin por las dos trayectorias a) y b) no son iguales, por consiguiente cuando hay fuerza de friccin el sistema no es conservativo. (La friccin no es conservativa). Ejemplo 12. Sistema Conservativo. La fuerza de la gravedad Supongamos que un bloque de masa m se mueve del punto P1(x1 ,y2) al punto P2(x2 ,y2) donde y es la direccin vertical. Calcular el trabajo realizado por la fuerza gravitacional con los tres casos mostrados en la figura.

+ dy j Fg = mg j , d r = dxi
+ dy Fg d r = ( mg j ) (dxi j ) = ( mg )dy
Luego

WP1P2 =

y3

y1

( mg )dy + y ( mg )dy
y2
3

= mg ( y3 y1 ) mg ( y2 y3 ) = mg ( y 2 y1 ) Resultado igual que en a) y b) Luego la fuerza de la gravedad es una fuerza conservativa. Trabajo en una trayectoria cerrada. Si completamos la trayectoria volviendo al punto inicial, tenemos una trayectoria cerrada y el trabajo es cero.

El trabajo para ir de 1 a 2 es Solucin. Por la trayectoria a)

Wr1r2 = F d r1 =
x1

x2

x2

x1

(F i + F j ) (dxi + dyj )
x y

WP1P2 = Fg d r
y1

y2

= Como

x2

x1

Fx dx + Fy dy
y1

y2

Aqu

Fg = mg j , d r = dy j
Luego

Wr1r2 = 0 mg ( y2 y1 )
= mg ( y2 y1 )
x1

F = mg j : Fy = mg . Fx = 0

WP1 P2 = mgdy = mg ( y2 y1 )
y2 y1

El trabajo para ir 2 a 1 es

Por la trayectoria b)
WP1P2 = Fg d r1 + Fg d r2 + Fg d r3
x1 y1 x2 x2 y2 x1

Wr2 r1 = F d r =
x2

Aqu

, d r2 = dy Fg = mg j , d r1 = dxi j , d r3 = dxi
Luego

= mg ( y1 y2 ) El trabajo total es

= 0 mg ( y1 y2 )

x1

x2

Fx dx + Fy dy
y2

y1

WP1P2 = 0 +

y2

y1

( mg )dy + 0 = mg ( y2 y1 )

Wr1r1 = Wr1r2 + Wr2 r1

Igual que en a) Por la trayectoria c).

= mg ( y2 y1 ) mg ( y1 y2 ) =0

WP1P2 = WP1P3 + WP3 P2


7

Esto no sucedera en el caso de una fuerza no conservativa, como la fuerza de friccin.

TRABAJO Y ENERGA

Hugo Medina Guzmn

LA FUNCION ENERGA POTENCIAL El trabajo realizado por la fuerza

+ Fy F = Fxi j + Fz k
Para mover una partcula de P1(x1. y1, z1) a P2(x2, y2, z2) es igual a:
P2

Si consideramos la energa potencial igual a cero en el nivel de referencia y = 0, la energa potencial a cualquier altura con respecto a y = 0 es:

U ( y ) = mgy
Tambin podamos haber determinado esta funcin a partir de:

W12 = F d r =
P1

(F dx + F dy + F dz )
P2 P 1 x y z

dU = Fg dy dU = ( mg )dy = mgdy

Integrando

Para un sistema conservativo el trabajo es independiente de la trayectoria seguida. Su integral debe ser un diferencial exacto, digamos - dU, tal que integrndolo, solamente los lmites determinan el valor de la integral. Esto es:

dU = mgdy + C
U ( y ) = mgy + C
Donde C es una constante relacionada con las condiciones de cada caso, por ejemplo aqu consideramos para y = 0 U (0 ) = 0 . La constante es C = 0

W12 =

P2

P1

( dU ) = ( U )P P

= (U 2 U1 ) = U

Aqu llamamos a U, energa potencial, cuyas unidades son las mismas que las de trabajo. Hemos determinado la funcin energa potencial a partir de una fuerza dada. Consideremos ahora el problema inverso, a partir de una funcin energa potencial determinar la fuerza

U ( y ) = mgy
Como comprobacin, a partir de esta energa potencial podemos encontrar la fuerza.

dU = F d r = Fx dx Fy dy Fz dz
Como U es funcin de x, y y z, podemos escribir esta derivada en funcin de sus derivadas parciales:

dU ( x , y , z ) =

U U U dx + dy + dz x y z

U mgy = =0 x x U mgy Fy = = = mg y y U mgy Fz = = =0 z z Fx =

Luego: F = mg j Ejemplo 14. Determinar la funcin energa potencial asociada a un resorte de constante de rigidez k. Solucin. Consideremos que el resorte est en el eje x, y se estira en esa direccin.

Relacionando con los componentes de la fuerza obtenemos

U U U , Fy = , Fz = x y z U U U F = x i + y j + z k Fx =

F = kxi
Tenemos que: Integrando

dU = Fx dx = ( kx )dx = kxdx
1 2 kx + C 2 1 2 kx 2 1 2 kx encontrar la fuerza 2

Ejemplo 13. La fuerza de la gravedad es un ejemplo de fuerza conservativa. Solucin. Tomemos la vertical a la tierra como el eje y, tal que:

U=

Fg = Fg j = mg j
El trabajo realizado por la gravedad cuando la partcula se desplaza desde el punto y1 al punto y2 es:

Si para la posicin de equilibrio x = 0, la energa potencial es cero, C es igual a cero y

U=

W12 = mg j dy j = mg dy
= mg ( y2 y1 )
y1 y1

y2

y2

Ahora realicemos el problema inverso: Dado U =

Como W12 U :

correspondiente:

mg ( y2 y1 ) = U = U ( y1 ) U ( y 2 )

O U = U ( y 2 ) U ( y1 ) = mgy2 mgy1

Fx =

U 1 = kx 2 = kx x x 2

TRABAJO Y ENERGA

Hugo Medina Guzmn

Fy =

U =0 y U Fz = =0 z

Colocando las energas iniciales a un lado y las finales al otro tenemos:

K1 + U1 = K 2 + U 2
Esta ecuacin es la forma matemtica de El principio de conservacin de la energa mecnica. Si definimos la energa mecnica total del sistema E como la suma de la energa cintica y potencial se puede expresar la conservacin de la energa mecnica como:

Luego F = kxi
Ejemplo 15. Energa potencial gravitatoria cerca de la tierra. Por la ley de Newton de la gravitacin universal, la fuerza de atraccin de dos masas es directamente proporcional al producto de estas e inversamente proporcional al cuadrado de la distancia.

E = K + U = Constante

F = G

mM r r2

Ejemplo 16. Fuerza de la gravedad: Se suelta una partcula de masa m desde la altura h sobre el suelo. Cuando la partcula est a una altura y del suelo, su velocidad es v. Su energa potencial es U = mgy Su energa cintica es K = La energa mecnica total es:

Donde m es la masa e un cuerpo, M la masa de la tierra, r la distancia entre las masas, G es la constante gravitatoria universal. Si r = R (radio de la tierra), la masa m est sobre la superficie de la tierra y

1 2 mv 2

F = G

mM = mgr r R2

La energa potencial es

dU = F d r = G U = G

mM dr r2

mM dr + C r2 mM U = G +C r
Para evaluar la constante C consideremos que el potencial U es cero para r infinito, de aqu C es igual a cero, Luego

1 2 mv + mgy 2 Para y = h , v = 0 E = 0 + mgh = mgh Para y = 0 , v = v 0 1 2 1 2 E = mv0 + 0 = mv0 2 2

E = K +U =

Para cualquier instante

1 2 mv + mgy = mgh 2 2 De aqu v = 2 g (h y )

E=

v = 2 g (h y )

U ( r ) = G

mM r

El grfico de la variacin de energa potencial y cintica es:

CONSERVACION DE LA ENERGA Hasta esta parte tenemos dos formas de encontrar el trabajo realizado sobre un objeto por una fuerza, la primera vlida para todo caso ya sea fuerza conservativa o no conservativa

Ejemplo 17. Una masa pequea m se suelta desde el reposo de la parte ms alta de una superficie esfrica de radio R, sin friccin. A qu ngulo con vertical dejar el contacto con la esfera?

W12 =

1 2 1 2 mv2 mv1 = K 2 K1 = K . 2 2
Solucin. Cuando la masa est a una altura h su energa es igual a cuando est en el punto ms alto.

Y la segunda para el caso de fuerzas conservativas

W12 = U1 U 2 = U
Luego podemos escribir

W12 = K 2 K1 = U1 U 2
9

TRABAJO Y ENERGA

Hugo Medina Guzmn

E= E=

1 2 1 2 mv0 + 0 = mv0 2 2

Para cualquier instante

1 mgR + 0 = mgh + mv 2 2 Con h = R cos 1 mgR = mgR cos + mv 2 2 2 v = gR(2 2 cos )


La segunda ecuacin de Newton cuando la masa esta en la posicin del ngulo :

1 2 1 2 1 2 mv + kx = kL 2 2 2 k 2 De aqu: v = L2 x 2 m k 2 v= ( L x2 ) m

El grfico de la variacin de la energa potencial y cintica es:

v2 Con ac = : R v2 N mg cos = m R
La masa deja la superficie esfrica cuando:

mv 2 mgR(2 2 cos ) = R R 2 v cos = = 2 2 cos Rg 2 cos = 3 = 48,2 mg cos =


Ejemplo 18. Fuerza de un resorte: Se jala una masa a sujeta a un resorte de constante k sobre una superficie sin friccin, desde la posicin de equilibrio x = 0 hasta una distancia L y se suelta. A una distancia x de la posicin de equilibrio la velocidad de la masa es v.

N =0

Ejemplo 19. Calcular la velocidad necesaria para que una partcula pueda escapar de la atraccin de la tierra. La energa total E de una partcula de masa m que est a una distancia r del centro de la tierra y que tiene una velocidad v es:

E = K + U , donde K =

1 2 mv y 2

mM r 1 2 mM Luego: E = mv G = Constante 2 r U = G
Si la partcula escapa de la atraccin de la tierra y se sita a una distancia infinita de sta su potencial es cero.

r , U = G

mM 0 r

En esta regin con la velocidad menor posible v = 0 Tenemos K = 0 Luego: E = K + U = 0 Como E es constante E = 0

1 2 kx 2 1 2 Su energa cintica es K = mv 2
Su energa potencial es U = Su energa mecnica total es:

La energa E de la partcula en la superficie de la tierra con la velocidad ve para que pueda escapar:

1 2 1 2 mv + kx 2 2 Para x = L , v = 0 1 1 E = 0 + kL2 = kL2 2 2 Para x = 0 , v = v 0

E = K +U =

mM 1 2 mve G =0 R 2 2GM ve = R E=
Como en la superficie de la tierra

mM GM = mg g = 2 2 R R Tenemos: v e = 2 gR F = G
10

TRABAJO Y ENERGA

Hugo Medina Guzmn

Siendo

m 6 y R = 6,4 10 m 2 s 4 m Obtenemos; ve = 1,12 10 s

g = 9,81

Ejemplo 20. Se tiene un resorte de longitud L y constante k conectado a la base de un bloque de masa m, Se suelta el bloque desde la altura H. Cul ser la distancia mas cercana al piso que alcanzar el bloque antes de rebotar? Solucin.

Solucin. Velocidad de la partcula: Tenemos que

ETotal = K + U =

1 2 mv + U ( x ) 2

1 2 mv = E U ( x ) 2 2 (E U ( x ) ) v= m

En el instante inicial la energa es solamente la potencia1 gravitatoria es U = mgH , la energa cintica es cero, tal que la energa total es E = mgH . En el instante final: La energa potencial es la correspondiente a la masa a una altura y, ms la del resorte comprimido una longitud (L y ) , es decir:

La energa cintica: - Es igual a cero en x1 y x5. - Tiene su valor mximo donde U(x) es mnimo, el punto x2 La partcula se mueve entre x1 y x5, fuera de estos valores la velocidad sera imaginaria. Como Fx =

dU ( x ) dt

, la pendiente del grfico de

1 2 U = U g + U r = mgy + k (L y ) 2
Como en ese instante ha cesado el movimiento, la energa cintica es cero, La energa total es:

U(x) en determinado punto corresponde a La fuerza actuante, tal que la fuerza se hace cero donde la pendiente es cero, como en x2, x3 y x4. La fuerza es positiva entre x1 y x2.entre x3 y x4. La fuerza es negativa entre x2 y x3, entre x4 y x5. Los puntos en que U es mnimo, son posiciones de equilibrio estable, como son x2 y x4. Ejemplo 22. En la figura, un auto de juguete de masa m se libera del reposo en la pista circular. Si se suelta a una altura 2R sobre el piso, cun arriba sobre el piso estar cuando sale de la pista, desprecie la friccin?

1 2 E = mgy + k (L y ) 2
Por la conservacin de la energa

1 2 mgH = mgy + k (L y ) 2
La solucin de esta ecuacin es:

mg mg 2k (H L ) y = L 1+ R k mg
Siendo el valor positivo de y la solucin significativa. Ejemplo 21. El grfico de la figura muestra la funcin potencial y la energa total de un movimiento. Qu podemos decir acerca del movimiento? Solucin. En la figura de arriba:

h = R (1 + sen )

Despreciando las prdidas por friccin la energa total es constante, de tal manera que: Siendo v la velocidad del auto a la altura h.

11

TRABAJO Y ENERGA

Hugo Medina Guzmn

1 mg (2 R ) = mg (h ) + mv 2 2 1 mg (2 R ) = mgR(1 + sen ) + mv 2 2 1 gR = gRsen + v 2 (1) 2


Aplicando la segunda ley de Newton en la altura h:

Solucin. a) Siendo v la velocidad de la masa en la parte superior del rizo. Por conservacin de la energa:

1 mg (H ) = mg (2 R ) + mv 2 2 1 2 gH = 2 gR + v (1) 2
Aplicando la segunda ley de Newton en ese punto:

mgsen N = mac
N = 0, condicin de cada.

v2 ac = R
Luego:

mg N = mac
N = 0, condicin de cada.

v2 mgsen = m R 2 v = gRsen (2)


Reemplazando (2) en (1):

ac =

v2 R v2 R

Luego:

mg = m v 2 = gR

1 gR = gR sen + gR sen 2
2 3 gR = gRsen sen = 3 2 Finalmente: `2 h = R (1 + sen ) = R1 + 3 5 = R = 1,67 R 3
Ejemplo 23. Una masa pequea resbala sobre una superficie inclinada pasando por un rizo de radio R. a) Cul es la altura mnima H de la que debe soltarse a fin de que el cuerpo no deje la superficie interior del rizo al dar la vuelta? b) Con que velocidad llega la masa al punto A? c) Cul es el valor del ngulo , con el que se puede retirar el segmento

(2) Reemplazando (2) en (1):

gH = 2 gR + H=

1 gR 2

5 R H = 2,5 R 2

b) Sea v la velocidad en el punto A su altura es

h = R(1 + cos )

Por conservacin de la energa:

AB de la circunferencia

1 mg (H ) = mgR(h ) + mv 2 2 1 mg (2,5R ) = mgR(1 + cos ) + mv 2 2 v 2 = 2 g (2,5R ) 2 gR(1 + cos ) v 2 = 3gR 2 gR cos


v = gR(3 2 cos )
c) La masa sale del punto A, como un proyectil con

de tal modo que la masa que sale de A alcance el punto B despus de viajar una cierta distancia en el aire.

+ vy velocidad inicial v = vxi j

12

TRABAJO Y ENERGA

Hugo Medina Guzmn

Solucin.

F 220 = = 344 N/m x 0,64 b) U 1 + K 1 = U 2 + K 2


a) k = En el tiempo t de su recorrido vertical debe alcanzar al punto B. Recorrido vertical:

1 2 1 kx + 0 = 0 + mv 2 v = 2 2
v= 344 (0,64) = 76,6 m/s 0,024

k x m

y = vsen t
0 = vsen t

1 2 gt 2 1 2 2v gt t = sen 2 g

Cuando llega a B, y = 0:

Su recorrido horizontal es

x = vxt = v cos t 2v Para t = sen debe de estar en B, luego: g 2v 2 Rsen = v cos g sen gR v2 = cos gR gR(3 2 cos ) = cos 1 3 cos 2 cos + = 0 2 2
Resolviendo:

Ejemplo 24. Puenting. Un saltador que pesa 800 N se ata con una cuerda elstica al tobillo y se salta de una torre alta. La cuerda tiene una longitud si estirar de 30 m, y un extremo se une al punto donde el salto comienza. La constante del resorte de la cuerda elstica es 200 N/m. Cunto recorrer el saltador antes de que la cuerda detenga su descenso?

Igualando esta expresin de la velocidad con la encontrada anteriormente:

Solucin. Sea el punto ms bajo del salto h = 0. La energa cintica inicial y la energa cintica en el punto ms bajo son ambas igual a cero. Tal que por la conservacin de la energa:

mgh = 0 +

1 cos = 1 2
En nuestro caso tomamos la solucin , con la que obtenemos = 60 Ejemplo 24. Un arco del tiro al arco ejerce la fuerza kx de la ley de Hooke en una flecha cuando la cuerda se jala una distancia x. Se supone que un arquero ejerce una fuerza de 220 N jalando a la flecha una distancia de 64 cm. a) Cul es la constante del resorte del arco? b) Cul es la velocidad de una flecha de masa 24 g cuando deja el arco?

1 2 kx , donde x = h 30 . 2

Sustituyendo mg = 800 N y k = 200 N/m, y resolviendo:

h 2 68h + 900 = 0
h = 68

(68)2 4(900)

= 50 m, o 18 m.

La solucin correcta es h = 50 m. La solucin h = 18 m corresponde al rebote que comprime la cuerda amortiguador auxiliar, pero una cuerda no se comprime como un resorte. Ejemplo 25. En la figura mostrada, el hombre y la plataforma tienen una masa m, el hombre se eleva una distancia h tirando la cuerda del lado derecho. a) En cunto aumenta su energa potencial gravitatoria? b) Qu fuerza debe ejercer para elevarse? c) Qu longitud de cuerda debe tirar para llegar a la posicin superior? d) Despreciando el rozamiento Qu trabajo habr realizado?

13

TRABAJO Y ENERGA

Hugo Medina Guzmn

W = Fd
La aceleracin de la caja es a =

F m

Como la caja parte del reposo su velocidad en la posicin final es:

v2 = 2ad =
Solucin. a) La energa potencial gravitatoria es

2 Fd m

El observador determina que el cambio de energa:

U ( y ) = mgy + C
Para la posicin inicial

U1 = mgy1 + C
Para la posicin final

U 2 = mgy2 + C
El aumento de la energa potencial gravitatoria es:

1 2 1 2 mv2 mv1 2 2 2 Fd Como v1 = 0 y v2 = m 1 2 Fd K = m = Fd 2 m K = K 2 K1 =


El observador sobre la plataforma concluye que:

U = U 2 U1 = mg ( y2 y1 ) = mgh

b) La fuerza para elevar el sistema, siendo esta conservativa,

K =

F =

U = mg y

1 2 Fd m = Fd 2 m W = K

b) Observador situado en tierra:

Como la polea divide en dos, la fuerza Fh que debe ejercer el hombre es: Fh =

mg . 2
El observador en tierra ve que la caja se mueve bajo la accin de la fuerza F, en este caso la caja se mueve la distancia d ' = Vt+ d ,. Siendo t el tiempo que demora el recorrido de la distancia d sobre la plataforma,

c) Para llegar a la posicin superior la cuerda debe ser tirada en una longitud dos veces h d = 2h. d) EL trabajo realizado por el hombre es:

mg Wh = Fh d = (2h ) = mgh 2
Justamente igual al cambio de energa. Ejemplo 26. Observadores en movimiento relativo. Sobre una plataforma en movimiento horizontal con una velocidad constante V. un hombre empuja una caja de masa m con una fuerza F una distancia d partiendo del reposo. Demostrar la validez de la conservacin de la energa desde los puntos de vista de observadores en marcos inerciales diferentes. Solucin. Las leyes de Newton se cumplen slo en marcos de referencia inerciales. Si se cumplen en uno en particular entonces se cumplen en todos los marcos de referencia que se muevan a velocidad constante en relacin a este mareo. a) Observador en la plataforma.

t=

2d = a

2dm 2dm , luego d ' = V +d F F

El trabajo es:

2dm + d W ' = Fd ' = F V F W ' = Fd + V 2 Fdm


El observador ve que la caja tiene una velocidad inicial

v'1 = V
y una velocidad final

v'2 = V + v2 = V +

2 Fd m

El observador en tierra determina que el cambio de energa es:

K ' = K '2 K '1 =


El observador en la plataforma ve que la caja, de masa m, se mueve bajo la accin de la fuerza F. El trabajo realizado para mover la distancia d es: 14

1 1 2 mv'2 mv'1 2 2 2
2

1 2 Fd 1 mV 2 V K ' = m + 2 m 2

TRABAJO Y ENERGA

Hugo Medina Guzmn

K ' = Fd + V 2 Fdm
Aqu se cumple tambin la conservacin de la energa:

La energa inicial es solo la energa cintica de la masa:

Ei =

W ' = K '

1 2 mv0 2 1 2 kx 2

La energa final es solo la energa potencial del resorte:

SISTEMAS NO CONSERVATIVOS. Supongamos que tambin intervienen fuerzas no conservativas, como la friccin. El trabajo total para mover la partcula de r1 a r2 es

Ef =
x2

El trabajo hecho por la friccin

1 2 1 2 W12 = mv2 mv1 = K 2 K1 2 2


Este trabajo es tambin igual a la suma del trabajo realizado por las fuerzas conservativas y del trabajo realizado por las fuerzas no conservativas, es decir:

W f = F f dx , x1 = 0 , x 2 = L + x ,
x1

F f = N = mg
Luego:

W12 =

L+ x

( mg )dx

= mg (L + x )

Como en un Sistema no Conservativo.


W12 NO CONSERVATIVAS = E2 E1

W12 = W12 CONSERVATIVAS + W12 NO CONSERVATIVAS


Como:

mg (L + x ) =

W12 CONSERVATIVAS = U 1 U 2

1 2 1 2 kx mv0 2 2

Ecuacin de segundo grado cuya solucin es:

W12 = U 1 U 2 + W12 NO CONSERVATIVAS


De las expresiones de trabajo total tenemos:

x=

mg
k

2m2 g 2
k
2

m 2 2gL v0 k

K 2 K1 = U 1 U 2 + W12 NO CONSERVATIVAS (K 2 U 2 ) (K1 U 1 ) = W12 NO CONSERVATIVAS

E 2 E1 = W12 NO CONSERVATIVAS
A diferencia que en un Sistema conservativo, no es igual a cero. Esta ltima expresin nos permite calcular el trabajo de fuerzas no conservativas, fuerzas que en general son complicadas y que en principio deberamos de calcular resolviendo integrales curvilneas. Ejemplo 27. A un bloque de masa m se le da un empujn tal que adquiere la velocidad v 0 a lo largo del eje x. Despus de resbalar distancia L golpea un resorte de constante k. Si el coeficiente de friccin entre el bloque y la masa es . Cunto se comprime el resorte? Solucin.

Ejemplo 28. Un cuerpo de masa 10 kilogramos cae desde una altura de 15 metros y alcanza el suelo en 2 segundos. Considerando constante la fuerza de resistencia del aire. a) Cul era la magnitud de la fuerza de resistencia? b) Cunta energa mecnica se ha perdido? c) Qu velocidad tena el cuerpo inmediatamente antes de chocar Contra el suelo? Solucin. a) Siendo el peso y la fuerza de resistencia del aire las fuerzas que intervienen y siendo ambas constantes tenemos que la aceleracin a del cuerpo es constante. Como h =

1 2 at 2

La aceleracin es a =

2h 2(15) m = 2 = 7,5 2 2 t 2 s

Aplicando la segunda ley de Newton:

mg Fg = ma

Fg = m( g a ) = 10(9,8 7,5) = 23 N

b) La energa que se ha perdido es el trabajo realizado por las fuerzas no conservativas.

W NO CONSERVATIVAS = Fg d = (23)(15) = 345 J

Sea x La longitud que se comprime el resorte. La distancia recorrida por la masa es (L + x).

c) Como W NO CONSERVATIVAS = E 2 E1 Siendo E1 = K1 + U1 = 0 + mgh = (10)(9,8)(15) = 1470 J

E2 = K 2 + U 21 =
15

1 2 2 mv2 + 0 = 5v2 2

TRABAJO Y ENERGA

Hugo Medina Guzmn

Tenemos:
2 2 5v2 1470 = 345 v2 =

1470 345 = 225 5

s=
=

v2 2 g (sen + cos )

m Finalmente: v2 = 15 s
Una manera directa de llegar al mismo resultado es considerar que la aceleracin efectiva de salida es

2(9,8)(sen30 +0,5 cos 30 )

(40)2

= 87,5 m

m , la velocidad despus de 2 segundos es: s2 m m v2 = at = 7,5 2 = 15 s s a = 7,5


LA CONSERVACIN DE LA ENERGA Y LA FRICCIN La ley de la conservacin de la energa se puede aplicar a los sistemas donde las fuerzas no conservativas como actan las fuerzas de la friccin. Si un sistema trabaja contra la friccin, la energa mecnica del sistema disminuir. As si Wf es el trabajo hecho contra la friccin, entonces energa inicial - la energa perdida por la friccin

POTENCIA Tan importante como saber cual es el trabajo realizado es conocer tambin la rapidez con la cual se realiza. Para proporcionar una medida cuantitativa de este concepto que incluye tanto el trabajo como el tiempo necesario para realizarlo se tiene a la Potencia. La potencia mide la rapidez con la que el trabajo se est realizando. Si se realiza un trabajo W en un intervalo de tiempo (de t1 a t2) la Potencia media es:

W12 W = t2 t1 t Cuando t 2 t1 , t 0 , tendremos Pm = P = lim W dW = dt t

La Potencia instantnea en el instante t.


t 0

E1 W f = E 2

U1 +

1 2 1 2 mv1 W f = U 2 + mv 2 2 2

Tambin como

dW = F d r
Tenemos

Ejemplo 29. Cerca de Lewiston, Idaho, hay una carretera muy inclinada donde circulan camiones cargados con madera. Han ocurrido varios accidentes serios cuando los carros perdieron sus frenos yendo para abajo de la colina a gran velocidad. Se han construido rampas de contencin que se espera puedan detener a los vehculos sin frenos. Suponga que un carro que viaja a 40 m/s encuentra una rampa inclinada para arriba 30 sobre horizontal. La grava floja en la rampa proporciona una fuerza friccional para ayudar a detener al carro mientras sube la rampa. La grava tiene un coeficiente eficaz de friccin de 0,50. Cun lejos a lo largo de la rampa el carro viajara antes de detenerse? Solucin. N = mg cos

dW d r P= = F dt dt

P = F v

El anlisis dimensional

[P] = [F ][L][T ]1 = [M ][L]2 [T ]1

F f = N = mg cos

U1 + K1 W f = U 2 + K 2

Su unidad en el sistema internacional es J/s llamado Watt Vatio cuyo smbolo es W. Un mltiplo muy usado es el kilowatt (kW) 1 kW = 103 W Existe una unidad de energa o trabajo en trminos de la unidad de potencia el kilowatt-hora (kwh), es la energa convertida o consumida en una hora a una razn constante de 1 kW. 1 kWh (103W)(3600s) = 3,6 x l06 .J Para tener una idea de cuanto es 1 Watt, imaginemos que tenemos que levantar una masa de 50 kg. a una altura de 1 metro, cada 5 minutos y realizar este trabajo durante una jornada de 8 horas. Si levanta cada 5 minutos, sern 12 veces por hora, siendo 8 horas por da, har un total de 12 x 8 = 96 veces al da. El trabajo realizado es:

0+

1 2 mv F f s = mgh + 0 2

h = s sen

1 2 mv (mg cos )s = mgs sen 2


16

W = 96mgh = 96(50Kg )(9,8m / s )(1m ) = 47040 J

TRABAJO Y ENERGA

Hugo Medina Guzmn Luego: P = (mg sen )( g sen t ) = Como

Para determinar la potencia tenemos que dividirlo por el nmero de segundos en un da.

47040J P= = 1,63 W 8 3600 s


Comparemos esta potencia con la potencia de un motor pequeo de 1 hp (horse power). El hp es la unidad de potencia en el sistema ingls 1 hp = 746 W Ejemplo 30. Si un objeto que parte del reposo se desliza por un piso liso inclinado un ngulo con respecto a la horizontal de altura h, hallar la potencia P gastada por la gravedad en funcin de la posicin y del objeto con respecto a la parte inferior plano inclinado. Solucin.

mg 2 sen 2 t 2(h y ) gsen 2

t=

Obtenemos:

P = mg 2 sen 2

2(h y ) gsen 2

= mgsen 2 g (h y ) Ejemplo 31. El flujo de agua de un ro es de 50 m3 por segundo, se tiene un desnivel de 200 metros y se quiere aprovechar construyendo una hidroelctrica a) Si la energa del agua que cae se utilizase totalmente Que potencia se podra obtener? b) Si toda la energa procedente de la cada del ro se convirtiese en energa elctrica y se vendiese a un sol el kilowatt-hora Cunto dinero se cobrara en un da? Solucin. a) El trabajo realizado por una masa m que cae desde una altura h es: W = mgh Como m = V, Donde es la densidad del agua. V es el volumen.

La potencia es:

P=
Con

dW , siendo W = Fd dt 1 2 1 at = g sen t 2 2 2

F = mg sen y d =
Tenemos

W = Vgh

La potencia que se obtiene al pie de la salida es

1 W = (mg sen ) g sen t 2 2 1 2 2 2 = mg sen t y 2 d 1 dW 2 2 2 = P= mg sen t dt 2 dt 2 2 = mg sen


Como ha recorrido la distancia s:

P=

dW d = Vgh dt dt

De estas cantidades la que vara con el tiempo es V.

dV m3 = 50 dt s
Luego

P = gh
Como

dV dt kg m , g = 9,8 2 , h = 200m 3 m s

s=

(h y ) = 1 gsen t 2
sen 2

= 1000
Obtenemos

Obtenemos:

t=
Luego

2(h y ) gsen 2
2(h y ) gsen 2

m3 kg m P = 1000 3 9,8 2 (200m ) 50 s m s


= 9,8 x 107 W b) Si tenemos una potencia P = 9,8 x 107 = 9,8 x 104 kW y consideramos que se consume las 24 horas del da. La energa obtenida es igual a todo el trabajo realizado.

P = mg 2 sen 2

= mgsen 2 g (h y ) Otra manera de obtener es considerar que:

dW = Pdt
t2 t1

P = Fv

Donde

F = mg sen y v = at = gsen t
17

W = (9,8 104 kW )(24h ) = 235,2 x 104 kW-h

W = P dt = P(t 2 t1 ) = Pt

TRABAJO Y ENERGA

Hugo Medina Guzmn

si el precio de cada kW-h es 1 sol, cada da se obtendrn 2,352 millones de soles. Ejemplo 32. En la figura, un bloque de masa m descansa sobre una faja que se mueve con velocidad constante v. El coeficiente de friccin entre el bloque y la faja es k. Tomando como tiempo inicial t = 0, una fuerza horizontal F aplicada al bloque le produce una aceleracin constante a. a) Determinar la fuerza F y la potencia disipada en friccin como funcin del tiempo. b) Si la fuerza F es ejercida por un hombre que se encuentra sobre la faja. Determinar la potencia que este libera en funcin del tiempo. e) Si la fuerza F es ejercida por un hombre que camina sobre el piso al costado de la faja. Determinar la potencia que este libera en funcin del tiempo.

MQUINAS Una mquina simple es un dispositivo usado para magnificar una fuerza o para cambiar una desplazamiento pequeo en grande. Las mquinas comunes son la palanca, el plano inclinado, el gato hidrulico, o una combinacin de engranajes. El trabajo se hace tpicamente en la mquina (el trabajo W1 de entrada), y entonces la mquina alternadamente hace un cierto trabajo W2 de salida. El estado de la energa de la mquina no cambia apreciable durante este proceso, as que si la friccin es insignificante, W1 = W2, basado en la idea de la conservacin de energa. Muy a menudo las fuerzas de entrada y de salida son constantes, en las cuales el caso W1 = W2, lo que lleva a:

F1 d1 = F2 d 2 F2 =

d1 F1 d2

Aqu F1 acta sobre una distancia d1 y F2 acta sobre una distancia d2. La ventaja mecnica de la mquina se define como

VM =
Solucin. a) Aplicando la segunda ley de Newton a la masa m en la figura

F2 F1

F F f = ma
Como

F f = k N = k mg , obtenemos:

F = ma + k mg
y la potencia disipada en friccin es P = F f v0 = ( k mg )v0 , siendo v0

= at

Ejemplo 33. La palanca de barra es un dispositivo usado para levantar objetos pesados (por ejemplo, un piano o una pieza grande de maquinaria). Consiste en una barra larga que se apoya en un fulcro una distancia corta del extremo de levantar de la barra. Suponga que el fulcro de una barra de la palanca est a 3 centmetros de la carga, y el punto donde usted empuja hacia abajo en el otro extremo est a 1,50 m del fulcro. Qu fuerza mnima tendra que ejercer para levantar una carga de 2000 N? Si mueve el extremo de la barra 4 centmetros hacia abajo, cunto levantar la carga? Solucin.

P = k mgat
b) La fuerza que hace el hombre sobre la faja es

F = ma + k mg

Su velocidad en funcin del tiempo es

v' = v + v0 = v + at
y la potencia que debe dar el hombre es

P = Fv' = (ma + k mg )at

Si la barra rota con un ngulo pequeo , entonces d1 = L1 y d 2 = L2

c) La tuerza que har el hombre sobre el piso es igual al caso anterior:

F ' = ma + k mg

La velocidad del hombre en funcin del tiempo en este caso es:

F1 L1 = F2 L2 L F1 = 2 F2 L1 (0,03) (2000) F1 = 1,50


= 40 N

v' = v + at

Luego la potencia que debe dar el hombre es:

P' = F ' v' = (ma + k mg )(v + at )

Para tringulos semejantes

d1 L1 L = d 2 = 2 d1 d 2 L2 L1
18

TRABAJO Y ENERGA

Hugo Medina Guzmn

d2 =

(0,03) (0,04) L(1,50)

= 0,008 m = 8 mm. Observe que una fuerza pequea de entrada da lugar a una fuerza grande de salida, pero el precio que se paga es que un desplazamiento grande de la entrada produce solamente un desplazamiento pequeo de salida. Ejemplo 34. Se bosqueja aqu un polipasto diferenciado de la clase usada para levantar un motor de auto. Las poleas tienen dientes que engranan con una cadena continua. Las poleas estn soldadas juntas, hay 18 dientes en la polea externa y 16 dientes en la polea interna. As cuando la polea hace una revolucin, 18 acoplamientos de la cadena se levantan y 16 acoplamientos bajan, dando por resultado la elevacin de la carga. Cul es la ventaja mecnica de esta mquina?

As el trabajo de la salida es W2 = F(L). Despreciando la friccin. W1 = W2 o F1(18 L) = F2(L) La ventaja mecnica del polipasto es VM = 18. Ejemplo 35. Un trailer est equipado de un sistema para sacar barcos del agua. Consiste en una manija larga de 30 centmetros unido al eje de un engranaje pequeo con 12 dientes. Este engranaje pequeo endienta con un engranaje ms grande con 36 dientes. Se une a este engranaje grande un tambor del radio 2 centmetros en el cual se enrolla la lnea atada al barco (la lnea es una cuerda.) Qu tensin se puede aplicar a la lnea cuando la manivela se empuja con una fuerza de 80 N? So1ucin. Considere que pasa cuando la manivela hace una revolucin. La mano mueve una distancia d1 = 2R1. El engranaje grande mueve 12/36 = 1/3 revoluciones. La lnea es jalada una distancia d2 = 2R2/3.

F1 d1 = F2 d 2 2R1 d R F1 = 3 1 F1 F2 = 1 F1 = 2R2 / 3 d2 R2
Solucin. Considere qu pasa cuando la polea superior hace una revolucin, es decir, cuando el trabajador jala 18 eslabones de la cadena hacia l con fuerza F1. Sea L = longitud de un eslabn. El trabajo de la entrada es W1 = F1(18 L). El lazo de la cadena que va bajo de la carga es acortado as por 18 eslabones y alargado por 16 eslabones, con un acortamiento neto de 18L - 16L = 2L que acorta al lazo 2L y levanta la carga L (intente esto con un pedazo de cuerda para convencerse de esta caracterstica).

30 F2 = 3 (80 ) = 3600 N. 2
La ventaja mecnica:

VM =

3600 = 45 80

La ventaja mecnica del torno (despreciando la friccin) es 45.

PREGUNTAS Y PROBLEMAS 1 Defina primero en palabras y luego en una expresin matemtica. a) El trabajo realizado por una fuerza cualquiera. b) La energa cintica de una partcula. 2 Una partcula P en el plano xy est sometida a la

x accin de la fuerza F = y i j . Calcular el trabajo efectuado por la fuerza para desplazar P sin friccin desde B (0,.b) a A (a, 0).
2 2

Respuesta.

W=

ab (a + b ) 3

3. Un depsito cilndrico de altura H tiene una masa m de agua que lo llena hasta la mitad, que ha de bombearse en su totalidad por encima del borde del mimo. Cunto trabajo ha de realizar la bomba?

19

TRABAJO Y ENERGA

Hugo Medina Guzmn

Respuesta.

W =

3 mgH 4

cos = 1 x 2 . El objeto se mueve entre 2 < x < 2 .


fuerza tambin vara tal que Cul es el trabajo realizado cuando el objeto se mueve de x = 0 a x = a? Respuesta.

4. Qu fuerza horizontal, constante debe aplicarse a un carro de masa 500 kg que viaja en una carretera horizontal a 36 km/h para que se detenga en 30 metros? Quin proporciona la fuerza? Respuesta. 2500 N, proporcionada por la carretera. 5. Un resorte est unido en A a un plano vertical fijo y a un bloque B que resbala sobre una varilla lisa horizontal Ox. La longitud del resorte no estirado es 45 cm y la constante del resorte es k = I000 N/m. Cul es el trabajo realizado por el resorte sobre B cuando se mueve 60 cm desde O por efecto de la fuerza F?

Aa + B

a2 a4 B 2 4

9. Un bloque que se mueve a lo largo del eje x comienza del reposo en x = A y se mueve a x = B luego vuelve a x = A donde queda en reposo nuevamente. Si una de las fuerzas actuante sobre el bloque es opuesta en direccin y proporcional a la magnitud de la velocidad, tal que Fv = b v con b Constante. Demostrar que el trabajo realizado por esta fuerza no es cero para una trayectoria cerrada.

+ xy j acta sobre la 10. La fuerza F = x yi partcula .P (x,y) que se mueve en el plano xy. a) Demostrar que F no es una fuerza conservativa. b) Determinar el trabajo de F cuando se mueve de A a C, a lo largo de los caminos ABC, ADC y AC.
2 2

Respuesta: 99,38 J 6. Un resorte de masa despreciable y constante k cuelga del cielorraso de un ascensor y lleva suspendido una masa in. Cuando el ascensor se mueve hacia arriba durante t segundos con una aceleracin uniforme

a=

1 g . la reaccin inercial 2
Respuesta. a) Si

hace que el resorte se alargue. a) Cunto trabajo realiza el ascensor sobre el sistema resorte-masa? b) Cunto trabajo realiza sobre el resorte? Respuesta. a)

Fx Fy a4 , b) WABC = , y x 3

WADC =

1 1 m2 g 2 mg 2 t 2 b) 8 k 4

a4 a4 . WAC = 3 2

7. En la figura se mueve el cuerpo A a lo largo de un plano horizontal liso por medio de la fuerza constante F = 5 N aplicada al extremo de una cuerda unida a A y que pasa por una pequea polea sin rozamiento en B. Calcular el trabajo realizado sobre A por la cuerda mientras A se desplaza 3 m,

11. El tubo de la figura se halla en un plano horizontal, su resorte comprimido inicialmente 10 cm.. y al dispararse una bolita entra en una canaleta circular de radio R, la friccin es constante igual a 1 Newton. Cuntas vueltas dar la bolita antes de detenerse? R= 50 cm k = 62 N/m

Respuesta. W = 120 J Respuesta. Una vuelta. 8. Una fuerza cuya magnitud vara con x de acuerdo a F = A + Bx acta sobre objeto que puede moverse solamente en el eje x. El ngulo con el que acta la 12. Se aplica una fuerza de 1 N a una partcula de 50 g que est inicialmente en reposo sobre una superficie.

20

TRABAJO Y ENERGA

Hugo Medina Guzmn

a) Cunto trabajo realiza sobre la partcula en l0 s si la superficie es lisa y la fuerza es horizontal? b) El mismo caso de a) pero la fuerza hace un ngulo de 60 con la horizontal. c) El caso b) pero con rozamiento entre la partcula y la superficie 0,25 y Cunto trabajo se consume en vencer el rozamiento? Respuesta. a) W = 1000 J , b) W = 2505 J , c) W = 143 J , W = 46 J 13. Encontrar la funcin energa potencial de un resorte si el origen se coloca en la pared y la longitud del resorte sin estirar es L.

U (r ) = 3 x 2 y +
Respuesta.

zy y2 x z y j k x x

z F = y 2 6 x i + 2 y 3x 2 x

Respuesta. Si

U(x) =

1 2 kx kLx + C 2

17. Una partcula de masa 4y penetra en una regin en la cual su energa potencial es la indicada en la figura y pasa valores grandes de x, a los cuales su energa potencial es cero, tiene una energa cintica de 16 x 10-7 J . a) Cul es su energa cintica en los puntos A, B y C? b) Estando en el punto A, la partcula pierde bruscamente la mitad de su energa total. (la grfica de la energa potencial no se altera). Describe cualitativamente el movimiento subsiguiente, dando el dominio de valores de x en el cual puede moverse la partcula.

1 2 kL 2 1 1 2 U ( x ) = k x 2 2 Lx + L2 = k ( x L ) 2 2 C=

14. Una partcula que se mueve a lo largo del eje x est sometida a la accin de una fuerza en un sistema conservativo a la que le corresponde la siguiente funcin energa potencial.

Respuesta.

U ( x ) = a + bx 2 cx 4
Determinar los coeficientes a. b y c, si se sabe que el potencial se anula en el origen, que x = 2 m en una posicin de equilibrio y que una partcula de 5 kg con una velocidad en el origen de 2 m/s queda en reposo en x = l m. Respuesta. a = 0, b = 80/7 J/m2 , c = 10/7 J/m4 15. La energa potencial entre dos molculas vecinas viene dada por:

E A = 8 10 -7 J , EB = 12 10-7 J , EC = 6 10-7 J

18. Un bloque de masa m es lanzado hacia arriba en un plano inclinado con una velocidad de magnitud friccin del bloque y el plano es . Si el bloque viaja una distancia L hasta detenerse y comienza a bajar volviendo a su posicin original. Calcular, a) El trabajo realizado por la fuerza normal durante el movimiento. b) El trabajo realizado por la fuerza de friccin durante el movimiento. c) El trabajo realizado por la fuerza de gravedad durante el movimiento. d) Encontrar L en funcin de

v0 . El ngulo del plano es y el coeficiente de

U (r ) =

A B + r 6 r12

siendo r la separacin entre las molculas. a) Cul es la fuerza entre ellas en funcin de r? b) Cul es la posicin de equilibrio de las dos molculas? c) Qu energa seria necesaria para alejarlas de su posicin de equilibrio indefinidamente? Respuesta. a)
1

v0 , y .

e) Cul es la velocidad del bloque cuando vuelve al punto inicial? Respuesta. a) 0, b) 2 mgL cos , c) 0,
2 v0 d) L = , 2 g ( cos + sen )

F(r ) = 6

A B + 12 13 , b) 7 r r

A2 2B 6 r = , c) E = 4B A
16. Hallar la fuerza conservativa que da origen a la funcin energa potencial.

e) v =

2 v0 4 gL cos

19. Se dispara un proyectil con una velocidad inicial de magnitud v0 y formando un ngulo con la

21

TRABAJO Y ENERGA

Hugo Medina Guzmn

horizontal. Usando la conservacin de la energa encontrar. a) La altura mxima alcanzada. b) La magnitud de la velocidad cuando el proyectil est a la mitad de su mxima altura. Respuesta. a) h =
2 v0 sen 2 , 2g 2

b)

v = v0

(1 + cos )
2

Respuesta. a) 0,5 mg , b) 0.3 mg , c) 2,5 mg d) 1,1 mg , e) 5,4 mg 24. Un bloque pequeo de masa m resbala partiendo de la parte superior de una esfera sin friccin de radio R. Cul es el ngulo en el que el bloque pierde contacto con la esfera.

20. Una fuerza F = 8t (t en segundos, F en Newton), acta la partcula P de masa m = 4kg durante un tiempo t = 6 s. S parte del reposo a partir del origen. a) Calcular el trabajo efectuado. b) Calcular la energa cintica al instante t. Respuesta. a) W = 2592 J, b) K = 2t4 J. 21. Un resorte de longitud l y constante k se sujeta a un bloque de masa m y al piso. Si el bloque se levanta a una altura 3 l y soltado desde el reposo. a) Cul ser la velocidad del bloque cuando est a una altura 2 l ? b) Cul ser la mxima compresin del resorte?

Respuesta. a)

cos =

2 3

k 2 Respuesta. a) v = 3 l + 2 gl , m
b)
2

kl mg 3kl 2 + 6mgl kl mg + y= + k k k

25. 1n saco se empuja suavemente por el borde de una pared en A y oscila en un plano vertical colgado del extremo de una cuerda de 4m que puede soportar una tensin mxima igual a dos veces el peso del saco. a) Determinar la altura a la que se rompe la cuerda. b) A qu distancia de la pared vertical caer al .suelo el saco?

22. Dos placas cuyas masas son m1 y m2, respectivamente, estn conectadas por un resorte. Qu fuerza deber aplicarse a la placa superior para elevar la placa inferior despus que se retira la presin? No tomar en cuenta la masa del resorte.

Respuesta. a) F > (m1 + m 2 )g 23. Una bolita de masa m desliza a partir del reposo hacia abajo por un carril doblado como se muestra en la figura, el rozamiento es despreciable, hallar: a) La reaccin normal del carril en A. b) La energa cintica de la bolita en B. c) La reaccin normal del carril en 8. d) La energa cintica de la bolita en C. e) La reaccin normal del carril en C.

Respuesta. a) y = 1,33 m 26. Una bola pequea de masa m = l g desliza hacia el fondo de un valle movindose sin rozamiento como se indica en la figura. Partiendo del reposo, la bola cae desde una altura h = 2m y abandona el fondo del valle formando un ngulo con la horizontal. En el punto ms elevado de su trayectoria la bola choca con un resorte montado sobre una pared y lo comprime 2 cm. La constante del resorte es k = 49 N/m. a) A qu altura y est el resorte? b) Cual es el ngulo ? 22

TRABAJO Y ENERGA

Hugo Medina Guzmn

a) Considerando que la fuerza de rozamiento es independiente de la velocidad, calcule su valor medio. b) Qu potencia debe consumirse para mantener el automvil en movimiento con una velocidad de 48 km/h? Respuesta. a) Ff = 110 N b), P = 2 hp Respuesta. a) y =1 m, b)

= 45

27. Una bola de acero de masa 1 kg est unida a un extremo de un alambre de 1m de largo y gira alrededor del otro extremo con una velocidad angular de 120 rpm. Cul es la energa cintica de la bola? Respuesta. 78,88 J 28. La faja transportadora de la figura se mueve con una velocidad constante v 0 y descarga los paquetes sobre la rampa AB. El coeficiente de rozamiento entre los paquetes y la rampa es 0,30. Sabiendo que los paquetes deben alcanzar el punto B con una velocidad de 4 m/s, determinar la velocidad v 0 requerida en la faja transportadora.

33. Un automvil de 1500 kg se desplaza 200 m mientras es acelerado uniformemente desde 50 hasta 73 km/h. Durante todo el movimiento el automvil se desplaza sobre una carretera horizontal, y la resistencia al movimiento es igual al 2 por ciento del peso del automvil. Determinar: a) La mxima potencia requerida. b) La potencia requerida para mantener la velocidad constante de 75 km/h. Respuesta. a) 25 kW , b) 6,13 kW 34. Un peso D y el contrapeso C tienen cada uno una masa de 350 kg. Determinar la potencia requerida cuando el peso: a) Se mueve hacia arriba con velocidad constante de 4m/s. b) Tiene una velocidad instantnea de 4m/s hacia arriba y una aceleracin hacia arriba de 0,9 rn/s2.

Respuesta. 3,02 m/s 29. Una locomotora ejerce un tiro constante en la barra de traccin de 160000 N mientras aumenta la velocidad de 48 a 72 km/h. Cul es la potencia que desarrolla la locomotora: a) al comienzo del periodo? b) al final del periodo? c) Cules la potencia .media durante el periodo? Respuesta. a) 2859 hp , b) 4290 hp c) 3574 hp 30. Una gra industrial puede levantar su mxima permitida de 25 toneladas a la velocidad de 20mm/s. Sabiendo que la gra es movida por un motor de 10 kW. Determinar su rendimiento. Respuesta. 49% 31. Cul es la velocidad mxima la que un motor capaz de suministrar 10 kW puede elevar un ascensor de masa 500kg, sin tomar en cuenta las fuerzas de rozamiento? Respuesta. v = 2,0 m/s 32. Si a un automvil de masa 1000 kg que se mueve sobre una carretera horizontal con una velocidad de 48 km/h se le apaga al motor, este recorre an 0,8 km antes de detenerse. carga Respuesta. 6,86 kW , 8.44 kW 35. Un bloque de 0,50 kilogramos es sujetado contra el resorte por una fuerza externa horizontal de 36 N. Se quita la fuerza externa, y el bloque se proyecta con una velocidad v1 = 1,2 m/s a partir de la separacin del resorte. El bloque desciende una rampa y tiene una velocidad v2 = 1,8 m/s en la base. La pista es sin friccin entre los puntos A y B. El bloque ingresa a una seccin rugosa en B, extendiendo hasta E. El coeficiente de friccin cintica es 0,30. La velocidad del bloque es v3 = 1,4 m/s en C. El bloque se mueve hasta C donde se detiene.

23

TRABAJO Y ENERGA

Hugo Medina Guzmn

38. Un cajn de 100 kilogramos est en una superficie spera inclinada 30. Una fuerza externa constante P de 800 N se aplica horizontalmente al cajn. La fuerza empuja el cajn una distancia de 3,0 m arriba de la pendiente, en un intervalo del tiempo de 2,0 s, y la velocidad cambia de v1 = 0,8 m/s a v2 = 2,2 m/s. a) La constante del resorte es: b) La compresin inicial del resorte en cm es: c) La altura h de la rampa en cm es: d) El trabajo realizado por la friccin entre los puntos B y C es: e) La distancia s que el bloque viaja entre los puntos B y D es: Respuesta. a) 1800 N/m, b) 2,0, c) 9, d) -0.32 J e) 0,55 m 36. Una barra sin masa de 1,5 m se fija libremente a un pivote sin friccin en O. Una bola de 3,0 kilogramos se une al otro extremo de la barra. La bola se sostiene en A, donde la barra hace un ngulo 30 sobre el horizontal, y se lanza. El montaje de la bola-barra puede girar libremente en un crculo vertical entre A y B

a) El trabajo realizado por el peso es: b) El trabajo realizado por la fuerza de friccin es: c) El trabajo realizado por la fuerza normal es: d) La potencia media producida por la fuerza externa P durante los 2,0 segundos es: Respuesta. a) -1500 J, b) - 400 J c) Cero , d) 1050 W 39. Una muchacha lanza una piedra de un puente. Considere las maneras siguientes que ella puede lanzar la piedra. La velocidad de la piedra con la que lanza es igual en cada caso. Caso A: Lanzada derecho para arriba. Caso B: Lanzada derecho para abajo. Caso C: Lanzada con ngulo de 45 sobre horizontal. Caso D: lanzada horizontalmente. En qu caso la velocidad de la piedra ser mayor cuando llega al agua? Respuesta. la rapidez es la misma en todos los casos. 40. Para hacer el trabajo sobre un objeto, A) es necesario que haya friccin. B) es necesario que no haya friccin. C) el objeto debe moverse. D) la fuerza que hace el trabajo debe estar dirigida perpendicularmente al movimiento del objeto. E) la fuerza aplicada debe ser mayor que la fuerza de la reaccin del objeto. Respuesta. C) el objeto debe moverse. 41. Un bloque de 8,0 kilogramos se lanza del reposo, vl = 0 m/s, en una pendiente rugosa. El bloque se mueve una distancia de 1,6 m abajo de la pendiente, en un tiempo de 0,80 s, y adquiere una velocidad de v2 = 4,0 m/s.

a) La bola pasa a travs de C, donde la barra forma un ngulo de 30 debajo de la horizontal. La rapidez de la bola cuando pasa por C es: b) la tensin en la barra cuando la bola pasa por el punto ms bajo D es: Respuesta. a) 5,4 m/ s, b) 120 N 37. Una fuerza externa constante P =120 N se aplica a una caja de 20 kilogramos, que est en una superficie horizontal spera. La fuerza empuja la caja una distancia de 8,0 m, en un intervalo del tiempo de 4,0 s, y la velocidad cambia de v1 = 0,5 m/s a v2 = 3,5 m/s.

a) El trabajo realizado por la fuerza externa es: b) El trabajo realizado por la friccin es: c) La razn de cambio promedio de la energa cintica de la caja, en los 4,0 segundos es: Respuesta. a) 830 J, b) -700 J, c) 30W 24

a)) El trabajo realizado por el peso es: b) La razn promedio a la cual la fuerza de friccin realiza trabajo en el intervalo de tiempo de 0,80 s es:

TRABAJO Y ENERGA

Hugo Medina Guzmn

c) La razn promedio a la cual la fuerza normal realiza trabajo en el intervalo de tiempo de 0,80 s es: d) La razn promedio a la cual el bloque gana energa cintica durante el intervalo de tiempo de 0,80 s es: Respuesta. a)) + 80 J, b) - 20 W, c) Cero, d) 80 W 42. Una persona de 60 kilogramo cae desde el reposo uno distancia 1,20 m sobre una plataforma de masa insignificante apoyada sobre un resorte duro. La plataforma baja 6 cm antes de que persona vuelva al reposo. Cul es la constante del resorte? Respuesta. 4,12 x 105 N/m 43. Un objeto est sujeto a una fuerza restauradora F = 6x3, donde x es el desplazamiento del objeto desde su posicin de equilibrio. Qu trabajo debe realizarse para mover al objeto desde x = 0 x = 0,15 m? Respuesta. 7,59 x 10-4 J 44. Dos resortes idnticos tienen longitudes sin estirar de 0,25 m y las constantes de la fuerza de 200 N/m. Los resortes se unen a un bloque pequeo y se estiran a una longitud de 0,30 m como en la figura A. Una fuerza externa P tira del bloque 0,02 m a la derecha y lo sostiene all. (Vase La Figura B)

a) El resorte 2 estira 0,06 m. La constante de fuerza del resorte 2 es: b) La constante de fuerza del resorte 1 es igual a 30 N/ m. La longitud sin estirar del resorte 1 es: Respuesta. a) 20 N/m, b) 0,53 m 46. Una barra ligera de 0,80 m se fija libremente a un eje vertical en A. Un disco de 2,0 kilogramos se une a la barra en B. Un resorte se une a la masa en B y a la manga en el eje en C. A La manga es sin friccin, permitiendo que se baje y suba libremente, de modo que el resorte sea siempre horizontal cuando est estirado. La longitud del resorte sin estirar es 0,45 m y la constante es 210 N/m.

a) El trabajo requerido para ensamblar los resortes y el bloque (figura A) es : b) La fuerza externa P, que mantiene al bloque en su lugar (figura B) es: c) El trabajo realizado por la fuerza externa P en jalar el bloque 0,02 m es: Respuesta. a) 0,50 J, b) E) 8 N, c) 0,08 W 45. El bloque A (0,40 kg) y el bloque B (0,30 kg) estn sobre una mesa sin friccin. El resorte 1 conecta al bloque A a una varilla sin fricin O y el resorte 2 conecta el bloque Ay el bloque B. Los bloques estn en movimiento circular uniforme alrededor de o, y los resortes tienen longitudes de 0,60 m y 0,40 m, como se muestra. La velocidad lineal del bloque B es 2.0 m/s.

a) El eje est girando y el resorte estirado tiene una longitud de 0,48 m. La aceleracin radial del disco es: b) El eje est girando y la varilla forma un ngulo de 40 con el eje. El resorte est estirado y horizontal. La aceleracin radial del disco es: c) El eje est girando y el resorte tiene una longitud de 0,45 m. La aceleracin radial del disco es: Respuesta. a) 10,5 m/s2 b) 15,0 m/s2, c) 6,7 m/ s2 47. Cierto coche que viaja 20 resbalones del mph a una parada en 20 metros del punto donde los frenos fueron aplicados. En qu distancia el coche parara aproximadamente la tena que va 40 mph? Respuesta. 80 metros 48. Un motor de la arena en una mina levanta 2.000 kilogramos de la arena por minuto una distancia vertical de 12 metros. La arena est inicialmente en el resto y se descarga en la tapa del motor de la arena con la velocidad 5 m/s en un canal inclinado de cargamento. En qu tarifa mnima se debe la energa proveer a esta mquina? Respuesta. 4,34 kW 49. La constante de un resorte es 500 N/m y su longitud sin estirar es 0,60 m. Un bloque de 4,0 kilogramos se suspende del resorte. Una fuerza 25

TRABAJO Y ENERGA

Hugo Medina Guzmn

externa tira hacia abajo lentamente el bloque, hasta que el resorte se ha estirado a una longitud de 0,72 m. se quita y el bloque sube. a) La fuerza externa sobre el bloque es: b) Cuando el resorte se ha contrado una longitud de 0.60 m, la velocidad del bloque hacia arriba es: : c) Cuando el resorte se ha contrado una longitud de 0.66 m, la aceleracin del bloque incluyendo su direccin es: Respuesta. a) 20 N, b) 0,4 m/ s, c) 2 m/s2, haca abajo 50. la constante de un resorte es 200 N/m y su longitud sin estirar es 10 centmetros. El resorte se pone dentro de un tubo liso de 10 centmetros de alto (la figura a). Un disco de 0,40 kilogramos se coloca sobre el resorte (figura b). Una fuerza externa P empuja el disco hacia abajo, hasta que el resorte tiene 4 centmetros de largo (la figura c). Se quita la fuerza externa, el disco se proyecta hacia arriba y emerge del tubo (figura d).

a) La compresin del resorte en la figura b es: b) La fuerza externa P en la figura c es: c) La energa potencial elstica del resorte en la figura c es: d) La aceleracin inicial del disco cuando la fuerza externa es removida es: e) La velocidad v del disco cuando emerge del tubo en la figura d es: Respuesta. a) 2,0 , b) 8N, c) 0,36 J, d) 20 m/s2, e) 0,80m/s

26

Sistema de partculas

Hugo Medina Guzmn

CAPTULO 6. SISTEMA DE PARTCULAS


INTRODUCCIN Hasta ahora hemos estado estudiando el movimiento de los objetos cualquiera que sea sin considerar su estructura. Ahora demostraremos que lo estuvimos haciendo bien considerando al objeto sin tomar en cuenta las fuerzas que actan sobre sus partes. Introduciremos el concepto de centro de masa de un sistema de partculas, tambin se introducir el concepto de cantidad de movimiento y se demostrar que este se conserva cuando el sistema se encuentra aislado de los alrededores, SISTEMA DE PARTICULAS La seleccin del contorno de un sistema es similar a seleccionar un sistema de coordenadas. SEGUNDA LEY DE NEWTON APLICADA A UN SISTEMA DE PARTICULAS La figura siguiente muestra un sistema de n partculas de masas m1, m2, ..mn, con posiciones especificadas

por r 1 , r 2 , . r n ,, respectivamente.

La segunda ley de Newton para la partcula mi es:

F i = m a i = F iexter + F i int
Donde: La figura muestra un sistema de partculas compuesto de tres masas. En el sistema existen dos tipos de fuerzas, a) Las fuerzas externas como la atraccin gravitacional de la tierra por ejemplo. b) Las fuerzas internas que las partculas ejercen unas sobre otras (estas fuerzas pueden ser gravitacionales, e1ctricas, etc.)

F i int = suma de las fuerzas internas sobre mi

F i ext = suma de las fuerzas externas sobre mi


La suma de las fuerzas internas sobre la masa mi es:

F 1int = F 12 + F 13 + ............ F 12 = Fij


( j i )

En general para la partcula i es:

F i int = Fij
( j i )

La fuerza total para el sistema es:

Fi = m a i = F i ext + Fij
i =1 i =1 i =1 i =1 ( j i )

i =n

i =n

i =n

En la figura hemos cambiado el contorno del sistema, excluyendo la masa m3. Como Una Consecuencia de esto las fuerzas internas Sobre m1 y m2 debido a m3 ya no son internas, se han sumado a las fuerzas externas previas, produciendo una nueva fuerza resultante. 1

Por la tercera ley de Newton cada una de las fuerzas


Fij tiene un F ji igual, pero de sentido contrario

Sistema de partculas

Hugo Medina Guzmn

Fij = F ji
De modo que

xCM = lim

1 mi 0 M

x m
i =1 i

1 xdm M

i = 0 j i )=1

Fij = 0

De igual forma se obtiene:

Consecuentemente solo queda

mi ai = Fi ext o
i =1 i =1

n d2 n m r = F ii i ext dt 2 i =1 i =1

CENTRO DE MASA Frecuentemente es muy prctico reemplazar un sistema de muchas partculas con una partcula simple equivalente de masa igual. La pregunta es donde colocar esta partcula simple con respecto al origen de x e y. Definamos el vector posicin del centro de masa por la ecuacin:

1 n 1 y i mi = ydm , mi 0 M M i =1 1 n 1 z CM = lim z i mi = zdm y mi 0 M M i =1 1 rCM = r dm M y CM = lim


MOVIMIENTO DEL CENTRO DE MASA.
n d2 n m r = Fiext 2 i i dt i = 0 i =1

Si en la ecuacin:

Sustituimos

m
i =1

ri = M rCM

rCM =

m
i =1 n i =1

ri
i

Obtendremos la ecuacin del movimiento del centro de masa


n n d2 M r = F M a = F CM iext CM iext dt 2 i =1 i =1
i

m
n

Llamando a partculas).

m
i =1 i

= M (masa total de las n

El punto indicado por rCM , vector posicin del centro de masa, se mueve se mueve como si en el estuviera concentrada toda la masa y las fuerzas externas del sistema. Ejemplo 1. Centro de masa de tres masas puntuales.

rCM =

m
i =1

ri

M
+ y CM rCM = xCM i j + z CM k

Como

Tenemos que: xCM =

1 M

m x ,
i =1 i i

yCM =

1 M

mi yi , zCM =
i =1

1 M

m z
i =1

i i

El centro de masa esta dado por: Si hacemos que el nmero de elementos n, se aproximen al infinito, la sumatoria se reemplaza por una integral y m por el elemento diferencial dm. Luego. = 2

xCM =

1 M

m x
i =1

i i

m(1) + 2m(1) + 3m(2) m + 2m + 3m

Sistema de partculas

Hugo Medina Guzmn

9m 3 = 6m 2 1 M

Sustituyendo:

xCM = 2 = 2 a
2 a2
a

2 a b 2 x (a x )dx = 2 0 ab a a
2 ax x dx = 2 a
2

x(a x )dx
a
0
a

yCM =

mi yi
i =1

(
0

x2 x3 + a 3 0 2

m(1) + 2m(3) + 3m(2 ) m + 2m + 3m 13m 13 = 6m 6 3 13 i+ j 2 6

a3 a3 a = 3 2 + 3

Realizando clculos similares encontramos:

y CM =

rCM =

b 3

Finalmente: rCM =

Ejemplo 2. Centro de masa de un tringulo.

a b i+ j 3 3

Ejemplo 3. Centro de masa de un arco semicircular.

xCM =

1 M

xdm

Por el sistema de coordenadas escogido, xCM = 0 , porque por cada elemento de masa a la derecha (+), existe otro elemento igual a la izquierda (-). Sin embargo para y CM es diferente.

Para evaluar

dm =

masa total rea de la lmina rea total

M 2M ydx = ydx = 1 ab ab 2
Luego: xCM =

y CM =

1 M

ydm , en este caso dm = dl


M y dl = Rd R

Donde

1 M

xdm =

1 M

2M ydx ab

Como y = Rsen , tenemos:

2 a = xydx ab 0
Para poder integrar tenemos que expresar la variable y en funcin de x. Por semejanza de tringulos:

y CM =

1 M

0 (Rsen )Rd
R 2
M 2R

R 2
M

send

[ cos ]0

R2 M = (2 ) M R

b y ax y = (a x ) = a b a
3

= 0,64 R

Sistema de partculas

Hugo Medina Guzmn

El centro de masa no se encuentra dentro del cuerpo. Las figuras siguientes muestran como localizar experimentalmente el centro de masa primero colgndolo de la parte superior y luego de otro punto cualquiera.

Despus que P1 y P2 se separan, las velocidades


respectivas son v '1 y v ' 2 diferentes de v1 , y v 2 .

Ejemplo 4. Explosin de una granada

Ahora nos preguntamos. Qu pasa durante el choque? El tiempo de contacto total t es muy pequeo, quizs solo de aproximadamente 0,001 segundos. La fuerza de contacto inicialmente es cero, aumenta hasta un valor muy grande y. finalmente disminuye hasta cero, cuando dejan de estar en contacto. La figura siguiente muestra una variacin tpica de la fuerza en el tiempo de contacto.

Una granada lanzada al aire que explota en varios fragmentos. La nica fuerza externa sobre la granada es la fuerza de la gravedad, entonces la granada sigue una trayectoria parablica. Si la granada no estallara continuara movindose a lo largo de la trayectoria parab1ica indicada en la figura. Como las fuerzas de la explosin son internas, no afectan al movimiento del centro de masa. Entonces. Despus de La explosin el centro de masa de los fragmentos sigue la misma trayectoria que tendra la granada s! no hubiera habido explosin. IMPULSO Y CANTIDAD DE MOVIMIENTO Supongamos el caso de dos partculas esfricas P1 y P2 de masas m1 y m2 con trayectorias contenidas en la misma recta, se aproximan una a otra con velocidades

Sea t f t i = t el tiempo que dura el choque, aplicando la segunda ley de Newton a las partculas P1 y P2.

v1 , y v 2 respectivamente.

dv F12 = m1 a1 = m1 1 y dt d v2 F21 = m2 a 2 = m2 dt
Cuando P1 y P2 entran en contacto, P1 ejerce sobre P2 la fuerza F12 y P2 ejerce sobre P1 la fuerza F21. De acuerdo con la tercera ley de Newton F12 = F21 .

O F12 dt = m1 d v1 y F21 dt = m2 d v 2 Integrando las dos relaciones durante el choque,

Sistema de partculas

Hugo Medina Guzmn

tf

ti tf

F12 dt = m1 d v1 y
v1

v '1

La partcula P1 ha sufrido en el intervalo t f t i = t , un cambio de la cantidad de movimiento

ti

F21 dt = m2 d v2
v2

v '2

tf

ti

F12 dt = m1 v'1 v1 = p f pi

Finalmente

tf

ti tf

ti

F12 dt = m1 v'1 v1 y F21 dt = m2 v' 2 v 2


y esta cantidad es tambin igual al impulso recibido en ese instante por la partcula

J = p f pi

Trabajando con el primer miembro

Luego: El cambio de la cantidad de movimiento es igual al impulso. Ejemplo 5. Una pelota de 100 gramos est en reposo sobre el piso, cuando recibe un puntapi que la lanza con una velocidad de 30 m/s. a) Qu impulso se dio a la pelota? b) Si el tiempo que el pie est en contacto con la pelota es 10-3 segundos. Cul es la magnitud aproximada de la fuerza impulsiva? Solucin. a) El impulso es igual al cambio de la cantidad de movimiento:

tf

ti

Fdt corresponde al rea bajo la curva mostrada

en la figura anterior, a sta cantidad la llamaremos IMPULSO J


J = F(t ) dt
ti

tf

Sus dimensiones son: [F] [T] = [M][L][T]-1 En el sistema internacional sus unidades son: Newton.segundo (N.s) Trabajando con el segundo miembro

J = p f p i = m v f m vi

En este caso

m1 v'1 v1 y m2 v' 2 v 2
Llamaremos a la cantidad

m/s m = 0,1 kg, vi = 0 , v f = 30i

mv = p,

J = (0,1)(30& i&) 0 = 3i

kg m s

CANTIDAD DE MOVIMIENTO LINEAL o Momentum lineal de la partcula (lo designaremos en la prctica simplemente como cantidad de movimiento), cuyas dimensiones son: [M] [[L]] = [M] [L] [T]-1 En el sistema internacional sus unidades son: kg.m.s-1

b) Se puede obtener un estimado de la fuerza que acta sobre la pelota, dividiendo e1 impulso tiempo t = t f t i en que acta la fuerza :

J por el

F=

J t

Como

J = 3i

kg m y t = 0,001 s s

Sistema de partculas

Hugo Medina Guzmn

F=

3i N = 3000i 0,001

Usando la expresin de centro de masa

Ejemplo 6. Se deja caer una pelota de masa m de una altura h sobre el nivel del suelo y rebota hasta una altura h1 a) Cul es la velocidad chocar con el suelo? b) Cul es la velocidad de chocar con el suelo?

m v
i =1 i

= M v CMi
n

De aqu

p total = mi vi = M v CMi
i =1

vi inmediatamente antes de

v f inmediatamente despus

La cantidad de movimiento total de un sistema es igual a la cantidad de movimiento de la masa total concentrada en el centro de masa del sistema. Derivando nuevamente la expresin anterior:

c) Cul es el impulso J que se le da a la pelota en el impacto con el suelo? Solucin. a) Como v 0 = 0 , x = 0 ,

d d p total = M v i CM = M a iCM = F i ext dt dt

Esta cantidad es muy importante, ya que si no hay fuerza externa,

y = h0

vi = 2 gh0 j

F i ext = 0

d p total = 0 dt

b) Como despus de chocar y = h1 , la velocidad

p total = CONSTANTE
Esto es la conservacin de la cantidad de movimiento. Si no hay fuerzas externas sobre un sistema. La cantidad de movimiento total del sistema es constante. Ejemplo 7. Tres partculas de masas 2 kg, 1 kg y 3 kg respectivamente con vectores posicin

v f despus de chocar es: v f = 2 gh1 j


c) El impulso de la pelota es:

J = m v f m vi = m 2 g

h1 h0 j

CONSERVACIN DE LA CANTIDAD DE MOVIMIENTO La cantidad de movimiento de una partcula de masa

[ ] ]cm (12 5t ) r = [(2t 3)i j + (4 + 6t 3t )k ]cm (t + 2 ) y r = [(12t 1)i j t k


cm , 5t 2 r1 = 5ti j + (3t 2)k

2 2 3

Donde t es el tiempo en segundos. Encontrar: a) La velocidad del centro de masa en t = 1 s y t = 2 s. b) La cantidad de movimiento lineal total del sistema en t = 1 s y t = 2 s. c) Analizar si el sistema de tres partculas es sistema aislado Solucin.

m y velocidad

v es:

p = mv

La cantidad de movimiento de n partculas es la suma de las cantidades de movimiento individuales,

p total = p i = mi vi
i =1 i =1

Sistema de partculas

Hugo Medina Guzmn

a) La posicin del centro de masa esta dada por la expresin:

ayudante ahogndose se encontrar el pescador cuando alcance el extremo del bote? Solucin. Consideremos aislado el sistema bote, pescador, ayudante, por lo tanto su cantidad de movimiento es constante.

rCM

m r + m2 r2 + m3 r3 = 1 1 m1 + m2 + m3

Reemplazando valores, obtenemos:

cm + t2 + 3 rCM = (3t 1)i j + t 3 + 2t k


La velocidad del centro de masa es

) (

)]

p = M v cm = CONSTANTE

Como en inicio el sistema est en reposo:

vCM =

d cm 2t rCM = 3i j + 3t 2 + 2 k dt s

)]

p = 0 v cm = 0

Para t = l s

v1M

cm 2 = 3i j + k s

Como v cm

d r cm = =0 dt

Para t = 2 s

r cm = CONSTANTE , la posicin del centro de masa permanece constante


En ste problema que es en una sola dimensin:

v2 M

cm 4 = 3i j + 8k s

xcm = CONSTANTE
Tomemos como punto de referencia la posicin del ayudante en el extremo del bote, al soltarse seguir en la misma posicin. Analicemos la posicin inicial.

b) La cantidad de movimiento del sistema es:

p = + m1 v1 + m2 v 2 + m3 v3 = M vCM

kg cm 2t p = 6 3i j + 3t 2 + 2 k s

)]

Para t = l s
kg cm 2 p1 = 6 3i jk s

El centro de masa del sistema pescador-bote est en:

Para t = 2 s
kg cm 4 p2 = 6 3i j 8k s

xcm =

mb xb + m p (2,5) mb + m p

Analicemos la posicin final. c) Como , p1 p 2 , p no es constante, luego el sistema no es aislado. Ejemplo 8. Un pescador de masa 70 kg est en un bote estacionario de masa 200 kg, cuando su ayudante que no sabe nadar y est en el agua cogido del extremo opuesto, se suelta. El pescador corre 2,5 m hasta alcanzar este extremo. A que distancia del 7

El centro de masa esta en:

Sistema de partculas

Hugo Medina Guzmn

xcm =

mb ( x b + x ) + m p x mb + m p

EI muchacho sale con una velocidad de mdulo

v1 =

Como la posicin del centro de masa del sistema es invariante, se tiene:

m2 v dirigida hacia el oeste, m1

mb xb + m p (2,5) mb + m p

mb ( xb + x ) + m p x mb + m p

(mb + m p )x = m p (2,5)
Reemplazando valores:

Ejemplo 10. Dos personas de masa m cada una, se encuentran paradas en los extremos opuestos de un bote de longitud d y masa 3m que se encuentra en reposo sobre un lquido sin friccin, tal como se muestra en la figura. Las personas caminan una hacia la otra con rapidez constante y se encuentran a d/4 del extremo izquierdo del bote. a) Si la persona de la izquierda se mueve con velocidad v 0 respecto al bote, cul es la velocidad que tiene la otra persona, respecto al bote? b) Cul es la velocidad del bote, respecto a tierra, durante el movimiento de ambas personas? c) Cunto avanzo el bote hasta el momento del encuentro?

x=

70(2,5) = 0,65m (200 + 70)

La posicin del pescador estar a 0,65 metros del ayudante. Ejemplo 9. Un muchacho de masa m1 y una muchacha de masa m2 , ambos con patines, se encuentran en reposo uno en frente del otro, El muchacho empuja a la muchacha, mandndola hacia

Solucin.

el este con una velocidad v . Describa el movimiento del muchacho. Solucin. Siendo un sistema cerrado la cantidad de movimiento se conserva,

p antes = p despus = 0 ,

a) El tiempo empleado para encontrarse es el mismo para las dos personas

v0

4=

3d v1

v1 = 3v 0 Hacia la izquierda

Si v 1 y v 2 son las velocidades del muchacho y la muchacha despus del empujn, respectivamente:

b) Por conservacin de la cantidad de movimiento

p antes = p despus p antes = 0

m1 v 1 + m2 v 2 = 0
Considerando el movimiento en el eje x, y la direccin al este como sentido positivo

+ m( 3v0 + vb )i + 3mvb i =0 p despus = m(v 0 + vb )i

v 2 = v = vi

vb =
c)

2 v0 i 5
d , 4v0

De aqu

El tiempo de caminata de las personas es t =


= 0 v1 = m1 v 1 + m2 vi

m2 vi m1
8

luego el bote se habr movido

Sistema de partculas

Hugo Medina Guzmn

2 d x = vb t = v0 5 4v 0

d = 10

Como

vCM

m v + m2 v 2 = 1 1 m1 + m2

SISTEMA DE REFERENCIA CENTRO DE MASA Cuando la fuerza externa resultante que acta sobre un sistema es cero, la cantidad de movimiento total es constante. Muchas veces es conveniente escoger un sistema de coordenada., con el origen situado en el centro de masa. Este sistema se denomina SISTEMA DE REFERENCIA CENTRO DE MASA Con respecto a este sistema la velocidad del centro de masa por supuesto es cero y la cantidad de movimiento total es cero. El anlisis de la mayor parte de los choques es ms sencillo en el sistema de referencia centro de masa. La transformacin de un sistema de referencia cualquiera a un sistema centro de masa no es difcil. Consideremos un sistema do dos partculas m1 y m2

m v + m2 v 2 u1 = v1 1 1 m1 + m2

m2 m1 + m2

v1 v 2

m v + m2 v2 y u 2 = v2 1 1 m1 + m2

m1 m1 + m2

v1 v 2

Como comprobacin, calculemos la cantidad de movimiento total con respecto al centro de masa, el cual debe ser igual a cero.

con velocidades v1 y v 2 respectivamente cuyo

p = m1 u 1 + m2 u 2
= p = m1

centro de masa se mueve con velocidad vCM , como se muestra en la figura.

m2 v1 v 2 m1 + m2

= m2 La cantidad de movimiento es:

m1 v1 v2 = 0 m1 + m2

p = m1 v 1 + m2 v 2 = (m1 + m2 ) v CM

En la seccin siguiente veremos ejemplos de aplicacin usando el sistema de referencia centro de masa. CHOQUES

Para transformar esta expresin al sistema Centro de masa, las velocidades de las partculas con respecto al centro de masa son como se muestra en la figura siguiente.

Se llama choque o colisin entre dos cuerpos a un fenmeno en el que los cuerpos Participantes son libres antes y despus de la interaccin, sobre los que no actan fuerzas resultantes. La interaccin dura un tiempo muy corto, durante el cual los cuerpos ejercen entre si fuerzas de cierta intensidad.

Las velocidades relativas al centro de masa son:

Por lo general en los choques s1o participan dos cuerpos, aunque esto no es estrictamente necesario.

u1 = v1 vCM y u 2 = v 2 vCM

Sistema de partculas

Hugo Medina Guzmn

Sean dos cuerpos de masas m1 y m2 con velocidades


Para la masa m 2 :

v1 y v 2 antes del choque y velocidades v'1 y v'2


despus del choque respectivamente. En todo choque entre dos cuerpos se conserva la cantidad de movimiento, esto es:

t0

ti

F2 dt = m2 v0 m2 v 2 = J 2 D = J 1D

Resolviendo para v1 y v 2 .
J J v1 = 1D + v 0 , v 2 = 1D + v0 m1 m2

p = p' p1 + p 2 = p'1 + p' 2


m1 v1 + m2 v 2 = m1 v'1 + m2 v' 2
Ahora nos introduciremos en el proceso complejo que acompaa al choque, el instante t = t f t i , en el que aparece la fuerza de interaccin, este periodo vamos a dividirlo en dos partes, los periodos de deformacin y restitucin. La figura muestra el grfico de la fuerza de interaccin en funcin del tiempo entre las masas m1 y m2.

La diferencia de estas velocidades es:


1 m +m 1 2 1 = + J v 2 v1 = J 1D 1D m m m m 2 1 1 2

Ahora aplicaremos la ecuacin Impulso-cantidad de movimiento por el periodo de restitucin (R). t0 t f . Para la masa m1 :


E1 tiempo t 0 es el instante de mxima deformacin en el que empieza la restitucin y las dos masas poseen la misma velocidad

t0

ti

F1 dt = m1 v'1 m1 v0 = J R

Para la masa m 2 :
t0 ti

F2 dt = m2 v' 2 m2 v 0 = J 2 R = J 1R

Resolviendo para v '1 y v ' 2 .


J J v'1 = 1R + v0 , v' 2 = 1R + v 0 m1 m2

v01 = v02 = v0
Vamos a aplicar la ecuacin impulso - cantidad de movimiento para el periodo de deformacin (D), ti t 0 : Para la masa m1 :

La diferencia de estas velocidades es:


1 1 v' 2 v'1 = J 1R m + m 2 1

= J 1R

t0

ti

F1 dt = m1 v 0 m1 v1 = J D

m1 + m2 m1 m2

De lo visto encontramos la relacin entre el impulso de restitucin y el impulso de deformacin.

10

Sistema de partculas

Hugo Medina Guzmn

J 1R

J 1D

(v' v' ) = 2 1 = (v 2 v1 )

a) Choque perfectamente elstico. En este caso no hay prdida en la energa mecnica asociada al impacto, la energa cintica permanece constante.

A esta relacin se le conoce como coeficiente de restitucin ( ) . Esta relacin fue propuesta por Newton y tiene validez solamente aproximada. EI valor de esta relacin depende de muchos factores tales como la geometra, las propiedades de los materiales, la velocidad, por ello debemos contentarnos con una determinacin experimental. Ejemplo 11. Una pelota de bisbol de 0,15 kg de masa se est moviendo con una velocidad de 40 m/s cuando es golpeada por un bate que invierte su direccin adquiriendo una velocidad de 60 m/s, qu fuerza promedio ejerci el bate sobre la pelota si estuvo en contacto con ella 5 ms?. Solucin. Datos: m = 0,15 kg vi = 40 m/s vf = - 60 m/s (el signo es negativo ya que cambia el sentido) t = 5 ms = 0,005 s p = J pf - pi = J mvf - mvi = F t F = m(vf - vi)/t F = 0,15 kg.(- 60 m/s - 40 m/s)/0,005 s = 0,15 kg.(- 100 m/s)/0,005 s = - 3000 N CASOS DE CHOQUE Perfectamente elstico

K 1 + K 2 = K '1 + K ' 2

1 1 1 1 2 m1v12 + m2 v 2 = m1v'1 + m2 v' 2 2 2 2 2 2


2 2 m1 v'1 v12 = m2 v' 2 2 v 2

Por conservacin de la cantidad de movimiento tenemos:

p1 + p 2 = p'1 + p' 2 , m1 v1 + m2 v 2 = m1 v'1 + m2 v' 2


m1 v1 v'1 = m2 v' 2 v 2
Asumiendo que el movimiento es en una sola direccin

m1 (v1 v'1 ) = m2 (v' 2 v 2 )


Dividiendo entre si las expresiones halladas por energa y por cantidad de movimiento obtenemos.

v1 + v'1 = v' 2 +v 2 (v' 2 v'1 ) = (v 2 v1 )

(v' 2 v'1 ) =1 (v 2 v1 )

= 1 , (v' 2 v'1 ) = (v 2 v1 )
Inelstico

El cual es por supuesto el coeficiente de restitucin de un choque perfectamente elstico = 1 . b) Choque perfectamente plstico. En un choque perfectamente Plstico, despus del choque las masas quedan juntas, es decir tienen la misma velocidad, tal que

<1

El coeficiente de restitucin y tiene un valor entre 0 y 1. Perfectamente plstico

v' 2 = v'1 , por lo tanto: v' 2 v'1 = 0 y = 0


Ejemplo 13. Medicin del coeficiente de restitucin .

= 0 , (v' 2 v'1 ) = 0
Explosivo

>1

Ejemplo 12. 11

Sistema de partculas

Hugo Medina Guzmn

Si se quiere medir el coeficiente de restitucin de 1os materiales, se realiza mediante una bola hecha con uno de los materiales y una superficie plana hecha con el otro material, la que se coloca sobre el suelo. Se suelta verticalmente la bola sobre la superficie desde una altura h1 . Conocemos la velocidad de la bola al momento del choque

sentido contrario. Luego del choque ambos cuerpos continan juntos con una velocidad final comn a ambos. La velocidad final ser: m1v1i + m2v2i = m1v1f + m2v2f Como v1f y v2f son iguales porque ambos cuerpos siguen juntos: v1f = v2f = vf m1v1i + m2v2i = (m1 + m2)vf

v1 = 2 gh1
La bola rebota verticalmente hasta una altura h2 , tal que la velocidad v '1 de la bola despus del choque es:

v'1 = 2 gh2
Como la superficie no tiene velocidad inicial ni velocidad final v 2 = 0 y v ' 2 = 0 . Encontramos que:

vf =

(m1v1i + m2v2i ) (m1 + m2 )

b) Velocidades de igual direccin y sentido contrario.

(v' 2 v'1 ) v'1 = (v2 v1 ) v1

Reemplazando valores:

2 gh2 2 gh1

h2 h1

Ejemplo 14. Choque plstico o inelstico a) Velocidades de igual direccin y sentido.

En este caso los cuerpos posean velocidades de igual direccin pero de sentido contrario antes del choque, como en el caso anterior luego del impacto continan juntos, con una velocidad final que estar dada por la diferencia de las cantidades de movimiento. La velocidad final ser: m1v1i - m2v2i = m1v1f + m2v2f Igualmente: v1f = v2f = vf m1v1i - m2v2i = (m1 + m2)vf

vf =
Supongamos un cuerpo 1 de masa m1 y velocidad v1 que se dirige a hacia el cuerpo 2 de masa m2 y velocidad v2, siendo ambas velocidades de igual direccin y sentido. Sobre cada cuerpo actu en el momento del choque, el impulso que le provoc el otro cuerpo, entonces hay dos acciones de igual intensidad y sentido contrario, en consecuencia ambas cantidades de movimiento sern iguales y de 12

(m1v1i m2v2i ) (m1 + m2 )

La velocidad final mantendr la misma direccin pero tendr el sentido de la velocidad del cuerpo que antes del choque tena mayor cantidad de movimiento. Ejemplo 15. Choque elstico a) Velocidades de igual sentido

Sistema de partculas

Hugo Medina Guzmn

Ejemplo 16. Choque plstico. Las dos partculas quedan en contacto despus del choque. Estudiar desde dos puntos de vista: a) Observado desde tierra, sistema laboratorio y b) Observado desde el centro de masa. Durante el choque cada cuerpo recibe una cantidad de movimiento que es igual a la velocidad perdida por el otro. Al recuperar su forma inicial, cada uno pierde o gana respectivamente, la cantidad de movimiento ganada o perdida en el momento del choque, la velocidad final de cada uno ser: Solucin. a) Sistema laboratorio. La figura muestra las dos partculas antes y despus del choque.

v1 f =

m2 (v2i v2 f ) + v1i m1

Si las masas son iguales

v1 f = v2i v2 f + v1i
b) Velocidades de distinto sentido Por conservacin de la cantidad de movimiento

m1 v1 + m2 v2 = (m1 + m2 )v' y
m v + m2 v2 (m1v1 + m2v2 ) v' = 1 1 = i (m1 + m2 ) (m1 + m2 )

La energa mecnica antes del choque es: En este caso los cuerpos literalmente rebotan, y la velocidad final de cada uno ser:

K = K1 + K 2 =

1 1 2 m1v12 + m2 v 2 2 2

v1 f =

m2 (v2i + v2 f ) v1i m1

La energa mecnica despus del choque es:

Si las masas son iguales

K '=

v1 f = v2i + v2 f v1i
El principio de conservacin del impulso es el mismo que el de conservacin de la cantidad de movimiento. Cabe aclarar que en la prctica podemos aplicar el principio de conservacin de la cantidad de movimiento durante los choques, siempre que el tiempo que el tiempo de duracin del impacto sea muy pequeo.

1 (m1 + m2 )v'2 = 1 (m1v1 + m2 v 2 ) 2 2 (m1 + m2 )

La relacin de la energa es:

m1 K' = K (m1 + m2 )

m2 v1 + m v 2 1 2 m2 2 v1 + m v 2 1

13

Sistema de partculas

Hugo Medina Guzmn

Por ejemplo la cada de un meteorito a la tierra, la que suponemos inmvil (v2 = 0) y m1 << m2, obtenemos K = 0 y

La figura muestra las dos partculas antes y despus del choque.

K' = 0 , ste es un choque perfectamente K

plstico. Si K fuera diferente de cero, la totalidad de la energa se transformara en calor. b) Sistema centro de masa. La figura muestra las dos partculas antes y despus del choque.

Por conservacin de la cantidad de movimiento:

m1 v1 = m1 v'1 + m2 v' 2
En ste caso: En sus componentes:

(1)

m1v1 = m1v'1 cos 1 + m2 v' 2 cos 2

m2 v1 v 2 y m1 + m2 m1 u2 = v1 v 2 m1 + m2 u1 =

0 = m1v'1 sen1 m2 v' 2 sen 2


Como es un choque elstico la energa mecnica se conserva:

Con v 2 = 0 , Obtenemos:

1 1 1 2 m1v12 = m1v'1 + m2 v' 2 2 2 2 2

(2)

u1 =

m2 m1 v1 y u 2 = v1 m1 + m2 m1 + m2

En las ecuaciones (1) y (2) conocidas las masas m1 y m2, tenemos como incgnitas v1, v1, v2, 1 y 2. Contamos con tres ecuaciones. Para resolver necesitamos conocer al menos dos de las cantidades anteriores. En el caso particular en que m1 = m2, podemos llegar a la relacin;

Despus del choque m1 y m2 entran en contacto constituyendo una sola partcula de masa (m1 + m2) que est en reposo en el sistema centro de masa,

u1 = u2 = 0.
Aqu tambin K = 0, = 0. Ejemplo 17. Choque elstico. Consideremos dos

v1 = v'1 + v' 2
Elevndola al cuadrado:
2 v12 = v'1 +v' 2 2 + 2v '1 v ' 2

partculas, una con masa m1 y velocidad v1 , la segunda con masa m 2 y velocidad v 2 = 0 Solucin. a) Sistema laboratorio.

Por la conservacin de la energa:


2 v12 = v'1 +v' 2 2

Luego, obtenemos:

14

Sistema de partculas

Hugo Medina Guzmn

v'1 v'2 = 0

m1u1 = m1u '1 u1 = u '1 y m2 u 2 = m2 u ' 2 u 2 = u ' 2


Para un choque elstico

Las velocidades v '1 y v '2 son ortogonales, esto nos dice que las trayectorias de las partculas despus del choque son perpendiculares entre s, tal que:

= 1 , como se espera.

1 + 2 = 2

Ejemplo 18. Reflexin de partcula sobre un plano. Consideremos dos partculas, una con masa m1 , que incide sobre una masa m 2 de superficie plana como se muestra en la figura. La masa m1 tiene

b) Sistema centro de masa. La figura muestra las dos partculas antes y despus del choque.

velocidades v1 y v '1 antes y despus del choque, la superficie inicialmente est inmvil v 2 = 0 y tiene

una velocidad v' 2 despus del choque.

Solucin. Por conservacin de la cantidad de movimiento: Por conservacin de la cantidad de movimiento:

m1 u1 + m2 u 2 = m1 u '1 + m2 u ' 2 = 0
De aqu:

m1 v1 = m1 v'1 + m2 v' 2
Para la energa tenemos que tomar en cuenta si el choque es elstico o no. a) Choque elstico. En ste caso la energa mecnica se conserva

m1 m1 2 u = m u '1 m u1 , u '2 = 2 2
2 2

Como es un choque elstico la energa mecnica se conserva:

K = K'

1 1 1 1 2 2 m1u12 + m2 u 2 = m1u '1 + m2 u ' 2 2 2 2 2 2


Reemplazando u 2 y u ' 2 en funcin de u1 y u '1 respectivamente.

1 1 2 1 m1v12 = m1v'1 + m2v'2 2 2 2 2


De aqu obtenemos: v '1 v1 =
2 2

m2 2 v' 2 m1

(m1u1 )2

1 1 1 2 1 + = (m1u '1 ) + 2m1 2m2 2m1 2m2

Expresin que podemos escribir como;

m2 v 2 v' 2 v1 + v'1 v1 v'1 = m1


De la conservacin de la cantidad de movimiento 15

De aqu se deduce:

Sistema de partculas

Hugo Medina Guzmn

m m1 v1 v'1 = m2 v' 2 v1 v'1 = 2 v' 2 m1

el otro con velocidad v 2 de tal modo que chocan.

Despus del choque el auto 1 sale con velocidad v '1 Reemplazando sta expresin en la de la energa, obtenemos: cuya direccin forma un ngulo indica en la figura.

, tal como se

v1 + v'1 = v' 2
Como

a) Hallar la velocidad del auto 1 luego del impacto. b) Determinar la posicin del centro de masa y las ecuaciones paramtricas del mismo. c) Determinar si el choque es elstico o no.

v1 cos 1 v1 = v1sen1i j, + v'1 cos '1 v'1 = v'1 sen '1 i j, v ' 2 = v ' 2 j
Reemplazando obtenemos:

m1 = m2 = 200 kg , v01 = 3 m/s ,

v02 = 1 m/s , v'1 = 2 m/s , = 53 , = 37 , d = 3m

v1 cos 1 v1sen1i j + v'1 sen '1 i + v'1 cos '1 j = v '2 j


De aqu:

v1sen 1 + v'1 sen '1 = 0 y v1 cos 1 v'1 cos '1 = v' 2


En el caso en que v ' 2 = 0 (la superficie no se mueve)

Solucin. a) por conservacin de la cantidad de movimiento


1 = '1

y v1 = v '1

p antes = p despus

b) Choque inelstico. En ste caso K > K '

m1 v1 + m2 v 2 = m1 v'1 + m2 v' 2
Aqu

1 1 1 2 m1v12 > m1v'1 + m1v' 2 2 2 2 2


Para encontrar la relacin de K y K podemos usar el coeficiente de restitucin .

, v1 = 3i + (1)sen53 v 2 = (1) cos 53 i j


=

K' , siendo 0 1 K

+ 0,8 j, 0,6i

+ (2)sen37 v'1 = (2)cos 37 i j + 1,2 = 1,6i j

Ejemplo 19. En un parque de diversiones dos amigos juegan con los autitos chocones. En cierto momento las direcciones de ambos vehculos forman un ngulo

. Un auto se dirige con velocidad v1

16

Sistema de partculas

Hugo Medina Guzmn Como m1 = m 2 = 200kg

Reemplazando:

+ 0,6i + 0,8 + 1,2 3i j = 1,6i j + v '2 0,4 v'2 = 2i j


v'2 = 2 + 0,4 = 4,16
2 2

xCM =

1 (x1 + x 2 ) , yCM = 1 ( y1 + y 2 ) 2 2

Antes del choque:

x1 = 3 + 3t ,

y1 = 0

= 2,04 m/s tan = 2 = 5 = 79 0,4

x 2 = 0,6 + 0,6t , y 2 = 0,8 + 0,8t


Luego

xCM = 1,8 + 1,8t , y CM = 0,4 + 0,4t


Despus del choque:

x1 = 1,6(t 1) , y1 = 1,2(t 1) x2 = 2(t 1) , y 2 = 0,4(t 1)


Luego

xCM = 1,8(t 1) , yCM = 0,4(t 1)


c) Para saber si es elstico o no, tenemos que analizar si la energa se conserva o no. La energa cintica antes del choque es:

b) Para determinar la posicin del centro de masa es necesario conocer la posicin inicial de la masa m 2 . Como m1 y m 2 emplean el mismo tiempo desde el inicio hasta el choque:

K=

t=

d 3m = = 1s v1 3 m s

1 1 2 m1v12 + m2 v 2 2 2 1 (200)(3)2 + 1 (200)(1)2 2 2

= La posicin inicial de m 2 es:

x20 = v2 xt = 0,6(1) = 0,6 m , y20 = v2 yt = 0,8(1) = 0,8 m

= 900 +100 = 1000 J La energa cintica despus del choque es:

Siendo la posicin inicial de m1

K'=

1 1 2 m1v'1 + m2 v' 2 2 2 2 1 (200)(2)2 + 1 (200)(2,04)2 2 2

x10 = 3m , y10 = 0
El centro de masa est dado por:

= 400 + 416 = 816 J

xCM =

x1 m1 + x 2 m2 y m + y 2 m2 , y CM = 1 1 m1 + m2 m1 + m2
17

Hay una disminucin de la energa cintica:

Sistema de partculas K = 816 1000 = - 184 J Luego el choque es inelstico. Ejemplo 20. El pndulo balstico. Este es el caso de un choque perfectamente plstico, se utiliza para medir la velocidad de un proyectil. Un proyectil de masa m y velocidad v se incrusta en el bloque de madera de masa M.

Hugo Medina Guzmn

V = 2 gy = 2(9,80 )(0,45 102 )


= 0,297 m/s La rapidez final v de la bala es

v=

p mv0 MV M = = v0 V m m m 1,00 = 450 (0,297) 5,00 10 3 = 390,6 m/s .

Aplicando la conservacin de la cantidad de movimiento.

Ejemplo 22. Un satlite artificial en vuelo explota en tres partes iguales. Una parte contina a lo largo de su lnea original de vuelo y las otras dos van en direcciones cada una inclinada 60 a la trayectoria original. La energa liberada en la explosin es dos veces ms que la energa que tena el sat1ite en el momento de la explosin. Determinar la energa cintica de cada fragmento Inmediatamente despus de la explosin. Solucin. La figura muestra el satlite antes y despus de la explosin.

m v = (m + M )V

La energa cintica despus del choque es:

1 (m + M )V 2 , sta se convierte en energa 2 potencial U = (m + M )gh


K'=
Luego (m + M )V
2

= (m + M )gh

V = 2 gh
La velocidad del proyectil es:

v=

(m + M ) V
m

(m + M )
m

Por conservacin de la cantidad de movimiento.

2 gh

p antes = p despus

Ejemplo 21. Una bala de 5,00 g se dispara contra un bloque de madera de 1,00 kg suspendido de un hilo de 2,000 m, atravesndolo. El centro de masa del bloque se eleva 0,45 cm. Calcule la rapidez de la bala al salir del bloque si su rapidez inicial es de 450 m/s. Solucin. La rapidez del bloque de madera despus de que la bala ha atravesado (pero antes de que el bloque comience a elevarse; esto asume una gran fuerza aplicada por un tiempo corto, una situacin caracterstica de las balas) es

La cantidad de movimiento debe conservarse en las tres dimensiones x, y, z , independientemente, de all que v1, v2, v3 y V deben ser coplanares. As obtenemos:

Mv =

M M M v1 + v 2 cos 60 + v3 cos 60 3 3 3

M M v 2 sen 60 v3sen 60 = 0 3 3

De estas dos ecuaciones encontramos que: 18

Sistema de partculas

Hugo Medina Guzmn b) W f =

v 2 = v3
3v = v1 + v 2

(1) (2)

mgd
Wf

La energa inicial es: Ei = La energa final es:

1 Mv 2 2

Ef =

1 1 3 Mv 2 + 2 Mv 2 = Mv 2 2 2 2

0,5 = 0,41 mgd 2,5 9,8 0,05 1 2 1 2 c) kx Wf = mv 2 2 1 (1040)(0,05)2 0,5 = 1 (2,5)v 2 2 2 v = 0,80 m/s

Esta energa es igual a la suma de las energas de los tres fragmentos.

3 1M 2 1M 2 1M 2 Mv 2 = v1 + v2 + v3 2 2 3 2 3 2 3
2 2 9v 2 = v12 + v 2 + v3

Ejemplo 24. Una pelota de masa m = 100 g se deja caer desde una altura h = 2m. La pelota rebota verticalmente hasta h despus de golpear el suelo. a) Calcular la cantidad de movimiento de la pelota antes y despus de golpear el suelo, b) si la duracin del golpe fue de 0,01 s, calcular la fuerza media ejercida por el piso sobre la pelota. Solucin. En la figura se muestra el esquema de la situacin.

(3)

De las ecuaciones (1), (2) y (3) obtenemos:

v1 = v , v 2 = 2v , v3 = 2v
La energa cintica de cada uno de los fragmentos inmediatamente despus de la explosin es: a) Cantidad de movimiento inicial:

K1 =

1 2 2 Mv 2 , K 2 = Mv 2 , K 3 = Mv 2 6 3 3

pi = mvi j
Cantidad de movimiento final:

Ejemplo 23. Un bloque de 2,5 kg, se desliza sobre una superficie rugosa, cuando contacta con el resorte tiene una velocidad de 1,2 m/s. el bloque se detiene momentneamente cuando el resorte se ha comprimido 5,0 cm. El trabajo realizado por la friccin, desde el instante en que el bloque hace contacto con el resorte hasta el instante en que hace el alto es 0,50 J. a) Cul es la constante del resorte (k)? b) Cul es el coeficiente de friccin? c) Despus de la compresin del resorte, el bloque se aleja de l, cual es la velocidad del bloque, despus de separarse del resorte?

p f = mv f j
Los valores de las velocidades inicial y final se pueden calcular usando el principio de conservacin de la energa. Inicial:

0 + mghi =
Final:

1 2 mvi + 0 vi = 2 ghi 2

1 2 mv f + 0 = 0 + mgh f 2
v f = 2 gh f =
3 2 g hi = 4
3 ghi 2

Por lo tanto, las cantidades de movimiento inicial y final son: Solucin. a) Energa antes = Energa despus
3 pi = m 2 ghi j , pf = m ghi j 2

1 (2,5)(1,2)2 = 1 k (0,05)2 + 0,5 2 2 k = 1040N/m


19

Reemplazando los valores, se tiene: pi = 0,63 kgm/s , p f = 0,54 kgm/s

Sistema de partculas

Hugo Medina Guzmn

b) Usando la aproximacin del impulso:

p pf J = Fm t = p Fm = p = f t t

Fm =

j 0,63 j 0,54 0,01 jN = 118

Ejemplo 25. Una bala de la masa m y con velocidad v pasa a travs de un pndulo de masa M. La bala emerge con una velocidad de v/2. El pndulo est suspendido por una barra rgida de longitud l y masa insignificante. Cul es el valor mnimo de v tal que la masa del pndulo gire un crculo completo? Solucin. En esta colisin, se conserva la cantidad de movimiento pero la energa no. Este es un ejemplo de una colisin inelstica que no es perfectamente inelstica. Para la colisin:

T + Mg = Mac
Condicin mnima para hacer movimiento circular

T =0

Luego

Mg = M

V '2 V ' 2 = gl (3) l

Reemplazando (3) en (2):

1 1 MV 2 = Mgl + Mg (2l ) 2 2 1 5 MV 2 = Mgl V = 5 gl 2 2


Reemplazando el valor de V en (1):

v p antes = p despus mv = m + MV 2

v=

2M m

5 gl

De aqu:

v=

2M V (1) m

MOVIMIENTO CON MASA VARIABLE PROPULSIN POR REACCIN Por la conservacin de la cantidad de movimiento si un cuerpo en reposo puede expulsar una parte de su masa en cierta direccin, el resto de la masa se mover en sentido opuesto, con igual cantidad de movimiento. Si este proceso puede mantenerse durante un tiempo, el resto de la masa, como es el caso de un cohete, aparecer para un observador externo en reposo. Como si se estuviese acelerando. Esto se expresa mediante la forma ms general de la segunda ley de Newton. Como:

Despus de la colisin se conserva la energa para el pndulo (la conservacin de la energa para la bala despus de la colisin no es til desde que su energa no cambia). Este tratamiento nos da la velocidad del pndulo el momento despus de la colisin:

1 1 MV 2 = MV ' 2 + Mg (2l ) (2) 2 2


Condicin para que pueda dar la vuelta

dp , F= dt

p = mv

Siendo la masa variable

20

Sistema de partculas

Hugo Medina Guzmn

dm v d v dm = m +v F= dt dt dt
Expresin que nos permite determinar el movimiento de un cuerpo cuya masa cambia durante su movimiento. Cuando aplicamos al caso de un cohete aparecen los problemas, evidentemente m es la masa del cohete que va cambiando. Cul es la velocidad de escape

Como el infinitsimo dmd v es de segundo orden, podemos despreciar ste trmino; luego

md v dm u = 0
d v dm Dividiendo por dt se obtiene: m u =0 dt dt dv Como v es la velocidad del cohete, es la dt

del combustible? No es igual a v , la velocidad del

cohete. Si no existe fuerza externa, F debe ser cero? Entonces no se mover el cohete. Analicemos el problema desde el punto de vista de la conservacin de la cantidad de movimiento. Sea el cohete mostrado en la figura siguiente, en el

aceleracin a . De ste modo:


dm dv m = ma = u dt dt

, instante t, tiene una masa m y una velocidad v = vi

con una cantidad de movimiento lineal m v .

Por la segunda ley de Newton se puede identificar la

cantidad u

dm como una fuerza, tal que la fuerza dt

de empuje es: En la figura siguiente se muestra el cohete en el instante t + dt , a expulsado una masa dm que sale

relativa al cohete. con una velocidad u = ui Ahora la masa del cohete es (m dm ) y su


velocidad es v + d v .

F =u

dm dt dm por se prdida de mas dt

Como u es negativa y

tambin es negativa, F es positiva, como esperbamos. Esta es una fuerza externa que produce aceleracin al cohete, al que ahora consideraremos como un sistema aislado sobre el que hay una fuerza externa. Velocidad del cohete. De la expresin md v dm u = 0 , obtenemos:

La cantidad de movimiento lineal total es:


(m dm ) v + d v + dm v u

dv=u

dm m
m

Por conservacin de la cantidad de movimiento lineal tenemos: p antes = p despus


m v = (m dm ) v + d v + dm v u

v (t ) = u

m0

dm m m = u [ln ]m0 = u ln m m0

Como u es negativa y m < m0 ,

ln

m es negativa. m0

m v = m v + md v dm v + dmd v + dm v dm u

Luego la velocidad es positiva. 21

Sistema de partculas

Hugo Medina Guzmn

Velocidad lmite del cohete. Una vez que se haya terminado el combustible la masa se reduce a m1 , y llegamos a la velocidad lmite.

v(t ) = gt uln

(m0 Ct )
m0

v m = u ln

b) Antes de encontrar la altura que alcanza en el momento que se acaba el combustible, encontraremos la altura para el tiempo t. Como v (t ) =

m1 m0

dz , tenemos: dt

Una vez alcanzada sta velocidad, sta permanecer constante. Ejemplo 26. Un cohete y su combustible tienen una masa inicial m0 . El combustible se quema a una

dz = gtdt uln
Integrando

(m0 Ct )
m0

dt

dm razn constante = C .Los gases son expulsados m

con una velocidad constante u respecto al cohete. a) Despreciando la resistencia del aire, hallar la velocidad del cohete en el instante t, despus de despegar de la tierra. b) Cul es la altura que alcanza en el momento que se acaba el combustible? Solucin. a) Las fuerzas que actan sobre el cohete son la fuerza de empuje y la fuerza de la gravedad. Aplicando la segunda ley de Newton.
dm F = m a u dt + m g = m a

dz = gtdt uln
0 0

(m0 Ct )
m0

dt

(m Ct ) u 1 z = gt 2 (m0 Ct )ln 0 1 C m0 2
El tiempo t1 en que se acaba el combustible es cuando m = m1 . Como m = m1 Ct , obtenemos:

t1 =

m0 m1 C

Reemplazando t1 en la expresin anterior encontramos la altura que alcanza en el momento en que el combustible se acaba. CANTIDAD DE MOVIMIENTO ANGULAR Y MOMENTO DE UNA FUERZA O TORQUE Consideremos una partcula P de masa m, su posicin con respecto al origen O en un instante dado est

d v dv , g = gk , Donde u = uk a= = k dt dt

dm = C m = m0 Ct y dt
Reemplazando uC (m0 Ct )g = (m0 Ct )

determinada por el vector r .

dv dt

La partcula tiene una velocidad v y su cantidad de

dv uC = g + (m0 Ct ) dt

movimiento lineal es

p = mv .

Integrando

dv = gdt +
0

uC dt (m0 Ct )

La velocidad en el instante t es: 22

Sistema de partculas

Hugo Medina Guzmn

dL d dr d p = r p = p+ r dt dt dt dt
Como
dr = v y p = mv dt

LA CANTIDAD DE MOVIMIENTO ANGULAR

O MOMENTUM ANGULAR L

dr p = v m v = 0 dt

Se define como el producto vectorial de r y p ,

Luego

L = r p

dL d p = r dt dt

La direccin de L es perpendicular al plano definido


por r y p , su sentido lo da la regla de la mano derecha, su mdulo es:

Por otra parte si F es la fuerza que produce el movimiento de la partcula, por la Segunda Ley de Newton tenemos:

L = rp sen = rmv sen


Como vsen es la velocidad tangencial ( vt ) y

dv dp = F = ma = m dt dt dL = r F dt

vt = r , siendo la velocidad angular. Podemos


escribir: Luego
2

L = mr
MOMENTO DE INERCIA (I ) . Llamando Momento de Inercia al producto mr , Tenemos:
2

A esta cantidad que produce un cambio en la cantidad de movimiento angular se le conoce como MOMENT0 DE UNA FUERZA o TORQUE

L = I , vectorialmente L = I
Las dimensiones de la cantidad de movimiento angular son:

dL = = r F dt
Tiene como mdulo = rF sen Su sentido est dado por la regla de la mano derecha. Ejemplo 27. Una partcula de masa m se mueve en el

[L] = [M ][L]2 [T ]1
Sus unidades en el sistema internacional:

Kg m o Js s

plano xy con una velocidad v a lo largo de una lnea recta. Cul es la magnitud y direccin de su cantidad de movimiento angular con respecto al origen O?

Derivando la cantidad de movimiento angular L con respecto al tiempo:

23

Sistema de partculas

Hugo Medina Guzmn

Ejemplo 29. Un cilindro slido Puede girar alrededor de un eje sin friccin como se ve en la figura. Una cuerda enrollada alrededor del radio exterior R1 ejerce una fuerza F1 hacia la derecha. Una segunda cuerda enrollada alrededor de la otra seccin cuyo radio es R2 ejerce una fuerza F2 hacia abajo. Cul es el torque que acta sobre el cilindro alrededor del eje z que pasa por O? Solucin.

La posici6n de la partcula es r .

La velocidad de la partcula es v . Su cantidad de movimiento lineal es

p = mv

Solucin. Su cantidad de movimiento angular es L = r p


Sobre el cilindro actan:

L = r m v = rmvsen k

( )

en r 1 = R1 F 1 = F1i j y F 2 = F2 j en

La magnitud es: L = rmvsen = mvd , donde

r 2 = R2 i

d = rsen

El torque neto sobre el cilindro es:

Luego L = (mvd )k
Podemos ver que la cantidad de movimiento angular con respecto a O' es cero. Ejemplo 28. En determinado instante, la Posicin de una partcula con respecto a un origen O de

= 1+ 2
+ R2 F2 k = r 1 F 1 + r 2 F 2 = R1 F1 k = (R2 F2 R1 F1 )k
Si F1 = 10 N, R1 = 2 m y F2 = 5 n, R2 = 1 m:

+4 coordenadas est dada por el vector r = 3i j (en metros) . En ella acta una fuerza + 32 F = 16i j (en Newton) . Encontrar el torque

= 15k Nm = [(5 N )(1m ) (10 N )(2m )]k


CONSERVACION DE LA CANTIDAD DE MOVIMIENTO ANGULAR En el caso de una partcula come en la seccin anterior, si el torque aplicado con relacin a un punto dado de referencia es cero, tenemos que:
dL = 0 , por consiguiente: L = CONSTANTE dt

originado por la fuerza F que acta sobre la partcula. Con referencia al origen O. Solucin.

+4 + 32 El torque es: = r F = (3i j ) (16i j)


=

+ (4)(16 )( k ) = 96k 64k (3)(32)k

Nm = 32k

La cantidad de movimiento angular con respecto al punto de referencia es constante. 24

Sistema de partculas

Hugo Medina Guzmn

Ejemplo 30. Una partcula de, masa M en el extremo de un hilo gira con velocidad v1 cuando el radio es r1, si disminuimos el radio de r1 a r2, qu sucede con la velocidad? Solucin.

Ejemplo 31. Clculo de la desviacin de un cuerpo situado en la lnea ecuatorial y que cae desde una altura h . Solucin.

La figura indica la forma como se puede realizar esta experiencia, para disminuir el radio basta jalar el hilo. Aplicando la conservacin de la cantidad de movimiento angular:

Lantes = Ldespus
r1 Mv1 = r2 Mv 2 v 2 = v1 r1 r2

En la figura, sea una partcula de masa m a una altura h sobre la superficie de la Tierra en un punto que, para simplificar, consideramos que se encuentra sobre el ecuador. Para los casos de inters fsico, la altura h ser por lo comn muy pequea en comparacin con el radio, R de la Tierra. Si se supone que la partcula parte del reposo en relacin a un punto de la superficie de la Tierra verticalmente por debajo de l entonces, inicialmente, el componente radial de la velocidad vr de la partcula desaparece y su componente tangencial v ser (R + h), en donde es la velocidad angular de la Tierra. Al soltarse, debido a la atraccin gravitacional de la Tierra, la partcula comienza a caer verticalmente haca abajo y, por ende, su distancia radial r del centro de la Tierra comienza a disminuir. De L = mrv se deduce que el componente tangencial de su velocidad v debe aumentar este proceso y de modo tal que haga que el producto rv sea constante. En trminos ms cuantitativos, esto quiere decir que durante su descenso hacia el suelo, la distancia radial r y la velocidad tangencial v se deben relacionar por medio de

Tambin podemos hallar el trabajo realizado para acortar el radio.

K antes =
K despus =

1 Mv12 , 2
1 1 r1 1 r1 2 Mv 2 v1 = M = M 2 2 r2 2 r2
2

mrv = m ( R + h) 2
2 v1
2

(1)

El trabajo realizado es igual al cambio de energa cintica.

Puesto que, inicialmente, la velocidad de la partcula es (R + h), de tal modo que su cantidad de movimiento angular L en relacin al centro de la Tierra es m(R + h)2 Anticipndonos al hecho de que la desviacin hacia el este ser muy pequea, podemos escribir para la distancia radial r del cuerpo al centro de la Tierra en cualquier instante t,

1 r1 W = K = M 2 r2
1 2 r1 = Mv1 r 2 2

2 1 2 v1 2 Mv1

r = R+h

1 2 gt 2

(2)

2 1

Y al sustituir esto en (1) obtenemos:

25

Sistema de partculas

Hugo Medina Guzmn

v =

( R + h) 2
1 R + h gt 2 2

( Ro + h )
1 2 1 2 gt /( R + h)

(3)

CONSERVACION DE LA CANTIDAD DE MOVIMIENTO ANGULAR DE UN SISTEMA DE PARTICULAS. Vamos a considerar un sistema de dos partculas, como se muestra en la figura.

Para calcular la magnitud de la desviacin hacia el este, sea vy en el instante t, la velocidad del cuerpo que cae en la direccin hacia el este, tal y como lo ve un observador fijo con respecto a la superficie de la Tierra. Entonces

v y = v o r =

( Ro + h )

1 2 1 gt /( R + h) 2 1 2 2 gt 1 2 = ( R + h) 1 + ( R + h) + gt 2 R + h 2 = gt
Donde la segunda igualdad se obtiene de la utilizacin de (2) y (3) y la tercera, a continuacin, mediante el hecho de que gt2 <<(R + h) y el empleo del teorema binomial. Al integrar esta frmula para vy se obtiene, para la desviacin total hacia el este y en el instante t,

(R + h

1 2 gt ) 2

d L1 Para la partcula 1: 1 = , donde dt 1 = r 1 F 1 = r 1 F 12 + F 1ext

F1 Es la fuerza total sobre la partcula 1.

y=

1 3 gt . 3
1

F12 Es la fuerza ejercida por la partcula 2 y

Finalmente, puesto que el tiempo que necesita la partcula para caer la distancia h es de ( 2hg ) 2 , la deflexin total hacia el este d, se puede expresar como sigue

F1 ext Es la suma de las fuerzas externas sobre la


partcula 1.
d L1 1 = = r1 F12 + r1 F1 ext = 12 + 1 ext dt

2 2h d = h 3 g

(4)

Por ejemplo, si se deja caer una partcula desde una altura de 100 metros, su desviacin hacia el este, segn esta frmula, se descubre que es (al sustituir los valores h = 100 metros y = 7,2 x 10-5rad/s) de 2,2 cm. Esta desviacin es muy pequea y slo se puede observar en condiciones controladas cuidadosamente. Es importante recordar la base fsica para la deflexin pronosticada en (4). Conforme la partcula desciende hacia la superficie de la Tierra su velocidad tangencial v debe aumentar para que el producto rv sea constante. Por consiguiente, de esto se desprende que su velocidad tangencial debe sobrepasar a la del punto de la superficie que se encontraba inicial e inmediatamente por debajo, y, en esta forma, se desva hacia el este.

Similarmente para partcula 2.


d L2 2 = = r2 F21 + r2 F2 ext = 21 + 2 ext dt

d L1 d L2 d Ltotal Sumando 1 + 2 = + = dt dt dt
Para los Torques internos tenemos:

12 + 21 = r1 F12 + r2 F21
Como F21 = F12 26

Sistema de partculas

Hugo Medina Guzmn

12 + 21 = r1 F12 r2 F12 = r1 r2 F12


Al estar sujetos los dos hombres a la cuerda su movimiento es circular y si consideramos que el piso est en el plano xy, De la figura: r1 + r12 = r2 r1 r2 = r12 Reemplazando

Tenemos:

L inicial = 2

12 + 21 = r12 F12
Como r12 y F12 son paralelos: r12 F12 = 0 , y

L final

l , pk = lpk 2 l lp ' = 2 p' k = k 4 2


Como L inicial = L final p ' = 2 p La cantidad de movimiento lineal final de cada hombre es el doble de la cantidad de movimiento lineal inicial. CONSTANTE, Independiente, del tiempo Ejemplo 33. Una partcula de masa m1 se desplaza

12 + 21 = 0
De aqu: 1 ext + 2 ext

d Ltotal d Ltotal = y ext = dt dt

Vemos si

ext = 0 Ltotal = CONSTANTE ,

independiente del tiempo. Ejemplo 32. Dos hombres se encuentran en una pista de patinaje, ambos sostienen una cuerda de longitud l . Qu pasa con la cantidad de movimiento lineal

sobre un plano horizontal con velocidad v 1 . Dos partculas de masas m2 y m3 unidas por una varilla de

masa despreciable se mueven con velocidad v 2 , como se indica en la figura.

se mueven en crculo y que la magnitud de sus cantidades de movimiento son iguales. Solucin.

p de cada uno de ellos, si ambos jalan la cuerda y acortan la distancia entre ellos a l / 2 ? Asumir que

Las nicas fuerzas externas al sistema son la fuerza de la gravedad y la reaccin normal del piso, estas fuerzas se cancelan. Las nicas fuerzas que, intervienen en el sistema son las internas, por lo tanto la cantidad de movimiento angular del sistema se conserva.

Suponiendo un choque totalmente plstico entre m1 y m2 m1 = m2 = m3 = 1 kg, v 1 = 20

l = 1m
27

m m , i , v 2 = 10 i s s

Sistema de partculas

Hugo Medina Guzmn

Calcular: a) La posicin del centro de masa respecto a la masa m2 en el momento del choque. b) La ley del movimiento del centro de masa. c.) La velocidad angular de rotacin alrededor del centro de masa despus del choque. Solucin. a) En el momento del choque, tomando como referencia la posicin de m2, el centro de masa est en: Posicin en el instante partida (t = 0).

b) Como parten del reposo, la cantidad de movimiento total del sistema es cero.

P total = M v cm = 0
Como v cm =
d r cm r cm = constante. dt

El centro de masa permanece en la misma posicin. c) Consideremos la cantidad de movimiento angular con respecto al centro de masa antes y despus del choque. Antes del choque.

Posicin en el instante de encuentro.

L = r 1cm p 1 + r 2cm p 2 + r 3cm p 3 r 1cm =


2d 1 i j , p1 = m1 v 1 = 20i 3 3 1 j , p 2 = m2 v 2 = 10i 3 2 j , p 3 = m3 v 2 = 20i 3

r 2 cm = (d v2t )i

+ r 3cm = (d v2t )i

xCM

2d 2d 2d m1 + m2 + m3 3 3 3 = m1 + m2 + m3
=

Nota: En los vectores posicin ( r 1cm , r 2 cm , r 3cm ) solo ponemos la posicin en el eje y, porque la posicin en x se va a anular con el producto vectorial. Reemplazando:

m(2d ) 2d = 3m 3 m1 (0) + m2 (0) + m3 (l ) m1 + m2 + m3

1 2 1 ) + ) + ) L = j (20i j ( 10i j ( 10i 3 3 3

yCM =

L=

20 10 20 k k+ k = 10k 3 3 3

l 1 = m 3 3
28

Despus del choque:

Sistema de partculas

Hugo Medina Guzmn

Como despus del choque el sistema gira alrededor del centro de masa con velocidad angular , podemos expresar la cantidad de movimiento angular en funcin del momento de inercia.

L' = I , I = I 1 + I 2 + I 3 2 2l l l I = m1 + m2 + m3 = 3 3 3 3
luego L' =

Solucin.
2

a) La cantidad de movimiento con que se acerca el muchacho es:

2 3

L = r p = rmv senk r sen = l 0 L = ml 0 vk

Por conservacin de la cantidad de movimiento angular L' = L

Como

rad = 2 = 15k 10k s 3

b) Cuando el muchacho se coge del poste, las fuerzas de reaccin centrpeta e impulsiva deben pasar por el poste por lo tanto no ejercen ningn torque sobre el muchacho y la cantidad movimiento angular se conserva.

= constante L = ml 0 vk 4 K. 5 1 2 mv 2

La energa cintica despus de cogerse del poste es

K'=

K es la energa cintica antes de cogerse, Ejemplo 34. Un muchacho va corriendo por la acera con una velocidad constante v con sus brazos estirados perpendicularmente a su recorrido. La distancia entre los extremos de los dedos de sus manos es 2l 0 . Cuando al correr pasa junto a un poste, se coge al mismo con la mano izquierda, levanta los pies del suelo, y gira por aire alrededor del poste. a) Si su masa es, M. Cul es el valor de su cantidad de movimiento angular respecto al poste cuando corre por la acera? b) Si la fuerza de reaccin del poste no s1o lo hace girar, sino que adems proporciona una fuerza impulsiva que hace frenar ligeramente su movimiento hacia adelante, de modo que su energa cintica se reduce a los cuatro quintos de su valor original. Cul es su momento de inercia respecto al poste?

K=

Luego:

K'=

4 41 2 K = mv 2 = mv 2 (1) 5 52 5
1 2 I , como L = Ik 2

Tambin K ' =

L I

Reemplazando ste valor de en K :

K'=

L2 (ml 0 v ) = 2I 2I

(2)

Igualando (1) y (2)

29

Sistema de partculas

Hugo Medina Guzmn

2 2 (ml 0 v ) mv = 5 2I

Luego su momento de inercia es I =

5 2 ml 0 4

PREGUNTAS Y PROBLEMAS

1. Una masa m1 se sita en (x1, y1, z1) y otra masa m2 en (x2, y2, z2). a) Hallar 1a distancia r0 entre m1 y m2. b) Hallar la distancia r1 entre m1 y el centro de masa. c) Hallar la distancia r2 entre m2 y et centro de masa. Respuesta. a)

r0 =
b) r1 =

(x 2 x1 )2 + ( y 2 y1 )2 + (z 2 z1 )2
m2 r0 m1 r0 , c) r2 = (m1 + m2 ) (m1 + m2 )

5. Suponga que la fuerza que acta sobre una pelota de tenis (m = 0,060 kg) en funcin del tiempo est dada por la grfica de la figura. Usando mtodos grficos estime: a) El impulso total dado a la bola. b) La velocidad de sta despus de haber sido golpeada, suponiendo que estaba en reposo en el momento de ser golpeada.

2. Dos partculas de masas m1 = 1 kg y m2 = 3 kg se mueven por el espacio, sus vectores posicin Son:

, r = sen t 2 i +k + t r1 = 3i j tk 2
a) Hallar el centro de masa. b) Cul es su aceleracin? c) Hallar su aceleracin vista por un observador que

. se mueve con velocidad constante v = j + 3k Respuesta. a) 1 + t r CM = 3 + 3sen t 2 i j + 3t t k 4 1 1 2 2 2 b) a CM = cos t + 2t sen t i + tk 2 16

Respuesta. a) 4,5 Ns b) 75 m/s

[(

)]

6. Un flujo de partculas idnticas de masa m y

velocidad uniforme v , inciden sobre un plano fijo de rea A, la direccin forma un ngulo con la normal. Despus del choque las partculas tienen una

c) igual que b) 3. Encontrar el centro de masa de una lmina delgada mostrada en la figura

velocidad v ' , la direccin es simtrica a la de v . Tambin v = v ' . a) Calcular el Impulso que se ejerce sobre cada partcula en el momento del choque. b) Calcular el valor de la fuerza comunicada a la superficie por unidad de tiempo. Siendo n el numero de partculas por unidad de volumen de chorro incidente.

, Respuesta. a) J = (2v cos )n


b) F = 2nAmv cos
2 2

Respuesta. y = 0,983m encima del centro del orificio

acta sobre un + t2 j + t 3k 4. Una fuerza F = ti


cuerpo en el intervalo de 0 t 6 s . Hallar el impulso sobre el cuerpo.

7. Un nadador de 70 kg se lanza al agua desde el podio de una piscina con una velocidad de 3m/s en la direccin de la figura. Calcular la fuerza ejercida sobre el podio durante los 0,8s que el nadador ejerce el esfuerzo sobre el mismo para impulsarse en el salto.

+ 72 j + 324k Respuesta. 181 + 72j + 324l 18i

30

Sistema de partculas

Hugo Medina Guzmn

818 j N Respuesta. F = 227i


8. Un recipiente de 0,25 kg con capacidad para 5 kg de agua se llena de un cao en 5 s. En el instante en que el recipiente est medio lleno, la balanza lee 3,0 kg. Si no se salpica el agua, Cul es la velocidad del agua que cae en dicho instante.

Encontrar la velocidad v del obs. d) Deducir del clculo anterior el alcance R del obs.

Respuesta. 2,45 m/s Respuesta. a) v = 9. Una bala de fusil de masa m y de velocidad constante v 0 , penetra en un bloque de madera fijo; la bala se detiene despus de recorrer una distancia d con un movimiento uniformemente retardado. e) Calcular la desaceleracin a de la bala, deducir la fuerza de desaceleracin. b) Calcular el tiempo de desaceleracin. c) Cul es el impulso comunicado a la bala por el bloque? Comparar con la cantidad de movimiento de la bala antes del choque. Realizar la aplicacin numrica para v 0 = 600 m/s, d =30 cm, m = 40 g

M V , m cos M V b) v ' = 1 + , m cos m c) u v = v' cos , m+M

+ v' sen v = (u + Mv' cos )i j


2v 2 Mv 2 + sen cos u g M +m

d) R =

v2 5 m Respuesta. a) a = 0 = 6 10 2 , 2d s 7 F = ma = 24 10 N , v 3 b) t = 0 = 10 s a
c) J = F t = 24 N, J = mv0 10. Un can fijo sobre un vagn que se puede desplazar si friccin sobre una va rectilnea horizontal con una masa total M. El can forma un ngulo con la horizontal. a) Si el vagn est en reposo el can dispara un obs de masa m, determinar la relacin entre las velocidades v y V del obs y del can. b) Si la velocidad del obs relativa al can es v (forma un ngulo con V ), determinar la relacin entre v ' y V . c) El vagn se desplaza con una velocidad rectilnea constante u sobre la va antes del disparo. El obs tiene una velocidad v relativa al can en movimiento a la velocidad V despus del disparo: Calcular la variacin de la velocidad (u - v) del vagn 31

11. Desde la plataforma de un tren que se mueve con una velocidad de 10 m/s se arroja un paquete de 25 kg, Este es cogido en el aire por una persona que est junto a la va. Desde el tren se observa que esta persona retrocede con una velocidad de 7,5 m/s. Cul es la masa de la persona? Respuesta. 75 kg. 12. Un muchacho est en medio de un lago congelado sin friccin de tal manera que no puede moverse. Para poder salir l lanza su sombrero de masa 0,5 kg hacia el norte con una velocidad de 12 m/s a 53 con la horizontal. Si la masa del muchacho es 60 kg y el radio del lago es 400 metros. Qu pasa7 Respuesta. El muchacho resbala hacia el sur y llega a la orilla 1 h 51 min despus. 13. Un paquete de 10 kg se descarga de una cinta transportadora con una velocidad de 3 m/s y cae en una vagoneta de 25 kg. Si la vagoneta est inicialmente en reposo y puede rodar libremente, Cul es su velocidad final?

Sistema de partculas

Hugo Medina Guzmn

Respuesta. a) xCM = - 3 1/3 m,

kg.m/s, p 2 = 0 , pCM = 25 i kg.m/s b) p1 = 25 i m/s c) v = 1 2/3 i m/s Respuesta. v = 0,732 i


14. Un hombre de 75 kg se lanza al agua desde la proa de su bote de 50 kg. La componente horizontal de su movimiento es 1 m/s respecto al bote. Hallar las velocidades del hombre y del bote respecto a un observador en el muelle. a) Si el bote est inicialmente en reposo b) Si el bote se mova inicialmente hacia adelante con una velocidad de 2 m/s. No considerar prdidas de energa debido al agua Respuesta. a) v1 = 0,4 m/s , v2 = 0,6 m/s b) v1 = 2,4 m/s , v2 = 1,4 m/s 15. Un can dispara un obs de 2,4 kg hacia arriba. A1canza su mxima altura, 313,6 m y se parte en dos, 0,8 kg y 1,6kg. Las dos partes llegan a tierra simultneamente. La pieza de 1,6 kg toca tierra a 480 m de la explosin (medida a lo largo del eje x). a) Cunto tiempo tomara al obs volver a tierra si no se hubiera partido? b) Cul es la velocidad de cada una de las piezas justamente despus de la explosin? c) Encontrar la cantidad de movimiento de cada pieza justamente antes de tocar tierra. Respuesta. a) 8 segundos 17. Dos bolas P1 y P2 de masas m1 y m2 estn suspendidas del cielorraso por dos hilos inextensibles de la misma longitud l ; P1 y P2 estn en contacto sin presin con los hilos verticales. Se saca P1 de la posicin de equilibrio a un ngulo 0 manteniendo el hilo tenso, luego se suelta sobre P2. Calcular: a) La velocidad de P1 justo antes del choque. b) Las velocidades v1 y v2 de P1 y P2 inmediatamente despus del choque perfectamente elstico. Discutir este resultado para valores relativos de las masas m1 y m2. c) Las alturas de las posiciones limites de P1 y P2 despus del choque. Aplicacin numrica; l = l m . 0 = 60, m2 = m1/2 Respuesta. a) v1 = b) v '1 =

2 gl cos 0 , v1 = 3,13 m / s ,

m1 m2 v1 , v'1 = 1,05 m / s m1 + m2

Para m1 > m 2 v1 y v'1 tienen el mismo sentido. Para m 2 > m1 v'1 tiene sentido contrario de v1 .

m/s (16), v (0,8 ) = 120i m/s b) v (1, 6 ) = 60i


125,44 c) p (1, 6 ) = 96i j kg.m/s ,

v' 2 =

2m1v1 , v '2 = 4,22 m / s m1 + m2


2 v'1 v '2 = 0,056 m , h2 = 2 = 0,91 m 2g 2g

v' 2 en todo caso tiene el mismo sentido que v1


c) h1 =

62,72 p (0,8 ) = 96i j kg.m/s

(El movimiento es en el plano xy; g = 9,8 m/s2) 18. En un Juego de billar, la bola A est movindose 16. Un bloque de masa 10 kg est en reposo en el origen segundo con masa 5 kg se mueve a lo largo del eje x con velocidad de magnitud v 0 = 5 m/s. Los bloques choca quedan unidos. y se mueven en el eje x. La superficie tiene friccin despreciable. a) Cuando el bloque de 5 kg est en x = -10 donde est centro de masa? b) Encontrar la cantidad de movimiento de la masa de 5 kg, de la masa de 10kg y del centro de masa antes del choque. c) Cul es la velocidad del sistema combinado?

con 1a velocidad v 0 = 31 m/s cuando choca con las bolas B y C que estn juntas en reposo. Tras el choque, se observan la tres bolas movindose en las direcciones que muestra 1a figura, con = 30. Suponiendo Superficies lisas y choques perfectamente elsticos, hallar los mdulos de la

velocidades, v A , v B y vC .

32

Sistema de partculas

Hugo Medina Guzmn

Respuesta. v A = 1,5 m/s , v B = 1,29 m/s , vC 2,25 m/s 19. Se dispara una bala de 39 g con una velocidad de 500 m/ contra un bloque A de 5 kg de El coeficiente de rozamiento entre el bloque A y la plataforma es 0,5. Si la masa de la plataforma es 4 kg y puede rodar libremente, hallar: a) La velocidad final de la plataforma y e1 bloque. b) La posicin final del bloque sobre la plataforma.

Respuesta. a) 0,943 m, b) 0,711 m, 0,37 m 23. Un objeto de 5 kg que se mueve con una velocidad de 1,2 m/s choca directamente con un objeto de 20 kg que est en reposo. Se observa que el objeto menor rebota con una velocidad de 0,6 m/s a) Cul es la prdida de energa cintica por el impacto? b) Cul es el coeficiente de restitucin? Respuesta: a) K = - 0,675 J, b) = 0,875 24. Una bola choca contra un plano liso formando un ngulo 1 con la normal del mismo y rebota con un

ngulo 2 . Encontrar La expresin correspondiente al coeficiente de restitucin Respuesta. a) 2,16 m/s b) El bloque se detiene a 0,33 m de B. 20. La figura muestra dos masas sobre una superficie con rozamiento despreciable. El coeficiente de restitucin entre Las dos masas es 0,73; determinar: a) Sus velocidades despus del choque. b) La prdida de energa durante el choque. Respuesta.

tan 1 tan 2

Respuesta: a) v A = - 0,563 m/s b) K = 41 J

i m/s, v B = 6,94 i

25. Ira partcula de masa m1 tiene un choque frontal perfectamente elstico con una partcula de masa m 2 inicialmente en reposo. Cul es la prdida relativa de energa cintica correspondiente a la partcula . m1 Respuesta.

K =4 K

m1 m1 m2 1 + m 2
2

21. Se deja caer una pelota al suelo desde una altura y. Si el coeficiente de restitucin es , escribir expresiones para el tiempo total que tardar la pelota en dejar de dar bote y la distancia total que recorrer en este tiempo. Respuesta. t =

2 y (1 + ) 1+ 2 , s= y g (1 ) 1 2

( (

) )

26. Una masa m1 se mueve a lo largo del eje x con una velocidad v 0 a lo largo de una mesa sin friccin. Choca con otra nasa, la cual est inicialmente en reposo. La masa m2 sale a lo largo del eje y. Si se pierde la mitad de la energa cintica original en el choque. Cual es el mdulo de la velocidad y con que ngulo sale despus de la colisin?

22. Una bola cae desde una altura h = 0,900 m sobre una superficie lisa. Si la altura del primer rebote es h1 = 0,800 m y la distancia d1 = 0,400 m, calcular: a) El coeficiente de restitucin. b) La altura y longitud del segundo rebote.

33

Sistema de partculas

Hugo Medina Guzmn

Respuesta. v 2 = m1v 0

3 , 2 2 m2 + m1 m2

m1v0 = cos 1 m v 2 2

2 1 2 (m1 + m 2 ) = cos m2 3

v ' v ' v ' B v ' A = 0,9 B A = 0,9 vA 3,43 v' B v' A = 0,9(3,43) = 3,09 (2)
Sumando (1) y (2):

v' B +1,6v' B = 3,09 + 3,43


La velocidad de B inmediatamente despus del choque es v' B = 2,51 m/s b) El diagrama del cuerpo libre de B, inmediatamente despus del choque

27. Se deja en libertad el bloque A cuando = 90 y desliza sin rozamiento, hasta chocar con la bola B. Si el coeficiente de restitucin es 0,90, calcular a) La velocidad de B inmediatamente despus del choque. b) La mxima traccin que soporta el hilo que sostiene a B c) La altura mxima a la que se eleva B.

Fr = mB ac T mB g = mB
v' 2 B T = mB 0,9 + g 2 2,51 = 2 + 9 , 8 0,9 = 33,6 N

v' 2 B 0,9

Solucin. a) Por conservacin de energa encontraremos v A .

c) Por conservacin de energa encontramos la altura mxima a la que se eleva B.

1 2 mAv A 2 v A = 2 gr = 2(9,8)(0,6 ) m A gr =
= 3,43 m/s Por conservacin de la cantidad de movimiento encontraremos v ' B

1 m B v' 2 B = m B gh 2 v' 2 2,512 = 0,321 m h= B = 2 g 2(9,8)


28. Un bloque de masa M est en reposo sobre una masa sin friccin. Lo podemos golpear con un bloque que se quede adherido o con un bloque muy duro con el que se producir un choque perfectamente elstico. Ambos bloques tienen masa m y pueden ser lanzados can velocidad V0 En cul de los casos el bloque M se mover ms rpidamente? (considerar el movimiento en una sola dimensin). Respuesta. a) v ' 2 = b) v ' 2 =

v A = 3,43 m/s v B = 0 m A v A + m B v B = m A v' A + m B v' B (1,25)(3,43) = 1,25v' A +2v B v' A +1,6v' B = 3,43 (1)
El coeficiente de restitucin es 0,90

m V0 , m+M

(v' 2 v'1 ) v' B v' A = (v 2 v1 ) vA

= 0,9
34

2m V0 m+M

En el segundo caso es el doble que en el primero.

Sistema de partculas

Hugo Medina Guzmn

29. Un cilindro A cae sobre otro B apoyado sobre un resorte de constante k = 3000 N/m desde una altura de 2m. Si el choque es perfectamente plstico, calcular: a) El desplazamiento mximo de B. b) La prdida de energa en el choque.

potencial del resorte comprimido es de 60 J y el

cuerda cuando = 30 , hallar la velocidad resultante de cada partcula

conjunto posee la velocidad inicial v 0 Si se rompe la

Respuesta. a) 3,47 cm , b) 8,18 J 30. Los parachoques se disean de tal manera que un automvil de 1600 g que golpea una pared rgida a la velocidad de 12 km/h no sufra dao. Suponiendo que ese choque es perfectamente plstico. Calcular: a) La energa absorbida por el parachoques durante el impacto. b) La velocidad a la que el automvil puede golpear a otro de iguales caractersticas, que est en reposo sin daarse. Respuesta. a) 8890 J b) 24 km/h 31. Se dispara una bala de 25g en direccin horizontal. La bala atraviesa el bloque A y queda alojada dentro de bloque B. Por dicha causa los bloques A y B comienzan a moverse con velocidades iniciales de 2,4 y 1.8 m/s. respectivamente. Hallar: a) La velocidad inicial v 0 de la bala. b) La velocidad de la bala en el trayecto entre el bloque A y el B.

+ 5,2 Respuesta. v A = 9i j m/s y 3,5 v B = 4i j m/s


34. Un depsito suelta arena a una banda transportadora razn de 75 kg/s. Si la banda se mueve con una rapidez constate v = 2,2 m/s. Qu fuerza se necesita para mantenerla en movimiento? No tomar en cuenta la friccin

Respuesta. 165 N 35. Un trineo lleno de arena se desliza sin friccin por una pendiente de 30. La arena se escapa por un agujero en el trineo a un ritmo de 2 kg/s. Si el trineo parte del reposo con una masa inicial de 40 kg. Cunto tard en recorrer 120m a lo largo de la pendiente? Respuesta. 7 segundos 36. Un cohete que consume combustible a un ritmo constante k se encuentra sometido a la accin de una fuerza externa constante de valor F adems de la fuerza de reaccin de los gases. La masa inicial del cohete ms combustible es m0 . La configuracin de la tobera de escape es de tal manera que la velocidad relativa de los gases es igual al negativo de la velocidad v del cohete. a) Escribir la ecuacin del movimiento. b) Obtener v (t ) . Respuesta. a) F = (m0 kt )

m s b) 3261 m/s Respuesta. a) 470i


32. Una explosin rompe un objeto en dos piezas una de las cuales tiene 1,5 veces la masa de la otra. Si se liberan 4500 J en la explosin. Cunta energa cintica adquiere cada pedazo? Respuesta. 1800 J, 2700 J 33. Cuando se rompe la cuerda que une las partculas A y B, el resorte comprimido las obliga a separarse (el resorte no est unido a las partculas). La energa 35

dv kv , dt

Sistema de partculas

Hugo Medina Guzmn

b) v (t ) =

Ft m0

b) Las velocidades de A y B cuando la varilla ha girado 90. c) Las velocidades de A y B cuando la varilla ha girado 180.

37. Un cohete experimental se proyecta de forma que pueda mantenerse inmvil sobre el suelo. El cuerpo del cohete tiene una masa de 1200 kg y la carga de combustible inicial es de 3600 kg,. e1 combustible se quema y se expulsa con una velocidad de 2500 m/s. Hallar la velocidad de consumo de combustible necesario. a) en el momento de encender el cohete. b) cuando se consume la ltima partcula de combustible.

3 mlv 0 k 4 1 3 1 1 b) v A = v 0 i + v0 j , v B = v0 i v0 j 4 4 4 4 1 1 c) v A = v 0 i , v B = v 0 i 2 2
, L = Respuesta: a) p = mv 0 i
Respuesta. a) 18,84 kg/s . b) 4.71 kg/s 38. Una bala de masa m se dispara con una velocidad

, si para x = x 0 , y = a (permanece v B = v B i
constante) Cul es su cantidad de movimiento angular con respecto al origen en funcin de x?

4l. Se tiene una varilla rgida de masa despreciable sujeta a un eje sin rozamiento de tal manera que la varilla pueda rotar libremente. Al otro extremo de la varilla hay un bloque de masa M. Si se dispara una bala de masa m con una velocidad v 0 tal como se muestra en la figura. Si la bala se incrusta en el bloque, cul ser la velocidad angular del bloque alrededor del eje7

Respuesta. L = mv0 ak
39. Un obs de masa m se dispara de un can en el origen, El obs se mueve en el plano y con una velocidad inicial de magnitud v 0 y un ngulo con el eje x. a) Cul es el torque sobre el obs, con respecto al origen en funcin del tiempo? b) Cul es la cantidad de movimiento angular del, obs con respecto al origen en funcin del tiempo?

dL c) Comprobar que = dt
Respuesta. a)

Respuesta.

mv0 (m + M )a

= v0 mgt cos i

1 2 b) L = v 0 mgt cos i 2
40. Dos esferas pequeas A y B estn unidas por una varilla rgida de longitud l y masa despreciable. Las dos masas reposan sobre una superficie lisa horizontal cuando se comunica repentinamente a A la

42. Una barra de longitud b est pivotada en su centro de tal manera que puede rotar en el plano horizontal. Dos nios estn sobre la barra en las posiciones mostradas en la figura 7.59. a cual est rotando con una velocidad angular en el sentido antihorario visto desde arriba. Si el nio de masa m1 empieza a moverse hacia el centro tal que su distancia a el es b 4 at , Cul debe ser el movimiento del nio de masa m2 para que la velocidad angular de la barra permanezca constante? (La masa de la barra es despreciable),
2

. Hallar: a) La cantidad de velocidad v 0 = v 0 i


movimiento lineal y la cantidad de movimiento angular del sistema respecto al centro de masa.

36

Sistema de partculas

Hugo Medina Guzmn

Respuesta. Debe cambiar su distancia al centro de acuerdo a la ecuacin

49. Un cuerpo de masa m1 = 2 kg se desliza sobre una mesa horizontal sin friccin con una velocidad inicial v1i = 10 m/s, frente a l movindose en la misma direccin y sentido se encuentre el cuerpo de masa m2 = 5 kg cuya velocidad inicial es v2i = 3 m/s, ste tiene adosado un resorte en su parte posterior, cuya constante elstica es k = 1120 N/m, cul ser la mxima compresin del resorte cuando los cuerpos choquen?. Respuesta. x = 0,28 m 50. Un bloque de 3,0 kilogramos, movindose sobre una superficie sin friccin con una velocidad de 1,2 m/s, tiene una colisin perfectamente elstica con un bloque de la masa M en el reposo. Despus de la colisin el bloque de 3,0 kilogramos retrocede con una velocidad de 0,4 m/s.

b 2 m1 2 b + at at 2 16 m2 2

43. Un taco golpea a una bola de billar ejerciendo una fuerza promedio de 50 N durante un tiempo de 0,01 s, si la bola tiene una masa de 0,2 kg, qu velocidad adquiri la bola luego del impacto?. Respuesta. vf = 2,5 m/s 44. Una fuerza acta sobre un objeto de 10 kg aumentando uniformemente desde 0 hasta 50 N en 4 s. Cul es la velocidad final del objeto si parti del reposo?. Respuesta. vf = 10 m/s 45. Se roca una pared con agua empleando una manguera, la velocidad del chorro de agua es de 5 m/s, su caudal es de 300 cm3/s, si la densidad del agua es de 1 g/cm3 y se supone que el agua no rebota hacia atrs, cul es la fuerza promedio que el chorro de agua ejerce sobre la pared?. Respuesta. F = 1,5N 46. Se dispara horizontalmente una bala de 0,0045 kg de masa sobre un bloque de 1,8 kg de masa que est en reposo sobre una superficie horizontal, luego del impacto el bloque se desplaza 1,8 m y la bala se detiene en l. Si el coeficiente de rozamiento cintico entre el bloque y la superficie es de 0,2, cul era la velocidad inicial de la bala?. Respuesta. v1i = 1073 m/s 47. Se dispara una bala de 0,01 kg de masa contra un pndulo balstico de 2 kg de masa, la bala se incrusta en el pndulo y ste se eleva 0,12 m medidos verticalmente, cul era la velocidad inicial de la bala?. Respuesta. v1i = 309,8 m/s 48. Una partcula A de masa mA se encuentra sujeta por medio de un resorte comprimido a la partcula B de masa 2mA, si la energa almacenada en el resorte es de 60 J qu energa cintica adquirir cada partcula luego de liberarlas?. Respuesta. Ec Bf = 20 J

a) La masa M es: b) La velocidad del bloque de masa M despus de la colisin es: c) Los bloques estn en el contacto para 0,20 s. La fuerza media en el bloque de 3,0 kilogramos, mientras los dos bloques estn en contacto, es: Respuesta. a) 6,0 kg, b) 0,8 m/ s, c) 24 N 51. El bloque de 8 kilogramos tiene una velocidad v y es detrs del bloque de 12 kilogramos que tiene una velocidad de 0,5 m/s. la superficie es de friccin despreciable. Los bloques chocan y se juntan. Despus de la colisin, los bloques tienen una velocidad comn de 0,9 m/s.

a) La prdida de energa cintica de los bloques debido a la colisin est la ms cercana a: b) El impulso sobre el bloque de12 kg debido a la colisin es Respuesta. a) 2,4 J, b) 4,8 N s 52. Una bola de acero de 72 g se lanza desde el reposo y cae verticalmente sobre una placa de acero. La bola golpes la placa y est en contacto con ella por 0,5 ms, la bola elsticamente, y vuelve a su altura original. El intervalo de tiempo para el viaje es 0,30 s. a) La fuerza promedio ejercida sobre la bola durante el contacto es b) Asumiendo que la placa no se deforma durante el contacto. La energa elstica mxima almacenada por la bola es: Respuesta. a) 420 N, b) 0,08 J

37

Sistema de partculas

Hugo Medina Guzmn

53. Una bala de la masa 0,01 kilogramos que se mueve horizontalmente golpea un bloque de madera de masa 1,5 kilogramos suspendida como pndulo. La bala se aloja en la madera, y juntos giran hacia arriba una distancia de 0,40 m. cul era la velocidad de la bala momentos antes del impacto con el bloque de madera? La longitud de la cuerda es 2 metros.

56. En una demostracin una bola de acero pequea de la masa m se sostiene sobre una superbola de masa M (superbola es una bola de goma del coeficiente de restitucin muy alto). La combinacin junta se suelta del reposo. Cuando el superbola golpea el piso rebota casi elsticamente, golpeando a bola de acero que todava est movindose hacia abajo. Esta colisin es tambin bastante elstica, y consecuentemente bola de acero se golpea y es lanzada derecho hasta una altura H. Si h es la altura de la cual los objetos fueron soltados, y M > > m, entonces bola de acero pequea se levantar a una altura: Respuesta. 9 h 57. Una muchacha de masa 50 kilogramos lanza una bola de la masa 0,1 kilogramos contra una pared. La bola golpea la pared horizontalmente con una velocidad de 20 m/s, y rebota con esta misma velocidad. La bola est en contacto con la pared 0,05 s, cul es la fuerza media ejercida sobre la bola por la pared? Respuesta. 80N 58. La bola A, de la masa 3,0 kilogramos, se une a una barra ligera de 0,4 m, que gira libremente en P. La bola B est suspendida de Q por una cuerda de 0,6 m y est en reposo. La bola A se levanta a cierto nivel y se suelta. La bola A desciende, y tiene una velocidad v1 = 3,6 m/s en el fondo, antes de chocar a la bola B. Las velocidades de las bolas A y B despus del choque son: v2 = - 1,2m/s y v3 =2,2 m/s...

Respuesta. 66,7m/s 54. Una bala de 10 g se dispara verticalmente en un bloque de 8 kilogramos. El bloque se levanta 3 mm. La bala penetra en el bloque en un intervalo de tiempo de 0,001 s. asume que la fuerza en la bala es constante durante la penetracin.

a) La energa cintica inicial de la bala es: b) El impulse en el bloque debido a la captura de la bala es: c) La penetracin de la bala en el bloque, es: Respuesta. a) 190 J, b) 2,0 Ns, c)) 10 cm.

55. Una bala de 8 g se tira en un bloque de 4,0 kilogramos, en reposo sobre una superficie horizontal sin friccin. La bala se aloja en el bloque. El bloque se mueve hacia el resorte y lo comprime 3,0 centmetros. La constante de la fuerza del resorte es 1500 N/m.

a) La masa de la bola B es: b) La magnitud del impulso sobre la bola A es: c) La bola A rebota y gira un ngulo , donde la velocidad v4 es cero. El valor de es: d) La bola B se eleva hasta la altura h, donde la velocidad v5 es cero. El valor de h es: Respuesta. a) 6,6 kg, b) 14.4 N. s, c) 35 d) 0,25 m 59. Una pieza en forma de L se corta de una hoja uniforme de metal. Cul de los puntos indicados es el ms cercano al centro de la masa del objeto?

a) La velocidad de la bala es: b) ) El impulso del bloque (con la bala), debido al resorte, durante el tiempo en el cual el bloque y el resorte estn en contacto est es: Respuesta. a) 290 m/s, b) 4,7 N.s

38

Sistema de partculas

Hugo Medina Guzmn

Respuesta. Completamente inelstico 65. Las masas de los bloques, y las velocidades antes y despus del choque estn dadas. Qu clase de choque es?

Respuesta. C 60. Las masas de los bloques, y las velocidades antes y despus del choque estn dadas. Qu clase de choque es? Respuesta. Parcialmente inelstico 66. Las masas de los bloques, y las velocidades antes y despus del choque estn dadas. Qu clase de choque es?

Respuesta. Parcialmente inelstico. Respuesta. Parcialmente inelstico 61. Las masas de los bloques, y las velocidades antes y despus del choque estn dadas. Qu clase de choque es? 67. Un resorte activa una bomba de juguete de 1,2 kg sobre una superficie lisa a lo largo del eje x con una velocidad de 0,50 m/s. en el origen O, la bomba estalla en dos fragmentos. El fragmento 1 tiene una masa de 0,4 kilogramos y una velocidad de 0,9 m/s a lo largo del eje y negativo.

Respuesta. Perfectamente elstico.

62. Las masas de los bloques, y las velocidades antes y despus del choque estn dadas. Qu clase de choque es? Respuesta. no posible porque la cantidad de movimiento no se conserva. 63. Las masas de los bloques, y las velocidades antes y despus del choque estn dadas. Qu clase de choque es?

a) La componente en x de la cantidad de movimiento del fragmento 2 debido a la explosin es: b) El ngulo , formado por el vector velocidad del fragmento 2 y el eje x es: c) La energa liberada por la explosin es: Respuesta. a) 0., N. s, b) 31, c) 0,32 J 68. Un cono trunco homogneo de metal tiene una base circular mayor de radio 4 cm y la menor de radio 2 cm. Su altura es 6 cm. A qu distancia de su dimetro mayor est situado el centro de masa?

Respuesta. Caracterizado por un incremento en energa cintica. 64. Las masas de los bloques, y las velocidades antes y despus del choque estn dadas. Qu clase de choque es? Respuesta. 2,36 cm 69. Cuatro masas puntuales se colocan como se muestra en la figura: Cules son las coordenadas del centro de masa? 39

Sistema de partculas

Hugo Medina Guzmn

72. Un carro de 19 kg est conectado por medio de un resorte comprimido con un carro 38 kg. Los dos carros se estn moviendo a la derecha a una velocidad de 25 m/s cuando el resorte se desenrolla y propulsa repentinamente el carro de 19 kg hacia adelante con una velocidad de 27 m/s. encontrar la velocidad del segundo carro con respecto al centro de la masa del sistema. Respuesta. (23, 2,8)

70. Un alambre uniforme de longitud 60 cm y masa 60 g , est doblado en un tringulo rectngulo. Cules son las coordenadas del centro de masa?

Respuesta. 1 m/s 7 3. Una fuerza de 5,3 N es necesaria para sujetar a un paraguas en un viento fuerte. Si las molculas del aire tienen una masa de 4,7 x 10-26 kilogramos, y cada una golpea al paraguas (sin rebotar) con una velocidad de 2,0 m/s en la misma direccin, cuntos tomos golpean al paraguas cada segundo? Asuma que el viento sopla horizontalmente para no tomar en cuenta la gravedad. Respuesta. 5,6 x 1025 por Segundo 74. Un cohete debe ser lanzado al espacio donde no hay campo gravitacional. el 81% de la masa inicial del cohete es combustible y este combustible se expulsa con una velocidad relativa de 2300 m/s. si se asume que todo el combustible ser utilizado, encuentra la velocidad final de la ltima porcin de combustible expulsado relativo a un observador estacionario. Respuesta. 1500 m/ s

Respuesta. (10, 3) 71. Una partcula de la masa 5,01 x 10-27 kilogramos, movindose a 1,88 x 105 m/s, choca con una partcula idntica que inicialmente est en el reposo. Despus de la interaccin, las partculas (que no pueden ser distinguidas) se mueven con los ngulos 55,4 y 34,6, ambos son medidos con respecto a la direccin original del movimiento. Qu velocidades finales tienen las partculas? Respuesta. 1,55 x 105 m/s a 346, 1,07 x 105 m/s a 55,4

40

Cuerpo rgido

Hugo Medina Guzmn

CAPTULO 7. Cuerpo rgido


INTRODUCCION En el capitulo anterior estudiamos el movimiento de un sistema de partculas. Un caso especial importante de estos sistemas es aquel en que la distancia entre dos partculas cualesquiera permanece constante en el tiempo, esto es un CUERPO RIGIDO. A pesar que no existen cuerpos que sean estrictamente rgidos, todos los cuerpos pueden ser deformados, sin embargo el modelo del cuerpo rgido es til en muchos casos en que la deformacin es despreciable. La descripcin cinemtica y dinmica de un cuerpo extenso aunque este sea rgido en un movimiento en tres dimensiones matemticamente es muy complejo y es tratado en libros avanzados de dinmica. Es complejo porque un cuerpo tiene seis grados de libertad; su movimiento involucra traslacin a lo largo de tres ejes perpendiculares y rotacin alrededor de cada uno de estos ejes. No llegaremos a hacer un tratamiento general directo, pero si desarrollaremos el movimiento del cuerpo rgido en dos dimensiones. MOVIMIENTO DE UN CUERPO RGIDO En esta parte expondremos algunos tipos de movimiento de los cuerpos rgidos. TRASLACION. Por traslacin entendemos al movimiento en el que lodos los puntos del cuerpo se mueven en la misma direccin, con la misma velocidad y la misma aceleracin en cada instante. La suma de las fuerzas que actan sobre las n partculas determinan la aceleracin del centro de masa.

aCM =

Fi
M

Tal como se mostr para un sistema de partculas, las fuerzas internas se anulan de pares, de forma que solamente importarn las fuerzas externas tal que

M aCM = Fext
El movimiento de traslacin del cuerpo rgido es como si toda su masa estuviera concentrada en el centro de masa y las fuerzas externas actuaran sobre l. Todo el estudio que hemos lecho anteriormente para la partcula corresponde a la traslacin de un cuerpo rgido. No importa ni la forma, ni el tamao. ROTACIN. Es el movimiento en que uno de los puntos se considera fijo. S se considera fijo un punto, el nico movimiento posible es aquel en el que cada uno de los otros puntos se mueve en la superficie de una esfera cuyo radio es la distancia del punto mvil al punto fijo. Si se consideran dos puntos fijos, el nico movimiento posible es aquel en que todos los puntos con excepcin de aquellos que se encuentran sobre la lnea que une los dos puntos fijos, conocida como EJE, se mueven en circunferencias alrededor de ste.

Por la definicin de centro de masa, tenemos:

rCM =

mi ri m
i

mi ri
M

Donde M es la masa total del cuerpo rgido y

M rCM = mi ri

Cualquier desplazamiento de un cuerpo rgido puede ser considerado como una combinacin de traslacin y rotacin.

Diferenciando dos veces

d2 d2 M 2 rCM = mi 2 ri dt dt
M a CM = mi ai = Fi
1

En los captulos anteriores ya hemos profundizado bastante sobre movimiento de traslacin

Cuerpo rgido

Hugo Medina Guzmn

estudiaremos aqu el movimiento de rotacin alrededor de un eje y el movimiento de rotacin traslacin. CANT1DAD DE MOVIMIENTO ANGULAR DE UN CUERPO RGIDO La cantidad de movimiento angular de una partcula respecto a un punto es

I) El teorema de Steiner o de los ejes paralelos. El momento de inercia del cuerpo respecto a un eje es igual al momento de inercia del cuerpo respecto a un eje paralelo al anterior y que pasa por el centro de masa es el producto de la masa del cuerpo por el cuadrado de la distancia entre los ejes.

I 0 = I CM + Md 2
Demostracin. La figura siguiente representa la seccin de un cuerpo en el plano del papel, CM es el eje normal al plano del papel a travs del centro de masa y O es un eje paralelo. Escogiendo un elemento diferencial de masa dm , escribamos la expresin para los momentos de inercia con respecto a los dos ejes.

L = r p = r m v r = rr , v =

En coordenadas polares:

dr r t& + r& dt dr L = rr m r + rt dt

L = mr 2 r t

tiene la direccin y sentido de r t

L = mr 2

Si consideramos al cuerpo rgido como n partculas que giran alrededor de un eje, la cantidad de movimiento angular de ste ser la suma de la cantidad de movimiento angular de cada una de las partculas.

2 I CM = rCM dm I 0 = r 2 dm M M

usando la ley de los cosenos, obtenemos:


2 r 2 = rCM + d 2 2rCM d cos

reemplazando
M

L total = m r + m r + ........ + m r
2 1 1 2 2 2 2 n n

2 I 0 = rCM + d 2 2rCM d cos dm


2 I 0 = rCM dm + d 2 dm 2d rCM cosdm M M M

= m r + m r + ........ + m r
2 1 1 2 2 2

2 n n

El primer trmino

m r
i =1

i i

2 rCM dm = I CM

El segundo trmino

La cantidad entre parntesis es el MOMENTO DE INERCIA DEL CUERPO RGIDO alrededor de un eje.

d 2 dm = Md 2
M

I = mi ri 2
i =1

El tercer trmino es cero porque es la suma en todo el cuerpo d los productos del elemento de masa y sus distancias al eje a travs del centro de masa, de aqu:

Es importante darse cuenta que el momento de inercia depende de la distribucin de la masa del cuerpo. En el caso de un cuerpo rgido continuo, los mi tienden a dm y

I 0 = I CM + Md 2
II. El teorema de la figura plana. El momento de inercia de una figura plana con respecto a un eje perpendicular a la misma es igual a la suma de los momentos de inercia de la figura plana con respecto a dos ejes rectangulares en el plano de la figura los cuales se intersecan con el eje dado Demostracin: En la figura siguiente el eje z pasa por O perpendicular al piano y. Elegimos un elemento diferencial de masa dm y escribimos los momentos de inercia de la figura para cada uno de los tres ejes.

se transforma en
M

, de aqu:

Como m = V , donde volumen del cuerpo:

I = r 2 dm

es la densidad y V el

dm = dV

Tenemos: I =

r 2 dV

Para muchos cuerpos de forma geomtrica simple sta integral puede evaluarse fcilmente. Dos teoremas que simplifican los clculos del momento de inercia son: 2

Cuerpo rgido = 7 mb + 5ma Aqu comprobamos


2 2

Hugo Medina Guzmn

Iz = Ix + Iy
b) Momento de inercia de una varilla delgada rgida de longitud l y masa m, con respecto a un extremo y con respecto al centro de masa. Solucin.

I x = y 2 dm , I y = x 2 dm , I z = r 2 dm
con
M

r dm = (x + y )dm = x dm + y dm
2 2 2

r =x +y
2 M
2

M 2

Tomemos un elemento diferencial dx, cuya masa es:

Iz = Ix + Iy
Ejemplo 1. A continuacin evaluaremos los momentos de inercia algunos cuerpos simples. a) Hallar el momento de inercia del sistema mostrado en la figura, las masas son puntuales unidas por varillas rgidas de masa despreciable.

dm =

El momento de Inercia de la varilla es:

M dx l
M

I O = x 2 dm = x 2
0

M l 2 M 3 x dx = x l 0 3l 1 3 = Ml 3
=

M dx l

[ ]

l 0

El momento de inercia de la varilla con respecto al centro de masa

Solucin. Momento de inercia respecto al eje x.

I x = y mi
2 i

I CM =
2 2 2

l 2 l 2

M M 3 x dx = x l 3l
2

[ ]

l 2

l 2

= m(0 ) + 2m(0 ) + 3m(b ) + 4m(b )


2

1 Ml 3 12

= 7 mb Momento de inercia respecto al eje y.


2

Aqu comprobamos:

I O = I CM

I y = xi2 mi
= m(0 ) + 2m(a ) + 3m(a ) + 4m(0 )
2 2 2 2

l + M 2

c) Momento de inercia un anillo de masa M y radio R, en el plano xy, Con respecto a los ejes x, y, z. Solucin.

= 5ma Momento de inercia respecto al eje z.


2

I z = ri 2 mi
= m(0 ) + 2m(a ) + 3m a + b
2 2 2

) + 4m(b)

La masa del elemento diferencial ds = Rd es:

Cuerpo rgido

Hugo Medina Guzmn

dm =

M M ds = d 2R 2
2

El momento de inercia del anillo con respecto al eje z es:

I z = R 2 dm = R 2
M
0

M d 2

MR 2 2 [ ]0 = mR2 = 2
Por el teorema de la figura plana

Iz = Ix + Iy
Por simetra

Consideremos la esfera como una serie de discos. Tomemos un disco diferencial como se muestra en la figura, su radio es r = dz. La masa del disco es:

Ix = Iy
Luego

R 2 z 2 , su espesor

Ix = Iy =

Iz 1 = MR 2 2 2

d) El momento de inercia de un disco de radio R y masa M con respecto al eje perpendicular que pasa por su centro. Solucin.

M 2 M r dz = R 2 z 2 dz V V 4 3 M es la masa de la esfera y V = R el 3 dm =
volumen de la esfera. El momento de inercia del disco con respecto al eje z es:

dI z =

2 1 1M dmr 2 = R 2 z 2 dz 2 2V

El momento de inercia de la esfera lo encontramos integrando esta expresin desde z = - R a z = R.

I z = dI z =
Consideremos como elemento diferencial al anillo de radio r y ancho dr, su masa es: =
R M 0 V 2 2 = MR 5

2 1M R 2 z 2 dz 2V 2 8 MR 5 R 2 z 2 dz = 15 V R

dm =

M 2M 2rdr = 2 rdr 2 R R 2M rdr R2

El momento de inercia de este anillo con respecto al eje perpendicular que pasa por O es

dI O = r 2 dm = r 2
=

Para encontrar el momento de inercia con respecto a un eje arbitrario como se muestra en la figura siguiente aplicamos el teorema de Steiner.

2M 3 r dr R2
R

El momento de inercia del disco es:

I O = dI O =
=

2M 3 2M r 4 r dr = R2 R2 4 0

1 MR 2 2 I P = I O + Md 2 = 2 MR 2 + Md 2 5 2 R 2 2 I P = Md 1 + 5d

e) El momento de inercia de una esfera con respecto a un eje que pasa por su centro. Solucin.

En el caso en que R << d podemos considera como 4

Cuerpo rgido

Hugo Medina Guzmn

si fuera una masa puntual y el momento de inercia se reduce a:

I O = Md 2
Ejemplo 2. Hallar el momento de inercia de un disco de masa M y radio R que gira alrededor de un eje paralelo a un dimetro y que pasa por el borde del disco. Solucin.

los torques producidos por las fuerzas externas que actan sobre el sistema es igual al cambio de la cantidad de movimiento angular.

dL = dt

Esto es vlido tambin para el cuerpo rgido, donde L es la cantidad de movimiento angular can respecto al eje x de la figura anterior.

d L d = I Como L = I dt dt

Siendo I el momento de inercia del cuerpo en torno al eje dado, es constante en el tiempo y

d =I dt

Por el teorema de las figuras planas Iz = Ix + Iy ; Adems por simetra I x = I y, Por tanto Ix = Iz/2 = MR2 Aplicando el teorema de Steiner I = MR2 + MR2 = 5/4 MR2 SEGUNDA LEY DE NEWTON PARA ROTACION En esta seccin vamos a analizar el movimiento de un cuerpo rgido que gira en torno a un eje fijo en el espacio.

d Como = , aceleracin angular del cuerpo dt

= I

Esta expresin tiene similitud a la ley de Newton

F = ma

Ejemplo 3. Una barra uniforme de longitud L y masa M, que gira libremente alrededor de una bisagra sin friccin, se suelta desde el reposo en su posicin horizontal, como se muestra en la figura. Calcular la aceleracin angular de la barra y su aceleracin lineal inicial de su extremo.

Solucin. Como el torque de la fuerza en la bisagra es cero, se puede calcular el torque en torno a la bisagra producido por la otra fuerza externa que acta sobre la barra, que es su peso, suponiendo que la barra es homognea y que el peso acta en su centro geomtrico. Entonces: El cuerpo gira en torno al eje x. Si = (t ) es el desplazamiento angular del punto del cuerpo desde la lnea referencial, la velocidad angular del cuerpo es:

= rMg =
Como

d dt

Como cada punto del cuerpo gira a la misma velocidad angular , el desplazamiento (t ) de cualquier punto describe el desplazamiento del cuerpo como un todo. Para el sistema de partculas vimos que la suma de 5

= I , y el momento de inercia de la barra 1 2 es I = ML . 3 1 Se tiene: I = LMg 2

1 LMg 2

Cuerpo rgido

Hugo Medina Guzmn

1 LMg 3g = 2 = 1 2L ML2 3
Para calcular la aceleracin lineal del extremo de la barra, usamos la ecuacin at = L . Reemplazando

5 r 2 b 2 gh 2 sen 7r 2 5b 2

a t = L =

3 g 2

Ejemplo 5. Se tiene un disco de masa M y radio R, que pueda girar libremente alrededor de un eje que pasa por su centro. Se enrolla una cuerda alrededor del disco, se tira la cuerda con una fuerza F. Si el disco est inicialmente en reposos Cul es su velocidad al tiempo t?

Ejemplo 4. Una esfera rueda sobre una barra, con seccin en forma de U, inclinada. Determinar la aceleracin.

Solucin. El momento de inercia del disco con respecto al eje es:

I=
Solucin. Las fuerzas que actan sobre la esfera son el peso, P, la reaccin normal del plano, R, y la fuerza de rozamiento Ff. Como la reaccin R y el rozamiento Ff estn aplicados en el eje instantneo de rotacin no realizan ningn torque, slo el peso:

1 MR 2 2

= FR Como = I
Tenemos

La direccin de la cuerda siempre es tangente al disco por lo que el torque aplicado es:

I
= 2F MR

Reemplazando

FR 1 MR 2 2

= hmg sen , siendo h = (r 2 b 2 )

12

Siendo Como

constante

= 0 + t
2F t MR

El momento de inercia de la esfera con relacin al eje instantneo de rotacin es

0 = 0 = t =

I=

2 2 mr + mh 2 5

Aplicando la ecuacin fundamental de la dinmica de rotacin:

Ejemplo 6. Se sujeta una masa M a una cuerda ligera enrollada alrededor de una rueda de momento de inercia I y radio R. Hallar La tensin de la cuerda, la aceleracin y su velocidad despus de haber descendido una distancia h desde el reposo.

hmgsen hgsen = 2 2 (2mr / 5 + mh ) (2r 2 / 5 + h 2 )

la aceleracin lineal ser: a = h

h 2 gsen gsen a= = 2 2 2 (2r / 5 + h ) (2r / 5h 2 + 1)

Solucin. La figura siguiente muestra los diagramas de cuerpo libre.

Cuerpo rgido

Hugo Medina Guzmn

Aplicando la segunda ley de Newton a la masa M Mg T = Ma (1) Aplicando la segunda ley de Newton para rotacin al disco TR = I ,

a) Por el teorema de las figuras planas, tenemos que: Iz = Ix + Iy ; Adems por simetra I x = I y, Por tanto

Ix =

a como a = R = R a TR = I o TR 2 = Ia R
Resolviendo (1) y (2) obtenemos

Iz 1 1 2 2 3 = LR = (2R )R = R 2 2 2 3 1 = 1,6.10 (0,05) = 6,28x10-5 kg m2

b) Al comunicarle un momento angular L = 7,9 x10-4 kg m2/s, (2)

0 =

L 7,9 10 4 = I 6,28 10 5

M a= g, M + I R2 T= I R2 Mg M + I R2

= 12,58 rad/s c) = 50 dina cm = 50x10-5 Nx10-2 m = 5x10-6 N m Por lo tanto la ecuacin del movimiento en trminos angulares ser:

= 0 + 0t + t 2 = 12,6t 0,0398t 2 , y
Siendo = 0 para t = 158 s. Ejemplo 8. Maquina de atwood tomando en cuenta la polea.

Siendo un movimiento con aceleracin constante


2 v 2 = v0 + 2as Conocemos: a , v0 = 0 , s = h : 2Mg v2 = h M + I R2

1 2 = 12,6 0,079t

v=

2 Mg h M + I R2

Ejemplo 7. Un anillo de 5 cm de radio, grosor despreciable y densidad 1,6 g/cm, se pone en rotacin alrededor de un dimetro cuando se le comunica un momento angular de 7900 g cm2/s. a) Hallar la expresin analtica y el valor numrico del momento de inercia respecto del eje de giro. b) Con qu velocidad angular empieza a girar? c) Si el rozamiento con el aire y los pivotes origina un par de fuerzas cuyo torque es de 50 dina cm, cul ser la ecuacin del movimiento que efecta el anillo?, cunto tiempo tarda en pararse? (Nota 1 N = 105 dinas) Solucin.

La polea es un disco de masa M y radio R. La figura muestra los diagramas de cuerpo libre de cada una de las partes de la mquina de atwood.

Cuerpo rgido

Hugo Medina Guzmn

Aplicando la segunda ley de Newton a cada una de las partes. Masa M1: T1 M 1 g = M 1 a (1) Masa M2: M 2 g T2 = M 2 a (2) Polea:

Planteando la segunda ley de Newton para cada masa: m1 g T1 = m1 a ,

T2 m 2 g = m2 a
Para la polea:

T2 R T1 R = I 1 a 1 = MR 2 = MRa (3) 2 R 2 T2 = M 2 ( g a ) y (m2 m1 ) a= g (m2 + m1 + M 2)

= T R T R = I = I R
1 2

Como el hilo no desliza,

a = R

Resolviendo (1), (2) y (3), obtenemos: T1 = M 1 ( g + a ) ,

Por lo tanto tenemos tres ecuaciones: m1 g T1 = m1 a ,

T2 m 2 g = m2 a , a T1 T2 = I 2 R
Que sumadas dan lugar a: (m1 m2) g = a(m1 + m2 + I/R2). Por lo tanto a vale:

Ejemplo 9. Una polea homognea de radio R, masa M y momento de inercia I, gira alrededor de su eje, debido a la accin de dos masas m1 y m2. R = 0,3 m, m1 =15 kg, m2 = 10 kg, M = 20 kg, I =18 kg m2. Calcular: a) La aceleracin angular de la polea. b) Las tensiones de las cuerdas. c) La tensin del soporte que fija el sistema al techo

a=

m1 m2 I m1 + m2 + 2 R

g =

5 18 25 + 0,3 2

9,8

= 0,22 m / s2 y

a 0,22 = 0,73 rad / s2 = R 0,3

b) De las ecuaciones anteriores obtenemos: T1 = m1 g m1 a = 15( g a ) = 143,7 N.

T2 = m 2 ( g + a ) = 100,2 N. c) Considerando todas las fuerzas que actan sobre la polea, que debe estar en equilibrio:

Solucin. a) Vamos a suponer que el sistema acelera hacia el lado de la masa mayor M.

F = 0
S = P + T1 + T2 = 20 x 9,8 + 146,67 + 102,22 = 445 N

Cuerpo rgido

Hugo Medina Guzmn

Ejemplo 10. La figura representa un cilindro macizo y homogneo de radio R = 20 cm y masa M = 20 kg. A su periferia va arrollado un hilo ideal de cuyo extremo libre cuelga una masa m = 8 kg. Por una hendidura muy fina se le arrolla otro hilo ideal a una distancia del eje horizontal r = 10 cm, a cuyo extremo libre se le aplica una fuerza constante F = 200 N. Calcular: a) Momento de inercia del cilindro respecto a un eje que coincida con una generatriz. b) Aceleracin con que sube la masa m. c) Aceleracin angular del cilindro. d) Tensin del hilo que sostiene la masa.

kg y de la garganta de la polea pequea pende otra masa de 100 kg que tiende a hacer girar a las poleas en sentido contrario al anterior. El momento de inercia del sistema formado por las dos poleas es de 10 kg m2. Al dejar el sistema en libertad, se pone en movimiento espontneamente. Se pide: a) En qu sentido se mueven las poleas? b) Valor de la aceleracin con que se mueve cada una. c) Aceleracin angular de las poleas. d) Tensin de la cuerda que sostiene la masa de 100 kg cuando el sistema est en movimiento. Solucin.

Solucin. a) Aplicando el teorema de Steiner, I = MR2 +MR2 = 3/2 MR2 a) Cuando las poleas estn inicialmente en reposo, los pesos coinciden con las tensiones. Por tanto T1 = 200 N, y T2 = 1000 N. El momento que ejerce T1 valdr 1 = T1 R1 = 200 Nm El que ejerce T2 valdr 2 = T2 R2 = 300 N m. Por tanto, al ser el momento de la fuerza T2 mayor, la polea girar de modo que la masa M1suba. b) y c) Planteando la ecuacin fundamental de la dinmica a cada masa y a la polea, tendremos:

b) Podemos plantear dos ecuaciones: T mg = ma y

1 a 1 Fr TR = I = MR 2 = MRa 2 R 2
Que conducen a:

1 Fr mgR = a mR + MR . 2
Por lo tanto la aceleracin a vale:

a=

= 1,2 m / s2 c)

20 15,68 Fr mgR = 1 1,6 + 2 mR + mR 2 a 1,2 = R 0,2

(3) De las tres ecuaciones obtenemos :

T1 M 1 g = M 1 a1 T1 M 1 g = M 1R1 M 2 g T2 = M 2 a 2 M 2 g T2 = M 2R2 2 1 = I T2 R2 T1 R1 = I

(1) (2)

M 2 gR2 M 1 gR1 2 M 2 R2 + M 1 R12 + I 30 20 = g = 2,51 rad / s2. 20 + 9 + 10

= 6 rad/s2. d) T = mg + ma = 8 (9,8 +1,2) = 88 N. Ejemplo 11. Dos poleas cuyos radios son 1 m y 0,3 m, estn acopladas pegada una a la otra en un plano vertical, formando un bloque que gira alrededor de su eje de rotacin comn. De la garganta de la polea grande pende una masa de 20 9

La aceleracin de cada masa ser: a1 = R1 = 2,51 m/s2,

a 2 = R2 = 0,75 m/s2 d) T2 = M 2 g M 2R2 = 904,7 N


Ejemplo 12. Un rollo de 16,0 kg de papel con radio R = 18,0 cm descansa contra la pared

Cuerpo rgido

Hugo Medina Guzmn

sostenido por un soporte unido a una varilla que pasa por el centro del rollo. La varilla gira sin friccin en el soporte, y el momento de inercia del papel y la varilla alrededor del eje es de 0,260 kg. m2. El otro extremo del soporte est unido mediante una bisagra sin friccin a la pared de modo que el soporte forma un ngulo de 30,0 con la pared. El peso del soporte es despreciable. El coeficiente de friccin cintica entre el papel y la pared es k = 0,25 . Se aplica una fuerza vertical constante F = 40,0 N al papel, que se desenrolla. a) Qu magnitud tiene la fuerza que la varilla ejerce sobre el rollo de papel al desenrollarse? b) Que aceleracin angular tiene el rollo?

(40,0 31,54)(18,0 102 ) = . I (0,260) = 4,71 rad/s 2

Ejemplo 13 Se debe aplicar una sola fuerza adicional a la barra de la figura para mantenerla en equilibrio en la posicin mostrada. Puede despreciarse el peso de la barra. a) Calcule las componentes vertical y horizontal de la fuerza requerida. b) Qu ngulo debe formar sta fuerza con la barra? c) Qu magnitud debe tener? d) Dnde debe aplicarse?

Solucin. En el punto de contacto, la pared ejerce una fuerza F f de la friccin dirigida hacia abajo y una fuerza normal N dirigida a la derecha. Esto es una situacin donde es cero la fuerza neta en el rodillo, pero el torque neto no es cero. La suma de fuerzas verticales

Solucin. a) La tensin en el resorte es W2 = 50 N, y la fuerza horizontal sobre la barra debe equilibrar la componente horizontal de la fuerza que el resorte ejerce sobre la barra, y es igual a (50 N) sen 37 = 30 N, a la izquierda en la figura. La fuerza vertical debe ser

Fvar cos = F f + W + F , F f = k N ,

50 cos 37 + 10 = 50 N, arriba
50 N arctan 30 N = 59

Las fuerzas horizontales Fvarsen = N . De aqu tenemos:

b)

Fvar cos = k N + F + W Fvarsen = N .


a) Eliminando N y resolviendo para Fvar da

c)

(30 N) 2 + (50 N) 2 = 58 N.
d) Tomando torques alrededor (y midiendo la distancia de) del extremo izquierdo

Fvar =

W +F cos k sin 40,0 + (16,0) (9,80) = 266 N = cos 30 (0,25)sen30

50 x = (40)(5,0 ) x = 4,0 m

Donde solamente las componentes verticales de las fuerzas ejercen torques. Ejemplo 14. Imagine que est tratando de subir una rueda de bicicleta de masa m y radio R a una acera de altura h; para ello, aplica una fuerza horizontal F. Qu magnitud mnima de F logra subir la rueda si la fuerza se aplica a) al centro de la rueda? b) En la parte superior de la rueda? c) En cul caso se requiere menos fuerza?

b) Con respecto al centro del rodillo, la barra y la fuerza normal ejercen el torque cero. La magnitud del torque neto es ( F F f ) R , y F f = k N Puede calcularse reemplazando el valor encontrado para Fvar en cualquiera de las relaciones anteriores;

F f = k Fvarsen = 33,2 N .
Luego,

10

Cuerpo rgido

Hugo Medina Guzmn

Aplicando la segunda ley de Newton en la masa C:

m C g T = mC a 8 g T = 8 R A
Aplicando la segunda ley de Newton de la rotacin en el conjunto giratorio:

TR B = I

Solucin. a) Tome los torques respecto a la esquina superior de la acera. La fuerza F acta a una distancia perpendicular Rh y el peso acta en una distancia perpendicular

Resolviendo el sistema formado:


2 8 gR B TR B = 8R B 2 8 gRB = 8aRB + I TR B = I 8 gR B 35,28 = = 0,66 rad/s2 = 2 8 R B + I 53,18 b) a 0 = R0 = 0,6 m/s2

R 2 (R h ) =
2

2 Rh h 2 .

Igualando los torques para encontrar la fuerza necesaria mnima,

c)

(4 s ) = t

= 2,65 rad/s

F = mg

2 Rh h 2 . Rh

aN = 2 RD = 6,34 m/s2
EQUILIBRIO ESTTICO En el captulo 5 vimos que para que una partcula estuviera en equilibrio esttico era suficiente que La fuerza resultante fuese cero.

b) El torque debido a la gravedad es el mismo, pero la fuerza F acta a una distancia perpendicular 2 R h, tal que la fuerza mnima es

F = mg

2 Rh h . 2R h
2

F = 0
Esta condicin tambin, es necesaria para que un cuerpo rgido este en equilibrio, pero no es suficiente que solamente el centro de masa este en reposo, el cuerpo puede girar Es necesario que el momento de: fuerzas o torque con respecto al centro de masa sea nulo.

c) Se requiere menos fuerza que cuando la fuerza se aplica en parte alta de la rueda. Ejemplo 15. Un disco homogneo A gira alrededor del eje y bajo la accin de la masa C unida a una cuerda que pasa por una polea sin peso ni rozamiento enrollada alrededor del tambor cilndrico macizo B, solidaria del disco A. A ste est unida una masa puntual D, como indica la figura. Las masas A, B, C y D son respectivamente 65, 15, 8 y 4 kg. Se supone que la cuerda permanece siempre horizontal. Calcular: a) Aceleracin angular del disco. b) Aceleracin tangencial de D. c) Aceleracin normal de D, 4 s despus de partir del reposo.

=0

A continuacin desarrollaremos algunos ejemplos de aplicacin. En muchos de ellos la fuerza de la gravedad ejercida sobre las diversas partes de un cuerpo puede sustituirse por una sola fuerza, el peso total actuando en el centro de gravedad. Si la aceleracin de la gravedad no vara a lo largo del cuerpo, el centro de gravedad coincide con el centro de masa. Ejemplo 16. Demostrar que cuando un cuerpo est en equilibrio y el torque con respecto al centro de masa es cero, el torque con respecto a cualquier punto tambin es cero. Solucin.

Solucin. a) Calculemos en primer lugar el momento de inercia del sistema A-B-D.

1 1 1 2 2 2 I = mA RA + m B RB + m D RD 2 2 2
= 51,56 kg m2 11

En la figura

rO es el vector del centro de masa a O

Cuerpo rgido

Hugo Medina Guzmn

ri es el vector del centro de masa al punto donde


acta Fi .

rOi es el vector del punto O al punto donde acta Fi .


De la figura vemos:

ri = rO + rOi

El torque total alrededor de O es

Solucin. a) Sustituir la fuerza vertical dada por otra igual paralela cuya lnea de accin pase por el centro de masa.

O = r Oix F i
i i i O i i

=
i

ri rO F i = i
=

r F r F
Como rO es constante

CM rO F i
i

O = CM rO F i
i

b) Hacer girar el plano del par, hasta desplazarlo hasta la lnea A B.

Para un cuerpo en equilibrio tal que

Fi = 0

O = CM

Si CM = 0 , el torque alrededor de cualquier punto debe ser cero y viceversa. Ejemplo 17. Par de fuerzas. Dos fuerzas iguales y opuestas que actan en la figura siguiente se denominan par de fuerzas, Segn se indica c) Se cambian los mdulos de las fuerzas a F de tal modo que:

F ' b = Fa F ' = F

a b

F es el valor de cualquiera de las fuerzas y d = ( x 2 x1 ) es la distancia entre ellas. El momento o torque producido por estas fuerzas con respecto a O es: O = Fx 2 Fx1 = F ( x 2 x1 ) = Fd Este resultado no depende de la seleccin del punto O, el momento producido por un par es el mismo respecto a cualquier punto del espacio. Ejemplo 18. Una fuerza vertical F que acta en A. en el slido rectangular mostrado en la figura, queremos sustituirla por otra cuya lnea de accin pasa por el centro de masa ms un par de fuerzas que acten horizontalmente aplicados en A y B. Ejemplo 19. Sobre una placa slida actan cuatro fuerzas de mdulos F1 = 28,3 N, F2 = 60 N, F3 = 20 N y F4 = 50 N. Como se indican en la figura. Hallar la tuerza resultante sobre la placa y determinar su lnea de accin.

12

Cuerpo rgido

Hugo Medina Guzmn

+ y r = xi j

Tal que

= r F = 6k

+ y = xi j

= (40 x 30 y )k De aqu:

) (30i + 40 j )

(40 x 30 y ) = 6

20 x 15 y = 3
Solucin. Utilizando el cuadriculado obtenemos:

0,2 r1 = 0,2i j,
2 + 28,3 2 i F1 = 28,3 2 j = 20i + 20 j 2

Esta es la ecuacin de la lnea de accin de la fuerza; si esta tuerza a de situarse en algn punto del borde inferior de la placa, y = - 0,2 m.. Obtenemos

x=

3 + 15 y 3 + 15( 0,2) = =0 20 20

0,2 r2 = 0,1i j , F2 = 60 j
+ 0,1 r3 = 0,2i j , F3 = 20i
3 3 + 0,2 50 r4 = 0,1i j , F4 = 50 i j 4 4 40 j = 30i

La figura siguiente muestra la fuerza resultante:

La fuerza resultante es

F = F1 + F2 + F3 + F4 = (20 20 + 30 )& i& + (20 + 50 40) j


=

(30i + 40 j )N

El torque resultante respecto al centro de masa es la suma de los torques individuales.

Ejemplo 20. Se tiene una escalera d masa M y largo L apoyada contra la pared .No hay friccin en la pared y el coeficiente de friccin del piso es . Cul es el mnimo ngulo de inclinacin para que no comience a resbalar?

= 1+ 2 + 3 + 4
Siendo:

1 = r1 F1

0,2 + 30 = 0,2i j 20i j =0

) (

2 = r2 F2
=

(0,1i 0,2 j ) (60 j ) = 6k


3 = r3 F3
=

Solucin. La figura siguiente muestra el diagrama del cuerpo libre de la escalera.

(0,2i + 0,1 j ) ( 20 j ) = 2k ( ) (20i 40 j ) = 2k

4 = r4 F4

+ 0,2 = 0,1i j Reemplazando: = 6k

Nm

Para determinar la lnea de accin de la tuerza, consideremos que el punto de aplicacin de la fuerza resultante es: 13

Condicin para que el centro de masa no acelere:

Cuerpo rgido

Hugo Medina Guzmn R es a reaccin de la pared. Como el sistema est en equilibrio

F F

x y

= 0 = N x N y , = 0 = Mg N y

De aqu obtenemos: N y = Mg , N x =

N y = Mg

Condicin de no rotacin: La suma de momentos de fuerza con respecto al centro de masa es cero.

Fx = R cos T cos = 0 Fy = Rsen Tsen Mg mg = 0


Con

F = 0

L L L N y cos N y sen N x sen = 0 2 2 2


Reemplazando las fuerzas: L L L Mg cos Mg sen Mg sen = 0 2 2 2

= 0

alrededor de cualquier punto.

Tomamos momentos con respecto a O.

TLsen mg

L Mgx = 0 2
L 2

De esta ltima ecuacin obtenemos

2sen = cos

1 = tan 1 2
Otra forma: En lugar de tomar el centro de masa como origen tomemos extremo inferior de la escalera. Tomando momentos con respecto a este punto.

x=

TLsen mg Mg

Si T = Tm obtenemos el valor mximo de x. Si estuviramos interesados en conocer R, sera mejor tomar momentos con respecto al otro extremo. Ejemplo 22. Un albail de 75 kg camina sobre un tabln de 3 m de largo y 80 kg apoyado sobre dos vigas distantes 2 m, tal como indica la figura. Cul es la mxima distancia x que puede recorrer, sin que caiga?

Mg Mg

L cos N x Lsen = 0 2

Reemplazando el valor de Nx:

L cos MgLsen = 0 2 1 = tan 1 2


Obtenemos la misma respuesta porque no importa con respecto a que eje tomemos el torque. Ejemplo 21. Una viga de masa m se empotra a la pared como se muestra en la figura y se sujeta por medio de un alambre. Si la tensin en el alambre excede Tm el alambre se rompe. Para qu valor de x, el alambre se romper por una masa M colocada sobre la viga?

Solucin. Para que el tabln gire, el torque del peso del albail respecto del punto O, ms el torque del peso de la parte de tabln que sobresale, debe ser mayor o igual que el torque del peso de la parte de tabln apoyada entre las vigas: Llamando a la densidad lineal del tabln:

M , haciendo d = 2 m, L = longitud del L

tabln, M = masa tabln, m = masa albail tendremos:

mgx + (L d )g
Solucin. La figura muestra el diagrama del cuerpo libre del sistema viga-masa.

(L d ) = dg d
2

mx + d 2 (L d )2 = M (2 Ld L2 ) ,
2L

M (2d L ) = 0,53 m x= 2m
Ejemplo 23. Un bal de masa M se empuja sobre un suelo con coeficiente de rozamiento a) Qu fuerza F se ejerce si el bal se mueve con aceleracin constante a? b) Si el bal se mueve con velocidad constante? 14

Cuerpo rgido

Hugo Medina Guzmn

c) Qu fuerza se necesita para inclinar el bal?

La componente horizontal de la fuerza ejercida en la barra por la bisagra debe equilibrar la fuerza

F aplicada, y as tiene magnitud 120,0 N y es


hacia la izquierda. Tomando torques alrededor del punto A

(120,0 N)(4,00 m) + FV (3,00 m)


Solucin. La figura siguiente muestra el diagrama del cuerpo libre del bal. La componente vertical es 160 N, el signo menos indica una componente hacia abajo, ejerciendo un torque en una direccin opuesta a la de la componente horizontal. La fuerza ejercida por la barra en la bisagra es igual en magnitud y contrario en la direccin a la fuerza ejercida por la bisagra en la barra Ejemplo 25. La caja es arrastrada sobre una superficie horizontal con rapidez constante por una fuerza. El coeficiente de friccin cintica es de 0,35. a) Calcule la magnitud de F. b) Determine el valor de h con el cual la caja comience a volcarse.

a) Aplicando la segunda ley de Newton.

F F

x y

= F k ( N 1 + N 2 )Ma , = N 1 + N 2 Mg = 0

Resolviendo las ecuaciones:

F = M (a + k g )

b) En el caso que el bal va con velocidad constante a = 0 y F = M k g c) Para analizar la inclinacin del bal tenemos que escribir la ecuacin de momentos con respecto al borde delantero, sin rotacin = 0, luego Solucin. a) F = F f = k = k mg = (0,35)(30,0 kg)(9,80 m s ) = 103 N b) Con respecto al borde delantero de la caja. El brazo de palanca del peso es
2

= bN
F= a= bMg 2h

hF +

b Mg = 0 2

Cuando el bal empiece a inclinarse, empezar a rotar en el sentido horario y N1 = 0, de aqu:

0,250 = 0,125 m 2
El brazo de palanca h de la fuerza aplicada es entonces

y la aceleracin:

F b k g = k g M 2h

h = (0,125)
=

1 mg = (0,125) k F

Ejemplo 24. El extremo A de la barra AB de la figura descansa en una superficie horizontal sin friccin, y el extremo B tiene una articulacin. Se ejerce en A una fuerza horizontal F de magnitud 120 N. Desprecie el peso de la barra. Calcule las componentes horizontal y vertical de la fuerza ejercida por la barra sobre la articulacin en B.

0,125 = 0,36 m. 0,35

TRABAJO Y ENERGIA EN ROTACIN. Consideremos un cuerpo que gira alrededor de un eje tal como se muestra en la figura

Solucin.

La energa cintica de un elemento de masa dm que gira a una distancia r del eje de rotacin es: 15

Cuerpo rgido

Hugo Medina Guzmn

1 dmv 2 , v = r 2 1 dK = dm 2 r 2 2
dK =
Integrando.

1 2 1 2 I2 I1 2 2 = K 2 K 1 = K

K = dK =
como

1 2 2 r dm 2

E1 trabajo neto realizado por las fuerzas externas al hacer girar un cuerpo rgido alrededor de un eje fijo es igual al cambio en la energa cintica de rotacin. Por la analoga que existe entre las expresiones para el movimiento lineal y el movimiento angular, podemos decir que un torque ser conservativo a condicin que exista una funcin potencial U = U ( ) de tal modo que el trabajo efectuado por , cuando el cuerpo sufre un desplazamiento angular ( 2 1 ) es la diferencia U (1 ) U ( 2 ) .

es constante.

1 K = dK = 2 r 2 dm M 2
El trmino integral es el momento de inercia del cuerpo con respecto al eje de rotacin

K=

1 2 I 2

Para relacionar la energa cintica, al trabajo efectuado sobre el cuerpo por un torque . Supongamos que se aplica una fuerza externa nica F, que acta en el punto P del cuerpo.

As pues se deduce que:

U (1 ) U ( 2 ) = K 2 K 1

K 1 + U (1 ) = K 2 + U ( 2 ) = constante

Cuando el sistema no es conservativo

W NO CONSERVATI VO = K 1 + U (1 ) K 2 + U ( 2 )
POTENCIA La rapidez con que se realiza este trabajo es:

) (

dW d = = dt dt

El trabajo realizado por F a medida que el cuerpo gira recorriendo una distancia infinitesimal ds = rd en un tiempo dt es:

Expresin que corresponde a la potencia instantnea.

P =

dW = F d s = Fsen rd Como Fsen r es el torque de la fuerza F


alrededor del origen se puede escribir el trabajo realizado para la rotacin infinitesimal como:

Ejemplo 26. Para la barra giratoria, calcular su rapidez angular y la rapidez lineal de su centro de masa y del punto mas bajo de la barra cuando est vertical.

dW = d

Cuando el cuerpo gira en torno a un eje fijo bajo la accin de un torque. El cambio de su energa cintica durante el intervalo dt se puede expresar como:

dK =

dK d 1 dt = I 2 dt dt dt 2 d = I dt = I dt = I dt dt

Solucin.

Como

= I y d = dt Obtenemos:
dK = d = dW
2

Si se ntegra esta expresin se obtiene el trabajo total

W12 = d =
1

2
1

Id

Usando el principio de conservacin de la energa, considerando que la energa potencial se calcula

16

Cuerpo rgido

Hugo Medina Guzmn

respecto al centro de masa y la energa cintica es de rotacin:

Ei = E f K i + U gi = K f + U gf
Cuando la barra esta inicialmente horizontal no tiene Ki y cuando esta vertical tiene solo Kf, entonces:

11 L 1 Mg = I 2 = ML2 2 2 2 23

Donde se ha usado la relacin v = R , despejando v se obtiene:

1 1 1 m1v 2 + m2 v 2 + I 2 + m1 gH 2 2 2 1 I m1 + m2 + 2 v 2 = (m2 m1 )gH 2 R


=

v=

=
vcm

3g L

2(m2 m1 )gH I m1 + m2 + 2 R

Para calcular la rapidez del centro de masa, se usa:

L 1 = r = = 3gL 2 2

En el punto mas bajo la rapidez es

v = 2v cm = 3 gL
Ejemplo 27. Para el sistema de la figura, las masas tiene momento de inercia I en torno a su eje de rotacin, la cuerda no resbala en la polea y el sistema se suelta desde el reposo. Calcular la rapidez lineal de las masas despus que una ha descendido H y la rapidez angular de la polea.

Ejemplo 28. Sobre un cilindro homogneo de radio R y masa M. tiene El cual tiene libertad de girar sin friccin sobre un eje, como se muestra en la figura. Si se le aplica en su borde una fuerza tangencial de magnitud F. a) Cul es la aceleracin angular del cilindro? b) Cual es la velocidad angular y la energa cintica del cilindro al tiempo t? c) qu cantidad de trabajo aplica la fuerza durante este intervalo t?.

Solucin. El momento de inercia del cilindro en torno a su eje es:

I=

1 MR 2 2 =

Solucin. Como no hay roce en la polea, se conserva la energa, que aplicada a cada masa m1 y m2, suponiendo que m2 se encuentra inicialmente en la parte superior del sistema, es:

Ei = E f

, = F0 R I 2F F0 R = 0 tenemos = 1 MR 2 MR 2 b) Siendo constante = 0 + t 2 F0 Si 0 = 0 , = t , = t MR a) Con = I La energa cintica:

1 11 2 F K = I 2 = MR 2 0 t 2 22 MR F2 2 = 0 t M
c) El trabajo realizado

K 1i + K 2i + U 1i + U 2i = K1 f + K 2 f + K p + U 1 f + U 2 f 0 + m2 gH
17

W = K = K 2 K 1 =
=

F02 2 t 0 M

F02 2 t M

Cuerpo rgido

Hugo Medina Guzmn

Otra forma de calcular es:

W = F0 R( 2 1 ) = F0 R 2 F0 Con = , 0 = 0 MR 1 1 2F F = t 2 = 0 t 2 = 0 t 2 2 2 MR MR
Finalmente

W = d , = F0 R (constante)
1

Luego W = MgR

2R
4g

1 M 2 R 2 4

Otra forma de evaluar el trabajo es por la conservacin de la energa.

W = K = K 2 K 1 =

1 2 I 0 2 11 1 2 2 2 2 = MR = M R 22 4

F F 2 t W = F0 R 0 t 2 = MR M
2 0

Ejemplo 29. Un carrete de hilo delgado tiene radio R y masa M. Si se jala el hilo de tal modo que el centro de masa del carrete permanezca suspendido en el mismo lugar. a) Qu fuerza se ejerce sobre el carrete? b) Cunto trabajo se habr realizado cuando el carrete gira con velocidad angular ? Solucin. La figura muestra al carrete suspendido.

Ejemplo 30. Una plataforma cilndrica uniforme de 180 kg de masa y 4,5 m de radio se frena de 3,2 rev/s al reposo en 18 s cuando se desconecta el. motor. Calcular la potencia de salida del motor (hp) para mantener una velocidad constante de 3,2 rev/s. Solucin. Como primer paso debemos conocer cul es el torque de frenado que tenemos que vencer para mantener la velocidad constante, ese torque lo calcularemos de la siguiente manera:

frenado = I frenado
I=

1 MR 2 . 2 2 1 frenado = = = t t 2 t1 t

frenado =

MR 2 1 MR 2 = 2 t 2t
MR 2 2 2t

La potencia es:

P = =
El carrete solo tiene movimiento circular ya que est en equilibrio vertical Aplicando las leyes de Newton:

Siendo M =180 kg, R = 4,5 m,

F = 0 = I
y

T Mg = 0 TR = I

= 3,2
t = 18 s.

rev 2 rad rad = 6,4 , s rev s


= 40889,73 W

1 2 Como I = MR , obtenemos: 2 1 MgR = MR 2 2 2g y = R


a) La fuerza que se ejerce sobre el carrete es

2 2 ( 180)(4,5) (6,4 ) P= 2(18)

Como 1 hp = 735,5 W

P = 55,6 hp
Ejemplo 31. Se sujeta una masa M a una cuerda ligera enrollada alrededor de una rueda de momento de inercia I y radio R. Hallar La tensin de la cuerda, la aceleracin y su velocidad despus de haber descendido una distancia h desde el reposo. Resolver desde el punto de vista de energa.

T = Mg

b) Como el trabajo realizado es:

W = , donde = TR = MgR 2
Siendo Si

constante = 0 + 2
2 2

0 = 0 2 = 2
=

2 2

2R
4g
18

Cuerpo rgido

Hugo Medina Guzmn

Solucin. Por el principio de conservacin de la energa Etotal = constante Al inicio del movimiento toda la energa es potencial, si consideramos como nivel cero el indicado en la figura (a).

Ei = Mgh

Las masas M1 y M2 inicialmente estn en reposo en la posicin y = 0 , despus de soltarlas una sube y la otra baja como muestra la figura. Las masas estarn movindose con velocidad v la Polea tendr una velocidad angular . Como no hay rozamiento por la conservacin de la energa

La energa final es pura energa cintica, de la nasa M con velocidad v antes de chocar y el disco con momento de Inercia I con velocidad angular = v R , figura (b).

1 1 v 1 1 Mv 2 + I 2 = Mv 2 + I 2 2 2 2 R 2 v 1 = M + 2 2 R Como E i = E f Ef =
v2 1 Mgh = M + 2 2 R 2M 2 y v = gh M +1 R2

E1 = E 2 1 1 0 = + M 1v 2 + M 2 v 2 + 2 2 1 2 I + M 1 gy M 2 gy 2 v 1 2 Siendo = , I = MR , tenemos: R 2 M 2 1 M 1 + M 2 + v = (M 1 M 2 )gy 2 2 2(M 1 M 2 ) v2 = gy M M1 + M 2 + 2


Para un movimiento uniformemente acelerado

v 2 = 2ay
Comparando:

a=

(M 2 + M 1 + M 2 )

(M 2 M 1 )

v=

2M gh M +1 R2

Ejemplo 32. Resolver la mquina le Atwood utilizando Conceptos de trabajo y energa, Solucin.

Ejemplo 33. Una canica slida uniforme de radio r parte del reposo con su centro de masa a una altura h sobre el punto ms bajo de una pista con un rizo de radio R. La canica rueda sin resbalar. La friccin de rodamiento y la resistencia del aire son despreciables. a) Qu valor mnimo debe tener h para que la canica no se salga de la pista en la parte superior del rizo? (Nota: r no es despreciable en comparacin con R.) b) Qu valor debe tener h si la pista est bien lubricada, haciendo despreciable la friccin?

19

Cuerpo rgido

Hugo Medina Guzmn

Solucin. a) De a a B, la distancia que la canica ha cado es

y = h (2 R r ) = h + r 2 R.

1 2 1 2 7 2 v + v = v 2 5 10 7 g (h 2 R + r ) = v 2 10 2 Reemplazando el valor de v : 7 g (h 2 R + r ) = g (R r ) 10 7 h 2 R + r = (R r ) 10 7 h = 2 R r + (R r ) 10 7 (R r ) = 2R r + 10 27 17 R r 10 10 = 2,7 R 1,7 r


= b) En ausencia de friccin no habr rotacin. Luego:

g (h 2 R + r ) =

El radio de la trayectoria del centro de masa de la canica es R r , .

mgy =

1 2 mv 2
2

Sustituyendo las expresiones para y y v en trminos de los otros parmetros da

h 2R + r =
La condicin para que la canica permanezca en la pista es

1 (R r ) 2

Resolviendo obtenemos

Fr = mac mg = m
v 2 = g ( R r ).

v2 (R r )

h=

5 3 R r. 2 2

La velocidad se determina del teorema del trabajo energa,

mgy =

1 2 1 2 mv + I 2 2

y = h (2 R r ) v = r
Se sabe que para una esfera

Se tiene:

Ejemplo 34. La figura muestra tres yoyos idnticos que inicialmente estn en reposo en una superficie horizontal. Se tira del cordel de cada uno en la direccin indicada. Siempre hay suficiente friccin para que el yoyo ruede sin resbalar. Dibuje un diagrama de cuerpo libre para cada yoyo. En qu direccin girar cada uno? Explica tus respuestas

I=

2 2 mr 5 1 2 1 2 2 v mv + mr 2 25 r
2

Reemplazando estos valores en la ecucin de la energa:

mg (h 2 R + r ) =

Solucin. En el primer caso, F y la fuerza de la friccin actan en direcciones opuestas, y la fuerza de friccin tiene el torque mayor que hace rotar el yo-yo a la derecha. La fuerza neta a la derecha es la diferencia F F f , tal que la fuerza neta es a la derecha mientras que el torque neto causa una rotacin a la derecha. 20

Cuerpo rgido

Hugo Medina Guzmn

Para el segundo caso, el torque y la fuerza de friccin tienden a dar vuelta al yoyo a la derecha, y el yo-yo se mueve a la derecha. En el tercer caso, la friccin tiende a mover al yoyo a la derecha, y puesto que la fuerza aplicada es vertical, el yoyo se mueve a la derecha.

mgh = =

1 2 1 2 mv + I 2 2

Rodar sin resbalar significa

Ejemplo 35. Una canica uniforme baja rodando sin resbalar por el trayecto de la figura, partiendo del reposo. a) Calcule la altura mnima h que evita que la canica caiga en el foso. b) El momento de inercia de la canica depende de su radio. Explique por qu la respuesta a la parte (a) no depende del radio de la canica. c) Resuelva la parte (a) para un bloque que se desliza sin friccin en vez de una canica que rueda. Compare la h mnima en este caso con la respuesta a la parte (a).

v 2 2 , I = mr 5 r 1 2 1 2 Tal que I = mv 2 5 7 mgh = mv 2 10 7v 2 7(17,82 m/s) h= = 10 g 10(9,80 m/s 2 )


= 23 m b)

1 2 1 2 I = mv , Independiente de r. 2 5

c) Todo es igual, excepto que no hay el trmino de energa rotacional cintica en K:

1 2 mv 2 1 mgh = mv 2 2 2 v = 16 m . h= 2g

K=

Comparado con la altura de la parte (a), 16 /23 = 0,7, es el 70 %. Ejemplo 36. Una esfera slida uniforme rueda sin resbalar subiendo una colina, como se muestra en la figura. En la cima, se est moviendo horizontalmente y despus se cae por un acantilado vertical. a) A qu distancia del pie del acantilado cae la esfera y con qu rapidez se est moviendo justo antes de tocar el suelo? b) Observe que, al tocar tierra la esfera, tiene mayor rapidez de traslacin que cuando estaba en la base de la colina. Implica esto que la esfera obtuvo energa de algn lado? Explique.

Solucin. a) Encuentre la velocidad v que necesita la canica en el borde del hoyo para hacerlo llegar a la tierra plana en el otro lado. La canica debe viajar 36 m horizontalmente mientras cae verticalmente 20 m. Use el movimiento vertical para encontrar el tiempo. Tome + y hacia abajo.

v 0 y = 0, a y = 9,80 m/s 2 , y y0 = 20 m, t = ?

1 a y t 2 t = 2,02 s 2 Luego x x 0 = v 0 x t v 0 x = 17,82 m/s.

y y 0 = v0 y t +

Utilice la conservacin de la energa, donde el punto 1 est en el punto de partida y el punto 2 est en el borde del hoyo, donde v = 17,82 m/s. Haga y = 0 en el punto 2, tal que

y 2 = 0 e y1 = h K1 + U 1 = K 2 + U 2

Solucin. a) Use la conservacin de la energa para encontrar la velocidad v 2 de la bola momentos antes que salga de la parte alta del acantilado. Sea el punto 1 en la base de la colina y el punto 2 en la cima de la colina. Tome y = 0 en la base de la colina, tal que

y1 = 0 e y2 = 28,0 m. K1 + U 1 = K 2 + U 2
21

Cuerpo rgido

Hugo Medina Guzmn

1 2 1 2 1 2 1 2 mv1 + I1 = mgy2 + mv2 + I2 2 2 2 2 Rodar sin resbalar significa = v r y 1 1 2 11 2 I = mr (v / r ) 2 = mv 2 5 25 2 7 7 2 mv12 = mgy 2 + mv 2 10 10 10 v 2 = v12 gy 2 = 15,26 m s 7
Considere el movimiento de proyectil de la bola, despus de salir de la cima del acantilado hasta justo antes de tocar tierra. Tome + y hacia abajo. Utilice el movimiento vertical para encontrar el tiempo en el aire:

en un eje sin friccin que pasa por su centro. Un bloque de masa M se suspende del extremo libre del hilo. El hilo no resbala en la polea, y el cilindro rueda sin resbalar sobre la mesa. Si el sistema se libera del reposo, qu aceleracin hacia abajo tendr el bloque?

v0 y = 0 , a y = 9,80 m/s 2 y y 0 = 28,0m , t = ? 1 y y 0 = v 0 y t + a y t 2 t = 2,39 s 2


Durante este tiempo la bola viaja horizontalmente x x 0 = v0 x t = (15,26 m s )(2,39 s ) = 36,5 m. Justo antes de tocar tierra,

Solucin. Hacer este problema usando la cinemtica implica cuatro incgnitas (seis, contando las dos aceleraciones angulares), mientras que usando consideraciones de la energa se simplifican los clculos. Si el bloque y el cilindro ambos tienen velocidad v, la polea tiene velocidad angular v/R y el cilindro tiene velocidad angular v/2R, la energa cintica total es 1 M ( 2 R) 2 MR2 K = Mv2 + (v 2 R ) 2 + (v R) 2 + Mv2 2 2 2

v y = v0 y + gt = 23,4 m/s y v x = v0 x = 15,26 m/s


2 2 v = vx + vy = 28,0 m s

3 Mv 2 . (1) 2

Esta energa cintica debe ser el trabajo hecho por la gravedad; si la masa que cuelga desciende una distancia y, K = Mgy. (2) De (1) y (2):

v2 =
b) En la base de la colina,

2 gy 3

Para aceleracin constante

v 2 = 2ay,
Por comparacin de las dos expresiones obtenemos:

v 25,0 m s = r r

La razn de la rotacin no cambia mientras la bola est en el aire, despus de dejar la parte alta del acantilado, tal que momentos antes de tocar tierra

a=

g 3

15,3 m/s = r
La energa cintica total es igual en la base de la colina y momentos antes de tocar tierra, pero momentos antes de tocar tierra poco de esta energa es energa cintica rotatoria, as que la energa cintica de traslacin es mayor. Ejemplo 37. Un cilindro slido uniforme de masa M y radio 2R descansa en una mesa horizontal. Se ata un hilo mediante un yugo a un eje sin friccin que pasa por el centro del cilindro de modo que ste puede girar sobre el eje. El hilo pasa por una polea con forma de disco de masa M y radio R montada 22

Ejemplo 38. Una barra de largo 2L y masa M est articulada en un extremo a un punto fijo O, inicialmente en reposo y horizontal. Si ella se suelta, comienza a rotar respecto a la articulacin bajo el efecto del peso de la barra. Determine la reaccin en la articulacin y la velocidad angular de la barra en funcin del ngulo que ella ha girado.

Solucin.

Cuerpo rgido

Hugo Medina Guzmn

Por conservacin de energa tenemos que


2 11 2 M (2 L ) Mgsen = 0 23

a) Momento de inercia de la barra con respecto a un extremo

1 I A = ML2 3
Por conservacin de energa.

Luego la velocidad angular de la barra es:

3g 3g sen = sen 2L 2L d2 L cos , Adems R H = M dt 2 d2 RV Mg = M 2 ( Lsen ) dt

L 11 2 2 = ML 2 23 3g = L L 1 3gL vCM = = 2 2 Mg
b) La aceleracin angular en dicho instante.

Entonces

R H = ML

d 1 sen 2 sen d

L 2 = 3g = = I A 1 ML2 2 L 3

Mg

= ML

1 d 2 3 g sen sen 2 sen d 2L

9 MLsen cos 4 d2 RV = Mg M 2 ( Lsen ) dt


= = Mg ML =

d 1 3g 2 sen cos 2 cos d 2L

Ejemplo 40. Una barra de longitud 2L y masa M se coloca sobre un plano horizontal liso. Si la barra es tirada por una fuerza constante F, inicialmente perpendicular a la barra y aplicada en un extremo, la barra comienza a moverse sobre el plano. La fuerza se mantiene aplicada a ese mismo extremo manteniendo su direccin original. Determine una ecuacin para el ngulo que gira la barra en funcin del tiempo.

5 9 Mg Mg cos 2 2 4
Solucin.

Ejemplo 39. Una barra de longitud L y masa M se coloca verticalmente sobre un plano horizontal liso, en reposo. Si ella es perturbada levemente comienza a caer. Determine: a) La velocidad del centro de masa de la barra justo cuando ella se coloca horizontal. b) La aceleracin angular en dicho instante.

El torque respecto al centro de masa conduce a

FLsen =
=

1 ML2 3

3F sen L

Solucin.

Ejemplo 41. Una barra de longitud L y masa M puede oscilar libremente en torno a uno de sus

23

Cuerpo rgido

Hugo Medina Guzmn

extremos que se mantiene fijo, bajo la accin de su peso. Escriba la ecuacin diferencial para el ngulo que ella gira. Solucin.

puramente energa cintica.

K=

1 2 11 1 I = MR 2 2 = MR 2 2 2 22 4

Por conservacin de energa.

2k 2 1 2 1 2 2 2 k = MR = 2 4 MR 2 2k 2 Finalmente = MR 2
TRASLACIONES Y ROTACIONES COMBINADAS Hasta ahora solo hemos tomado en consideracin la rotacin del cuerpo en torno a un eje fijo en el espacio. La finalidad de esta seccin es estudiar el caso en que el eje de rotacin si acelera tambin vamos a presentar tres mtodos analticos de resolver este caso. Primer mtodo Aplicamos la segunda ley de Newton para traslacin relativa ejes no rotantes a travs del centro de masa. Para ilustrar este mtodo y los otros tambin, consideremos un cuerpo de radio R, masa M y momento de inercia respecto a su entro masa I, al que se le obliga a rodar sin deslizamiento a lo largo de una superficie horizontal por medio de una fuerza F que acta en su centro de masa, La tuerza de friccin F f y la reaccin N actan tal como se

Por conservacin de energa

E=

2 11 L ML2 Mg cos 23 2

Derivando respecto al tiempo


L 1 ML2 + Mg sen = 0 3 2

Finalmente

3g sen = 0 2L

Ejemplo 42. Un pndulo de torsin consiste en un disco uniforme de masa M y radio R suspendido de una barra delgada y vertical de masa despreciable y que puede torcerse al dar vuelta al disco alrededor de su eje, como se indica en la figura. La barra tiene una Constante de elasticidad torsional k. inicialmente se hace girar el disco un ngulo respecto del equilibrio y luego se le suelta desde el reposo. Determinar su velocidad de rotacin cuando llega nuevamente a la posicin de equilibrio.

muestra en la figura siguiente.

Solucin. Con la ley de Hooke para rotacin,

EL cuerpo se mueve con una aceleracin horizontal a que es la que corresponde a su centro de masa, y a su vez rota con aceleracin angular . Como rueda sin deslizamiento la relacin entre el desplazamiento lineal y el desplazamiento angular es x = R . La velocidad es es:

= k

El trabajo para torcer un ngulo



0 0

W = d = ( k )d =

1 2 k 2

dx d =R v = R dt dt dv d =R a = R dt dt

La aceleracin es

Este trabajo queda como energa potencial.

U ( )

1 = k 2 2

Al liberarse esta se convierte en energa cintica. Al pasar por el punto de equilibrio la energa es 24

Aplicando la segunda ley de Newton para traslacin

Cuerpo rgido

Hugo Medina Guzmn

F F f = Ma
Aplicando la segunda ley de Newton para rotacin alrededor del centro de masa

En este caso como la aceleracin del centro masa es a, la aceleracin angular del cuerpo alrededor de O es = a R . Aplicando la segunda ley de Newton para traslacin:

RF f = I CM

Eliminando F f y

, obtenemos:

F F f = Ma
Aplicando la segunda ley de Newton para rotacin a alrededor de O:

I M + CM a = F R2
La aceleracin

FR = I O

a=

F I M + CM R2

Como

=a R
+ MR 2

y I O + I CM + MR :
2

con la segunda ecuacin,

(I

CM

a )R = FR

a=

Si para t = 0: x0 = 0 , v0 = 0 , Siendo a = constante La velocidad es:

F I M + CM R2

Tercer mtodo Este mtodo Consiste en usar las ecuaciones de la energa directamente.

v = v 0 + at

Es un Sistema Conservativo

F v= t M + I CM R 2

K + U = Constante

El desplazamiento es:

x = x0 + v0 t + x=

1 2 at 2

1 F t2 2 M + I CM R 2

Resolveremos por este mtodo el ejemplo anterior. Puesto que no hay deslizamiento la tuerza de friccin sobre el cuerpo no trabaja sobre el mientras rueda. Siendo un sistema conservativo la fuerza F se puede deducir de una funcin Potencial U = - Fx donde x es la coordenada horizontal del centro de nasa. La energa E del cuerpo es:

Segundo mtodo En este mtodo escribimos la ecuacin para traslacin igual que en el anterior mtodo, pero para la rotacin se aplica la segunda ley de Newton con respecto al eje de rotacin que pasa a travs del punto de reposo instantneo (punto de apoyo en el movimiento) si tal punto no existe no puede usarse este mtodo Como ilustracin veamos el ejemplo anterior. El punto contacto es el punto fijo instantneo O, consideremos que este no desliza y todos los otros puntos de eje momentneamente rotan alrededor de el.

E = K +U 1 1 K = I CM 2 + Mv 2 , U = Fx 2 2 1 1 2 2 Luego: E = I CM + Mv Fx 2 2 v Siendo = R 1 I E = v 2 CM + M Fx 2 2 R

De aqu podemos evaluar la velocidad considerando que para el instante inicial x = 0, y v = 0, por consiguiente E = 0.

1 2 I CM v 2 + M Fx = 0 y 2 R 1 2 I CM v 2 + M = Fx 2 R
v= 2 Fx I CM +M R2

25

Cuerpo rgido

Hugo Medina Guzmn

Siendo un movimiento con aceleracin constante

v = 2ax
De esto

Eliminando

a=

F I CM +M R2

y F f obtenemos: Mgsen a= M + I CM R 2

Considerando que para t = 0: s = 0, y v = 0.

Otra forma de calcular la aceleracin. Considerando que

E = Constante

dE =0 dt

Mgsen t , v= 2 M +I CM R 1 Mgsen 2 t s= 2 2 M I R + CM
Para un anillo:

dE 1 2 I CM = v 2 + M Fx = 0 dt 2 R

I CM = MR 2 , s =
Para un disco:

1 gsen t 2 4

dv I dx v CM =0 +MF 2 dt R dt dv dx Como =a y =v dt dt I va CM + M Fv = 0 2 R F a= I M + CM R2
Ejemplo 43. Analizar el movimiento de un cuerpo de radio R, momento de inercia respecto a su centro de masa I que rueda sin deslizar hacia abajo en plano inclinado de ngulo .

I CM = I CM = s=

1 1 MR 2 , s = gsen t 2 3 2 2 5 MR 2 , s = gsen t 2 5 14

Para una esfera:

Para un plano sin friccin (sin rodadura)

1 gsen t 2 2

Por la ecuacin de energa Si para t = 0: K 0 = 0 y U 0 = 0

E = K0 + U0 = 0
Llamando h a la cada del centro de masa desde la posicin de reposo, tenemos:

1 1 Mv 2 + I CM 2 , 2 2 U = Mgh = Mgs sen = 0 , =v R I 1 2 v M + CM Mgs sen 2 R2

K=

Solucin. Como se muestra en la figura hay dos fuerzas que actan sobre el cuerpo, Mg acta en el centro de gravedad y la fuerza de contacto que se descompone en la reaccin normal N y la fuerza de friccin Ff. Vamos a resolver por el primer mtodo. Traslacin:

v=

2 Mgsen s M + I CM R 2

Ejemplo 44. Usar la conservacin de la energa para describir el movimiento de rodadura de un cuerpo rgido de masa M que rueda por un plano inclinado y rugoso.

Mgsen F f = Ma RF f = I CM

Rotacin:

Por la condicin de no deslizamiento:

=a R

26

Cuerpo rgido

Hugo Medina Guzmn

Estudiar el movimiento.

Solucin. Se supone que el cuerpo rgido parte del reposo desde una altura h y que rueda por el plano sin resbalar la conservacin de energa da:

E = cte K + U g = cte

K i + U gi = K f + U gf
Pero Ki = 0 y Ugf = 0, entonces

Solucin. Vamos a resolver primero por las ecuaciones del movimiento de Newton. Traslacin.:

Mg T = Ma RT = I CM I CM =

Rotacin.: Como:

1 1 2 Mgh = I cm 2 + Mvcm 2 2
Como vcm= R = vcm/R, se reemplaza en la ecuacin anterior

a 1 MR 2 , = : 2 R

v2 1 1 2 + Mvcm = Mgh I cm cm 2 2 2 R
Despejando cm se obtiene:

1 a 1 RT = MR 2 = MRa 2 R 2
De aqu se obtenemos:

vcm =

2 gh I + I cm MR 2

T=
2 MR 2 , se puede 5

1 2 Ma y a = g 2 3

Por ejemplo, para una esfera slida uniforme de momento de inercia I cm =

El yo-yo funciona segn este principio, est proyectado para que a sea mucho menor que g. Resolviendo por conservacin de la energa E = K +U =

calcular su vcm en el punto ms bajo del plano y su aceleracin lineal.

2 gh 2 gh 10 = = gh 2 2 7 ( 2 5)MR 1+ 1+ 5 MR 2 10 vcm = gh 7
2 = vcm

1 11 v Mv 2 + MR 2 Mgy 2 22 R
Como E = constante

La aceleracin lineal se puede calcular con la ecuacin

dE =0 dt dy dv Tambin v = y a= dt dt a=
2 g 3

Con esto encontramos que

2 cm

=v

2 cmi

+ 2a cm x a cm

De la geometra de la figura, se tiene: h = x sen , donde x es la longitud del plano, reemplazando en acm:

v2 = cm 2x

a cm

5 gxsen 5 7 = = gsen 2x 7

Ejemplo 46. Estudiar el movimiento de un disco homogneo de radio R y masa M, sobre el que acta una fuerza horizontal F aplicada un punto variable a lo largo de una lnea vertical que pasa por el centro, segn se indica en la figura. Supngase el movimiento sobre un plano horizontal.

Ejemplo 45. Un disco homogneo de radio R y masa M tiene una cuerda enrollada alrededor, segn vemos en a figura. Sujetando el extremo libre de la cuerda a un soporte fijo, se deja caer el disco.

27

Cuerpo rgido

Hugo Medina Guzmn

(0) F 3F f = F 1 2 = F F f = 3 R
En la ecuacin (3)

Solucin. En la figura vemos que la fuerza F se aplica a una distancia h sobre el centro. Suponiendo que F f acta hacia la izquierda.

1 F F (0 ) + R = I CM = MR 2 2 3 1 F R = MR 2 3 2 2F = 3MR
El cilindro rueda hacia la derecha. d) Si F se hace muy grande tal que Ff tiende a aumentar, tan pronto como sobrepase el valor mximo posible de la tuerza de rozamiento (N), el disco deslizar. Se debe hacer una nueva hiptesis, esta vez se tienen tambin las ecuaciones (1), (2) y (3) pero a R.
Ejemplo 47. Un carrete de radio interior R1 y radio exterior R2 se halla sobre un suelo spero. Se tira de l con una tuerza F mediante un hilo arrollado en torno a su cilindro interior. Se mantiene un ngulo con la horizontal. Se observa que hay un ngulo Crtico 0 , tal que < o , el carrete rueda sin

Aplicando las leyes de Newton del movimiento: Traslacin

(2) Rotacin alrededor del centro de masa

F F f = Ma N Mg = 0

(1)

Fh + F f R = I CM =
Considerando

1 MR 2 (3) 2

a R
(3a)

2F

h + 2 F f = Ma R

Igualando (1) y (3a)

h + 2F f R h 3 F f = F 1 2 R F F f = 2F
Discusin: a) F f = 0 , cuando 1 2

deslizar en el sentido del cual se tira de l, y para > o el carrete rueda sin deslizar en sentido contrario, Cul es el valor del ngulo critico.

R h =0 h= R 2
Solucin. Aplicando las leyes de Newton del movimiento; Traslacin:

Esto quiere decir si F se aplica a R/2 del centro, la fuerza de rozamiento es cero. b) Si h = R

F R 3F f = F 1 2 = F F f = R 3
el rozamiento es en sentido contrario al indicado y la ecuacin (3) se convierte en:

F cos F f = Ma = MR2 (1) Fsen Mg + N = 0


F f R2 + FR1 = I CM

1 F F (R ) R = MR 2 2 3 2 1 F = MR 3 2 4F = 3MR
Esto indica que el cilindro rueda hacia la derecha. c) Si disminuye h hasta que h = 0. 28

Rotacin:

F f R2 = FR1 I CM
Ff = F I R1 R1 CM (2) R2 R2 Eliminando la fuerza F f ., reemplazando (2) en

(1):

Cuerpo rgido

Hugo Medina Guzmn

R1 I CM F cos F R R1 R = Ma 2 2 I R F cos F 1 R1 + CM = MR2 R2 R2

Ff =

1 Ma 2

Que sustituida en la primera da:

2 R2 cos R1 MR2 I CM = R2 R2

=F

(R2 cos R1 )

(MR

2 2

I CM

d dt

2F , 3M 2F a b) = , = R 3MR F 1 c) F f = Ma = 2 3
a) a =
Ejemplo 49. Un disco de masa M y radio 2R se apoya sobre un plano horizontal spero de modo que puede rodar sin resbalar con su plano vertical. El disco tiene un resalto de radio R como se indica en la figura, en el cual se enrolla una cuerda que se tira con una fuerza horizontal constante F, determine: a) La aceleracin del centro de masa del disco. b) La aceleracin angular del disco. c) La fuerza de roce.

La rotacin har que el movimiento del carrete ser hacia adelante cuando

d >0 dt R2 cos R1 > 0 R cos > 1 R2

El movimiento ser hacia atrs cuando

R2 cos R1 < 0 R cos < 1 R2


El ngulo crtico es cuando

d <0 dt

R2 cos R1 = 0 R cos = 1 R2

d =0 dt

Solucin.

Ejemplo 48. Un disco de masa M y radio R se apoya sobre un plano horizontal spero de modo que puede rodar sin resbalar con su plano vertical. Si se tira del centro del disco con una fuerza horizontal constante F, determine: a) La aceleracin del centro de masa del disco. b) La aceleracin angular del disco. c) La fuerza de roce. Solucin.

Ahora F F f = Ma , N Mg = 0

F f 2 R + FR =

1 2 M (2 R ) 2 2 a = 2 MR = MRa 2R

Simplificando:

2 F f + F = Ma = F F f Ff = 0
De donde resulta: a) a = Aqu b)

F m

F F f = Ma , N Mg = 0 ,
1 1 MR 2 = MRa 2 2
29

Ff R =

F 2MR c) F f = 0

Entonces

Cuerpo rgido
Ejemplo 50. Un disco de masa M y radio R tiene enrollada una cuerda en su periferia y cae partiendo del reposo mientras la cuerda que se sostiene de su extremo se desenrolla. Determine: a) La aceleracin de bajada del disco. b) La tensin de la cuerda.

Hugo Medina Guzmn

A partir del punto A en que el piso es spero deslizar primeramente sobre el plano spero, pero acabar rodando sin deslizar. En la parte intermedia habr una aceleracin a que disminuye a la velocidad de v1 a v 2 y una aceleracin angular que disminuye a 1 , la hace igual a cero y cambia su rotacin hasta que llega la velocidad angular a un valor tal que 2 = v2 R . Aplicando las leyes de Newton en la figura siguiente.

Solucin.

Traslacin:

N = Ma , N Mg = 0

Aqu Mg T = Ma ,

1 1 MR 2 = MRa 2 2 1 De donde Mg Ma = Ma 2 2 a) a = g 3 1 1 b) T = Ma = Mg 2 3

TR =

1 MR 2 2 2 g De esto obtenemos: a = g , = R La velocidad es: v = v1 + at = v1 gt


Rotacin: RN = I CM = La velocidad angular es:

= 1 t =

v1 2 g t R R

Parta encontrar el tiempo en que el disco deja de resbalar, debe cumplirse: v = R

R = k vi j = Ri

Ejemplo 51. Se da a un cilindro homogneo de radio R y masa M con una velocidad horizontal v1

y una velocidad angular

las agujas del reloj 1 = v1 R en la parte sin rozamiento de la superficie horizontal. Ms all del punto A, cambia la superficie de manera que a la derecha de A el coeficiente de rozamiento es .

en sentido opuesto a

R 2 v1 2v1 = 3gt t = 3 g con este valor de t 2 v1 v1 v 2 = v1 g 3 g = 3

v 2 g (v1 gt ) = t R 1 R

La velocidad final es un tercio de la inicial

Solucin. En la parte lisa el cuerpo se mueve con velocidad horizontal constante v1 hacia la derecha, rotando

con velocidad angular

en el sentido antihorario. 30

Ejemplo 52. Se lanza una bola de billar con una velocidad inicial v 0 sobre una mesa horizontal,

Cuerpo rgido

Hugo Medina Guzmn

existiendo entre la bola y la mesa un coeficiente de rozamiento . Calcular la distancia que recorrer hasta que empiece a rodar sin deslizamiento. Qu velocidad tendr en ese instante? Aplicar para el caso v 0 = 7 m/s, = 0,2.

v0 gt = v=

2v 5gt t= 0 2 7 g

la velocidad en ese instante es

5 v0 = 5 m/s, t = 1,02 s 7 1 gt 2 2
2

La distancia recorrida

x = v0 t
Solucin. La fuerza de rozamiento N = mg se opone al movimiento, siendo adems la fuerza resultante, por lo que: mg = ma , a = g La velocidad de la bola comenzar a disminuir de tal modo que: v = v 0 at = v 0 gt .

2 2 2v0 12v 0 1 2v0 g = = 7 g 2 49g 7 g

= 6,12 m.
Ejemplo 53. Un tambor tiene un radio de 0,40 m y un momento de la inercia de 5,0 kg m2. El torque producido por la fuerza de friccin de los cojinetes de anillo del tambor es 3,0 Nm. Un anillo en un extremo de una cuerda se desliza en una clavija corta en el borde del tambor, y una cuerda de 15 m de longitud se enrolla sobre el tambor. El tambor est inicialmente en reposo. Una fuerza constante se aplica al extremo libre de la cuerda hasta que la cuerda se desenrolla y se desliza totalmente de la clavija. En ese instante, la velocidad angular del tambor es de 12 rad/s. El tambor despus decelera y se detiene.

Al mismo tiempo, sobre la bola que inicialmente no rueda, (0 = 0) acta un momento de fuerza:

= F f R = mgR
que producir una aceleracin angular

mgR
2 mR 2 5

5g 2R

Por lo que la velocidad angular ir aumentando:

= t =

5gt 2R

La velocidad de un punto de la periferia de la esfera vale v P = R , que ir aumentando con el tiempo, porque aumenta con el tiempo.

a) Cul es la fuerza constante aplicada a la cuerda? b) Cul es la cantidad de movimiento angular del tambor en el instante en que la cuerda deja el tambor? c) Cul es el trabajo negativo realizado por la friccin? d) Qu tiempo el tambor estuvo en movimiento? Movimiento con la cuerda? Solucin. a) Trabajo de la fuerza F + trabajo de la friccin = Energa cintica ganada al terminarse la cuerda

Por tanto, observamos que la velocidad de la bola disminuye, y la velocidad de la periferia de la bola aumenta. En el momento en que la velocidad de la periferia se iguale a la velocidad de traslacin, se conseguir la rodadura, es decir el no deslizamiento.

1 I O 2 2 15 1 2 F (15) 3,0 = (5,0 )(12) 0,4 2 F = 31,5 N

Fs + f =

b)

L = I O = (5)(12)
= 60 kg.m2/s

v = v P v = R

c) Movimiento con la cuerda

31

Cuerpo rgido

Hugo Medina Guzmn

15 W f 1 = f = 3 = - 112,5 J 0,4
Movimiento sin la cuerda

1 1 2 2 W f 2 = I OO = (5,0)(12) = 360 2 2
Trabajo total

m s2 mg T = ma T 20 m= = g a 9,8 0,75
= 0,75 = 2,21 kg b)

W f = W f 1 + W f 2 = 482,5 J
d)

FR f I O 31,5(0,4 ) 3,0 = 5,0


31,5(0,4) 3,0 rad = 1,92 5,0 s

= I O

1 =

a 0,75 = R 0,4 rad = 1,875 2 s O = I O TR = I O


20(0,4 ) 1,875 2 = 4,27 kg m

Por otra parte

o = 1t1 t1 =

0 12 = = 6,25 s 1 1,92

IO =

TR

Movimiento sin la cuerda

= I O 3 = 5 2

rad 3 2 == = 0,6 s 5 0 = 0 + 2t 2 0 12 t2 = = = 20s 0,6 2


El tiempo total es 26,25 s
Ejemplo 54. Una rueda tiene un radio de 0,40 m y se monta en cojinetes sin friccin. Un bloque se suspende de una cuerda que se enrolla en la rueda. La rueda se libera de reposo y el bloque desciende 1,5 m en 2,00 segundos. La tensin en la cuerda durante el descenso del bloque es 20 N. a) Cul es la masa del bloque? b) Cul es el momento de inercia de la rueda?

Ejemplo 55. El radio de una rueda de 3,0 kilogramos es 6,0 centmetros. La rueda se suelta del reposo en el punto A en un plano inclinado 30. La rueda gira sin deslizar y se mueve 2,4 m al punto B en 1,20s. a) Cul es el momento de inercia de la rueda? b) Cul es la aceleracin angular de la rueda?

Solucin.

a) I O = b)

1 1 2 mR 2 = (3kg )(0,06m ) 2 2

= 0,0054 kg m2

mgsen30 Ff = ma F f R = I O

Solucin.

a)

1 2 at 2 2h 2(1,5) a= 2 = t (2)2

h=

I F f = O R I mgsen30 O = mR R mgsen30 3(9,8)(0,5) = = 0,0054 IO + 3(0,06) + mR 0,06 R rad 14,7 = 54,4 2 = s 0,27
Ejemplo 56. Una masa de 20 kg se halla sobre un plano inclinado 30, con el que tiene un rozamiento cuyo coeficiente vale 0,3, unida a una cuerda sin masa e inextensible que pasa por una polea de MP = 160 kg, cuyo radio geomtrico es de 20 cm y radio

32

Cuerpo rgido de giro rg = 15 cm. De dicha cuerda pende una masa de 40 kg que es abandonada libremente. Calcular: a) Aceleracin con que se mueve el sistema. b) Tensiones en la cuerda. c) En qu rango de valores de la masa que pende, el sistema estar en equilibrio? Momento de inercia de la polea I P = Mrg .
2

Hugo Medina Guzmn

= 10 + 5,2 = 15,2 kg.

Solucin. a) Partiendo de la suposicin de que la masa colgante acelera hacia abajo, plantearemos las tres ecuaciones correspondientes al movimiento de las tres masas: m2g - T2 = m2a

Si la masa m2 se hace an menor, llegar un momento en que ser arrastrada por m1. Esto producira una inversin en el sentido de la fuerza de rozamiento. El valor mximo de m2 deber cumplir ahora:

m2 = m1sen + m1 cos

T1 m1gsen + m1 g cos = m1 a , a T2 R T1 R = I = M P rg2 R


Sumando las tres ecuaciones siguientes m0 g T2 = m2 a ,

= 10 5,2 = 4,8 kg. Por tanto, entre 0 y 4,8 kg el sistema acelerar de modo que m2 suba; entre 4,8 y 15,2 kg, permanecer en equilibrio; y para ms de 15,2 kg m2 acelerar hacia abajo.
Ejemplo 57. Porqu una esfera que rueda se detiene? En esta parte vamos a tratar de explicar la resistencia al rodamiento. La figura siguiente muestra una esfera de masa M y radio R la cual est rodando con una velocidad angular y avanza con una velocidad v = R .

T1 m1gsen + m1 g cos = m1 a

rg T2 T1 M P R a
Obtenemos:

m2 g m1gsen + m1 g cos
2 rg = a m1 + m2 + M P R

a=

m2 m1sen + m1 cos

rg m1 + m2 + M P R 40 10 5,2
=

15 60 + 160 20

Solucin. La fuerzas que actan sobre la esfera son el peso Mg 1a reaccin del piso N y la fuerza de friccin F f . Si aplicamos la segunda ley de Newton a la

traslacin.

Ff = M g

= 1,62 m/s2 b)

T2 = m2 ( g a ) = 327 N,
2

debe haber una aceleracin a y v decrecera. Si aplicamos segunda ley de Newton a la rotacin. la aceleracin angular depende de Ff. por consiguiente Ff acta incrementando . En resumen: en traslacin Ff. acelera, en rotacin Ff. desacelera, esto aparentemente es una contradiccin. Por otra parte Mg y N estn en la lnea vertical que por el centro de masa y no causan efecto en el 33

rg T1 = T2 a = 181 N. R
c) El valor mnimo que hace que la masa m2 acelere hacia abajo se produce cuando a = 0, es decir:

RF f = I CM

m2 = m1sen + m1 cos

Cuerpo rgido

Hugo Medina Guzmn

movimiento horizontal. Si la esfera y el plano son rgidos, de modo que la esfera est en contacto solo en un punto, tampoco originan alrededor del centro de masa. .porque actan a travs de l Para resolver la Contradiccin suprimamos la idealizacin de que todos los cuerpos son rgidos, la esfera se aplana un poco y el nivel de La superficie se hunde Ligeramente (ver la figura a continuacin)

Sobre la varilla acta el peso Mg y la reaccin R. La velocidad angular en este instante se puede encontrar aplicando la ecuacin de la energa.

La reaccin N acta delante del centro de masa, produciendo un torque N = dN de resistencia al rodamiento.

N RF f = I CM
N = Mg , F f = Ma , =
a R a : R

Como

N RMa = I CM

I N = a CM + RM R 2 2 Para una esfera: I CM = MR 5 7 Luego: N = MRa , como N = Mg 5 N 7R d= a = Mg 5 g


Ejemplo 58. La figura muestra una varilla homognea de masa M y longitud L en posicin vertical. La cual se deja caer desde el reposo. a) A que ngulo entre la varilla y la vertical, la varilla ya no presionar al piso? b) Con qu coeficiente de friccin el extremo de La varilla no resbalar hasta este momento?

L L 1 = Mg cos + I O 2 2 2 2 1 2 Como I O = ML 3 L L 11 Mg = Mg cos + ML2 2 2 2 23 3g (1 cos ) 2 = L 6g 3g 2 sen = = 2sen 2 L L 2 6g = sen 2 L 2 Mg


Aplicando la segunda Ley de Newton para traslacin a lo largo de la varilla.

F = ma

R Mg cos = M 2 L 2

L 2

Cuando La varilla deja de presionar R = 0, y:

Mg cos = M 2
reemplazando el valor de

encontrado

L 6g sen 2 Mg cos = M 22 L
Simplificando

cos = 6 sen 2 tan =

cos 2

= 5 sen 2

5 5 De aqu: = 48,2
b) Para que la varilla no resbale tenemos en la figura siguiente. Las componentes de R son:

+ R cos R = Rsen i j
Solucin. a) La figura siguiente muestra .la varilla cuando forma un ngulo con la vertical.

34

Cuerpo rgido

Hugo Medina Guzmn

La condicin para que la varilla no resbale es:

F f Rsen

Con F f =

N y N = R cos R cos Rsen tan . = 48,2 1,12

Por la conservacin de la cantidad de movimiento angular.

El coeficiente de rozamiento del piso debe ser cuando menos igual a tan para que llegue sin deslizar hasta el ngulo Para

I1 1 I2 Siendo I 2 < I 1 , resulta 2 > 1 I 2 2 = I 11 2 =


Su velocidad aumenta.
Ejemplo 60. Esta vez el mismo estudiante sentado sobre el mismo banco, sostiene en sus manos en posicin vertical al eje de rotacin de una rueda de bicicleta, la rueda gira alrededor de ese eje vertical con velocidad angular 0 , el estudiante y el banco

CONSERVACION DE LA CANTIDAD DE MOVIMIENTO ANGULAR. Anteriormente hemos visto que:

F=

dp dL y tambin = dt dt

y mostramos que para un cuerpo rgido.

estn en reposo (a). El estudiante gira el eje de la rueda en ngulo con la vertical (b), como no hay torque respecto al eje vertical, la cantidad de movimiento angular con respecto al eje vertical debe conservarse.

ext

d L total = dt

Si no hay torque externo con respecto a algn eje la cantidad de movimiento angular ser constante con respecto a ese eje.

L total = Constante I = Constante

o expresado en funcin del momento de inercia apropiado. Esta relacin nos va a ser muy til como veremos a continuacin.
Ejemplo 59. Un estudiante est sentado sobre un banco giratorio montado sobre cojinetes sin friccin que puede girar libremente alrededor de un eje vertical como se muestra en la figura (a). El estudiante sostiene en las manos extendidas dos pesas. Su momento de inercia en esta posicin es I1 y su velocidad angular 1 . No actan sobre l torques no equilibrados y en consecuencia su cantidad de movimiento angular tiene que conservarse. Cuando el estudiante acerca las manos al cuerpo, su momento de inercia vara, figura ( b) ahora es I2 y su velocidad angular ser 2

Inicialmente se tiene

L = I 0 0 k

Cuando se inclina la rueda (respecto al eje vertical)

L' = L estudiante +banco + L rueda


= I e e + I 0 0 cos k
Siendo I e el momento de inercia del estudiante y banco respecto al eje vertical, angular con respecto a ese eje. Como L = L'

su velocidad

=I k I e e + I 0 0 cos k 0 0
35

Cuerpo rgido

Hugo Medina Guzmn de 2 rad/s2 durante 6 segundos, momento en el cual encoge los brazos y acerca sus piernas al eje hasta tener un momento de inercia de 4 kg.m2 . Determinar su velocidad de giro final.

I e = 0 0 (1 cos ) k Ie

Es la velocidad angular del estudiante con el sentido de giro inicial de la rueda. Cuando la rueda se invierte se invierte totalmente = 2 , y:

e =

2I 0 0 k Ie

Ejemplo 61. Una persona est sentada en una silla giratoria manteniendo los brazos extendidos con una pesa en cada mano. Gira con una frecuencia de 2 Hz. El momento de inercia de la persona con los pesos es de 5 kg m2. Hallar: a) la nueva frecuencia cuando encoja los brazos y disminuya el momento de inercia a 2 kg m2. b) La variacin de energa cintica del sistema. c) De dnde procede este incremento de energa cintica?

Solucin.

Despus de un tiempo t de iniciar el giro, su velocidad angular ser:

(t ) = at 2 =

1 2

1 (2)(6)2 = 36 rad/s 2

al acercar brazos y piernas al eje, el torque de las fuerzas sigue siendo nulo, por lo que se conserva la cantidad de movimiento angular, I

(I )Antes = (I )Despus
Solucin. a) Al encoger los brazos, estn actuando fuerzas y torques de fuerzas internas, por lo que podemos admitir que se conserva la cantidad de movimiento angular.

Despus =

I Antes Antes I Despus

L1 = L2 I 11 = I 2 2 I I 2 = 1 2 , 2 f 2 = 1 2 f1 , I2 I2 I 5 f 2 = 1 f1 = 2 = 5 Hz I2 2

7 36 = 63 rad/s 4

b) K =

L = I 11 = 5(2 2) = 20 kg m 2 s ; 1 1 K = 200 2 = 60 2 J . 2 5

1 1 L2 L2 2 I 2 2 I 112 = 2 2 2I 2 2I1

Ejemplo 63. Un muchacho de 25 kg corre con velocidad de 2,5 m/s hacia un tiovivo en reposo de radio 2 m cuyo, momento de inercia vale 500 kg m2. Hallar la velocidad angular y frecuencia del conjunto despus de que el muchacho suba al tiovivo justo en el borde.

El signo positivo nos indica que hay un aumento de energa cintica. c) Este incremento de energa cintica procede de la energa qumica almacenada en los msculos del brazo.
Ejemplo 62. Un patinador, con los brazos extendidos y las piernas abiertas y con un momento de inercia respecto a su eje vertical de 7 kg.m2 , inicia un giro sobre si mismo con una aceleracin

Solucin. La cantidad de movimiento angular del muchacho respecto al centro del tiovivo es: L1 = mvR = (25)(2,5)(2 ) = 125 kg m2/s

36

Cuerpo rgido

Hugo Medina Guzmn

El momento de inercia del conjunto tiovivomuchacho es I = Im + IT = 25x22 + 500 = 600 kg m2 Planteando la igualdad entre la cantidad de movimiento angular inicial y final, tendremos: L1 = L2 , mvR = (I m + I T )

' =

2,0 = 2,256 1,5 rad = 1,33 s

IO I 'O

Reemplazando: Energa despus =

mvR 125 = = (I m + I T ) 600


= 0,208 rad/s

1 1 2 I 'O ' 2 = (2,256)(1,33) 2 2 2,05 = 0,5 4,05

f =

=2J Se pierde 4,05 -2 = 2,05

= 0,033 Hz

fraccin de energa =

= 1,99 r.p.m.
Ejemplo 64. Una tornamesa con radio de 8,0 m y momento de inercia de 2,0 kg.m2. La placa tornamesa rota con una velocidad angular de 1,5 rad/s sobre un eje vertical que pasa a travs de su centro en cojinetes sin friccin. Una bola de 0,40 kg se lanza horizontalmente hacia el eje de la tornamesa con una velocidad de 3,0 m/s. La bola es cogida por un mecanismo con forma de tazn en el borde de la tornamesa. a) Cul es cantidad de movimiento angular de la bola alrededor del eje de la tornamesa? b) Qu fraccin de energa cintica se pierde durante la captura de la bola? Ejemplo 65. Una barra rgida de masa M y largo L gira en un plano vertical alrededor de un eje sin friccin que pasa por su centro. En los extremos de la barra se unen dos cuerpos de masas m1 y m2. Calcular la magnitud del momento angular del sistema cuando su rapidez angular es y la aceleracin angular cuando la barra forma un ngulo con la horizontal.

Solucin. El momento de inercia por el eje de rotacin del sistema es igual a la suma de los momentos de inercia de los tres componentes, con los valores de la tabla se obtiene:

Solucin. a) La cantidad de movimiento angular de la bola alrededor del eje de la tornamesa es cero b)

Como el sistema gira con rapidez angular , la magnitud del momento angular es:

1 L I = ML2 + m1 12 2 L2 M = m1 + m2 + 4 3
L2 4

L + m2 2

Energa antes =

1 2 1 mv + I O 2 2 2 1 1 2 2 = (0,4 )(3,0 ) + (2,0 )(1,5) 2 2


= 4,05 J

L = I =

M m1 + m2 + 3

Para calcular la aceleracin angular usamos la relacin

1 2 Energa despus = I ' O ' 2


Para calcular esta energa necesitamos conocer I0 y .

t = I = t = m1 g
=

t
I

, al calcular el torque total

en torno el eje de rotacin, se obtiene:

I ' O = I O + mR 2 = 2,0 + (0,4)(0,8)


= 2,256 kg/ m
2

L L cos m2 g cos 2 2

Lantes = Ldespus I O = I 'O '


37

1 (m1 m2 )gL cos 2

Cuerpo rgido Reemplazando en los valores de I y de t , se obtiene la aceleracin angular:

Hugo Medina Guzmn

t
I

2(m1 m2 )g cos L(m1 + m2 + M 3)

a) El momento de inercia ser la suma del momento de inercia de una varilla, ms el de la esfera. Como la esfera est a L+R del eje, aplicamos Steiner:

Ejemplo 66. En la figura las masas m1 y m2 se conectan por una cuerda ideal que pasa por una polea de radio R y momento de inercia I alrededor de su eje. La mesa no tiene roce, calcular la aceleracin del sistema. Solucin. Primero se calcula en momento angular del sistema de las dos masas ms la polea:

2 1 2 me R 2 + me (L + R ) , I V = mV L2 5 3 I = I e + IV 2 1 2 2 2 = me R + me (L + R ) + mV L 5 3 2 1 2 2 2 I = (0,25)(0,1) + (0,25 )(0,85) + (0,5)(0,75) Ie =


5 3

= 0,27 kg.m2 b) L = I = 0,27 = 0,54

2 = 0,27( 2f ) T

12 = 0,345 kgm2 / s 60

v L = m1vR + m2 vR + I R
Luego se calcula el torque externo sobre el sistema, la nica fuerza externa que contribuye al torque total es m1g, entonces el torque es = m1 gR . Entonces se tiene:

Ejemplo 68. Un cilindro de 50 kg y 20 cm de radio, gira respecto de un eje vertical que coincide con su eje de simetra, debido a una fuerza constante, aplicada a su periferia que, despus de 40 s de iniciado el movimiento, alcanza 200 r.p.m. Calcular: El valor de la fuerza y el torque de la fuerza aplicada.

dL dt d v m1 gR = (m1 + m2 )vR + I dt R dv I dv m1 gR = (m1 + m2 )R + dt R dt I m1 gR = m1 + m2 + 2 Ra R m1 g a= I m1 + m2 + 2 R

Solucin. La frecuencia de rotacin adquirida vale:

f =

200 Hz 60 200 20 rad = 60 3 s

La velocidad angular:

= 2f = 2 =

Ejemplo 67. Una varilla de 500 g y 75 cm de longitud, lleva soldada en un extremo una esfera de 10 cm de radio y 250 g de masa. Calcular: a) El momento de inercia cuando gira, alrededor de un eje perpendicular a la varilla que pasa por el extremo libre. b) La cantidad de movimiento angular del conjunto si gira a 12 rpm.

La aceleracin angular:

rad = t 6 s2

Por otra parte el momento de inercia del cilindro vale:

I=

1 1 2 mR 2 = (50)(0,2 ) = 1 kgm2. 2 2

Luego el torque de la fuerza aplicada

= FR = I = (1)
La fuerza tangencial:
Solucin.

= 0,52 Nm.

F=

0,52 = 2,6 N 0,2

38

Cuerpo rgido
Ejemplo 69. Un anillo de masa M y radio R (ICM = MR2), cae en rodadura pura sobre un plano inclinado que forma un ngulo con la horizontal. a) Hacer el DCL. del anillo. b) Hallar la aceleracin del centro de masa del anillo. c) Encontrar el valor de la friccin entre el plano inclinado y el anillo. d) Cul debe ser el mnimo valor del coeficiente de rozamiento esttico entre el plano y el anillo para que este se encuentre en rodadura pura?

Hugo Medina Guzmn

longitud l I CM =

1 Ml 2 se sostiene de un 12

extremo mediante un pivote sin friccin. La barra se encuentra inicialmente en reposo en forma vertical cuando un proyectil de masa m impacta sobre ella y queda incrustado instantneamente. La velocidad inicial del proyectil es v 0 . Hallar: a) La cantidad de movimiento angular del sistema respecto del pivote justo antes de la colisin. b) La velocidad angular de giro del sistema despus que el proyectil se incrusta en la barra. c) La altura mxima que alcanzar el CM de la barra. d) El trabajo del proyectil cuando se incrusta contra la barra.

Solucin. a) El DCL. del anillo.

b) Segunda ley de Newton para la traslacin

Solucin. a) La cantidad de movimiento angular del sistema respecto del pivote justo antes de la colisin.

Mgsen F f = Ma I = F f R MR 2 F f = Ma

Lantes = mv0 d
b) La velocidad angular de giro del sistema despus que el proyectil se incrusta en la barra.

Segunda ley de Newton para la rotacin

a = Ff R R

Reemplazando el valor de Ff en la primera ecuacin.

Mgsen Ma = Ma Mgsen = 2Ma 1 Finalmente a = gsen 2 1 Mgsen 2 Lantes = Ldespus mv0 d = = 1 Ml 2 + (d )d 3 mv0 d 1 2 2 Ml + md 3

c) El valor de la fuerza de friccin entre el plano inclinado y el anillo.

F f = Ma =

d) El mnimo valor del coeficiente de rozamiento esttico entre el plano y el anillo para que este se encuentre en rodadura pura debe de cumplir

1 Mgsen 2 Mgsen 1 k = = tan 2Mg cos 2 Ff = k N =


Ejemplo 70. Una barra uniforme AB de masa M y

c) La altura mxima que alcanzar el CM de la barra. Energa justo despus del choque

39

Cuerpo rgido

Hugo Medina Guzmn

1 l I O 2 Mg + mgd 2 2 1 2 2 = I O = Ml + md 3
= Mg

velocidad angular del bloque despus del choque.

Energa cuando alcanza el punto ms alto

l + mgd (1 cos ) 2
Solucin. Cantidad de movimiento angular antes del choque con respecto al eje O.

Por conservacin de energa: Energa justo despus del choque = energa cuando alcanza el punto ms alto.

1 l I O 2 Mg + mgd 2 2 l = Mg + mgd (1 cos ) 2 1 l I O 2 = Mg + mgd cos 2 2 1 I O 2 2 cos = l Mg + mgd 2


2

= mbv0 k L antes = r p = rmv0sen k

Para encontrar la cantidad de movimiento angular despus del choque, segn la figura siguiente.

(mv0 d ) 11 2 2 Ml + md 2 23 1 2 2 Ml + md 3 = l Mg + mgd 2
2 2 m 2 v0 d 1 l 2 M + md Ml 2 + md 2 g 3 2

L despus = I p + (m + M )b 2

Por conservacin de la cantidad de movimiento angular

L antes = L despus

= I p + (m + M )b 2 rmv0 sen k

hmx = l(1 cos )

rmv0 sen k I p + (m + M )b 2

d) El trabajo del proyectil cuando se incrusta contra la barra.

l 1 2 W = E = mv 0 Mg mgd 2 2

Ejemplo 72. Se tiene una plataforma circular que puede rotar sin friccin alrededor de un eje perpendicular al centro. E1 momento de inercia de la plataforma con respecto al eje es I p . Un insecto

1 l I O 2 + Mg mgd 2 2 1 2 1 2 = mv0 I O 2 2
- Ejemplo 71. Un bloque de masa M se pega a una plataforma circular, a una distancia b de su centro. La plataforma puede rotar, sin friccin, alrededor de un eje vertical alrededor de su centro. Siendo I p su

de masa m se coloca sobre la plataforma a una distancia b del eje. El sistema se hace girar con una velocidad angular 0 en el sentido horario. El insecto empieza a correr en una circunferencia de radio b alrededor del eje con una velocidad de magnitud constante v 0 , medida relativa a tierra. a) Cual es la cantidad de movimiento angular total si el insecto corre con la plataforma? b) Cul ser si corre en oposicin a la rotacin de la plataforma? c) Es posible que el pequeo insecto pueda detener la gran plataforma? Cmo? Solucin. La cantidad de movimiento angular del sistema antes que el insecto comience a correr es:

momento de inercia con respecto a sta. Si un proyectil de masa m que se mueve con una velocidad horizontal v 0 , como se muestra en la figura, incide y queda en el bloque. Encontrar la 40

Cuerpo rgido

Hugo Medina Guzmn

L = I p + mb 2 0 = I p + mb 2 0 k

Resolver usando la conservacin de la cantidad de movimiento angular.

a) Cuando el insecto corre en el mismo sentido del giro con mdulo de velocidad v 0 su cantidad de movimiento angular es:

L' = (I p + mb 2 ) ' mbvo k

Pero como la cantidad de movimiento angular es constante. La cantidad de movimiento angular total es:

Solucin. En la parte lisa no hay fuerza de friccin, en la parte spera aparece la tuerza de friccin, cuya lnea de accin est en el plano. Por tanto, la cantidad de movimiento angular del disco respecto a un punto de referencia en el plano permanecer Constante durante todo el movimiento (por ejemplo A). La cantidad de movimiento antes de llegar a A.

L' = L = (I p + mb 2 ) 0 k

L = r M v1 = I 0 1

1 = v1 k MR 2 , 1 = 1 k R 2

= Rv1 k , Como r v1 = rv1sen k


b) En este caso, como en el caso anterior

L' = L

I0 =

L' = I p + mb 2 0 k
c) Si es posible, tomando el caso a)

+ L = MRv1 k

1 = 1 MRv1 k MRv1 k 2 2

L' = I p + mb 2 ' mbvo k 2 = I + mb k

La cantidad de movimiento angular despus de pasar A y haber 1legado a rodar sin deslizar. Se traslada con velocidad v 2 tal que

2 =

La plataforma se detiene cuando

= I + mb 2 k mbv0 k p 0
Esto sucede cuando

' = 0 , es decir:

v2 . R

v0 =

(I

+ mb 2 ) mb

En el sentido indicado en el caso a).


Ejemplo 73. Se da a un cilindro homogneo de radio R y masa M con una velocidad horizontal v1

L' = r M v 2 = I 0 2

y una velocidad angular

las agujas del reloj 1 = v1 R en la parte sin rozamiento de la superficie horizontal. Ms all del punto A, cambia la superficie de manera que a la derecha de A el coeficiente de rozamiento es . 41

en sentido opuesto a

= Rv 2 k , Como r v 2 = rv 2 sen k
I0 =
1 = v2 k MR 2 , 2 = 2 k 2 R

1 MRv 2 k = 3 MRv 2 k L ' = MRv 2 k 2 2

Cuerpo rgido

Hugo Medina Guzmn

Igualando L' = L , tenemos:

3 = 1 MRv1 k v 2 = v1 MRv 2 k 3 2 2

= I f =

1 R MR 2 + m 2 4

Ejemplo 74. Un proyectil de masa m y velocidad v0 se dispara contra un cilindro slido de masa M y radio R. El cilindro est inicialmente en reposo montado sobre un eje horizontal fijo que pasa por su centro de masa. El proyectil se mueve perpendicular al eje y se encuentra a una distancia D < R sobre el eje. Calcular la rapidez angular del sistema despus que el proyectil golpea al cilindro y queda adherido a su superficie.

1 MR 2 + mR 2 f = 2 0 2 1 R 2 MR + m 2 16 M +m 1 2 0 = 2 M m + 2 16
b)

Ki =
=
Solucin. El momento angular del sistema se conserva, entonces
Kf =

1 11 2 2 I i0 = MR 2 + mR 2 0 2 22

1 (M + 2m )R 202 4
2 1 1 1 R 2 2 = + I f2 MR m f f 2 2 4 2

Li = L f 1 mv0 D = I = MR 2 + mR 2 2 mv0 D = 1 MR 2 + mR 2 2
Ejemplo 75. Un disco de masa M y radio R gira en un plano horizontal en torno a un eje vertical sin roce. Un gato de masa m camina desde el borde del disco hacia el centro. Si la rapidez angular del sistema es 0 cuando el gato est en el borde del disco, calcular: a) la rapidez angular cuando el gato ha llegado a un punto a R/4 del centro, b) la energa rotacional inicial y final del sistema. Solucin. Llamando Id al momento de inercia del disco e Ig al momento de inercia del gato, el momento de inercia total inicial y final del sistema es:

= 1 1

R 2 MR + m 2 2 4

M 2 + m 2 0 M 2 + m 16
2

m M + 2m 2 2 = 1 R 0 M +
2

8 M + m / 8

4 M + m/8 M + 2m Como >1 M + m/8

M + 2m 2 2 =1 (M + 2m )R 0

La energa rotacional aumenta.


Ejemplo 76. La barra horizontal de la figura tiene un momento de inercia respecto al eje de rotacin de 5x10-3 kg m2, y cada una de las bolas que pueden deslizar sobre ella pesan 50 g y se consideran de dimensiones despreciables. El conjunto est girando libremente alrededor del eje O-O con las bolas dispuestas simtricamente respecto al eje y sujetas por un hilo AB de 20 cm. Si se rompe el hilo cuando el conjunto gira a 20 rad/s, determinar la nueva velocidad angular cuando las bolas lleguen a los topes del extremo de la barra.

1 MR 2 + MR 2 2 2 1 R 2 I f = MR + m 2 4
Ii = Id + I g =
a) Como no hay torques externos sobre el sistema en torno al eje de rotacin, se puede aplicar la conservacin de la cantidad de movimiento angular

I ii = I f f

1 I f = MR 2 + mR 2 0 2
Solucin.

42

Cuerpo rgido

Hugo Medina Guzmn

Empecemos calculando el momento de inercia del conjunto, cuando las bolas estn separadas 20 cm. I1 = Ibarra + Ibolas = Ibarra + 2 m r12 = 5x10-3 kg m2 + 0,1x0,12 = 6x10-3 kg m2 Cuando se alejen hasta los topes: I2 = Ibarra + Ibolas = Ibarra + 2 m r22 = 5x10-3 kg m2 + 0,1x0,252 = 11,25x10-3 kg m2 La rotura del hilo libera fuerzas exclusivamente internas, por lo que se conservar la cantidad de movimiento angular del sistema:

f =

2 180 = 120 rpm. (2 + 1)

L1 = L2 I 11 = L2 2 I 6 2 = 1 1 = 20 = 10,67 rad / s 11,25 I2

Ejemplo 77. Un disco de 2 kg de masa y 10 cm de radio gira alrededor de su eje a 180 r.p.m.. Encima, pero sin que exista contacto, se encuentra otro disco de 1 kg de masa, del mismo radio y en reposo. Cuando el disco superior se deja caer, ambos se mueven solidariamente. Calcular la velocidad angular final.

GIROSCOPOS Y TROMPOS - MOVIMIENTO DE PRECESION El girscopo es una rueda montada en rodamientos sin friccin, en tal forma que la rueda tiene libertad de rotar en cualquier direccin con respecto al marco que lo sujeta. Para lograr esto se necesitan tres gmbalos (correspondientes a los tres espacios dimensionales). Como los rodamientos no tienen friccin no se ejercen torques sobre la rueda. Esto significa que una vez iniciado el giro, el eje de rotacin permanecer fijo no importando que movimiento se de al mareo exterior. La direccin en el espacio del eje no variar. Hasta ahora vimos el movimiento rotacional en que el eje de rotacin est fijo, o tiene movimiento de traslacin sin cambio en su direccin. La mayora de los movimientos rotacionales quedan en estas categoras, pero en el caso de un trompo o girscopo en rotacin no se cumple lo anterior. Si se hace girar rpidamente el rotor de este aparato y luego se coloca un extremo libre del eje de rotacin sobre un soporte fijo, como se muestra en la figura. El girscopo no caer del soporte sino que se mantiene en posicin casi horizontal mientras que el eje de su rotor gira lentamente en un plano horizontal, esta rotacin lenta del eje se conoce como PRECESION.

Solucin.

Cuando el disco superior se posa sobre el inferior, el torque de las fuerzas sigue siendo nulo por lo que se conserva la cantidad de movimiento angular, I .

(I )Antes = (I )Despus
I1i = (I1 + I 2 ) f f = I1 i I1 + I 2

Veamos como se origina la precesin. Consideremos un girscopo simplificado mostrado en la figura siguiente, un disco cilndrico muy macizo de masa M y radio a que tiene libertad para girar sin friccin en torno a una varilla muy ligera y delgada, a lo largo de su eje.

Como el Momento de inercia de un disco es .m.R2 se obtiene:

1 m1 R 2 m1 2 f = i = 1 (m1 + m2 ) i 1 2 2 m1 R + m2 R 2 2
En este caso particular: 43 Un extremo de la varilla se apoya en A. que est a una distancia l del disco. Si se mantiene la varilla

Cuerpo rgido

Hugo Medina Guzmn es de 2,7 kg m2. La velocidad angular inicial de la rueda es de 55 rad/s en sentido antihorario. En un momento dado la profesora gira 180 el eje de la rueda pasando a girar con -55 rad/s en sentido contrario al anterior. Calcular: a) La velocidad angular adquirida por la silla y el sentido de giro. b) El trabajo realizado por la profesora.

horizontal, y se hace girar al disco con una velocidad angular en torno a su eje y luego, se suelta. Como actan dos nicas fuerzas el peso Mg y la reaccin del apoyo R, podra pensarse que el disco caera. Si L0 fuera cero sucedera esto, pero el torque que produce Mg es:

= Mgl = (li ) Mgk j


d L = dt = Mgl j dt

este torque produce un cambio en la cantidad de movimiento angular

la magnitud. de este cambio es: Por otra parte: dL = L0 d De aqu Mgldt = L0 d y Como L0 = L0 =

dL = Mgldt

d Mgl = dt L0

d Mgl = 1 dt Ma 2 2

1 Ma 2 ; 2 2 gl = 2 a

Solucin. a) Dado que no hay momentos externos sobre la silla giratoria podemos considerar que el momento angular no vara. L1 = I RUEDA 1 ,

Por consiguiente el disco no caer, en lugar de ello girar en el plano horizontal xy (ver la figura siguiente) en torno al eje vertical a travs del punto de apoyo A.

L2 = I RUEDA ( 1 ) + I SILLA 2 I RUEDA 1 = I RUEDA ( 1 ) + I SILLA 2 2I 2 = RUEDA 1 I SILLA 2(0,02) 2 = 55 = 8,15 rad /s 2,7

(Positivo, por tanto en el sentido de rotacin inicial de la rueda) b)

W = E = E 2 E1
= La velocidad angular de esta precesin es:

1 1 1 2 2 2 I SILLA 2 + I RUEDA ( 1 ) I RUEDA 1 2 2 2

d 2 gl = = dt I a 2

1 2 = 89,6 J I SILLA 2 2

Ejemplo 78. Una profesora de fsica se encuentra sentada en una silla giratoria manteniendo en sus manos una rueda de bicicleta como se indica en la figura. El momento de inercia de la rueda respecto a su eje es de 0,2 kg m2, y el momento de inercia de la profesora ms la rueda respecto del eje de la silla

El trabajo es por tanto la energa adquirida por la silla, ya que la energa de la rueda no vara. Dicho trabajo, positivo, es producido por la fuerza muscular (interna) de la profesora.

PREGUNTAS Y PROBLEMAS 1. El centro de masa de una pelota de radio R, se mueve a una rapidez v. La pelota gira en torno a un eje que pasa por su centro de masa con una rapidez angular . Calcule la razn entre la energa

rotacional y la energa cintica de traslacin. Considere la pelota una esfera uniforme.


2. Un volante en la forma de un cilindro slido de radio R = 0,6 m y masa M = 15 kg puede llevarse

44

Cuerpo rgido

Hugo Medina Guzmn

hasta una velocidad angular de 12 rad/s en 0,6 s por medio de un motor que ejerce un torque constante. Despus de que el motor se apaga, el volante efecta 20 rev antes de detenerse por causa de la friccin (supuesta constante). Qu porcentaje de la potencia generada por el motor se emplea para vencer la friccin? Respuesta. 2.8%.
3. Un bloque de masa m1 y uno de masa m2 se conectan por medio de una cuerda sin masa que pasa por una polea en forma de disco de radio R, momento de inercia I y masa M. As mismo, se deja que los bloques se muevan sobre una superficie en forma de cua con un ngulo como muestra la figura. El coeficiente de friccin cintico es para ambos bloques. Determine a) la aceleracin de los dos bloques y b) la tensin en cada cuerda.

. Respuesta. a) 2(Rg/3), b) 4(Rg/3), c) (Rg).


6. Un peso de 50 N se une al extremo libre de una cuerda ligera enrollada alrededor de una pelota de 0,25 m de radio y 3 kg de masa. La polea puede girar libremente en un plano vertical en torno al eje horizontal que pasa por su centro. El peso se libera 6 m sobre el piso. a) calcular la tensin de la cuerda, la aceleracin de la masa y la velocidad con la cual el peso golpea el piso. b) Calcular la rapidez con el principio de la conservacin de la energa. Respuesta. a) 11,4N, 7,6 m/s2, 9,5 m/s, b) 9,5 m/s. 7. Una ligera cuerda de nylon de 4 m est enrollada en un carrete cilndrico uniforme de 0,5 m de radio y 1 kg de masa. El carrete est montado sobre un eje sin friccin y se encuentra inicialmente en reposo. La cuerda se tira del carrete con una aceleracin constante de 2,5 m/s2. a) Cunto trabajo se ha efectuado sobre el carrete cuando ste alcanza una velocidad angular de 8 rad/s? b) Suponiendo que no hay la suficiente cuerda sobre el carrete, Cunto tarda ste en alcanzar esta velocidad angular? c) Hay suficiente cuerda sobre el carrete? Respuesta. a) 4 J, 1,6 s, c) s. 8. Una barra uniforme de longitud L y masa M gira alrededor de un eje horizontal sin friccin que pasa por uno de sus extremos. La barra se suelta desde el reposo en una posicin vertical. En el instante en que est horizontal, encuentre a) su rapidez angular, b) la magnitud de su aceleracin angular, c) las componentes x e y de la aceleracin de su centro de masa, y d) las componentes de la fuerza de reaccin en el eje. Respuesta. a) (3g/L), b) 3g/2L, c) (3/2 + )g, d) (-3/2 + )Mg.

Respuesta. a) (m2sen - )(m1 + m2cos )g/(m1 + m2 + M), b) T1 = m2g + m1a, T2 = T1 + Ma. 4. Una masa m1 y una masa m2 estn suspendidas por una polea que tiene un radio R y una masa m3. La cuerda tiene un masa despreciable y hace que la polea gire sin deslizar y sin friccin. Las masas empiezan a moverse desde el reposo cuando estn separadas por una distancia D. Trate a la polea como un disco uniforme, y determine las velocidades de las dos masas cuando pasan una frente a la otra.

5. Un disco slido uniforme de radio R y masa M puede girar libremente sobre un pivote sin friccin que pasa por un punto sobre su borde. Si el disco se libera desde el reposo en la posicin mostrada por el crculo. a) Cul es la rapidez de su centro de masa cuando el disco alcanza la posicin indicada en el crculo punteado? b) Cul es la rapidez del punto ms bajo sobre el disco en la posicin de la circunferencia punteada? c) Repetir para un aro uniforme

9. Los bloques mostrados en la figura estn unidos entre si por una polea de radio R y momento de inercia I. El bloque sobre la pendiente sin friccin

45

Cuerpo rgido

Hugo Medina Guzmn

se mueve hacia arriba con una aceleracin constante de magnitud a. a) Determine las tensiones en las dos partes de la cuerda, b) encuentre el momento de inercia de polea.

Respuesta. x = Respuesta. a) T1 = m1 (a + gsen ) ,

L (1 s ) 2

T2 = m2 ( g a ) g 2 g b) m2 R m1R 2 m2 R 2 m1R 2 sen a a

12. Determinar la tensin en el cable AB que Impide que el poste BC deslice. En la figura se ven los datos esenciales. La masa del poste es de 18 kg. Suponer que todas las superficies son lisas.

10. Un cuerpo plano est sometido a cuatro fuerzas como se indica en la figura. a) Hallar el mdulo y direccin del torque actuante respecto a un eje perpendicular al plano y que pasa por el punto A. b) Respecto a un eje que pasa por el punto B. e) Respecto a un eje que pasa por el punto C. d) Determinar la fuerza equivalente y su lnea de accin. e) Sustituir esta fuerza por otra que est aplicada en A y un par de fuerzas o cupla aplicadas en los puntos B y C y hallar el valor mnimo de estas fuerzas.

Respuesta. T = 46,2 N 13. Un hombre de 70 kg, sostiene un objeto de 31,9 kg. Como se indica en la figura. La polea carece de rozamiento. La plataforma sobre la que est situado el hombre est colgada mediante dos cuerdas en A y otras dos en B. Cul e tensin de una de las cuerdas en A?

Respuesta. a) = 23 Nm, b) = 23 Nm, c) = 24 Nm,

+ d) F = i j , y = x 23 ,
e) FB =
23 ( 3i + 4 j ) = FC 25

Respuesta. 124,5 N 14. Reemplace la fuerza de 1000 N de la figura por una fuerza que pasa por A y una cupla cuyas fuerzas actan verticalmente a travs de B C.

11. Un marco cuadrado de lado L. Se cuelga de un clavo rugoso de coeficiente de rozamiento esttico s . A qu distancia del vrtice est clavado si el

marco est a punto de deslizar? 46

Cuerpo rgido

Hugo Medina Guzmn

Respuesta. 3,82 s 18. Si se aplica La fuerza F a una cuerda ligera atada a un bloque con el sistema de poleas mostrado en la figura. Cul es el mximo peso que puede levantar?

+ 600 Respuesta. FA = 800i j , FB = 467 j,


FC = 467 j
15. Un hombre de 60 kg que camina a 2 m/s atraviesa un tabla de 30 kg y 10 m de larg a) Cul es la fuerza sobre el soporte B en funcin d tiempo? b) Si la mxima fuerza que puede resistir B es 490 Cundo y dnde caer al ro el hombre? Considerar que el peso del hombre siempre acta en direccin de la vertical que pasa por su centro de masa.

Respuesta. 3F 19. El rodillo que se ve en la figura tiene una masa de 339 kg Que fuerza F es necesaria para subir el rodillo sobre el bloque?

Respuesta: a) FB = (12t + 15)9,8 N, b) t = 2,92 s, x = 5,83 m de A. 16. Un hombre de masa m quiere subir por una escalera. La escalera tiene masa M, largo L y forma un ngulo con e piso. El coeficiente de friccin entre la escalera y e peso es , mientras que la pared no tiene friccin. a)A qu altura de la escalera puede llegar antes que comience a resbalar? b) Si el ngulo es el mayor sin que la escalera sola puede estar sin resbalar, cul es la altura a la que puede llegar el hombre? Respuesta. a)

Respuesta. F =3949,4 N 20. La lnea de accin de una fuerza de 1N est en el plano xz y corta el eje z en un punto que dista 0,6 m del origen. a) Cul es el torque respecto al eje y si el ngulo comprendido entre la direccin de la fuerza y el eje z es 60? b) Si el ngulo e l80? c) Si el ngu1o es 330? Respuesta. a) = 0,52 N m , b) = 0 c) = - 0,3 N m 21. Dos discos de masa 10 kg y radio R = 0,3 m cada uno estn conectados mediante una cuerda. En el instante mostrado en la figura, la velocidad angular del disco B es de 20 rad/s en sentido horario. Calcular cunto sube el disco A cuando la velocidad angular del disco B sea de 4 rad/s.

(m + M )Lsen 1 ML cos M cos 2 b) L 2


17. El disco A tiene una masa de 2 kg y un radio de 7,5 cm, se coloca en contacto con una correa que se mueve con una velocidad v = 15 m/s. Sabiendo que el coeficiente de rozamiento entre el disco y la correa es 0,2, calcular tiempo necesario para que el disco alcance una velocidad angular constante.

47

Cuerpo rgido

Hugo Medina Guzmn

Respuesta.

2 3mgR

Respuesta. 1,54 m 22. Un cilindro de masa ni y radio r rueda sin deslizar sobre la cara interior de una superficie cilndrica de radio R. Sabiendo que la esfera parte del raposo en la posicin indicada en la figura, obtener: a) La velocidad de la esfera al paso por B. b) El mdulo de la reaccin normal en cada instante.

25. Una esfera de l00 kg de masa y 0,6 m de dimetro baja rodando, partiendo del reposo, por un plano inclinado 25. recorriendo 30 m.. a) Cul es su energa cintica al cabo de los 30 m? b) Cul es la velocidad de su centro de masa? Respuesta. a) 1268 kg m, b) 13,3 m/s 26. Un pasajero viaja de pie en un mnibus. El mnibus se mueve con una velocidad de 50 km/h cuando el conductor aplica los frenos. El mnibus desacelera de modo uniforme durante una distancia de 15 ni hasta detenerse. Qu ngulo respecto a la vertical deber inclinarse el pasajero para evitar su cada? Respuesta. 33,27 hacia atrs. 27. a) Cmo podra distinguirse una esfera de oro de otra de plata si ambas tuviesen el mismo peso, el mismo radio y las dos estuvieron pintadas del mismo color? b) Cmo podra distinguir un huevo duro de uno fresco si estuvieran juntos? 28. Un carrete cilndrico hueco y uniforme tiene radio interior R/2, radio exterior R y masa M . Est montado de manera que gira sobre un eje horizontal fijo. Una masa m se conecta al extremo de una cuerda enrollada alrededor del carrete. La masa m desciende a partir del reposo una distancia y durante un tiempo t. Demuestre que el torque debido a la fuerza de roce entre el carrete y el eje es:

Respuesta. a)

4 g (R r )(1 cos ) , 3

b)

mg (7 4 cos ) 3

23. A que altura sobre la mesa debe golpearse una bola de billar con un taco mantenido horizontalmente para que la bola comience su movimiento sin rozamiento entre ella y la mesa?

= R m g 2 2 M 2 t 4 t

Respuesta. 7/5R 24. Un cilindro homogneo de masa m y radio R descansa sobre un plano horizontal. Se aplica un torque, segn se indica en la figura. Hallar el valor del coeficiente de rozamiento entre la rueda y el plano para que aparezca rodadura pura.

29. Un cilindro de 10 kg de masa rueda sin deslizar sobre una superficie horizontal. En el instante en que se su centro de masa tiene una rapidez de 10 m/s, determine: a) la energa cintica traslacional de su centro de masa,

48

Cuerpo rgido

Hugo Medina Guzmn

b) la energa rotacional de su centro de masa, y c) su energa total. Respuesta. a) 500 J, b) 250 J, c) 750 J.
30. Una esfera slida tiene un radio de 0,2 m y una masa de 150 kg. Cunto trabajo se necesita para lograr que la esfera ruede con una rapidez angular de 50 rad/s sobre una superficie horizontal? (Suponga que la esfera parte del reposo y rueda sin deslizar). 31. Un disco slido uniforme y un aro uniforme se colocan uno frente al otro en la parte superior de una pendiente de altura h. Si se sueltan ambos desde el reposo y ruedan sin deslizar, determine sus rapideces cuando alcanzan el pie de la pendiente Qu objeto llega primero a la parte inferior? 32. Una bola de boliche tiene una masa M, radio R y un momento de inercia de (2/5)MR2. Si rueda por la pista sin deslizar a una rapidez lineal v, Cul es su energa total de funcin de M y v? Respuesta. 0,7Mv2. 33. Un anillo de 2,4 kg de masa de radio interior de 6 cm y radio exterior de 8 cm sube rodando (sin deslizar) por un plano inclinado que forma un ngulo de = 37 con la horizontal. En el momento en que el anillo ha recorrido una distancia de 2 m al ascender por el plano su rapidez es de 2,8 m/s. El anillo continua ascendiendo por el plano cierta distancia adicional y despus rueda hacia abajo. Suponiendo que el plano es lo suficientemente largo de manera que el anillo no ruede fuera en la parte superior, qu tan arriba puede llegar? 34. Una barra rgida ligera de longitud D gira en el plano xy alrededor de un pivote que pasa por el centro de la barra. Dos partculas de masas m1 y m2 se conectan a sus extremos. Determine la cantidad de movimiento angular del sistema alrededor del centro de la barra en el instante en que la rapidez de cada partcula es v. Respuesta. ( m1 + m2)vD. 35. Un pndulo cnico consta de masa M que se mueve en una trayectoria circular en un plano horizontal. Durante el movimiento la cuerda de longitud L mantiene un ngulo constante con la vertical. Muestre que la magnitud de la cantidad de movimiento angular de la masa respecto del punto de soporte es:

a) el origen, b) el punto ms alto de su trayectoria, c) justo antes de chocar con el suelo.


Respuesta. a) 0, b)
3 mv0 sen 2 cos , 2g

c)

3 2mv0 sen 2 cos g

37. Un disco slido uniforme de masa M y radio R gira alrededor de un eje fijo perpendicular su cara. Si la rapidez angular es , calcular la cantidad de movimiento angular del disco cuando el eje de rotacin a) pasa por su centro de masa, y b) pasa por un punto a la mitad entre el centro y el borde. 38. Una partcula de 0,4 kg de masa se une a la marca de 100 cm de una regla de 0,1 kg de masa. La regla gira sobre una mesa horizontal sin friccin con una velocidad angular de 4 rad/s. Calcular la cantidad de movimiento angular del sistema cuando la regla se articula en torno de un eje, a) perpendicular a la mesa y que pasa por la marca de 50 cm, b) perpendicular a la mesa y que pasa por la marca de 0 cm. Respuesta. a) 0,43 kgm2/s, b) 1,7 kgm2/s. 39. Una mujer de 60 kg que est parada en el borde de una mesa giratoria horizontal que tiene un

momento de inercia de 500 kgm2 y un radio de 2 m. La mesa giratoria al principio est en reposo y tiene libertad de girar alrededor de un eje vertical sin friccin que pasa por su centro. La mujer empieza a caminar alrededor de la orilla en sentido horario (cuando se observa desde arriba del sistema) a una rapidez constante de 1,5 m/s en relacin con la Tierra. a) En qu direccin y con qu rapidez angular gira la mesa giratoria b) Cunto trabajo realiza la mujer para poner en movimiento la mesa giratoria? Respuesta. a) 0,36 rad/s, antihorario.
40. Una barra uniforme de masa M y longitud d gira en un plano horizontal en torno de un eje vertical fijo sin friccin que pasa por su centro. Dos pequeas cuentas, cada una de masa m, se montan sobre la barra de manera tal que pueden deslizar sin friccin a lo largo de su longitud. Al principio las cuentas se fijan por medio de retenes ubicados en las posiciones x (donde x < d/2) a cada lado del centro, tiempo durante el cual el sistema gira una rapidez angular . Repentinamente, los retenes se quitan y las pequeas cuentas se deslizan saliendo de la barra. Encuentre, a) la rapidez angular del sistema en el instante en que las cuentas alcanzan los extremos de la barra, y

L=

gM 2 L3 sen 4 cos

36. Una partcula de masa m se dispara con una rapidez vo formando un ngulo con la horizontal. Determine la cantidad de movimiento angular de la partcula respecto del origen cuando sta se encuentra en:

49

Cuerpo rgido

Hugo Medina Guzmn

b) la rapidez angular de la barra despus de que las cuentan han salido de ella.
41. Un bloque de madera de masa M que descansa sobre una superficie horizontal sin friccin est unido a una barra rgida de longitud l y masa despreciable. La barra gira alrededor de un pivote en el otro extremo. Una bala de masa m que se desplaza paralela a la superficie horizontal y normal a la barra con rapidez v golpea el bloque y queda incrustada en l. a) Cul es la cantidad de movimiento angular del sistema bala-bloque? b) Qu fraccin de la energa cintica original se pierde en la colisin? Respuesta. a) mvl , b) M/(M+m). 42. Una cuerda se enrolla alrededor de un disco uniforme de radio R y masa M. El disco se suelta desde el reposo con la cuerda vertical y su extremo superior amarrado a un soporte fijo. A medida que el disco desciende, demuestre que a) la tensin en la cuerda es un tercio del peso del disco. b) La magnitud de la aceleracin del centro de masa es 2g/3, y c) la rapidez del centro de masa es (4gh/3). Verifique su respuesta a la pregunta c) utilizando mtodos de energa. 43. Una pequea esfera slida de masa m y de radio r rueda sin deslizar a lo largo de la pista mostrada en la figura. Si parte del reposo en la parte superior de la pista a una altura h, donde h es grande comparada con r a) Cul es el valor mnimo de h (en funcin de R) de modo que la esfera complete la trayectoria? b) Cules son las componentes de fuerza de la esfera en el punto P si h = 3R?

45. A una bola de boliche se le da una rapidez inicial vo en una canal de manera tal que inicialmente se desliza sin rodar. El coeficiente de friccin entre la bola y la canal es . Demuestre que durante el tiempo en que ocurre el movimiento de rodamiento puro, a) la rapidez del centro de masa de la bola es 5vo/7, y b) la distancia que recorre es 12 vo2/49 g. (Sugerencia: Cuando ocurre el movimiento de rodamiento puro, vcm = R. Puesto que la fuerza de friccin proporciona la desaceleracin, a partir de la segunda ley de Newton se concluye que acm = g.) 46. El alambre de un carrete de masa M y radio R se desenrolla con una fuerza constante F. Suponiendo que el carrete es un cilindro slido uniforme que no desliza, muestre que, a) la aceleracin del centro de masa es 4F/3M, y b) la fuerza de friccin es hacia la derecha y su magnitud es igual a F/3. c) Si el cilindro parte del reposo y rueda sin deslizar, Cul es la rapidez de su centro de masa despus que ha rodado una distancia D? Respuesta. c) (8FD/3M).

44. Un proyectil de masa m se mueve a la derecha con rapidez v0. El proyectil golpea y queda fijo en extremo de una barra estacionaria de masa M y longitud D que est articulada alrededor de un eje sin friccin que pasa por su centro. a) Encuentre la rapidez angular del sistema justo despus de la colisin. b) Determine la prdida fraccionaria de energa mecnica debida a la colisin.

47. Suponga un disco slido de radio R al cual se le da una rapidez angular o alrededor de un eje que pasa por su centro y despus se baja hasta una superficie horizontal y se suelta, como en la. Suponga tambin que el coeficiente de friccin entre el disco y la superficie es . a) Calcular la rapidez angular del disco una vez que ocurre el rodamiento puro. b) Calcular la prdida fraccionaria de energa cintica desde el momento en que el disco se suelta hasta que ocurre el rodamiento puro c) Muestre que el tiempo que tarda en ocurrir el movimiento de rodamiento puro es R o/3 g. d) Muestre que el tiempo que recorre el disco antes de que ocurra el rodamiento puro es R2 o 2/18 g.

50

Cuerpo rgido

Hugo Medina Guzmn

Respuesta.

48. La figura muestra un carrete de alambre que descansa sobre una superficie horizontal. Cuando se tira, no se desliza en el punto de contacto P. El carrete se tira en las direcciones indicadas por medio de los vectores F1, F2, F3 y F4. Para cada fuerza determine la direccin en que rueda el carrete. Advierta que la lnea de accin de F2 pasa por P.

M r2 m + R2 M r2 reposo solo si m + R2 1

= sen -1

. Estar en

51. Los discos A y B son del mismo material y tienen el mismo espesor, pudiendo girar 1ibemente alrededor de un eje vertical. El disco B se encuentra en reposo cuando se deja caer sobre el disco A. el est girando con una velocidad angular de 400 rpm. Sabiendo que la masa del disco A es de 4 kg, calcular: a) La velocidad angular final de los discos. b) La variacin de la energa cintica experimentada por el sistema. R A = 0,1 m, RB = 0,15 m,

49. El carrete mostrado en la figura tiene un radio interior r y un radio externo R. El ngulo entre la fuerza aplicada y la horizontal puede variar. Demuestre que el ngulo crtico para el cual el carrete no rueda y permanece estacionario est dado por cos = r/R. (Sugerencia: En el ngulo crtico la lnea de accin de la fuerza aplicada pasa por el punto de contacto.)

Respuesta. a) 334 rpm, .b).- 6,5l J 52. Una bala de 3g se dispara, con una velocidad horizontal de 550 m/s, contra. Una varilla de madera AB de longitud L = 0,750 m. La varilla que inicialmente est en reposo, se encuentra suspendida de una cuerda de longitud L = 0,750 m. Sabiendo que h = 0,150 m, calcular las velocidades de cada uno de los extremos de la varilla inmediatamente despus de que la bala se haya incrustado.

50. Se tiene un carrete sobre un plano inclinado, el cual tiene enrollado un hilo delgado y su extremo libre sujeta una masa m por medio de una polea sin friccin y masa despreciables. Se asume que la masa del carrete M est distribuida uniformemente en un crculo de radio R. Determinar el ngulo de inclinacin al cul el centro de gravedad del carrete estar en reposo.

, v B = 6,22i Respuesta. v A = 0,566i


53. Un tabln masa M se apoya sobre un pequeo pivote D. Un gimnasta A de masa m est de pie sobre el extremo C del tabln, un segundo gimnasta B de la misma masa m salta desde la altura h y cae

51

Cuerpo rgido

Hugo Medina Guzmn

sobre el tabln en E. Suponiendo que este choque es perfectamente inelstico, determinar la altura que alcanzar el gimnasta A. (El gimnasta A permanece de pie completamente rgido).

Respuesta. = 2,1 rad/s 55. Una rueda de bicicleta de 82 cm de dimetro tiene una platina de acero enrollada en su parte exterior de modo que la masa resultante del sistema puede suponerse que est situada toda ella en la periferia de la rueda, siendo M = 7,3 kg sosteniendo los dos extremos del eje con las manos en la posicin horizontal. El eje sobresale 15,2 cm a cada lado de la rueda. Mientras la rueda est girando con una velocidad angular de 25,12 rad/s se hace girar el eje con las manos en un plano horizontal alrededor de su centro. Calcular el valor y direccin de la fuerza que deber ejercer en cada mano para producir una velocidad angular de precesin de 0,628 rad/s alrededor del centro. Respuesta. un par de fuerzas de 64,6 N aplicadas en cada extremo del eje.

Respuesta.

(2m + M 3)2

m2h

54. Un disco macizo de 1,2 kg de masa y 10 cm de dimetro est montado en un extremo de un eje de masa despreciable que est pivotado alrededor de un punto a 6 cm del, centro del disco en el otro extremo del eje, a una distancia de 10 cm del pivote, se cuelga un objeto de 0,96 kg de masa. Si la velocidad angular de giro del disco es 37,37 rad/s. Cul es la velocidad de precesin?

52

S-ar putea să vă placă și